You are on page 1of 472

A-PDF Merger DEMO : Purchase from www.A-PDF.

com to remove the watermark

Answer one question from this Paper.

Answers should be between 500 and 800 words in length.

1 ‘A life spent primarily on the pursuit of material things is a wasted life.’ To what
extent do you agree?

2 Which is more important in the modern world - logic or imagination?

3 ‘Far too much attention is given to young people and not enough to the
elderly.’ To what extent is this statement reflective of the situation in your
society?

4 ‘Politics should stay out of the world of sports.’ Is that possible?

5 Our scientific power has outrun our spiritual power. How far do you agree with
this statement?

6 Do you agree that today the Arts are more about form than function?

7 ‘The only thing necessary for evil to triumph is for good men to do nothing.’
Assess the validity of this statement in light of modern-day history.

8 How far do you agree that ethics are more important than profits in business
today?

9 Is the idea of keeping our environment for posterity a mere dream?

10 ‘Yes, We Can.’ (Barack Obama) Is such optimism too idealistic in today’s


world?

11 Globalisation has rendered the concepts of nationalism and patriotism


obsolete. Discuss.

12 ‘Our generation is better educated than our parents’ generation but we are not
wiser than them.’ Discuss.

2
ANGLO-CHINESE JUNIOR COLLEGE
JC2 PRELIMINARY EXAMINATION 2010

GENERAL PAPER 8806/02


Paper 2 16 August 2010

INSERT 1 hour 30 minutes

READ THESE INSTRUCTIONS FIRST

This insert contains the passages for Paper 2.

This document consists of 3 printed pages.

GP 8806_2 Anglo-Chinese Junior College


General Paper Department
[Turn over]
ACJC 2010
Passage 1: Matt Frei writes about the Noughties.

1 As the first decade of the 21st Century clatters to a close, some of us have been wondering how
best to sum up the Noughties 1. The name is unfortunate because ‘naughty’ is probably one
adjective that doesn't spring to mind unless you see everything through the prism of one particular
golfer. Time magazine has called the 00s ‘The Decade from Hell’. That seems a little unkind. After
all, it hasn't been that bad for China, India or TV personality Simon Cowell. Perhaps we should just 5
call the decade ‘unnerving’ or ‘unsettling’ or even ‘unhinged’.

2 9/11 was THE event that set the tone. But I don't buy the idea that the signature phenomenon of
this decade has been the conflict between Lady Liberty and the forces of extremism. I think the
biggest impact of the decade, the one that has set the tone and defined the era, has come from
YOU! Time magazine nailed it in 2006 when it made YOU ‘Person of the Year’ by putting a mirror 10
on its front cover. Even Barack Obama kept saying it during an election campaign in which he used
the web to recruit 13 million foot-soldiers and fundraisers: "It's all about you!" I still remember the
first personal e-mail I got from him. "Dear Matt," it started. An e-mail just for me … and a few million
others. At one stage of the campaign, I was on first-name terms with virtually every Presidential
candidate. The Internet has been largely responsible. From Googlemania to Wikipedia to MySpace, 15
Facebook and YouTube, YOU have become the star.

3 And when it wasn't about YOU, it was about ME. First came the iPod. Then the iPhone. What's
next? The iWife or iHusband? Whether it's voting on American Idol or bargaining on eBay, you
have grasped the power of the mouse and the text message. You have done so in the world of
entertainment and politics. In Iran, you students and housewives used your mobile phones and 20
computers to record and broadcast footage that almost toppled the government after disputed
elections. In China, you have overcome rural isolation and poverty thanks to the Internet. You may
have lost your job in the garment factory in Guangdong, but you returned to your village armed with
a web address and expectations. In Botswana, mobile phones are now doing what landlines could
never do. And an undersea cable has just brought faster-than-fast broadband to Kenya. 25

4 But the power of you hasn't just been enhanced and underpinned by mobile phone technology or
by the Internet. Greater connectivity has gone hand-in-hand with a greater sense of entitlement,
especially the entitlement to information. Remember how the British MPs' expenses scandal was
first uncovered by an American researcher who was incensed that authorities were denying her
access to public information about what MPs spent on their second homes? 30

5 The decade also spawned a sense of entitlement to prosperity, lubricated for much of the decade in
much of the world by easy credit. Even if we were pay cheque paupers, so many of us could
pretend to live like millionaires. We all know how that has ended. And this is where we get to the
fundamental conflict of the unnerving Noughties. You have increasingly locked horns with the
institutions that are supposed to represent you or act in your interest. You have lost faith in the 35
banks to which you have entrusted your money, in the politicians to whom you have pledged your
votes and the companies that have put food on your table. You have become inquisitive, suspicious
and picky. And when you can do something about it on Twitter or your own blog, you have
flourished.

6 But there are some things over which you have no control: like how your garbage is collected, how 40
wars are fought on your behalf or how politicians govern in your name. And that has made you
frustrated. The biggest conflict in the past decade has been between You and Your expectations. If
the Noughties have been unnerving, what can we expect from the next decade? They will after all
be called ‘The Teens’.

1
Noughties: A proposed, slightly humorous, name for the decade from 2000 to 2009.
2
Passage 2: Bridget Delaney writes on “Ah, the Noughties, the decade when we cast off reality."

1 The past 10 years were the 'decade from hell', according to Time magazine; Martin Amis dubbed it
the age of horrorism and The New York Times said Tiger Woods personified the sordid reality of
the Noughties. But for me, the plastic, malleable Barbie doll is the symbol of our age. She doesn't
look or act real, yet we can project any number of desires and scenarios onto her. In the Noughties,
we wanted to be her. 5

2 The narrative of this decade has lacked authenticity. We have stopped prizing things of enduring
value, things that might be perceived as natural and flawed, yet are still substantial - and swapped
them for inauthentic things: a beauty ideal that is more plastic than fantastic and 'reality TV'
celebrities whose rise and fall is based on constructed personas. Our houses have come to
resemble Barbie houses - with more bedrooms than we need, his 'n' hers matching four-wheel- 10
drives, heavy ''it'' handbags and statement heels - all of which led people to record personal credit
card debt in recent years. We wanted to look like Barbie too. Women’s magazines and
entertainment blogs maintained an almost Taliban-like policing of women's bodies, except that
these gatekeepers were monitoring flaws. A starlet who showed a bit of dimpled thigh, or unwaxed
leg or, heaven forbid, a belly or double chin, would wear the scarlet letter of a magazine spread 15
with the offending body part circled in red with the legend: ''Britney (or substitute name) fat.''
Nothing less than perfection would do. Botox was normalised, hair extensions were fitted and spray
tans became a sort of de facto national complexion.

3 At the fag-end of this decade, some actors looked more youthful than they did in the '90s. But their
performances were less real, their foreheads immobile, their lineage curious (if she is playing the 20
mother, why does she look younger than the daughter?) and they had disabled their ability to
emote, surely an essential part of their craft. Dominating the art of inauthenticity was reality TV
which was the most contrived and predictable genre of television. There was always a ‘bad girl’, the
''good listener'', the alpha-male, the peace-maker, the girl with the ridiculous boob job. People
became characters, their real selves edited out or turned into an exaggeration or a caricature that 25
would play well in the half-hour package at night. The boring boy always won. When the audience
occasionally voted for someone authentic, it seemed to throw this new order. Casey Donovan, the
Australian Idol winner who was not the blonde, skinny Barbie ideal of pop stars had the best voice,
yet in the world of record companies,photo shoots and glossy magazines, she sank quickly.

4 Producers may have sensed our desire for something real, so they gave us Susan Boyle - the 30
middle-aged Scot; unworldly and unadorned, living with her cat in her village and, so the legend
goes, 'never been kissed'. But even then, watching her perform on a reality TV show, gave a sense
that something had been staged, that this claim of being authentic was not as real as we were
being told. Maybe our cynicism too is a by-product of the age.

5 This was an age when even the news event of the decade was relayed and remembered in a form 35
that seemed inauthentic. The destruction of the World Trade Centre by terrorists who had hijacked
commercial aircraft provided the most startling, original and previously unimaginable footage
beamed around the world. Soon enough this event became rendered with a sort of unreality. We
borrowed our descriptions from Hollywood: ''Just like Die Hard 2'', or ''like that Tom Cruise movie
with the giant insects''. There seemed to be nothing rich in our culture, no allegory or collective tale 40
or wisdom that we could reach for. Our collective jewellery box, our store of stories and culture and
wisdom, is now filled with tat.

6 The past decade has seen to that. We have collected costume jewellery for our Barbie lives -
plastic, disposable and ultimately worthless. But a new decade is a new beginning. Let's throw
away our toys from the Noughties and start acting like grown-ups. 45

3
4
2

Read the passages in the insert and then answer all the questions which follow below. Note For
Examiner’s
that up to fifteen marks will be given for the quality and accuracy of your use of English Use
throughout this Paper.

Note: When a question asks for an answer IN YOUR OWN WORDS AS FAR AS POSSIBLE
and you select the appropriate material from the passages for your answer, you must still use
your own words to express it. Little credit can be given to answers which only copy words or
phrases from the passages.

Questions on Passage 1

1 From paragraph 1,
(i) why does the writer consider the term ‘Noughties’ (line 2) unfortunate? Use your own
words as far as possible.

[1]

(ii) why is Time Magazine’s description of life in the Noughties ‘unkind’ (line 4)? Use your
own words as far as possible.

[1]

2 Explain the irony expressed in the following lines: ‘I still remember the first personal
email I got from him … "Dear Matt," it started. An email just for me… and a few million
others.’ (lines 12 - 14) Use your own words as far as possible.

[1]
3

For
3 From paragraph 3, explain how the individual made use of technology to bring about Examiner’s
personal empowerment. Use your own words as far as possible. Use

[3]

4 In paragraph 4, the writer gives the example of the American researcher who uncovered
the British financial scandal. (lines 28-30) What is his intention in citing this example?

[2]

5 In paragraph 6, the author referred to the next decade as the ‘Teens’. (line 44) What can
be deduced about the next decade from the use of this term?

[2]
4
For
Examiner’s
Questions on Passage 2 Use

6 What does the phrase ‘an almost Taliban-like policing of women's bodies’ (line 13) say
about the standards of beauty adopted by women’s magazines and entertainment blogs?
Use your own words as far as possible.

[2]

7 Using material from paragraphs 2 to 4, summarise how life in the Noughties was
superficial and fake. Use your own words as far as possible.

Write your summary in no more than 120 words, not counting the opening words
printed below.

Life in the Noughties was superficial and fake because

[8]

Number of words: ________________


5
For
Examiner’s
8 From paragraph 6, identify one metaphor and explain how it reflects the artificiality of life Use
in the Noughties.

[2]

Questions on Both Passages

9 Give the meaning of the following words as they are used in Passages 1 and 2. Write
your answer in one word or a short phrase.

From Passage 1:

a) signature (line 7)

b) spawned (line 31)

c) lubricated (line 31)

From Passage 2:

d) malleable (line 3)

e) unadorned (line 31) [5]

.
6
For
Examiner’s
10 The two passages describe the developments and trends that dominated the Noughties. Use
Matt Frei explains how the individual gained empowerment while Bridget Delaney says
that it was a period when life lacked authenticity and superficial values reigned.

Were the experiences of your society in the Noughties similar or different from the
developments and trends described in the passages? What are the implications for your
society?

In your answer, develop some of the points raised by the authors and give your own
views and some account of the experiences which have helped you form them.
7

For
Examiner’s
Use
8
For
Examiner’s
Use

[8]

Requirement
Explanation
Evaluation
Coherence
Band/Marks

End of Paper
ANDERSON JUNIOR COLLEGE
JC 2 Preliminary Examination
Higher 1

GENERAL PAPER 8806/01


PAPER 1 13 September 2010
1hour 30 minutes
Additional Materials: Answer Paper

READ THESE INSTRUCTIONS FIRST

Write your name, PDG, GP Tutor’s name, question number and the question on
your answer script.
Write in dark blue or black pen on both sides of the paper.
Do not use paper clips, highlighters, glue or correction fluid.

Answer one question.


Note that 20 marks out of 50 will be awarded for your use of language.

At the end of the examination, fasten all your work securely together.
All questions in this paper carry equal marks.

This document consists of 2 printed pages.

© AJC 2010 JC2PRELIM 09_8806_01/T4 [Turn over


2

Answer one question.

Answers should be between 500 and 800 words in length.

1 ‘Technology has improved the lives of the disadvantaged.’ How far is this true?

2 Should pragmatism always be prioritised in Singapore?

3 ‘Today’s youth are losing the ability to think.’ Discuss.

4 How important is the pursuit of happiness?

5 To what extent should governments intervene in family matters?

6 Should current restrictions on drugs be relaxed?

7 To what extent should the scientific screening of a foetus be encouraged?

8 Is capital punishment an effective deterrent?

9 ‘The idolatry of sports celebrities is undesirable.’ Discuss.

10 ‘Education is the key to alleviating poverty.’ How far do you agree?

11 Should we be concerned about the surge in the popularity of cosmetic surgery?

12 Are some works of art more meaningful than others?

© AJC 2010 8806/01/S/10


INSERT

In Passage 1, Carl Honoré writes:

1 Google seems like the last employer on earth that would promote slowness at work.
After all, this is a company that went from a twinkle in its founders' eyes to global
supremacy in just a few years. It pumps out new products at a dizzying rate. It is the
reigning superpower on the speed-obsessed Web. Yet Google also understands the
need to step off the spinning hamster wheel in the workplace. The company famously 5
encourages its staff to devote twenty percent of their time to personal projects. That
does not mean brushing up on World of Warcraft or updating Facebook pages or
flirting with that hot new manager in Accounts. It means getting the creative juices
flowing by stopping the usual barrage of targets, deadlines and distractions. In other
words, allowing staff to slow down. 10

2 The idea is that Google employees can tackle problems that really interest them at
their own pace, free to think deeply, pursue hunches and flights of fancy, make
mistakes, meander down dead ends that may ultimately illuminate a better route
forward. And it seems to work. Many of Google's most innovative products, from Gmail
to AdSense, have grown from projects hatched during the twenty percent time. 15

3 The moral of the story is that, even in the high-speed modern world, slowness and
creativity go hand in hand. We are only just starting to understand how the brain
works, but already it seems clear that there are different ways of thinking. Malcolm
Gladwell has shown how the mind can sometimes make incredibly accurate split-
second decisions. Others have identified modes of thought known as Fast Thinking 20
and Slow Thinking. The former is rational, analytical, linear, logical. It delivers clear
solutions to well-defined problems. It is how computers think. It is what human beings
do under pressure, when the clock is ticking, when the boss is hovering nearby with a
Quarterly Assessment clipboard in hand. By contrast, Slow Thinking is intuitive, woolly
and creative. It is what we do when the pressure is off, and there is time to let ideas 25
simmer on the back burner. It yields rich, nuanced insights and sometimes surprising
breakthroughs.

4 Research has shown that time pressure leads to tunnel vision and that people think
more creatively when they are calm, unhurried and free from stress and distractions.
We all know this from experience. Your best ideas, those eureka moments that turn 30
the world upside down, seldom come when you're juggling emails, rushing to meet the
5pm deadline or straining to make your voice heard in a high-stress meeting. They
come when you're walking the dog, soaking in the bath or swinging in a hammock.

5 Of course, Slow Thinking can be pointless without the rigours of Fast Thinking. You
have to grasp, analyse and harness the ideas that bubble up from the subconscious – 35
and often you must do so quickly. Einstein appreciated the need to marry the two
modes of thought: "Computers are incredibly fast, accurate, and stupid. Human beings
are incredibly slow, inaccurate, and brilliant. Together they are powerful beyond
imagination." This balancing of fast and slow fits into a wider cultural shift. Everywhere,
people are discovering that slowing down at the right moment can help us work, play 40
and live better. Yet this Slow revolution can be a tough sell in an economic downturn.
When recession bites, our reflex is to work harder, longer, faster. But there is so much
to be gained by resisting the urge to put the pedal to the metal.

6 Returning to business as usual is not the answer to this crisis. The future will belong to
those who can innovate their way back into shape – and innovation comes from 45
knowing when to slow down. The secret is to relax and let the mind drift. You'll be
amazed by where it takes you.
Adapted from: ‘In Praise of Slow Thinking’ 2009
http://www.factsandarts.com/articles/in-praise-of-slow-thinking/
In Passage 2, Lucy Kellaway writes:

1 When I was a student it didn't occur to me that time was something that I could
manage. The hours and minutes ticked away and you could either spend them in the
library where they sometimes dragged a bit, or in the pub where they skipped ahead
rather more briskly. But now we are mesmerised by time. Every day the papers bring
fresh evidence of how badly we are managing it. Recently I've learned that Madonna 5
goes to bed with her BlackBerry tucked under her pillow. And that 15 million Britons
binge every night on junk sleep.

2 What is so odd is that despite all this we actually have more time than we've ever had
before. We live for longer, we work fewer hours than we did a hundred years ago.
Thanks to tumble driers, hoovers and microwaves, we can dispatch our chores in a 10
trice. We should have plenty of time left over for twiddling our thumbs. Yet like most
people, I march through my life in a daze of busyness. I check my e-mails before
breakfast and then more times during the day than I care to admit. There are articles to
write, phone calls to be made, sandwiches to be eaten over the keyboard and so on.

3 The answer we are told is to slow down. Our bodies are at risk from the stress we are 15
under and our frazzled minds are losing the ability to think at all. But would slowing
down really make things better? Certainly, busyness is not what is pushing us over the
edge. It is what is keeping us sane. In my view, the pressure of time bearing down is
usually a force for good. It encourages us to pull our fingers out and get things done -
and getting things done is surely satisfying. Dashing around doing things raises the 20
heartbeat agreeably. And if, as we are told, it is a drug - then so what? It doesn't
destroy your lungs or your liver or your bank balance and it doesn't make you go round
snatching people's handbags to get your next fix.

4 Busyness is frowned on because it is meant to leave no time for thought, but this is
nonsense too. For me at least, busyness acts as an efficient thought-filter. It is 25
perfectly possible to have a stroke of inspiration when doing something else. Yet being
busy also crowds out brooding. It is hard to fret about your own mortality when you are
desperately trying to find a plumber to fix the boiler.

5 When I have idle time I tend to use it wondering what's for lunch, and it seems I'm not
alone in my banality. A psychologist in Las Vegas recently did an experiment in which 30
he gave 2,000 people bleepers and told them to write down exactly what was in their
heads when the bleepers went off. One woman thought about Christmas tree
decorations. Another spent the whole day silently repeating to herself the names of two
snacks - Twinkies, Granola.

6 And in case anyone still believes that modern busyness is bad, they should read Jane 35
Austen. In her time a worrying affliction of the middle classes was crazy idleness. In
Bath, where everyone repaired for the season to be relatively busy after the languor of
the countryside, the women thought nothing of doing nothing all morning and then
every afternoon going to the Pump Room to watch others doing very little. And far
from such stillness clearing the mind for great thoughts, much time was spent worrying 40
over whether to wear the sprigged muslin or the plain.

7 The slow hysterics point the finger at technology, which they say is stealing our time
and making us feel nervous and constantly on call. We are meant to disapprove when
we read that Madonna has a BlackBerry under her pillow, and take it as a sign that her
soul is empty. But I fail to see why taking a BlackBerry to bed with you is any more of a 45
barrier to intimacy or relaxation than a copy of the Economist.

8 Man has always fretted about life being too fast. In his bestseller Time: A User's Guide,
Stefan Klein argues that it is all a matter of control. If we are in control of our time, then
all goes well. If we are not, then we end up frazzled.

Adapted from: ‘Speed up, life's too short’, BBC News, 6 June 2008
http://news.bbc.co.uk/2/hi/uk_news/magazine/7440153.stm
Candidate Name PDG GP Tutor’s Name

/09

ANDERSON JUNIOR COLLEGE


JC 2 Preliminary Examination 2010

GENERAL PAPER 8806/02


Paper 2 13 September 2010

Candidates answer on the Question Paper. 1 hour 30 minutes


Additional Materials: 1 insert

READ THESE INSTRUCTIONS FIRST

Write your Name, PDG and GP tutor’s name on all the work you hand in.
Write in dark blue or black pen on both sides of the paper.

Answer all questions.


The Insert contains the passages for comprehension.
Note that 15 marks out of 50 will be awarded for your use of language.

At the end of the examination, fasten all your work securely together.
The number of marks is given in brackets [ ] at the end of each question or part of question.

For Examiner’s Use

Content /35

Language /15

Total /50

This document consists of 7 printed pages and 1 blank page and 1 insert
[Turn over

1
Read the passages in the insert and then answer all the questions which follow. Note For
that up to fifteen marks will be given for the quality and accuracy of your use of English Examiner’s
Use
throughout this Paper

Note: When a question asks for an answer IN YOUR OWN WORDS AS FAR AS
POSSIBLE and you select the appropriate material from the passages for your answer,
you must still use your own words to express it. Little credit can be given to answers
which only copy words and phrases from the passages.

Questions on Passage 1

1 Explain why the writer claims that ‘Google seems like the last employer on
earth that would promote slowness at work’ (line 1). Use your own words as
far as possible.

_________________________________________________________________

_________________________________________________________________

_________________________________________________________________

_________________________________________________________________

_________________________________________________________________ [2]

2 From paragraph 1, what is Google’s position as a search engine? Use your


own words as far as possible.

_________________________________________________________________

_________________________________________________________________

_________________________________________________________________ [1]

3 Explain the paradox in ‘… dead ends that may ultimately illuminate a better
route forward’ (lines 13-14).

_________________________________________________________________

_________________________________________________________________

_________________________________________________________________

_______________________________________________________________ [2]

4 Slow thinking is ‘intuitive, woolly and creative’ (lines 24-25). What are the
possible results of this type of thinking? Use your own words as far as
possible.

_________________________________________________________________

_________________________________________________________________

_________________________________________________________________

_________________________________________________________________

_________________________________________________________________ [2]
2
For
5 According to the writer, when do people experience their ‘eureka moments’ Examiner’s
Use
(line 30)? Use your own words as far as possible.

_________________________________________________________________

_________________________________________________________________

_________________________________________________________________ [1]

6 Explain the ‘cultural shift’ in line 39. Use your own words as far as possible.

_________________________________________________________________

_________________________________________________________________

_________________________________________________________________

_________________________________________________________________ [1]

Questions from Passage 2

7 In paragraph 1, why do you think the writer says that time passes slowly in the
library but moves more quickly in the pub?

_________________________________________________________________

_________________________________________________________________

_________________________________________________________________ [1]

8 Explain in your own words what the writer means by ‘binge … on junk sleep’
(line 7).

_________________________________________________________________

_________________________________________________________________

_________________________________________________________________

_________________________________________________________________

_________________________________________________________________
[2]

9 What is the irony in paragraph 2?

_________________________________________________________________

_________________________________________________________________

_________________________________________________________________ [1]

3
10 Using material from paragraphs 3 – 5 in Passage 2, summarise the arguments For
against busyness and the writer’s reasons for defending busyness. Examiner’s
Use

Write your summary in no more than 120 words not counting the opening
words which are printed below. Use your own words as far as possible.

Some people are against busyness because _____________________________

__________________________________________________________________

__________________________________________________________________

__________________________________________________________________

__________________________________________________________________

__________________________________________________________________

__________________________________________________________________

__________________________________________________________________

__________________________________________________________________

__________________________________________________________________

__________________________________________________________________

__________________________________________________________________

__________________________________________________________________

__________________________________________________________________

__________________________________________________________________

__________________________________________________________________

__________________________________________________________________

__________________________________________________________________

_________________________________________________________________

__________________________________________________________________

__________________________________________________________________

__________________________________________________________________

__________________________________________________________________

__________________________________________________________________

__________________________________________________________________

__________________________________________________________________

__________________________________________________________________ [8]
4
11 What point is the writer making in the quote “much time was spent worrying For
over whether to wear the sprigged muslin or the plain” (lines 40-41)? Examiner’s
Use

__________________________________________________________________

__________________________________________________________________

__________________________________________________________________ [1]

From Passage 1 and 2

12 Give the meaning of the following words as they are used in the passage.

Write your answers in one word or a short phrase.

From Passage 1

a hunches (line 12) _________________________________________________ [1]

b hatched (line15) ___________________________________________________ [1]

c tunnel vision (line 28) _______________________________________________ [1]

From Passage 2

d mesmerised (line 4) ________________________________________________ [1]

e agreeably (line 21) _________________________________________________ [1]

[Total 5]

5
13 Carl Honoré in Passage 1 advocates slowing life down while Lucy Kellaway in For Examiner’s
Passage 2 believes that life is more fulfilling when we are busy. Use

Which writer’s ideas do you agree with more?

Would your society benefit more by slowing down?

You should refer to arguments from both passages as well as ideas of your
own.

__________________________________________________________________

__________________________________________________________________

__________________________________________________________________

__________________________________________________________________

__________________________________________________________________

__________________________________________________________________

__________________________________________________________________

__________________________________________________________________

__________________________________________________________________

__________________________________________________________________

__________________________________________________________________

__________________________________________________________________

__________________________________________________________________

_________________________________________________________________

__________________________________________________________________

__________________________________________________________________

__________________________________________________________________

__________________________________________________________________

__________________________________________________________________

__________________________________________________________________

__________________________________________________________________

__________________________________________________________________

__________________________________________________________________

__________________________________________________________________

__________________________________________________________________

6
__________________________________________________________________ For Examiner’s
Use
__________________________________________________________________

__________________________________________________________________

__________________________________________________________________

__________________________________________________________________

__________________________________________________________________

__________________________________________________________________

__________________________________________________________________

__________________________________________________________________

_________________________________________________________________

__________________________________________________________________

__________________________________________________________________

__________________________________________________________________

__________________________________________________________________

__________________________________________________________________

__________________________________________________________________

__________________________________________________________________

__________________________________________________________________

__________________________________________________________________

__________________________________________________________________

__________________________________________________________________

__________________________________________________________________

__________________________________________________________________

__________________________________________________________________

__________________________________________________________________ R

__________________________________________________________________ Ev

__________________________________________________________________ Ex

__________________________________________________________________ Co
__________________________________________________________________

__________________________________________________________________

________________________________________________________________ [8]
7
8
CATHOLIC JUNIOR COLLEGE
JC2 Preliminary Examination
Higher 1 30 August 2010

GENERAL PAPER 8806/01


Paper 1
1 hour 30 min

READ THESE INSTRUCTIONS FIRST

Write your name, class and question number on all the work you hand in.
Write in dark blue or black pen on both sides of the paper.
Do not use paper clips, highlighters, glue or correction fluid on your work.

Answer ONE question,


You are reminded of the need for good English and clear presentation in your answers.

At the end of the examination, fasten all your work securely together with the string provided.
All questions in this paper carry equal marks.

Note that 20 marks out of 50 will be awarded for your use of language.

This document consists of 2 printed pages including this cover page.

© Permission to reproduce paper should be sought from Catholic Junior College.


2

Answer ONE question from this paper.


Answers should be between 500 and 800 words in length.

1. “Small country, small minds.” Is this a fair assessment of Singaporeans?

2. “Only the best and brightest should have the right to have children.” What are your views?

3. “The mass media is nothing but a profit-making machine.” Evaluate this claim.

4. How far is society responsible for the crimes of its people?

5. “Every generation of youth is characterised by the need to rebel.” How far does this statement
reflect your generation?

6. “Men are as much the victims of traditional gender roles as women.” Comment.

7. “Those who allow themselves to be governed like sheep will eventually be ruled by wolves.” In
the light of this statement, what is the value of political dissent?

8. ”Success breeds success.” What are your views?

9. “The world will be a better place without religion.” Discuss.

10. “Art should not offend.” To what extent do you agree with this statement?

11. “Technology has created a culture of convenience.” Is this more a blessing than a curse?

12. How far have examinations undermined the essential aims of education today?

END OF PAPER
Candidate’s Name: _______________________________________________

Candidate’s Home Tutorial Class/Banded Group: _______________________

CATHOLIC JUNIOR COLLEGE


JC2 PRELIMINARY EXAMINATION 2010

GENERAL PAPER 8806/02


PAPER 2

MONDAY 30 AUGUST 2010 1 hour 30 minutes

INSTRUCTIONS TO CANDIDATES

This paper contains the passages for Paper 2 and the Answer Booklet.

Write your Name and Home Tutorial Class in the spaces at the top of this page.

Answer all questions.

Attach this cover sheet to the completed Answer Booklet.

If you remove the passages for easy reference, please reattach them to the back of the Answer
Booklet before handing it up.

INFORMATION FOR CANDIDATES

The number of marks is given in brackets [ ] at the end of each question.

EXAMINER’S USE ONLY


Content
/35
Language
/15
Total
/50

This paper consists of 10 printed pages including this page


PAPER 2 (50 marks)

Read the passages and then answer all the questions which follow below. Note that up to fifteen
marks will be given for the quality and accuracy of your use of English throughout this paper.

Note: When a question asks for an answer IN YOUR OWN WORDS AS FAR AS POSSIBLE and
you select the appropriate material from the passage(s) for your answer, you must still use your
own words to express it. Little credit can be given to answers which only copy words or phrases
from the passage(s).

Questions on Passage 1

1. Explain why the author finds his insight about the pursuit of happiness ‘liberating’ (line 1). Use
your own words as far as possible.

…………………………………………………………………………………………………………………...

……………………………………………………………………………………………………………. [1 m]

2. From paragraph 2, identify three ways by which happiness can be cultivated. Use your own
words as far as possible.

(i) ………………………………………………………………………………………………………………..

.…………………………………………………………………………………………………………………..

(ii)………………………………………………………………………………………………………………..

…………………………………………………………………………………………………………………...

(iii)………………………………………………………………………………………………………………..

……………………………………………………………………………………………………………..[3 m]

3. Explain the purpose of the author in giving the example of Buddha (line 25-26). Use your own
words as far as possible.

…………………………………………………………………………………………………………………...

…………………………………………………………………………………………………………………...

…………………………………………………………………………………………………………………...

……………………………………………………………………………………………………………. [2 m]

2
4. Why does the author say that happiness ‘cannot take us entirely by surprise’ (line 30)? Use
your own words as far as possible.

…………………………………………………………………………………………………………………...

……………………………………………………………………………………………………………..[1 m]

5. Explain what the author means by ‘desire industry’ (line 34).

…………………………………………………………………………………………………………………...

…………………………………………………………………………………………………………………...

…………………………………………………………………………………………………………………...

…………………………………………………………………………………………………………… [2 m]

Questions on Passage 2

6. Explain what the author means by ‘wastelands of mechanistic behaviour’ (lines 13-14).

…………………………………………………………………………………………………………………...
.
…………………………………………………………………………………………………………………...

…………………………………………………………………………………………………………………...

……………………………………………………………………………………………………………..[2 m]

7. Why does the author repeat ‘I’d hate for us…’ in paragraph 6? Use your own words as far as
possible.

…………………………………………………………………………………………………………………...
.
…………………………………………………………………………………………………………………...

…………………………………………………………………………………………………………………...

………………………………………………………………………………………………………………......

…………………………………………………………………………………………………………………..

……………………………………………………………………………………………………………..[3 m]

3
8. Using the material from paragraphs 3 to 5, summarise the negative effects of the overemphasis
on happiness.

Write your summary in no more than 120 words, not counting the opening words which are
printed below. Use your own words as far as possible.

One negative effect of the overemphasis on happiness is possibly that…………………………….

…………………………………………………………………………………………………………………...

…………………………………………………………………………………………………………………...

…………………………………………………………………………………………………………………...

…………………………………………………………………………………………………………………...

…………………………………………………………………………………………………………………...

…………………………………………………………………………………………………………………...

…………………………………………………………………………………………………………………...

…………………………………………………………………………………………………………………...

…………………………………………………………………………………………………………………...

…………………………………………………………………………………………………………………...

…………………………………………………………………………………………………………………...

…………………………………………………………………………………………………………………...

…………………………………………………………………………………………………………………...

…………………………………………………………………………………………………………………...

…………………………………………………………………………………………………………………...

…………………………………………………………………………………………………………………...

…………………………………………………………………………………………………………………...

…………………………………………………………………………………………………………………...

……………………………………………………………………………………………………………..[8 m]

4
Questions on Passages 1 and 2

9. Give the meaning of the following words as they are used in the passage. You may write your
answer in one word or a short phrase. [5 m]

From Passage 1:

(a) perverted (line 8) ……………….…………………….………………..………………………….

(b) resolve (line 28) ..………………..………………………….………………..……………………

(c) span (line 32) ………………..…..……………………….…………….………………………….

From Passage 2:

(d) felicity (line 6) …………..……………………..……………………….…………………………..

(e) apocalyptic (line 9) ………..……….………………….………………….…………………........

10. Schoch argues for the pursuit of happiness while Wilson criticizes the American obsession
with happiness. With which of the two authors are you most in sympathy? Explain the reasons
for your choice. How relevant are the views raised by both authors to your society?

…………………………………………………………………………………………………………………...

…………………………………………………………………………………………………………………...

…………………………………………………………………………………………………………………...

…………………………………………………………………………………………………………………...

…………………………………………………………………………………………………………………...

…………………………………………………………………………………………………………………...

…………………………………………………………………………………………………………………...

…………………………………………………………………………………………………………………...

…………………………………………………………………………………………………………………...

…………………………………………………………………………………………………………………...

…………………………………………………………………………………………………………………...

…………………………………………………………………………………………………………………...

…………………………………………………………………………………………………………………...
5
…………………………………………………………………………………………………………………...

…………………………………………………………………………………………………………………...

…………………………………………………………………………………………………………………...

…………………………………………………………………………………………………………………...

…………………………………………………………………………………………………………………...

…………………………………………………………………………………………………………………...

…………………………………………………………………………………………………………………...

…………………………………………………………………………………………………………………...

…………………………………………………………………………………………………………………...

…………………………………………………………………………………………………………………...

…………………………………………………………………………………………………………………...

…………………………………………………………………………………………………………………...

…………………………………………………………………………………………………………………...

…………………………………………………………………………………………………………………...

…………………………………………………………………………………………………………………...

…………………………………………………………………………………………………………………...

…………………………………………………………………………………………………………………...

…………………………………………………………………………………………………………………...

…………………………………………………………………………………………………………………...

…………………………………………………………………………………………………………………...

…………………………………………………………………………………………………………………...

…………………………………………………………………………………………………………………...

…………………………………………………………………………………………………………………...

…………………………………………………………………………………………………………………...

…………………………………………………………………………………………………………………...

…………………………………………………………………………………………………………………...
6
…………………………………………………………………………………………………………………...

…………………………………………………………………………………………………………………...

…………………………………………………………………………………………………………………...

…………………………………………………………………………………………………………………...

…………………………………………………………………………………………………………………...

…………………………………………………………………………………………………………………...

…………………………………………………………………………………………………………………...

…………………………………………………………………………………………………………………...

…………………………………………………………………………………………………………………...

…………………………………………………………………………………………………………………...

…………………………………………………………………………………………………………………...

…………………………………………………………………………………………………………………...

…………………………………………………………………………………………………………………...

…………………………………………………………………………………………………………………...

…………………………………………………………………………………………………………………...

…………………………………………………………………………………………………………………...

…………………………………………………………………………………………………………………...

…………………………………………………………………………………………………………………...

…………………………………………………………………………………………………………………...

…………………………………………………………………………………………………………………...

……………………………………………………………………………………………………………. [8 m]

END OF PAPER

7
BLANK PAGE

8
Passage 1: Richard Schoch writes about the pursuit of happiness…

1 For me, the most liberating insight about our pursuit of happiness, liberating because it helps us
to overcome so many threatening anxieties, is that we do not have to become someone else to
be happy. In a way this truth is trivial: how can we be other than who we are? But what is far from
trivial is the disturbing and all too prevalent belief that to find happiness we must turn our backs
on all that is familiar, forge a new life, journey to distant lands, perform extraordinary acts, 5
exchange a dismal present for a fantastic future, or wish upon an auspicious star. Such efforts
are wasteful because they squander the opportunity that is always before us: to become not
someone else – that is the perverted goal of the ‘makeover ethos’ – but a better version of the
person we already are. Whoever we are, and whatever circumstances we face – and for nearly all
of us, they will be ordinary ones – the possibility of happiness always surrounds us. We are 10
always in the right place, though we do our best to forget it.

2 So let us cultivate a happiness that is authentically ours and let us be happy with the things that
will make us so. To be authentically happy means to take possession of ourselves, in order to
become more real. As Voltaire famously put it, ‘We must make our garden grow.’ Action is the
heart of an authentic existence because only in action do we attain fulfillment. Of course, ‘action’ 15
is not just striking out in the world but a directed realization of the kind of person we imagine
ourselves to be. In that way, we give our lives a coherent shape, and rescue ourselves from
dissolution and waste. Through purposeful action, we become our future and find our
contentment. We accomplish ourselves. Is it too much, too paradoxical, to say that happiness
feels like growing up? We make happiness as best we can within life as it is, and do not import it 20
from some magical elsewhere.

3 To search for happiness is not to embark upon a voyage to an exotic distant land, but to return
home. Whether actual or symbolic, the homecoming is a common feature in many different
stories about the quest for happiness. Ghazali returned to his birthplace in Persia after spending
years in prayerful solitude in foreign lands. The Buddha was born – and died – in a grove of sala 25
trees. Though each story is distinctive they all reveal the general truth for all of us, no matter who,
no matter where – that happiness must feel like something that we once knew, perhaps only
dimly, but now are finding again, although this time with a greater resolve and surer purpose. In a
way, we are discovering a part of ourselves that we had never known.

4 Happiness cannot take us entirely by surprise, cannot steal upon us, because it is an enterprise 30
that requires our investment. For all its connotations of blissful epiphany, happiness is a rather
pragmatic affair. It must lie within our reach and fall within the span of our days.

5 That we shall find happiness cannot be guaranteed. Which is another way of saying that,
however much the ‘desire industry’ tries to persuade us otherwise, we are not entitled to be
happy. We are entitled only to work for it. This entitlement we all share, simply because we are 35
human. The chance for happiness is an irreducible part of our being, a part from which no one
can be disqualified.

Adapted from Richard Schoch: The Secrets of Happiness (2007)

9
Passage 2: Eric Wilson writes about the obsession with happiness

1 The psychological world is now abuzz with a new field, positive psychology, devoted to finding
ways to enhance happiness through pleasure, engagement, and meaning. Mainstream publishers
are now learning from the self-help industry and printing thousands of books on how to be happy
and on why we are happy. Doctors now offer a wide array of drugs that might eradicate
depression forever. It seems truly, perhaps more than ever before, an age of almost perfect 5
contentment, a brave new world of persistent good fortune, joy without trouble, felicity with no
penalty.

2 Surely all this happiness can’t be for real. How can so many people be happy in the midst of all
the problems that beset our globe—not only the collective and apocalyptic ills but also those
particular irritations that bedevil our everyday existences, those money issues and marital spats, 10
those stifling vocations and lonely dawns? Are we to believe that four out of every five Americans
can be content amid the general woe? Aren’t we suspicious of this statistic? Don’t we fear that
this rabid focus on exuberance leads to half-lives, to bland existences, to wastelands of
mechanistic behaviour?

3 I for one am afraid that this overemphasis on happiness at the expense of sadness might be 15
dangerous, a wanton forgetting of an essential part of a full life. I further am wary in the face of
this possibility: to desire only happiness in a world undoubtedly tragic is to become inauthentic, to
settle for unrealistic abstractions that ignore concrete situations. Without the agitations of the
soul, would all of our magnificently yearning towers topple? Would our heart-torn symphonies
cease? 20

4 The predominant form of American happiness breeds blandness. This kind of happiness appears
to entertain a craven disregard for the value of sadness. This brand of supposed joy, moreover,
seems to foster an ongoing ignorance of life’s enduring and vital polarity between agony and
ecstasy, dejection and ebullience. Trying to forget sadness and its integral place in the great
rhythm of the cosmos, this sort of happiness insinuates in the end that the blues are an aberrant 25
state that should be cursed as weakness of will or removed with the help of a little pink pill.

5 Most of us have been duped by the American craze for happiness. We might think that we’re
leading a truly honest existence, one attuned to vivid realities and blooded hearts, when we’re
really just behaving as predictably and artificially as robots, falling easily into well-worn “happy”
behaviours, into the conventions of contentment, into obvious grins. Deceived, we miss out on the 30
great interplay of the living cosmos, its luminous gloom, its terrible beauty.

6 The American dream might be a nightmare. What passes for bliss could well be a dystopia of
flaccid grins. Our passion for felicity hints at an ominous hatred for all that grows and thrives and
then dies—for all those curious thrushes moving among autumn’s brownish indolence, for those
blue dahlias seemingly hollowed with sorrow, for all those gloomy souls who long for clouds 35
above high windows. I’d hate for us to awaken one morning and regret what we’ve done in the
name of untroubled enjoyment. I’d hate for us to crawl out of our beds and walk out into a country
denuded of gorgeous lonely roads and the grandeur of desolate hotels, of half-cracked geniuses
and their frantic poems. I’d hate for us to come to consciousness when it’s too late to live.

Adapted from Eric Wilson: Against Happiness (2008)

10
Answer Scheme for GP Paper 2 (CJC Prelim 2010)

Passage 1
1. Explain why the author finds his insight about the pursuit of happiness „liberating‟
(line 1). Use your own words as far as possible. (1)

Lifted Paraphrase
the most liberating insight about our A: The pursuit of happiness enables us to
pursuit of happiness, liberating conquer/remove/cope with/deal with (1/2)
because it helps us to overcome
(1/2) so many threatening anxieties get past – acceptable
(1/2), is that we do not have to
become someone else to be happy. B: various kinds of
(l.1-3) worries/concerns/fears/insecurities/troubles. (1/2)

nervousness/stress/problems/barriers/perplexities/
woes (great sorrow and grief) = 0
gripes/negative emotions = 0
break down challenges/obstacles = 0
stop worrying - ½

2. From paragraph 2, identify three ways by which happiness can be cultivated. Use
your own words as far as possible. (3)
(i)
(ii)
(iii)

Lifted Paraphrase
To be authentically happy means to take A: The first way is to recognize who we
possession of ourselves, in order to become actually/truly are (1/2) and become more
more real. (l.13-14) genuine/true to ourselves (1/2)
OR being in control of our natural selves
Lift – Authentic = 0 (1/2) and true to our identities (1/2)
Accepting individuality = 0

Through purposeful action, we become our B: The second way is through the doing of
future (l. 18) something deliberate/meaningful (1/2) to
fulfill/achieve our potential.(1/2)

- to take initiative (1/2)

We make happiness as best we can within C: The third way is by accepting our
life as it is, and do not import it from some current situation /what we have/content
magical elsewhere. (l.20-21) with what we have (1/2) and not
search/look for happiness in other
places.(1/2)
3. Explain the purpose of the author in giving the example of Buddha (line 25-26).
Use your own words as far as possible. (2)

Lifted Paraphrase
Whether actual or symbolic, the A: The example of the Buddha illustrates
homecoming is a common feature in the point that going back to the place of
many different stories about the quest origin/place of birth/going back to our
for happiness. Ghazali returned to his roots is a source of happiness.(1)
birthplace in Persia after spending years in
prayerful solitude in foreign lands. The
Buddha was born – and died – in a grove of B: It also emphasises the truth that
sala trees. Though each story is distinctive happiness is something we were formerly
they all reveal the general truth for all of aware of/formerly experienced (1)
us, no matter who, no matter where – that
happiness must feel like something that - something familiar (1/2)/formerly felt
we once knew…(l. 23-29) (1/2)

4. Why does the author say that happiness „cannot take us entirely by surprise‟? (line
30-31). Use your own words as far as possible. (1)
Lifted Paraphrase
Happiness cannot take us entirely by It is because we already have some
surprise, cannot steal upon us, because it is knowledge of it (implied by „enterprise‟)
an enterprise that requires our
investment (1/2). (l.30-31) OR

It is because happiness is a course of


action/strategy/undertaking/endeavour that
needs planning/effort/work (1)

Contribution – 0
Commitment - 0
5. Explain what the author means by the „desire industry‟ (line 34). (2)

Lifted Paraphrase
That we shall find happiness cannot be It is the consumerist culture that is based
guaranteed. Which is another way of on commercial exploitation/manipulation
saying that, however much the „desire (1)
industry‟ tries to persuade us otherwise, we
are not entitled to be happy. (l.34) of consumers‟ wants/dreams/needs (1)

As long as the student conveys a holistic


meaning of the understanding of ‘desire
industry’, full marks can be given.

Passage 2

6. Explain what the author means by “wastelands of mechanistic behaviour” (lines 13-
14). (2)

Lifted Paraphrase
Don‟t we fear that this rabid focus on A: It refers to the
exuberance leads to half-lives, to bland sterile/desolate/barren/empty lives (1) of
existences, to wastelands of mechanistic people
behaviour? (l. 13-14)
- mundane (1/2)
- purposeless/meaningless (1/2)
- useless (0)

B: who act like robots/are automatons/are


puppet-like/are rigid conformists (1)
- routine/predictable - 0

OR

do not feel or do not


think/emotionless/unspontaneous/lacks
vibrancy/devoid of emotions (1)
7. Why does the author repeat “I‟d hate for us…” in paragraph 6? Use your own words
as far as possible. (3)

Lifted Paraphrase
I‟d hate for us to awaken one morning and A. The author wants to emphasise/stress/
regret what we‟ve done in the name of highlight/underscore/reinforce (1)
untroubled enjoyment. I‟d hate for us to Reiterate = 0 (means „repeat‟)
crawl out of our beds and walk out into a
country denuded of gorgeous lonely roads B. that the author does not want to see a
and the grandeur of desolate hotels, of half- bleak world where people see happiness as
cracked geniuses and their frantic poems. pure bliss (1)
I‟d hate for us to come to consciousness
when it‟s too late to live. (l.36-39) C. that he would be extremely
upset/appalled if people are incapable of
perceiving the beauty in the dark side of
life (1)/ losing the beauty of the dark side
of life (1)

D. that people would realize/regret that


they missed out on opportunities of
experiencing a full existence that includes
sorrow and joy, if they are obsessed with
the pursuit of happiness (1)

Any 3 out of the 4 points


Summary (Passage 2)
8.. Using materials from paragraphs 3 to 5, summarise the negative effects of the
overemphasis on happiness.
Write your summary in no more than 120 words not counting the opening words which
are printed below. Use your own words as far as possible. (8)

One negative effect of the overemphasis on happiness is possibly that…


Lifted Paraphrase
A. this overemphasis on happiness at the expense It is possibly threatening/hazardous
of sadness might be dangerous (l.15-16)
B. overemphasis on happiness at the expense of It is ignoring the importance of
sadness …a wanton forgetting of an essential sadness/melancholy/blues and sadness
part of a full life (l.16) is a necessary condition for a complete
existence
C. to desire only happiness in a world undoubtedly Life/ We would be artificial/false
tragic is to become inauthentic (l.17) One might not be true to oneself

D. to settle for unrealistic abstractions that ignore and one would be replacing specific
concrete situations (l. 18) circumstances/experiences with
theoretical/conceptual ideas
E. The predominant form of American happiness It gives rise to
breeds blandness (l. 21) unexciting/humdrum/boring lives/
monotony
F. This brand of supposed joy, moreover, seems to It appears to generate a perpetual lack
foster an ongoing ignorance of life‟s enduring of awareness of the dynamic contrast
and vital polarity between agony and ecstasy, between suffering and rapture
dejection and ebullience.(l.23-24)
G. this sort of happiness insinuates in the end that Sadness/Melancholy/Blues is seen as
the blues are an aberrant state (l.25) something abnormal/deviant

H. that should be cursed as weakness of will (l.26) and is denounced/condemned/strongly


rejected as a sign of feeble-
mindedness/being feeble-minded/lack
of determination
I. removed with the help of a little pink pill. (l.26) Treated as a sickness that can be
remedied/cured through medication
J. Most of us have been duped by the American To be tricked/deceived into believing
craze for happiness. We might think that we‟re that we are having a genuine/real way
leading a truly honest existence, (l.27-28) of life
K we‟re really just behaving as predictably and Living like automatons/machines
artificially as robots (l.29)
L Deceived, we miss out on the great interplay of We suffer from the omission of the
the living cosmos, its luminous gloom, its tremendous synergy/vital
terrible beauty. (l.30) connection/interaction between the
awesome/fearsome/frightening and the
beauty
M. Without the agitations of the soul, would all of Without sadness, we would lose our
our magnificently yearning towers topple? source of creativity in artworks and
Would our heart-torn symphonies cease? (l.19- architecture
20)
9. Vocabulary
1m 1/2m 0m
Passage 1 deviating from Corrupted abnormal
perverted (line 8) what is right or Misguided anything sexual
correct perverse
twisted corrupt
distorted skewed
warped unethical
wrong/incorrect
mistaken
resolve (line 28) determination commitment Boldness
firmness Courage
tenacity conviction
steadfastness
willpower
will
span (line 32) Extent of time Range of time Range
Length of time Period Distance
Period of time Length
Duration Time
Time frame Living time
Lifetime Living years
Limits
Passage 2 Bliss Ecstasy Joy (0) – Lift
felicity (line 6) Blissfulness Rapture Good feeling
Happiness Jubilation Unrestrained enjoyment
exuberance Mirth/glee
(These refer to short- Contentment/satisfaction
term feelings)

apocalyptic (line Catastrophic Extremely Global


13) Causing destructive/disastrous Momentous
worldwide Deadly
destruction Devastating Threatening
Devastating on a Threatening to the world
large scale Near-catastrophic
World-ending
Doomsday Armageddon
Calamitous (metaphor for
Cataclysmic apocalypse)

(Devastating
impact must be
present)
Application Question

10. Schoch argues for the pursuit of happiness while Wilson criticizes the American obsession with happiness.
With which of the two authors are you most in sympathy? Explain the reasons for your choice. (R1) How relevant are the views raised
by both authors to your society? (R2) (8)

Main Theses:
Schoch: Happiness cannot be guaranteed and has to be found within ourselves. We need to be happy with who we are. It has to be
something that is authentic and can be found close to home (L.10)

Wilson: He is concerned about how we may be superficial in our blind pursuits of happiness and laments that we have forgotten about
the value of sadness – which is essential to a full life.

In sympathy (R1) Not in sympathy (R1) Relevant to Singapore Not relevant to


(R2) Singapore (R2)
P1 True – happiness is the External circumstances SM Goh has spoken on For some Singaporeans,
The possibility of condition of the heart and a do play a part in affecting the new 5Cs that the pursuit of materialism
happiness always state of mind. It is not our level of happiness – Singaporeans should is an answer to their quest
surrounds us – necessarily related to how events such as adopt, including for happiness. The 5Cs –
whoever we are and happenings or social strata. natural disasters, the charity and car, condos, country club,
whatever financial crisis have consideration - which cash and credit card -
circumstances we Bhutan as one of the caused pain and are ways Singaporeans define the image and
face. (Para 1, l.9) poorest countries in the sufferings in both can achieve happiness status of Singaporeans.
world but it is one of the developed and without the material Without material comforts
happiest countries in Asia, developing countries tangibles. and luxury, it is not
according to the Happiness possible to be happy.
Index. It pursues happiness Money is a means to an
not through the Gross end – to attain happiness
National Product but through material comforts.
through the Gross National
Happiness
We make happiness Certain philosophers such How happiness cannot be Singapore youths are
within life and do as Aristotle recognize the just from a personal influenced heavily by the
not import it from pursuit of happiness comes perspective of life but „desire industry‟, which
some magical from a life of virtue and imported from social promotes material pursuits
elsewhere (Para 2, has to be cultivated networks. and that happiness can be
l.22) through goals and bought e.g. how teenagers
activities, instead of being Researchers found that are obsessed with the
a fleeting emotion. happiness tended to latest gadgets (i-phones
spread through close and i-pads) to provide
relationships like friends, them with entertainment
siblings, spouses, and and temporary happiness.
next-door neighbors
(British Medical Journal,
2008)
To search for More are also seeking In the globalised world, it More Singaporeans are Many Singaporeans have
happiness is not to happiness from self- is inevitable that many placing emphasis on migrated to find greener
embark upon a actualisation and personal are seeking happiness in and inner happiness pastures and do not see the
voyage to an exotic growth in the aspects of the form of immigration, from their work and merit of finding happiness
distant land (Para work and religion. travel and even escapism studies that can be at home.
3, l. 22) to the „virtual world‟ found at home.
Surveys by Gallup, the through the Internet.
National Opinion Research Singapore is
Center and the Pew encouraging more
Organization in the U.S. opportunities for the
conclude that spiritually youths to live their
committed people are twice dreams through
as likely to report being increased social
"very happy" than the least opportunities (e.g.
religiously committed Local Arts and Sports
people. School to develop
talents)
We are entitled to True – the pursuit of He is pessimistic and How Singaporeans
work for it…The happiness is an inalienable making assumptions that have learnt to unwind
entitlement of human right from the we are not entitled to be in the competitive
happiness we all American declaration of happy. It may be an society – Some
share, simply happiness – life, liberty and unfair legitimization to seeking actively for
because we are pursuit of happiness; all of say that happiness has to activities to find their
human (l.35) us have a chance to pursue be worked for when it happiness e.g. laughter
happiness should be a fundamental clubs in Singapore to
right of every human. release the endorphins
He is realistic in that Singaporeans are
happiness has to be worked taking on the voluntary
for – long-term happiness work to seek their inner
cannot come without effort fulfillment and
and purpose. happiness. Some
Singaporeans even
leave their top-paying
jobs to find fulfillment.
P2 Students may sympathize What is wrong with Mechanistic, artificial
Don’t we fear this with the half-lives (l.13) hedonism – why should way of life in
rabid focus on and bland existence and the people not view life with Singapore where most
exuberance leads to social woes (such as the rose-tinted glass? things are measured in
half-lives, bland recent sub-prime mortgage Hedonists could also lead terms of material
existence, to crisis, woes over academic lives that are interesting happiness and survival;
wastelands of inflation and the endless and fulfilling, especially workers on the
mechanistic paper chase to secure jobs) if they use money to hedonistic treadmill
behaviour? (l.13- e.g. How young South support their cause or without understanding
14) Koreans are obsessed with expand their horizons to the simple pleasures of
university entry enrich themselves. life.
examination as their path
to happiness that makes
their lives barren to the
point that they seek suicide
as a release

This sort of Sadness helps us to May be too much to Local artistes and
The receptivity towards
happiness appears appreciate better the expect of the common directors have used the
the trend of positive
to entertain a moments of happiness and masses. They have to be theme of melancholia psychology coming into
craven disregard could cause people to see melancholic and Singapore – e.g. self-help
in their work to create
for the value of beauty in their current philosophical by nature, humane pieces of work books on happiness,
sadness – and melancholia (e.g. how to argue for the value of that have touched
weight-loss, stress-
ignores the Vincent van Gogh, Emily sadness. Most people international and local
management programmes
essential part of a Dickinson and other artistic would want to pursue audience. – is evident that
full life (Para 3 & geniuses saw beauty in the happiness and actively Singaporeans want to be
4) world through a glass avoid sadness. Singapore filmmakers rid of depression.
darkly; The creator of such as Eric Khoo
"Peanuts," Charles M. The author romanticizes have used the theme of
Schulz, was known for his sadness – depression is sadness in melancholic
gloom, while Woody Allen an unpleasant state to be films such as „Be with
plumbs existential in and requires treatment. me‟ (inspired by the
melancholia for his films) life of deaf-and-blind
teacher Theresa Poh
Lin Chan) to render
hope in fragile lives.
This sort of Instead of offering
happiness sympathy and condolences
insinuates the blues to people who are feeling
are an aberrant sad, we have attached a
state and should be stigma to being sad –
removed with a depression is listed as a
pink pill (Para 4) mental illness in the
Diagnostic and Statistical
Manual of Mental
Disorders - and the normal
range of human emotion is
not being tolerated.
DUNMAN HIGH SCHOOL
UNIVERSITY OF CAMBRIDGE LOCAL EXAMINATIONS SYNDICATE
General Certificate of Education Advanced Level
Higher 1
YEAR 6 PRELIMINARY EXAMINATION

CANDIDATE QUESTION
NAME NUMBER

CIVICS INDEX
GROUP 6C NUMBER

GENERAL PAPER 8806/01


Paper 1 3 September 2010

Monday 1 hour 30 minutes

Additional Materials: Answer Paper

READ THESE INSTRUCTIONS FIRST

Write your name, Civics Group, Index number and Question number in the spaces provided
on the question paper and on all the work you hand in.
Write in dark blue or black pen on both sides of the writing paper.
Do not use staples, paper clips, highlighters, glue or correction fluid.

Answer one question.


Note that 20 marks out of 50 will be awarded for your use of language.

At the end of the examination, fasten all your work securely together.
All questions in this paper carry equal marks.

For Examiner’s Use

Content / 30

Language / 20

Total / 50

This document consists of 2 printed pages.


[Turn over

© DHS 2010 8806/01/Y6/10/Prelim


2

Answer one question.

Answers should be between 500 and 800 words in length.

1 ‘We have guided missiles and misguided men.’ Discuss.

2 Is education for all a realistic aim?

3 ‘Sport is over-rated.’ Is this a fair comment?

4 ‘Love of one’s country is as much a good thing as a bad thing.’ What is your view?

5 ‘Money is everything.’ How far is this true in your society?

6 Should the State limit freedom?

7 ‘As the world progresses, the quality of life declines.’ To what extent do you agree?

8 ‘Modern technology has made us cleverer, but not wiser.’ Discuss.

9 ‘The book is dead.’ Comment.

10 Discuss the importance of play.

11 ‘The government, not the people, should be responsible for protecting the environment.’ Do
you agree?

12 ‘Many developed countries are paying increasing attention to the arts in their society.’ How far
is this true of Singapore?

© DHS 2010 8806/01/Y6/10/Prelim


CANDIDATE NAME CLASS INDEX NUMBER
6C

DUNMAN HIGH SCHOOL


UNIVERSITY OF CAMBRIDGE LOCAL EXAMINATIONS SYNDICATE
General Certificate of Education Advanced Level
Higher 1

YEAR 6 PRELIMINARY EXAMINATION

GENERAL PAPER 8806/02


Paper 2 3 Sep 2010

INSERT
1 hour 30 minutes

READ THESE INSTRUCTIONS FIRST

This Insert contains the passage for Paper 2.

This document consists of 3 printed pages and 1 blank page.


[Turn over
© DHS 2010 8806/02/Y6/10/PRELIM/INSERT
2

Hanna Rosin predicts that the world of the future will be a woman’s world.

1 Man has been the dominant sex since the dawn of mankind. For a long time,
evolutionary psychologists have claimed that we are all imprinted with adaptive
imperatives from a distant past: men are faster and stronger and hardwired to fight for
scarce resources, and that shows up as a drive to win on Wall Street; women are
programmed to find good providers and to care for their offspring, and that is manifested 5
in more nurturing and docile behaviour, ordaining them to domesticity. For centuries,
young women have been forced into marriages arranged by their parents, and rigid
patriarchal societies have been abusing wives who failed to produce male heirs.

2 But over the course of the past century, we have seen massive and unprecedented
changes at the workplace. As more and more governments embraced industrial 10
revolution and encouraged women to enter the labour force, and as more and more
women advanced rapidly from industrial jobs to professional work, gender roles were
questioned and adapted. And before long, ethics were no longer prescriptive but a matter
of preference. What used to be considered as sins – abortion, the bearing of children
outside wedlock – are now widely regarded as manifestations of freedom of choice. And 15
marriage is no longer sacrosanct but merely provisional, a contract which can be easily
terminated. As people indulge in such audacious choices, the traditional order begins to
crumble.

3 Earlier this decade, the balance of the workforce tipped towards women, who now hold
the majority of many nations’ jobs. The working class is slowly turning nations into a 20
matriarchy. Women dominate professional schools, the workplace and political arena
today. Indeed, the world is in some ways becoming a kind of travelling sisterhood:
women leaving home and entering the labour force, climbing up the corporate ladder,
and creating domestic jobs for other women, from local and abroad, to fill.

4 There are obvious reasons behind this shift. Given the phenomenal and ever-increasing 25
rate of technological changes we are witnessing, the global economy is evolving in a way
that is eroding the historical preference for men worldwide. The post-industrial economy
today is indifferent to men’s size and strength. Thinking and communication skills have
eclipsed brute strength and stamina as the keys to economic success. The attributes that
are most valuable today – social and emotional intelligence, the ability to be finely tuned 30
to verbal and body language – are not predominantly male. In fact, the opposite is true.
We see women in poorer parts of India learning English faster than men to meet the
demands of new global call centres, and women own more than forty per cent of private
businesses in China. In 2009, Iceland elected Jóhanna Sigurðardóttir as her Prime
Minister, who is the world’s first openly lesbian head of state. She campaigned explicitly 35
against the male elite and vowed to end the ‘age of testosterone’.

5 Today, the list of growing jobs is heavy on nurturing professions and places a premium
on service, in which, ironically, women seem to benefit from old stereotypes and habits.
Theoretically, there is no reason men should not be qualified for these professions. But
they have proved remarkably unable to adapt. Feminism has pushed women to do things 40
once considered against their nature: first, women enter the workforce, then they
continue to work while married, and then they work even with small children at home.
Many professions that started out as a province of men are now filled mostly with women
– teachers and soldiers come to mind. Yet, the same is not true of the opposite! Nursing
schools have tried hard to recruit men in the past few years with negligible success. 45
Education institutions, eager to recruit male role models, are having a similarly hard time.
The range of acceptable masculine roles has changed comparatively little, and has
perhaps even narrowed, as men have shied away from some careers women have
entered.

© DHS 2010 8806/02/Y6/10/PRELIM/INSERT


3

6 And women today are knocking on the door of leadership. Once it was thought that 50
leaders should be aggressive and competitive, and that men are naturally more of both.
But psychological research seems to indicate that men and women are just about equally
aggressive and competitive, with slight variations. Men tend to assert themselves in a
controlling manner, while women tend to take into account the rights of others. Of course,
when highlighting such variations, researchers may risk perpetuating crude gender 55
stereotypes: women are more empathetic and better consensus seekers. These suggest
that women bring in a superior moral sensibility to bear on a cutthroat business world.
As it is, the perception of the ideal business leader is starting to shift. The old model of
command and control which men thrive in, with one leader holding all the decision
making power, is considered archaic. The new paradigm sees leaders behave like good 60
coaches, who are able to channel their charisma to motivate others to be industrious and
creative. As such, we can resoundingly cheer that women boost corporate performance,
and we can expect innovative, successful firms to be the ones that have women in top
positions. Similarly, we see the reason why industries that are like the ghost of the
economy past – shipbuilding, steelworks and machinery – are those that have always 65
favoured the brawn.

7 It is little wonder that patriarchal societies are unable to cope with the changes, resisting
them and resorting to means to preserve the status quo. For one, there is still a prevalent
wage gap between men and women – which can be convincingly explained by
discrimination. Yes, women still do most of the parenting. And yes, the higher echelons 70
of society are still dominated by men. Moreover, a knee-jerk reaction from the timid and
conservative people, which debatably stems from anxiety and fear, against the new world
of the alpha female is to attack and vilify these accomplished women: Roman Polanski’s
novel The Ghost Writer rewrites the traditional political wife as a cold-blooded killer; Lady
GaGa’s music video for the song Telephone rewrites Thelma and Louise not as a story 75
about elusive female empowerment but ruthless abuse of power where she and her
girlfriend kill a ‘bad boyfriend’ and random others on a homicidal spree. The alpha female
does not have her defenders.

8 Should we despair? I think not. Reassuringly, a white collar economy values brain power
which men and women have in equal amounts. Perhaps most importantly – for better or 80
worse – it increasingly requires formal education credentials, which women are more
prone to acquire, since they are naturally more academically inclined. Indubitably,
enlightened organisations that tap on the talents of women can be assured of economic
success, for they are armed with brain power and not the ‘qualities’ of the dominant sex.

9 Surely, the fashionable James Bond is a dying species rather than a permanent 85
establishment. We see how the ongoing variations of the alpha male have been
degenerated from the once esteemed debonair gentlemen, to the often unemployed,
romantically challenged perpetual adolescent, or a happy misanthrope (the film ‘The 40-
Year-Old Virgin’ comes to mind), or a beer-loving couch potato (think Homer Simpson).
The reality is that the modern economy is becoming a place where women hold the 90
cards. It may be happening slowly and unevenly, but it is unmistakably the scenario for
our none-too-distant future.

© DHS 2010 8806/02/Y6/10/PRELIM/INSERT


4

BLANK PAGE

Acknowledgements:

Adapted from © Hanna Rosin, The End Of Men, The Atlantic Monthly Group, 2010.

© DHS 2010 8806/02/Y6/10/PRELIM/INSERT


DUNMAN HIGH SCHOOL
UNIVERSITY OF CAMBRIDGE LOCAL EXAMINATIONS SYNDICATE
General Certificate of Education Advanced Level
Higher 1
YEAR 6 PRELIMINARY EXAMINATION

CANDIDATE
NAME

INDEX
CLASS 6 C NUMBER

GENERAL PAPER 8806/02


Paper 2 3 Sep 2010

1 hour 30 minutes
Candidates answer on the Question Paper.
Additional Materials: 1 Insert

READ THESE INSTRUCTIONS FIRST

Write your Centre Number, index number and name on all the work you hand in.
Write in dark blue or black pen on both sides of the paper.
Do not use staples, paper clips, highlighters, glue or correction fluid.

Answer all questions.


The Insert contains the passage for comprehension.
Note that 15 marks out of 50 will be awarded for your use of language.

At the end of the examination, fasten all your work securely together.
The number of marks is given in brackets [ ] at the end of each question or part question.

For Examiner’s Use

Content / 35

Language / 15

Total / 50

This document consists of 6 printed pages and 1 Insert.


[Turn over
© DHS 2010 8806/02/Y6/10/PRELIM
2

Read the passage in the Insert and then answer all the questions which follow. Note that up to For
fifteen marks will be given for the quality and accuracy of your use of English throughout this Examiner’s
Use
Paper.

NOTE: When a question asks for an answer IN YOUR OWN WORDS AS FAR AS POSSIBLE
and you select the appropriate material from the passage for your answer, you must still use
your own words to express it. Little credit can be given to answers which only copy words and
phrases from the passage.

1 What do the words ‘drive’ (line 4) and ‘docile’ (line 6) suggest about the different nature of men
and women?

[2]

2 According to the author, what happens when ‘ethics were no longer prescriptive but a matter of
preference’ (lines 13-14)? Use your own words as far as possible.

[2]

3 What does the phrase ‘travelling sisterhood’ (line 22) suggest about women?

[1]

4 What qualities does the author imply women have from the examples given from lines 32-34?

[1]

5 ‘Feminism has pushed women to do things once considered against their nature’ (lines 40-41).

According to the author, what would women do if they were to follow their supposed nature?
Use your own words as far as possible.

[2]

© DHS 2010 8806/02/Y6/10/PRELIM [Turn over


3

For
Examiner’s
Use
6 Using your own words as far as possible, explain how women are different from men although
they are ‘equally aggressive and competitive’ (lines 52-53).

[2]

7 ‘which debatably stems from anxiety and fear’ (line 72).

What does ‘debatably’ say about the suggestion made by the author?

[1]

8 Why does the author find the ‘white collar economy’ reassuring to women (line 79)? Use your
own words as far as possible.

[2]

9 ‘… the ‘qualities’ of the dominant sex’ (line 84).

Why does the author place inverted commas around ‘qualities’?

[1]

10 Give the meaning of the following words as they are used in the passage. You may write your
answer in one word or a short phrase.

(a) audacious (line 17) [1]

(b) perpetuating (line 55) [1]

(c) knee-jerk (line 71) [1]

(d) indubitably (line 82) [1]

(e) esteemed (line 87) [1]

© DHS 2010 8806/02/Y6/10/PRELIM [Turn over


4

11 Using material from paragraphs 4 to 7, summarise what the author has to say about the For
Examiner’s
changes in the economy that are disadvantageous to men, and how men are trying to ‘preserve Use
the status quo’.

Write your summary in no more than 120 words, not counting the opening words which are
printed below. Use your own words as far as possible.

The post-industrial economy today

[8]

© DHS 2010 8806/02/Y6/10/PRELIM [Turn over


5

12 Having reviewed women’s progress in modern post-industrial society, Hanna Rosin thinks the For
Examiner’s
scenario for the near future is a place where women hold the cards and men play a reduced Use
role.

To what extent do you agree or disagree with her predictions and views? Support your answer
by referring both to what you have read in the passage and to your own experiences.

© DHS 2010 8806/02/Y6/10/PRELIM [Turn over


6

For
Examiner’s
Use

[8]

© DHS 2010 8806/02/Y6/10/PRELIM


1
2010 Year 6 Preliminary Examination
H1 General Paper (8806/02) - Suggested Answer and Mark Scheme

DUNMAN HIGH SCHOOL


UNIVERSITY OF CAMBRIDGE LOCAL EXAMINATIONS SYNDICATE
General Certificate of Education Advanced Level
Higher 1
YEAR 6 PRELIMINARY EXAMINATION

CANDIDATE
NAME

INDEX
CLASS 6 C NUMBER

GENERAL PAPER (8806/02)

SUGGESTED ANSWER AND MARK SCHEME

Final Version: 7 Sep 2010

© DHS 2010 8806/02/Y6/10/PRELIMANS


2
2010 Year 6 Preliminary Examination
H1 General Paper (8806/02) - Suggested Answer and Mark Scheme

1 What do the words ‘drive’ (line 4) and ‘docile’ (line 6) suggest about the different nature of
men and women? [2]

Inferred answer:
‘drive’ suggests that men are aggressive, motivated to achieve goals (ORA) [1]
‘docile’ suggests that women are submissive, tend to be a follower (ORA) [1]

2 According to the author, what happens when ‘ethics were no longer prescriptive but a matter
of preference’ (lines 13-14)? Use your own words as far as possible. [2]

Lifted Paraphrased

What used to be considered as sins – - People are permitted to behave in ways


abortion, the bearing of children outside which were previously considered as a
wedlock – are widely regarded as crime / immoral / an offence. [1]
manifestations of freedom of choice.
- People no longer see marriage as
And marriage is no longer sacrosanct but sacred vow [½] but a temporary
merely provisional, a contract which can be agreement which can be annulled easily.
easily terminated. [½]

3 What does the phrase ‘travelling sisterhood’ (line 22) suggest about women? [1]

Inferred answer:
Lines 23-24 give the contextual clue that women are economically/socially mobile. (ORA) [1]

Since the contextual clues given in lines 24-25 are quite clear, the following answers from
scripts are not accepted: women look out for one another; women are becoming more
independent; women have close relationship with each other…

4 What qualities does the author imply women have from the examples given from lines 32-
34? [1]

Lifted Inferred

We see women in poorer parts of India - astute learner


learning English faster than men to meet the - proficient in language
demands of new global call centres, … - linguistically inclined (ORA) [½]

… and women own more than forty per cent - enterprising, industrious,
of private businesses in China. - good at doing business
- astute, have business acumen (ORA)
[½]

Generic qualities e.g. risk taking. may be


accepted only if candidate relates them
to the context of business or enterprise

© DHS 2010 8806/02/Y6/10/PRELIMANS


3
2010 Year 6 Preliminary Examination
H1 General Paper (8806/02) - Suggested Answer and Mark Scheme

5 ‘Feminism has pushed women to do things once considered against their nature’ (lines 40-
41).
According to the author, what would women do if they were to follow their supposed nature?
Use your own words as far as possible. [2]

Lifted Paraphrased

… first, women enter the workforce, then - stay at home (or not working) [1]
they continue to work while married, and
then they work even with small children at - being a housewife / mother OR
home. - take care of children [1]

6 Using your own words as far as possible, explain how women are different from men
although they are ‘equally aggressive and competitive’ (lines 52-53). [2]

Lifted Paraphrased

Men tend to assert themselves in a - Men are likely to be more rigid /


controlling manner, domineering / authoritative. [1]

while women tend to take into account the - While women are more respectful /
rights of others. democratic/accommodating / cooperating
/ sensitive… ORA [1]

7 ‘which debatably stems from anxiety and fear’ (line 72).


What does ‘debatably’ say about the suggestion made by the author? [1]

Inferred answer:
The word suggests that the author is open minded about the suggestion, that it is a
possibility, certainly not as a hard fact. (ORA) [1]

8 Why does the author find the ‘white collar economy’ reassuring to women (line 79)? Use your
own words as far as possible. [2]

Lifted Paraphrased

Reassuringly, a white collar economy values The white collar economy


brain power which men and women have in - places importance on intellectual power
equal amounts. which women possess [½]

Perhaps most importantly – for better or - and it requires qualifications [½]


worse – it increasingly requires formal
education credentials,

which women are more prone to acquire, - which women are likely to attain [½]

since they are naturally more academically - because they are better at studies [½].
inclined.

© DHS 2010 8806/02/Y6/10/PRELIMANS


4
2010 Year 6 Preliminary Examination
H1 General Paper (8806/02) - Suggested Answer and Mark Scheme

9 ‘… the ‘qualities’ of the dominant sex’ (line 84).


Why does the author place inverted commas around ‘qualities’? [1]

Inferred answer:
The author is being sarcastic, referring to those ‘qualities’ as weaknesses instead. [1]

Candidates should also be duly rewarded for answering this question in the context of the
passage, responding that the author intends to show that these traits are no longer favoured
in the post-industrial economy.

10 Give the meaning of the following words as they are used in the passage. You may write your
answer in one word or a short phrase. [5]
- bold
- daring
(a) audacious (line 17) - unorthodox

Not courageous, gutsy, risky, outrageous


- continuing
- furthering
- keeping alive
- maintaining
(b) perpetuating (line 55)
- promoting
- spreading

Not advocating, encouraging


- automatic
- instinctive
- reflexive
- reactive
(c) knee-jerk (line 71)
- spontaneous
- unconsidered

Not kicking, immediately, natural, involuntary, uncontrollable


- certainly
- definitely
- unquestionably
- without a doubt
(d) indubitably (line 82)
- undoubtedly
- undeniably

Not inevitably
- admired
- honoured
- respected
(e) esteemed (line 87) - revered
- well-regarded

Not recognised, well known

© DHS 2010 8806/02/Y6/10/PRELIMANS


5
2010 Year 6 Preliminary Examination
H1 General Paper (8806/02) - Suggested Answer and Mark Scheme

11 Using material from paragraphs 4 to 7, summarise what the author has to say about the
changes in the economy that are disadvantageous to men, and how men are trying to ‘preserve
the status quo’.

Write your summary in no more than 120 words, not counting the opening words which are
printed below. Use your own words as far as possible.

Lifted Suggested Paraphrased Answer

From Para 4: The post-industrial economy today…


The post-industrial economy today is indifferent to a) does not unduly favour / care about men’s
men’s size and strength. attributes.

Thinking and communication skills have eclipsed b) Soft skills are more important than physical
brute strength and stamina as the keys to abilities for financial achievements.
economic success.

The attributes that are most valuable today – social c) Traits that are emphasised / deemed
and emotional intelligence, the ability to be finely important are not men’s forte.
tuned to verbal and body language – are not
predominantly male.

From Para 5:
Today, the list of growing jobs is heavy on nurturing d) Today, job opportunities are in training and
professions and places a premium on service, in development and focuses on / emphasises
which, ironically, women seem to benefit from old service.
stereotypes and habits.

Theoretically, there is no reason men should not be e) While men are suitable for these jobs, they
qualified for these professions. But they have are unable to adjust to perform them.
proved remarkably unable to adapt.

Many professions that started out as a province of f) Occupations / jobs that previously belong
men are now filled mostly with women – teachers to men are now performed / taken up by
and soldiers come to mind. Yet, the same is not women.
true of the opposite!

The range of acceptable masculine roles has g) Men refrained from / refused taking up jobs
changed comparatively little, and has perhaps even that are now done by women [½]
narrowed as men have shied away from some h) hence limiting their functions /
careers women have entered. responsibilities in society. [½]

From Para 6:
However, the perception of the ideal business i) The authoritative business leader, which
leader is starting to shift. The old model of men tend to be, is deemed out-dated /
command and control which men thrive in, with obsolete / irrelevant/old-fashioned.
one leader holding all the decision making power,
is considered archiac.

The new paradigm sees leaders behave like good j) New thinking deems leaders as teachers
coaches, who are able to channel their charisma who influence others to be productive and
to motivate others to be industrious and creative. innovative.

Similarly, we see the reason why industries that k) Labour intensive industries which employ
are like the ghost of the economy past – the men are declining.
shipbuilding, steelworks and machinery – are
those that have always favoured the brawn.

© DHS 2010 8806/02/Y6/10/PRELIMANS


6
2010 Year 6 Preliminary Examination
H1 General Paper (8806/02) - Suggested Answer and Mark Scheme
Lifted Suggested Paraphrased Answer

Men attempt to stop and manage such


From Para 7: changes by…
For one, there is still a prevalent wage gap l) resorting to practise unfair salary policy for
between men and women – which can be women (income inequality / income
convincingly explained by discrimination. disparity between the two genders)

Yes, women still do most of the parenting. m) expecting women to take care of children
[½]
And yes, the higher echelons of society are still
dominated by men. n) holding on to top positions. [½]

Moreover, a knee-jerk reaction from the timid and o) They present capable women in a bad light
conservative people, which debatably stems from where possible / demonise women through
anxiety and fear, against the new world of the the mass media.
alpha female is to attack and vilify these
accomplished women:… 1 mark each (except points G, H, M & N)

Sample Summary Answer

The post-industrial economy today does not favour men's attributes. Rather, soft skills are
more crucial for financial achievement. Furthermore, the traits emphasised are not men’s forte.
Moreover, the economy today emphasises on training and development and services sector,
which men are unable to adjust to perform in. They leave those jobs that are increasingly
dominated by women. Their style of authoritative leadership is deemed out-dated as we expect
leaders to influence others to be productive and innovative. Labour intensive industries which
employ the men are also declining. To keep the status quo, men resorted to giving women lower
pay, expecting them to take care of children and they hold on to top positions. They also
demonise capable women through the mass media.
(118 words)

© DHS 2010 8806/02/Y6/10/PRELIMANS


7
2010 Year 6 Preliminary Examination
H1 General Paper (8806/02) - Suggested Answer and Mark Scheme

12 Having reviewed women’s progress in modern post-industrial society, Hanna Rosin thinks
the scenario for the near future is a place where women hold the cards and men play a
reduced role.

To what extent do you agree or disagree with her predictions and views? Support your
answer by referring both to what you have read in the passage and to your own experiences.
[8]

Key Ideas from Paragraphs Think about…

Paragraph 1
ƒ The biological make-up of men and women - To what extent does our biological make
ƒ The entrenched gender role up determine our gender role?
ƒ Cultural practices that discriminate against - Have gender roles changed that drastically
women today? (then and now)

Paragraph 2
ƒ Changing trend at the workplace: - Is the trend that more women taking up
ƒ encouragement from government professional (instead of industrial work)
ƒ industrial and professional work necessarily true? (developed vs.
ƒ ethics no longer prescriptive but a developing countries)
matter of preference
- Shifting values: are we being ‘audacious’
when we exercise our rights, or is it more
of a case of us discarding archaic
traditions, rituals or practices that no
longer have symbolic, practical
significance in society today?

Paragraph 3
ƒ Domination of women at majority of many - While it is true that more women are in the
nation’s jobs workforce today, does it mean that they
ƒ travelling sisterhood dominate many of the nation’s jobs? (What
happens to the male workforce? Are they
unemployed now and roaming on the
streets?)

- travelling sisterhood: foreigners taking


domestic jobs left by women for the
corporate world – does this not contradict
with women leaving blue collar (industrial)
jobs for white collar (professional) jobs
stated in para 2? Is this mobility true
everywhere?

Paragraph 4
ƒ Changes in the economy in favour of - Is it true that men are naturally inept at
women and less in favour of men social skills and communication?
ƒ The attributes that are most valuable
today – social and emotional
intelligence, the ability to be finely
tuned to verbal and body language –
are not predominantly male.

© DHS 2010 8806/02/Y6/10/PRELIMANS


8
2010 Year 6 Preliminary Examination
H1 General Paper (8806/02) - Suggested Answer and Mark Scheme
Key Ideas from Paragraphs Think about…
Paragraph 5
ƒ List of growing jobs heaving on nurturing - Is the emphasis on service industry
professions and places a premium on applicable to the entire world? What about
service developing countries in Asia and South
ƒ feminism – women breaking away from America?
their nature - Does feminism mean that women are no
longer obliged to fulfil their gender role at
all in reality?

Paragraph 6
ƒ Leadership styles between men and - What does the reality in the corporate
women world tell us about male and female
ƒ Changing perception of what makes a bosses and leaders?
good leader - What are the critical success factors for
ƒ female leaders are the reason why companies today? Is it really because of
companies are successful today women? (cause and effect)
ƒ industries that are like the ghost of the - What are the factors that caused firms in
economy past are those that have certain industries to fail? Is it really
favoured the brawn because of the male leaders and
employees?

Paragraph 7
ƒ evidence of discrimination against women - Are those claims of discrimination against
ƒ Examples taken from mass media: Roman women justified?
Polanski and Lady GaGa - Could there be any other logical
explanation for the income gap between
men and women, for women being the
preferred parent to look after children and
women not taking up higher positions in
work and in society?
- Are the 2 selected examples which the
media vilify women appropriate or
representative?

Paragraph 8
ƒ white collar economy favours women given - Any scientific claim that women are more
the natural qualities of women academically inclined than men? What
about meritocracy?

Paragraph 9
ƒ the degeneration of the alpha male - Is it true? Examples cited are drawn from
the mass media – are they representative
of the male persona today?
ƒ Ian Fleming’s James Bond is a cult
classic that is enduring, with new
generation of male actors taking over
the role (from Sean Connery to Daniel
Craig)
ƒ The enduring popularity of super
heroes who are males: Superman,
Iron Man, Spider Man.

© DHS 2010 8806/02/Y6/10/PRELIMANS


HWA CHONG INSTITUTION
PRELIMINARY EXAMINATION 2nd SEPTEMBER 2010 8:15 – 9:45 AM

GENERAL PAPER 8806/1


PAPER 1 (1 HOUR 30 MINUTES)
INSTRUCTIONS TO CANDIDATES
Answer one question from this Paper.
Answers should be between 500 and 800 words in length.
Write your name, class and question number clearly.
Write in dark blue or black pen on both sides of the paper.
Do not use highlighters, glue, correction tape or fluid.
Staple this question paper to the back of your answer at the end of the examination.

1. Does foreign aid usually do more harm than good?

2. ‘There can be no keener revelation of a society's soul than the way in which it treats its
children.’ (Nelson Mandela) Comment on this observation with reference to your own
society.

3. Does the prospect of a genetically engineered future fill you with hope or despair?

4. ‘Desperate situations demand desperate measures.’ Discuss the applicability of this


advice to any two major problems confronting the world today.

5. Do you agree with the criticism that most people have been so brainwashed by
advertising that they are unable to appreciate true beauty?

6. ‘Most international organizations are all bark and no bite.’ To what extent do you
agree with this criticism?

7. ‘Live our dreams, fly our flag.’ What would your ‘dream Singapore’ be like? What
significant changes would be needed to make your dream come true?

8. ‘The use of any performance enhancing technology or substance in sporting


competitions is cheating.’ Discuss.

9. Do you agree that the most important human right is the freedom of speech?

10. ‘The notion that nuclear power is the answer to our energy problems is not only
gravely mistaken but also extremely dangerous.’ What are your views?

11. ‘Population problems eventually solve themselves – government meddling only


makes things worse.’ Discuss.

12. What distinguishes a truly great work of art from a merely average one?
Candidate's Name CT Group GP Tutor

HWA CHONG INSTITUTION


JC2 PRELIMINARY EXAMINATION 2010

GENERAL PAPER 8806/2

PAPER 2 Duration: 1 hour 30 minutes


Date: 2 September 2010
Time: 1015 – 1145 hrs

INSERT

READ THESE INSTRUCTIONS FIRST

This insert contains the passages for Paper 2.

This insert consists of 3 printed pages.


HCI C2 Preliminary Examination GENERAL PAPER 2 September 2010

Passage 1: Harry Yarger writes on HARD POWER......

1 Will military power become redundant in the coming decades? Ineffectual US interventions in Kosovo
and Bosnia, the wars in Afghanistan and Iraq and the 9-11 terrorist attacks have been seminal events
calling into question common assumptions on the efficacy of what is termed “hard power”. But whilst it
may be true that the number of large-scale, interstate wars continues to decline, and fighting is unlikely
among developed nations with elected governments on many issues, as U.S. President Barack Obama 5
said in accepting the Nobel Peace Prize in 2009: “We must begin by acknowledging the hard truth that
we will not eradicate violent conflict in our lifetimes. There will be times when nations — acting
individually or in concert — will find the use of force not only necessary but morally justified.”

2 When people speak of military power, they tend to think in terms of the resources that underlie the hard-
power behaviour of fighting and threatening to fight — soldiers, tanks, planes, ships and so forth. In the 10
end, if push comes to shove, such military resources matter. Napoleon famously said, “God is on the
side of the big battalions,” and Mao Zedong argued that, “power comes from the barrel of a gun.”
Fundamentally, the world has changed very little since Napoleon’s time: our planet is still segmented into
sovereign states with hard borders and vested interests, which means that every nation still needs an
army not only to secure its welfare but also to ensure its survival. In a world with few international laws 15
and even fewer enforceable ones, whether we like it or not, might remains right. Even on an international
level, where order is more tenuous, residual concerns about the coercive use of force, even if a low
probability, can have important effects. Military force at least helps to provide a minimal degree of order.

3 In arguing for the increasing irrelevance of hard power, many analysts point to the fact that the number of
interstate wars between “legitimate” armies has decreased drastically, rendering the use of coercive 20
force obsolete. What they fail to recognise is that the nature of conflict in the 21st century has
metamorphosised. Most “wars” today occur within, rather than between states, and many combatants do
not wear uniforms. Contemporary military theorists refer to this as “fourth-generation warfare” that has no
definable fronts. Here, the distinction between civilian and military disappears altogether with commercial
aircraft being used as weapons of mass destruction and suicide bombers killing innocent bystanders. In 25
the face of such an implacable and fanatical adversary, hard power is not just our last but our only resort.
As if all this were not enough, technology has added a new foe that only the presence of hard power can
deter: the prospect of cyber attacks by which an enemy — state or non-state — can create enormous
destruction (or threaten to do so) without the need for an army to physically cross another country’s
border. 30

4 Moreover, there is much more to military resources today than guns and battalions, fighting or
threatening to fight. Military power is also used to provide protection for and assistance to allies. Even if
the actual use of force among states has become less probable, its possibility can never be discounted,
and it is this consideration that leads rational actors to purchase expensive insurance. This leads to a
larger point about the economic role of military power in world politics. Economic security relies upon 35
stable systems and institutions, which in turn rest upon the bedrock of hard power. In a well-ordered
modern state, the credibility and reliability of this hard power depend on its monopoly on the legitimate
use of force, which allows domestic markets to operate. In chaotic conditions, markets crash and
economies crumble.

5 Military strength provides a degree of security that is to political and economic order as oxygen is to 40
breathing: little noticed until it begins to become scarce. Once that occurs, its absence dominates all
else. In this sense, the role of hard power in determining political calculations and shaping world politics
remains paramount. The recent fuzzy liberal notion that so-called “soft power” has replaced hard power
as the shaping force of world politics is as dangerous as it is unrealistic. Soft power is not only reliant on
hard power for its very existence but also totally useless without its protection. As much as Kim Jong Il 45
may enjoy watching Disney cartoons and eating fast food, Mickey Mouse is not going to persuade him to
trade his nuclear weapons for a Happy Meal.

Adapted from “Perspectives on the use of military power in the 21st century.”

2
HCI C2 Preliminary Examination GENERAL PAPER 2 September 2010

Passage 2: Shashi Tharoor writes on SOFT POWER......…

1 It is increasingly axiomatic today that the old calculations of “hard power” are no longer sufficient to guide
a country’s conduct in world affairs. Traditionally, power in world politics was seen in terms of military
might but even in the past, this was not enough; after all, the United States lost the Vietnam War, and the
Soviet Union was ignominiously defeated in Afghanistan. While the use of military power may serve to
compel temporary submission, it also presents some glaring shortcomings with regard to its ultimate 5
efficacy and its wielder’s legitimacy.

2 Military power strategies that do not take into account a country’s international image may have serious
consequences. When a country behaves like an irate teacher brandishing a big stick, its credibility
abroad deteriorates, engendering attitudes of resentment and mistrust. At the same time, international
cooperation diminishes, such that the country’s ability to obtain its objectives is actually damaged rather 10
than enhanced. The consequences of American reliance on hard power in removing Saddam Hussein
and its subsequent handling of the crisis in Iraq provide unfortunate examples. Polling data reveal
growing anti-Americanism and disillusionment with U.S. foreign policy. Worldwide opinion has mobilized
against the United States which has in turn inhibited America’s capacity to attain its goals on many
fronts. 15

3 What the United States needs to employ is not hard power, which has proved self-defeating, but soft
power – a term recently coined by Harvard professor Joseph Nye. According to Nye, “power is the ability
to alter the behaviour of others to get what you want, and there are three ways to do that: coercion,
payments and attraction. If you are able to attract others, you can economise on the sticks and carrots.”
Soft power is the power of attraction. 20

4 Nye argues that the “soft power” of a country rests primarily on three resources: its culture, its political
values, and its foreign policies. In terms of culture, the United States possesses wealth in abundance. It
is the home of MTV, Hollywood and Disneyland, McDonald’s and Starbucks, Macintosh and Microsoft –
in short, home of most of the major products that dominate daily life around the globe. The attractiveness
of these assets, and of the American lifestyle they epitomise, permits the United States to maximise its 25
ability to attract and persuade others not only to adopt its values but also to support its policies. In terms
of political values, the U.S. indisputably offers desirable democratic role models in terms of its egalitarian
systems, ethical values, and efficient institutions. If its international actions mirrored its internal ideals, the
world would indeed view it in a much more favourable light. Instead of being regarded as an arrogant and
aggressive bully, it would be regarded with admiration and respect. 30

5 The potential of soft power goes beyond the United States. Other nations could successfully acquire and
employ it. In today’s information era, three types of countries are likely to use soft power as leverage:
those whose dominant cultures and ideals are closer to prevailing global norms; those with the most
access to multiple channels of communication and thus possess more influence over how issues are
framed; and those whose reputation is enhanced by their domestic and international performance. When 35
France lost the war of 1870 to Prussia, one of its most important steps to rebuild the nation’s shattered
morale and restore its prestige was to create the Alliance Francaise to promote French language and
literature throughout the world. French culture has remained a major selling point for French diplomacy
ever since. The United Kingdom has the British Council and Germany its Goethe Institute. Today, even
China, that most combative and cranky of nations, has gone soft by marketing Confucius as a cultural 40
icon.

6 Insofar as a country's role on the world stage is increasingly decided by its international image, soft
power has become not just influential but crucial in determining success. Rather than relying on the
coercive (and usually counter-productive) use of military might, a state needs to garner cooperation and
compliance through the attractiveness of its culture and ideology as well as through the proliferation of its 45
norms and values. This use of soft power entails a more nuanced approach, involving the skillful use of
diplomacy to not only augment a state’s international standing but also make legitimate its political
status. The sooner politicians realise that soft power is a far more nimble, amenable and amiable
creature to employ than the blundering, unruly, intimidating beast of hard power currently straining at its
leash, the better.

Adapted from “The Elephant, The Tiger And The Cell Phone.”
3
HCI C2 Preliminary Examination GENERAL PAPER 2 September 2010

4
Candidate's Name CT Group GP Tutor

HWA CHONG INSTITUTION


JC2 PRELIMINARY EXAMINATION 2010

GENERAL PAPER 8806/2


PAPER 2 Duration: 1 hour 30 minutes
ANSWER BOOKLET Date: 2 September 2010
Time: 1015 - 1145 hrs

READ THESE INSTRUCTIONS FIRST

Write your name, civics group and the name of your GP tutor in the spaces provided at the top of this page.
Write in dark blue or black pen in the spaces provided on the Question Paper.
Do not use paper clips, highlighters, correction fluid or tape.

Answer ALL questions.


You may attach additional pieces of writing paper if necessary.
Staple the passages to the back of this answer booklet at the end of the examination.

There are 12 questions in this paper.


The number of marks is given in brackets [ ] at the end of each question.

(Note that 15 marks out of 50 will be awarded for your use of language.)

For Examiner’s Use


Content /35
Language /15
TOTAL /50

This answer booklet consists of 8 printed pages.


HCI C2 Preliminary Examination GENERAL PAPER 2 September 2010

For
Examiner's
Read the passages and then answer all the questions which follow below. Note that up to fifteen Use
marks will be given for the quality and accuracy of your use of English throughout this paper.

NOTE: When a question asks for an answer IN YOUR OWN WORDS AS FAR AS POSSIBLE and
you select the appropriate material from the passages for your answer, you must still use
your own words to express it. Little credit can be given to answers which only copy words
or phrases from the passages.

Questions on Passage 1

1 Give two arguments from paragraph one which support the view that military power might be
less relevant in the future. Use your own words as far as possible.

.....................................................................................................................................................

.....................................................................................................................................................

.................................................................................................................................................[2]

2 “…it is this consideration that leads rational actors to purchase expensive insurance” (line 34).
Who or what are the “rational actors” being referred to here? What is the nature of the
“expensive insurance” they buy?

.....................................................................................................................................................

.....................................................................................................................................................

.................................................................................................................................................[1]

2
HCI C2 Preliminary Examination GENERAL PAPER 2 September 2010

3 Summarise Yarger’s reasons for arguing that military power is still of crucial importance in the
world today. Use material from paragraphs 2-4 of the passage. Write your summary in no
more than150 words, not counting the opening words which are printed below. Use your own
words as far as possible.

Yarger argues that military power is still of crucial importance in today’s world because………

.....................................................................................................................................................

.....................................................................................................................................................

.....................................................................................................................................................

.....................................................................................................................................................

.....................................................................................................................................................

.....................................................................................................................................................

.....................................................................................................................................................

.....................................................................................................................................................

.....................................................................................................................................................

.....................................................................................................................................................

.....................................................................................................................................................

.....................................................................................................................................................

.....................................................................................................................................................

.....................................................................................................................................................

.....................................................................................................................................................

.....................................................................................................................................................

.....................................................................................................................................................

.....................................................................................................................................................

.....................................................................................................................................................

.....................................................................................................................................................

.....................................................................................................................................................

.................................................................................................................................................[8]

3
HCI C2 Preliminary Examination GENERAL PAPER 2 September 2010

4 “Military strength provides a degree of security that is to political and economic order as
oxygen is to breathing: little noticed until it begins to become scarce” (lines 40-41). Explain
carefully how this analogy effectively conveys the writer’s argument. Use your own words as
far as possible.

.....................................................................................................................................................

.....................................................................................................................................................

.....................................................................................................................................................

.................................................................................................................................................[2]

5 “As much as Kim Jong Il may enjoy watching Disney cartoons and eating fast food, Mickey
Mouse is not going to persuade him to trade his nuclear weapons for a Happy Meal” (lines 45-
47). What point is the writer making here? Use your own words as far as possible.

.....................................................................................................................................................

.................................................................................................................................................[1]

Questions on Passage 2

6 What two reservations about the effectiveness of military power does the writer express in the
last sentence of paragraph one? Use your own words as far as possible.

......................................................................................................................................................

......................................................................................................................................................

..................................................................................................................................................[2]

7 “If you are able to attract others, you can economise on the sticks and the carrots” (line 19).
Explain carefully what Professor Nye means here. Use your own words as far as possible.

.....................................................................................................................................................

.....................................................................................................................................................

..................................................................................................................................................[1]

4
HCI C2 Preliminary Examination GENERAL PAPER 2 September 2010

8 What does the writer believe is America’s current weakness where soft power is concerned
and what does he believe America needs to do to rectify this problem? Use your own words
as far as possible.

......................................................................................................................................................

......................................................................................................................................................

......................................................................................................................................................

..................................................................................................................................................[2]

9 “Today, even China, that most combative and cranky of nations, has gone soft by marketing
Confucius as a cultural icon” (lines 39-40). Why does the author include the word “even”
here? Use your own words as far as possible.

......................................................................................................................................................

..................................................................................................................................................[1]

10 Give two advantages of soft power compared to hard power suggested in the last sentence of
the passage. Use your own words as far as possible.

......................................................................................................................................................

......................................................................................................................................................

..................................................................................................................................................[2]

5
HCI C2 Preliminary Examination GENERAL PAPER 2 September 2010

11 Vocabulary

Give the meaning of the following words as they are used in the passages. Write your answer
in one word or a short phrase.

a) seminal (Passage 1, line 2)…………………………………………………………………….……

b) ignominiously (Passage 2, line 4)………………………………………………………………..…

c) assets (Passage 2, line 25)…………………………………………………………………....……

d) leverage (Passage 2, line 32).………………………………………………………………………

e) prestige (Passage 2, line 37) …………………………………..……………….…………………

[5]

6
HCI C2 Preliminary Examination GENERAL PAPER 2 September 2010

12 Harry Yarger argues that hard power is far more important than soft power in today’s world
whilst Shashi Tharoor disagrees.

Which writer’s arguments do you find more persuasive and why? To what extent and how
effectively does your country employ both types of power? For
Examiner's
Use
Refer to relevant material from both passages, as well as your own knowledge and opinions
to support your points.

...................................................................................................................................................

...................................................................................................................................................

...................................................................................................................................................

...................................................................................................................................................

...................................................................................................................................................

...................................................................................................................................................

...................................................................................................................................................

...................................................................................................................................................

...................................................................................................................................................

...................................................................................................................................................

...................................................................................................................................................

...................................................................................................................................................

...................................................................................................................................................

...................................................................................................................................................

...................................................................................................................................................

...................................................................................................................................................

...................................................................................................................................................

...................................................................................................................................................

...................................................................................................................................................

...................................................................................................................................................

...................................................................................................................................................

...................................................................................................................................................

7
HCI C2 Preliminary Examination GENERAL PAPER 2 September 2010

...................................................................................................................................................

...................................................................................................................................................

....................................................................................................................................................

...................................................................................................................................................

...................................................................................................................................................

...................................................................................................................................................

...................................................................................................................................................

...................................................................................................................................................

...................................................................................................................................................

...................................................................................................................................................

.....................................................................................................................................................

...................................................................................................................................................

...................................................................................................................................................

...................................................................................................................................................

...................................................................................................................................................

...................................................................................................................................................

...................................................................................................................................................

...................................................................................................................................................

...................................................................................................................................................

...................................................................................................................................................

...................................................................................................................................................

...................................................................................................................................................

.....................................................................................................................................................

...................................................................................................................................................

...................................................................................................................................................

................................................................................................................................................. [8]

8
HCI C2 Preliminary Examination GENERAL PAPER 2 September 2010

9
Hwa Chong Institution (College Section)

PRELIMINARY EXAMINATION 2010


GENERAL PAPER 8806/2

PAPER 2 ANSWER SCHEME Duration: 1 hour 30 minutes

Passage 1

1. Give two arguments from paragraph one which support the view that military power might
be less relevant in the future? Use your own words as far as possible. (2 m)

Lifting Paraphrased

Ineffectual US Interventions in Kosovo Inferred :


and Bosnia, and the wars in Afghanistan
and Iraq have been seminal events calling Despite the large scale military action in
into question common assumptions on areas of contention / countries with
the efficacy of what is termed „hard contentious issues, these have largely
power‟. been futile in resolving conflict

and have had little effect on


discouraging/deterring terrorist
the 9-11 terrorist attacks attacks/attacks on countries‟ own soils.

The number of occurrences/outbreaks of


major / sweeping / wide-ranging / global /
It is true that the number of large-scale international / worldwide conflict /
inter-state wars continues to decline, combat has decreased significantly

and
and Confrontation / warfare appears less
probable / conceivable among rich /
fighting is unlikely among developed
nations with representative governments. affluent democracies.

( Any 2 out of 4 )

1
Hwa Chong Institution (College Section)

2.―…it is this consideration that leads rational actors to purchase expensive insurance‖
(line34). Who or what are the ‗rational actors‘ being referred to here and what is the nature of
the ‗expensive insurance‘ they buy? (1m)

Lifting Paraphrased

Inferred ―Rational actors‖ refer to countries/


governments that logically/sensibly
conclude they need military deterrence
to adequately defend themselves/ensure
their survival ( 1/2m )

―Expensive insurance‖ refers to


costly/extensive investments in
weaponry / arms/ armaments / hardware
for combat/warfare. ( 1/2m )

2
Hwa Chong Institution (College Section)

3. Summarise Yarger‘s reasons for arguing that military power is still of crucial importance in
the world today. Use material from paragraphs 2-4 of the passage. Write your summary in
no more than150 words, not counting the opening words which are printed below.

Use your own words as far as possible. (8 m)

Yarger argues that military power is still of crucial importance in today’s world
because…

Lifting Paraphrased

Whether we like it or not .. might remains Military power is still the ultimate / final or
right . decisive factor

Fundamentally, the world has changed …as the traditional political system of / the
very little since Napoleon‘s time: because independent / autonomous
our planet is still segmented into nations/territories has not altered/
sovereign states with hard borders remains ;

And vested interests, with their own selfish aims / objectives


priorities

…every nation still needs an army not To ensure its well-being, existence, self
only to secure its welfare but also to protection (any one of the 3 acceptable for
ensure its survival. 1/2m ) , each country still needs a physical
defence/ military forces / militia.

In a world with few international laws and There is a paucity of / are not many global/
world-wide legislation / rules / regulations,
directives

even fewer enforceable ones, …(might and those that do exist are ineffectual /
remains right.) lack clout / cannot be successfully
implemented / put in effect

Even on an international level, where order Even if it is unlikely , armed conflict


is more tenuous, residual concerns about between states is still considered possible /
the coercive use of force, even if a low feared / insecurity/doubt/anxiety still remains
probability, can have important effects

Military force at least helps to provide a The existence of hard power continues to act
minimal degree of order. as a deterrent

the nature of conflict in the 21st century Also the type of hostilities / confrontations
has metamorphosised has radically been transformed / altered

OR

New kinds of hostilities/ confrontations /


antagonists have emerged

Most ―wars” today occur within, rather Nowadays, fighting takes place/ transpire

3
Hwa Chong Institution (College Section)

than between states and inside / intra countries

…many combatants do not wear Involving terrorists/ insurgents / radicals


uniforms. Here, the distinction between who are less easy to identify / not easily
civilian and military disappears recognizable as conventional soldiers /
altogether
impossible to distinguish from ordinary
citizens / the population at large.

... Contemporary military theorists refer to There is no demarcated / designed war


this as ―fourth-generation warfare‖ that has zone / theatre of war / battlefield / arena
no definable battlefields or fronts.

Here, the distinction between civilian and and they employ methods/tactics that are
military disappears altogether with very different / unconventional
commercial aircraft being used as
weapons of mass destruction and
suicide bombers

Killing innocent bystanders to attack non-military targets / innocent


citizens.

In the face of an implacable and fanatical Because this new enemy is extremist /
adversary, hard power is not just our last irrational/ uncompromising / obdurate, force
but our only resort. is the only option.

As if all this were not enough, technology The latest weapon is using the internet/
has added a new foe that only the computer/ electronic attacks to disrupt a
presence of hard power can deter: the country , which only the prospect of
prospect of cyber attacks by which an military retaliation can prevent /
enemy — state or non-state — can create discourage/ dissuade / daunt.
enormous destruction (or threaten to do
so) without the need for an army to
physically cross another country‘s border.

Military Power is also used to provide Hard Power can be used to safeguard and
protection for and assistance to allies. help a country‟s friends

Repeat of point 7: Despite the reduced chance / possibility /


likelihood of conflict / hostilities can never be
Even if the actual use of force among ruled out / ignored
states has become less probable, its
possibility can never be discounted,
and it is this consideration that leads so countries still need to invest in military
rational actors to purchase expensive hardware
insurance
This leads to a larger point about the Order in financial/fiscal/money / trading
economic role of military power in world markets depends on unwavering/ firm/
politics. Economic security relies upon robust / sturdy /solid organizations/
stable systems and institutions, which establishments / enterprises which are built
in turn rest upon the bedrock of hard on the foundation of hard/ military
power. might/strength.

4
Hwa Chong Institution (College Section)

In a well-ordered modern state, the This hard power must be in the hands or
credibility and reliability of this hard control of an officially recognized
power depend on its monopoly on the government to instil confidence / maintain
legitimate use of force, which allows credibility / prevent financial turmoil
domestic markets to operate. In chaotic /disarray/instability / turbulence/ confusion.
conditions, markets crash and
economies crumble.

Max 16 out of 20 points for 8m

Suggested Summary

Yarger argues that military power is still important in today’s world because…

it is still the decisive factor as the traditional political system of autonomous nations with
selfish aims remains. To ensure its well-being, each country still needs military muscle.
There is a paucity of global legislation and that which does exist lacks clout. Although
unlikely, armed conflict between states is still feared so countries need to invest in military
hardware. The existence of armies acts as a deterrent. Radically different types of
confrontation have emerged with new kinds of hostilities and antagonists. Fighting now
occurs within countries, involving insurgents not recognizable as conventional soldiers.
There is no demarcated war zone and unconventional tactics are employed to attack
innocent citizens. Force is the only option against such an uncompromising foe. The latest
weapon is using electronic attacks to disrupt a country which only the prospect of military
retaliation can discourage. Military might can also be used to safeguard or help a country‘s
friends. Order in financial markets depends on robust enterprises built on the foundation of
hard power which must be in the control of an officially recognized government to instil
confidence and prevent turmoil.

(All 20 points in full – 183 words)

5
Hwa Chong Institution (College Section)

4 ―Military strength provides a degree of security that is to political and economic order as
oxygen is to breathing: little noticed until it begins to become scarce‖ (lines 40-41). Explain
carefully how this analogy effectively conveys the writer‘s argument. Use your own words
as far as possible. (2m)

Lifting Paraphrased

Military strength provides a degree of Just as oxygen is essential / vital /


security that is to political and economic indispensable / crucial for respiration to
order as oxygen is to breathing: little ensure individual survival (1/2m) ,
noticed until it begins to become scarce.
so military power is needed for national
survival .(1/2m)

Like oxygen, the presence of military


power as a deterrent is usually taken for
granted/is inconspicuous/unobtrusive
(1/2m)

But, if they were to diminish/disappear,


both their absences would immediately
and keenly be felt ( 1/ 2m)

5. ―As much as Kim Jong Il may enjoy watching Disney cartoons and eating fast food,
Mickey Mouse is not going to persuade him to trade his nuclear weapons for a Happy Meal‖
( lines 45-47). What point is the writer making here? Use your own words as far as
possible. (1m)

Lifting Paraphrased

As much as Kim Jong Il may enjoy The writer stresses that the attractions of
watching Disney cartoons and eating fast Western popular culture, hardline/
food, Mickey Mouse is not going to obdurate / uncompromising/ militaristic
persuade him to trade his nuclear weapons despotic leaders/ governments will not
for a Happy Meal.
change their policies/become more
amenable/compliant because they are
political realists. (1m)

OR

Despite its attractions, no amount of


soft power will persuade such leaders.

(1m)

6
Hwa Chong Institution (College Section)

Passage 2

6. What two reservations about the effectiveness of military power does the writer express in
the last sentence of paragraph one? Use your own words as far as possible. (2m)

Lifting Paraphrased

While the use of military power may serve Military Power might be utilized to force /
to compel temporary submission, it also coerce / drive others yield to demands
presents some glaring shortcomings with ( 1/2m) but this is only of a brief/ short
regard both to its ultimate efficacy and
term/ short-lived / momentary nature.
its wielder‟s legitimacy.
(1/2m)

It also will incite/ elicit questions /


concerns ( 1/2m) about the authority /
source of the weight/ mandate / legality
/justice/ethics of such use (1/2m) of
force to bend others to their will.

7. ―If you are able to attract others, you can economise on the sticks and the carrots‖ (line 19)
Explain carefully what Professor Nye means here. (1)

Lifting Paraphrased

If a country has / is able to wield its soft


power well (1/2m) ,

it need not resort to the more usual


practice of enticing someone to do our
biding by dangling a reward / incentive /
enticements (1/2m) before them and,
either alternately

OR

at the same time, urging them to do so by


threatening punishments /disincentives
(1/2m)

to discourage undesired behaviour.


(1/2m)

7
Hwa Chong Institution (College Section)

8) What does the writer believe is America‘s current weakness where soft power is
concerned and what does he believe America needs to do to rectify this problem? Use your
own words as far as possible. (2m)

Lifting Paraphrased

In terms of political values, the U.S. Instead of capitalizing/leveraging on its


indisputably offers desirable democratic attractive political system which
role models in terms of its egalitarian emphasizes on equality, transparency
systems, ethical values, and efficient and accountability (1/2 m), it does not
institutions. …its international actions utilize this same yardstick to measure /
( does not ) mirrored its internal ideals, control / its actions outside of its
domestic domain. (1/2m)

OR

US foreign policies smack of double


standards/hypocrisy. (1m)

Inferred America must employ the same


Instead of (behaving like)… an arrogant standards to herself as she applies to
and aggressive bully, … it would (then ) others/internally in its foreign policy
be regarded with admiration and respect.
( 1/2m ) and not act like a belligerent
tyrant / being intimidating / browbeating
others into agreeing with her ( or her
policies) (1/2m)

9. ―Today, even China, that most combative and cranky of nations, has gone soft by
marketing Confucius as a cultural icon‖ (lines 39-40). Why does the author include the word
―even‖ here? Use your own words as far as possible . (1m)

Lifting Paraphrased

Inferred The author is suggesting surprise / that it


is remarkable (1/2m) that a country like
China , well known for / having a
reputation of being a traditional /
uncompromising wielder /user /believer
of military power / country which has
not been bothered about
rapprochement, has jumped onto the
bandwagon of soft power/ use of more
persuasive tactics ( 1 /2 m)

OR

China is a country one would least


expect to be willing to bend / be
accommodating enough to use “softer”

8
Hwa Chong Institution (College Section)

tactics. (1m )

10. Describe two advantages of soft power compared to hard power suggested in the last
sentence of the passage. Use you own words as far as possible. (2)

Lifting Paraphrased

..soft power is a far more nimble, It is more flexible/versatile as opposed to


amenable and amiable creature to employ clumsy (1/2m)
than the blundering, unruly, intimidating
beasts of hard power currently straining at compliant/easier to manage /handle
their collective leashes, the better. /restrain as opposed to unmanageable
/uncontrollable/ unrestrainable (1/2m)

attractive/endearing/amicable as opposed
to terrifying/fearsome (1/2m)

and less hostile/belligerent/ferocious.


(1/2m)

9
Hwa Chong Institution (College Section)

11. Vocabulary

Give the meaning of the following words as they are used in the passages. Write your
answer in one word or a short phrase. (5m)

a. seminal (Passage 1, line2)

1 mark 1/2 m 0 mark

strongly influential (on later determining, decisive,


developments),shaping, powerful, significant ,
formative important , leading ,
prominent , momentous,
major, noteworthy

b. ignominiously (Passage 2, line 4)

1 mark 1/2 m 0 mark

in a humiliating manner, contemptibly / dishonorably /


humiliatingly/abjectly/ shabby / sordid / sorry /
shamefully / disgracefully approbiously /tragically
/infamously

c) assets (Passage 2, line 25)

1 mark 1/2 m 0 mark

valuable possessions/ possessions, resources,


attributes property , belongings ,
worldly goods, advantages,
qualities, goodwill

d) leverage (Passage 2 , line 32)

1 mark 1/2 m 0 mark

influence/ power exerted to influence/power force, pull, stress upon ,


gain influence/ control/ emphasize upon, tool to
advantage achieve certain ends

e) prestige ( Passage 2, line 37)

1 mark 1/2 m 0 mark

high reputation/admiration Status dignity , glory , pride,


and influence / stature/ high respect , international
status/ standing/ image

10
Hwa Chong Institution (College Section)

Application Question

12. Harry Yarger argues that hard power is far more important than soft power in today‘s
world whilst Shashi Tharoor disagrees.

Which writer‘s arguments do you find more persuasive and why? To what extent and how
effectively does your own country employ both types of power?

Refer to relevant material from both passages as well as your own knowledge and opinions
to support your points. (8m)

From passage A : The following are some possible points for and against Harry Yarger's argument.

Harry Yarger argues that...

Points from Passage A Agree Disagree

 Fundamentally, the  The concept of sovereign  The idea that the world has
world has changed states is still very much changed very little, as in
very little since relevant today. International countries being segmented
Napoleon’s time: our conflicts and tensions between into sovereign states, is an
planet is still countries (Palestine and Israel, over generalisation. While
segmented into Iran/North Korea and the sovereign states will
sovereign states with U.S./U.N ) are often the result undoubtedly continue to exist
hard borders and of competing and conflicting for a long to come, the
vested interests; interests between sovereign revolution of the information
states. technology has broken down
 Territorial disputes (past and “hard borders” between
present) also validate Yarger's countries. This has resulted in
argument of "hard borders and the notion of the global
vested interests." The village.
territorial dispute over
Kashmir, (between India,
Pakistan and China), is a case
in point, as is Pedra Branca.
 ...every nation still  Much can be said about  Singapore spends a significant
needs an army not seeking alliance with the major proportion of its GDP on
only to secure its super powers, as these building its military forces
welfare but also to countries seek protection from ($11.46b), military might has
ensure its survival. these powers when they have not been used exclusively to
none. This is especially ensure its survival. Though
pertinent for Singapore. Even Singapore has one of Asia’s
though the government best-equipped militaries, the
spends quite a considerable large proportion of the
sum on defence, the fact of defence budget is used for the
the matter is, Singapore relies upkeep of military equipment
heavily on U.S. support and and the payment of salaries
alliance for protection (who for the conscription-based
has the most advanced military armed forces. Furthermore,
equipment and the biggest the survival of the nation has
defence budget in the world). instead been dependent on
Singapore knows that other factors, such as sound
adversaries that seek to economic policies, corrupt-

11
Hwa Chong Institution (College Section)

threaten Singapore will have free governance and strong


to face the hard power of the educational and social
U.S. policies.
 Military force at least  The deterrent value with super  The causal link between
helps to provide a powers with military forces military force and order is
minimal degree of cannot be denied. There is also highly questionable as real
order. the element of the balance of world examples point to the
terror. Arguably, the size and contrary. For example,
might of the U.S. military force countries like Monaco,
 In the face of such an acts as a deterrent against Western Samoa and the
implacable and rogue nations. Any country Vatican State City do not
fanatical adversary, that seeks to invade another possess armies and these
hard power is not just will always have to factor in countries are not in chaos.
our last but our only retaliation by the U.S. and Conversely, there are
resort. other allied forces. In this countries that possess a
sense then, there is a certain sizeable military force and are
degree of order with the in a perpetual state of
presence of military force. disorder, such as Haiti and
 This is particularly illuminating Congo. In addition, there is
in the context of Singapore, the fact that given how fragile
whose geopolitical climate is order is in the world today,
complex. Although Singapore military force might ironically
is a member of ASEAN, precipitate an arms race,
Singapore cannot rely on its evident during WW1.
neighbours to fight off acts of  Singapore has also not relied
aggression, given that on hard power to deal with
members of the ASEAN often “implacable and fanatical”
lack the political and economic adversaries. While Singapore
will to do so. In addition, the takes an extremely firm stand
amount of money Singapore towards foreign elements that
spends on its annual budget on threaten her survival, the use
defence is indicative of the of hard power has taken a
importance of hard power to back seat in relation to
ensure protection against education.
potential adversaries.

 This leads to a larger  The U.S. spends in excess  There is the need to
point about the of $800 billion annually on distinguish between a
economic role of national security (including standing army and civil
military power in the cost of the wars in Iraq authorities, which the writer
world politics. and Afghanistan, and the fails to do so. A strong / non-
Economic security Departments of Homeland corrupt police force enforcing
relies upon stable Security and Veterans law and order with an
systems and Affairs). But what is the independent judiciary will
point? Why do Americans
institutions, which in ensure stability and security of
spend so much more on its
turn rest upon the a state; it need not necessarily
military than does any other
bedrock of hard country, or any other be the existence of a military
power. In a well- combination of countries? army.
ordered modern  One possible argument is  In its quest for economic
state, the credibility that the purpose of security, Singapore is
and reliability of this American military power is committed to the
hard power depend to provide global public maintenance of "an open

12
Hwa Chong Institution (College Section)

on its monopoly on goods, to defend other global and regional trading


the legitimate use of countries so that they don‘t regime". Unlike traditional
force, which allows have to defend themselves, notions of national security
domestic markets to and to ensure international based on military strength, the
operate. In chaotic order for long term security imperative of a
conditions, markets economic security. vulnerable island-state is
crash and economies dependent upon economic
crumble. strength rather than military
prowess. Singapore's national
leaders recognised the
fundamental importance of
economics in ensuring the
viability of a vulnerable small
state.

 Soft power is not only  In a sense, soft and hard  Soft power can make the use
reliant on hard power power are related because of hard power redundant,
for its very existence they are both aspects of the because hard power and all
but also totally ability to achieve one’s that go with it, bribes and
useless without its purpose by affecting the threats, can only get a nation
protection behaviour of others. No matter so far. In other words, soft
how attractive the style of a power need not be seen as
country’s foreign policies (or effective only in relation to
cultural power), it cannot be the strength of hard power.
separated from its substance, Corporations in the U.S. like
which is an integral part of Hollywood, Disney, Coca Cola
hard power. People will only and Nike are some examples
admire your culture if you of the influence of soft power,
have the power to ‘coerce’ while the failures in
them. In other words, the Afghanistan and Iraq point to
country with soft power needs the inefficacy of hard power.
at least some kind of resources
(hard power) to communicate
and promote its soft power.
 To put it differently, soft
power is nothing other than
the ‘soft’ face of hard power.
Behind or beneath it is hard
power. Soft power is merely a
manifestation or presentation
of hard power. It is important
to distinguish potential and
real soft power. A county with
rich sources in soft power does
not necessarily have the
‘power’ at its disposal. In other
words, the existence of soft
power sources provides a
deposit but the country needs

13
Hwa Chong Institution (College Section)

to have the ability, means and


other resources, quite often,
power, to tap into the deposit
and convert this potential
power into real power.
Without hard power soft
power cannot work properly or
cannot work at all.

From Passage B : The following are some possible points for and against Shashi Tharoor's argument.

Shashi Tharoor argues that...

Points from Passage B Agree Disagree

 Traditionally, power  The fact that the two  While valid in principle, the
in world politics was superpowers lost these wars extent of this assertion is
seen in terms of points to the inefficacy of hard questionable. “Traditionally”,
military might but power. If hard power was the military might has defined
even in the past, this most effective means of those in power and those
was not enough; asserting power, backed by without power but the two
after all, the United strong military might (which examples cited do not validate
States lost the the U.S. and Russia possessed completely the assertion that
Vietnam War, and at that point in time), in “this was not enough.”
the Soviet Union was theory, these two countries Perhaps the defeats in the
ignominiously would not have lost the wars. Vietnam War and Afghanistan
defeated in Thus, while in theory, hard were not so much a question
Afghanistan. power would have guaranteed about the inefficacy of military
victory to the superpowers power but rather tactical
that possessed hard power, in errors in the deployment of
reality, the opposite held true. military power and the loss of
political will (with waning
public support for Vietnam).
 The attractiveness of  It can be argued that the U.S.  There are limits to what soft
these assets, and of should enhance the co-optive power could achieve. In a
the American power of its culture and the context dominated by hard
lifestyle they attraction of its lifestyle in power considerations, soft
epitomise, permits order to become power is meaningless; big soft
the United States to preponderant not only in hard power does not bend hearts; it
maximise its ability power, but also in soft power. twists minds in resentment
to attract and This will establish its and rage. Hence, the argument
persuade others not ideological domination of soft power that rests on
only to adopt its throughout the entire world. affection, attraction and desire
values but also to Soft power plays a strong is too simplistic and unrealistic.
support its policies. reactive role in international People may like some aspects
politics. Its positive impact can of American values but hate
help make a country make others, evidenced by the
feasible national strategy, mixed perception of the U.S. in
guide national enthusiasm, China: people generally admire
shape united will and strong American technological
cultural power. Thereby it can superiority and super brands

14
Hwa Chong Institution (College Section)

promote the development of but detests its policies on


comprehensive national Taiwan.
power, improve the country’s
international status, and
increase its international
contribution and influence.

 In terms of political Any country, in drawing up its  The socio-political reality of


values, the U.S. national strategy, must pay the situation in U.S. paints a
indisputably offers attention to creating better different picture. The racial
desirable democratic surroundings; to making its riot in Los Angeles, the
role models in terms Columbine High and Virginia
development model, values,
of its egalitarian Tech massacres, and the
systems, ethical lifestyle and corresponding notoriously bureaucratic
values, and efficient systems attractive, appealing institutions with overlapping
institutions. and inspiring; and to roles and responsibilities (eg,
incorporating both tangible FEMA’s poor response to
and intangible power in order Hurricane Katrina) debunk the
writer’s point of contention.
to assure the achievement of
Ironically as well, for the land
national interests. Therefore, of the free, the rate of
soft power is always the first incarceration is alarmingly
option or tool for countries to high.
deal with various affairs in  In addition, the writer’s
contemporary international argument is largely
relations. ethnocentric and possibly
condescending as it is based
on false assumptions that
American culture is superior
and that western values and
culture will continue to define
the world. Western core values
of democracy, liberty and
consumerism, no matter how
attractive or even admirable at
first sight, may not necessarily
be suitable or achievable in
other countries.
 The sooner  Due to the increasing influence  Here, the writer cleverly
politicians realise of soft power in international appeals to emotion rather than
that soft power is a relations, major powers in the reason by alluding to hard
far more nimble, world stress the enhancement power as an “unruly,
amenable and of their soft powers. As early intimidating beast” and soft
amiable creature to as the 1980s, former Japanese power as a “nimble, amenable
employ than the PM Nakasone proposed a and amiable creature.” The
blundering, unruly, strategic plan to “create a dichotomy is arguably a false
intimidating beast of culturally developed country.” one as both forms of power
hard power currently French President Chirac are needed to “legitimate” a
straining at their suggested making a cultural country’s political status.
collective leashes, Europe and establishing a  Both hard and soft power
the better. European cultural community. possess limitations, and history

15
Hwa Chong Institution (College Section)

Russian President Putin began has proven that the exclusive


to carry out his “cultural employment of one or the
expansion” strategy as acting other yields limited results.
President. In September 1992, Singapore recognises this and
former U.S. President Bush has gone on the offensive in
highlighted in his Agenda for developing and employing
American Revival, “our both forms of power. The
political and economic government spends a
connections are supplemented significant sum of the budget
by the attractiveness of on military expenditure while
American culture in the it has also spends a
world.” considerable amount of
 In the face of aggressive resources developing its soft
cultural expansion of U.S.-led power.
Western countries, developing
countries feel challenged.
President Jiang Zemin pointed
out recently that “it is vital for
most developing countries to
maintain and upgrade the
excellent traditions of their
national cultures, carry
forward their national ethics,
absorb the good cultural
achievements of others, and
keep cultural development
abreast of the times.”

To what extent and how effectively does your own country employ both types of power?

Hard Power Soft Power

 Singapore has astutely recognised the need  In many ways, while Singapore attempts to
to use both forms of power. This is why it employ soft power via attracting foreign
has invested heavily in building up its talents, its policies are arguably
military force. Singapore’s economic discriminatory and self-serving. This
importance and military capability rank it difference in the treatment between white
among Southeast Asia’s middle powers and blue collar immigrants is telling.
despite its small size and population. Unique Professionals from the I.T., medical and
among the ASEAN states, Singapore has sporting fields are often offered and granted
ignored the economic crisis affecting the permanent residency while those from the
region since 1997, and has continued its construction industry face basic problems of
military build-up, a relentless process that inadequate housing and exploitation. Hence,
began in 1965 following Singapore’s there is a certain degree of hypocrisy in the
independence. Singapore fears sudden use of soft power.
political developments in the region that  In addition, what has Singapore achieved
might require its armed forces to be used with the use of soft power? While the impact
either as a deterrent, or as a means of of hard power is normally direct and
national defence. This indicates that immediate, straight and visible, the effect of
Singapore’s leadership perceives that under soft power is indirect and takes much longer
certain circumstance, conflict could in fact to appear; it may take years to produce the

16
Hwa Chong Institution (College Section)

occur, and military defence capabilities must desired outcome. Do initiatives like
be credible at all times. The fact that it takes “Uniquely Singapore” and the SIWW actually
a very long time to build up a military work in augmenting Singapore’s regional and
capability, especially if that military international clout?
capability has to be relative to potential  The inefficacy of soft power is all the more
adversaries as well as unforeseen enemies, evident given the nebulous understanding
has meant that Singapore’s military and definition of soft power. Soft power, by
development has been continuous and definition and implementation, is intangible
sustained. This also reflects Singapore’s basic and uncontrollable. The relevance of
insecurity as a city-state in a volatile region. effectiveness of soft power depends on the
Hence, it recognizes that ultimately, the only perception of the target country audience on
form of deterrence is via hard power. the host country; ultimately, they are the
 To invoke Lord Palmerston regarding his deciders of what is attractive to them and
British foreign policy: "Nations have no what is not. Additionally, soft power is still
permanent friends or allies, they only have power and it can still make enemies. The
permanent interests”. Singapore has taken instrumental nature of soft power can lead
this cue seriously given the volatile economic to feelings of manipulation, and no country
and geopolitical setting she finds herself in. likes to be manipulated, be it real or
As a result, defence interactions between perceived.
Singapore and the US are extensive and  The Mas Selemat affair is a case in point.
expanding. Singapore has been a strong Despite being members of ASEAN, which
supporter of the US military presence in the aims to promote bilateral relations within
Asia-Pacific region. When the US withdrew the South East Asia region, Singapore did not
from Clark and Subic, Singapore was the first have enough clout for Mas Selemat to be
to offer facilities. In 1990 Singapore and the extradited from Malaysia to Singapore for
US signed an MOU to allow the US navy and trial. This serves to highlight the inefficacy of
airforce access to Singapore’s military soft power that is achieved via alliance with
facilities. Also, COMLOG WESTPAC, a US other countries, such as the ASEAN.
logistics coordinating unit which serves US  While there is an inherent need to spend on
forces deployed in the Asia-Pacific, was military defence, Singapore arguably spends
relocated to Singapore in July 1992. The far too much money in this area. One
Singapore-US MOU, now into its eighth year, suspects that the money could be better
has served both countries well. More than a used on education or health or even
hundred ships from the US Navy call at transportation. What is the point of
Singapore each year and US fighter aircraft possessing one of the most (or even the
regularly deploy to Singapore. most) impressive military forces in South
 Access to Singapore’s facilities has enabled East Asia if citizens in Singapore find the
the US military to deal quickly with costs of health care, education and
contingency situations in the Persian Gulf, transportation increasingly less affordable?
Indian Ocean, Korean Peninsula, and  The ongoing development of the Singapore
Somalia. For instance, during the Gulf War, military force is also questionable. A cursory
the US Armed Forces used Singapore as a comparison amongst the ASEAN states,
transit point for US ships, troops and aircraft notably Singapore, will indicate an increase
on their way to the Gulf. The use of Paya in all categories, such as military personnel,
Lebar Air Base helped the US to support its tanks, armoured personnel carriers (APCs),
airlift operations to Somalia. Over the years, medium-range howitzers, missile-armed
the SAF has bought many types of military naval vessels, combat helicopters and
equipment from the US, e.g., F-16s, Chinook combat aircraft. This phenomenon, namely,
helicopters and KC-135 aircraft. the concerted regional arms build-up, has
 In recent years, Singapore has drawn upon caused some to express concerns about the
the U.S. experience in the first Gulf War, and prospects of an arms race in the region. For
has noted the RMA debate in the United instance, the initial acquisition of F-16

17
Hwa Chong Institution (College Section)

States, which has touted the new combat aircraft by Singapore, then Indonesia
information, sensing, precision attack, and Thailand, as well as Malaysia’s
stealth and aerial warfare technologies subsequent interest in acquiring an
employed in the Gulf as being the precursor advanced strike fighter, point to the need to
of a fundamental change in the way wars will counter or at least not be left behind by
be fought. Singapore has taken note of these one’s neighbours.
developments, and is paying special
attention to enhancing its command,
control, communication and intelligence
systems in order to fully exploit the modern
weapons systems that it possesses or will
soon acquire.
 The use of soft power is also concomitant in
the social pervasiveness of the Singapore
Armed Forces (SAF), reinforced by glitzy
television and print commercials for careers
in the SAF that are managed by leading
advertising firms. The SAF also has its own
radio station, Power 98, which, in between
playing song requests, provides snippets of
information on the SAF's training and
operations. The SAF therefore is, ceteris
paribus, extremely well represented in
everyday life.
 There also exist very telling political
indicators of the military's pre-eminence in
Singapore, the most obvious being the
defence budget. Singapore's high and
sustained defence spending over the years
illustrates the Ministry of Defence's
(MINDEF) importance in the minds of
policymakers. In 1998, MINDEF was
allocated a total of SGD$6.1 billion, making
the island's defence spending per capita the
highest in the Asia-Pacific. In 1999, this was
increased to SGD$7.3 billion, making up 25%
of the overall budget. Of even greater
significance is the 2004 budget speech,
where Prime Minister Lee Hsien Loong
announced a permanent 2% cut in budget
allocations in FY2004 for all ministries except
MINDEF. That the country's defence
spending is generally not reduced in bad
times or good is a clear indication of
Singapore's commitment to the
uninterrupted growth of its Armed Forces.
 More visible to the man on the street are the
continual reminders during official speeches
of the need to keep the SAF strong and
credible. The most notable of these in recent
times was President S.R. Nathan's address at
the Istana on 12 Jan 2005, where he

18
Hwa Chong Institution (College Section)

proclaimed that: "…… a strong SAF remains a


priority. The SAF provides us with a credible
deterrence and effective defence. The SAF is
effective, because of our collective
determination to fight to protect ourselves,
our families and our homes when we are
endangered. The dedication and sacrifices of
generations of national servicemen
demonstrate that despite many years of
peace, this resolve remains unshaken. The
SAF will maintain a high level of operational
readiness as it modernises and transforms
into the 3rd Generation SAF."

19
INNOVA JUNIOR COLLEGE
JC 2 PRELIMINARY EXAMINATIONS 2
in preparation for General Certificate of Education Advanced Level
Higher 1

CANDIDATE
NAME

CLASS INDEX NUMBER

GENERAL PAPER 8806/01


Paper 1 2 September 2010
1 hour 30 minutes

Additional Materials: Answer Paper

READ THESE INSTRUCTIONS FIRST

Write your name and class on all the work you hand in.
Write in dark blue or black pen on both sides of the paper.
Do not use staples, paper clips, highlighters, glue or correction fluid.

Answer one question.


Note that 20 marks out of 50 will be awarded for your use of
language.

At the end of the examination, fasten all your work securely


Question
together.
All questions in this paper carry equal marks.
No.
FOR EXAMINER’S
USE
FOR MODERATOR’S USE
(Only when necessary) Content
Moderator’s Remarks:
/30

Language
/20
FINAL MARK:
TOTAL
/50

This document consists of 2 printed pages.

Innova Junior College [Turn over


2

PAPER 1

Answer one question from this Paper.

Answers should be between 500 and 800 words in length.

1. To what extent is patriotism harmful?

2. ‘The supreme purpose of learning from history is to create a better world.’


Discuss.

3. ‘The pursuit of wealth is the scapegoat for social problems.’ Is this true in your
society?

4. ‘To save the environment, a change in mindset is more important than a change
in policy.’ Do you agree?

5. Should healthcare be the government's top priority?

6. ‘Poverty is the mother of crime.’ How far do you agree?

7. ‘It never pays to be nice.’ Discuss.

8. Consider the value of food in today’s society.

9. When is the use of force acceptable?

10. ‘Artists should have total freedom of expression.’ How far do you agree?

11. How far should the government protect the rights of the minority in your society?

12. Science and technology have shaped us to be what we are today. Do you agree?

END
INNOVA JUNIOR COLLEGE
JC 2 PRELIMINARY EXAMINATIONS 2
in preparation for General Certificate of Education Advanced Level
Higher 1

GENERAL PAPER 8806/02


Paper 2 2 September 2010
INSERT 1 hour 30 minutes

READ THESE INSTRUCTIONS FIRST

This insert contains the passage for Paper 2.

This document consists of 3 printed pages.

Innova Junior College [Turn over]


2
Jess Worth writes about the problems with ethical consumerism.

1 'Ethical' shopping is all the rage these days. Consuming with a conscience - once seen as the
preserve of beardy-weirdy tree-hugging freaks and barely registering on the radar of corporate
execs and politicians - has suddenly burst noisily into the mainstream. You can now buy a more
socially or environmentally responsible version of just about anything: clockwork mobile phone
chargers, organic anti-wrinkle cream, recycled silk designer handbags, solar-powered bird baths... 5

2 The so-called 'ethical consumerism' phenomenon is nothing new. Many people partake in the more
positive pastime of trying to buy things that don't cause harm to people and the planet. Depending
on your circumstances, this could include buying fair trade and organic food and drink, supporting
local independent shops and farmers' markets, buying energy-efficient appliances, or shopping
online for sweatshop-free clothes. 10

3 But we seem to have reached a tipping point. Although ‘ethical' sales still only account for a tiny
part of the global economy, analysts and companies firmly believe the future of retail will be green,
and are rebranding and repositioning themselves accordingly. Rob Harrison from Ethical Consumer
magazine has been charting this trend: 'The big companies have moved into the ethical market
defensively. They seem convinced it will become dominant in developed economies - there'll be a 15
broad ethical mainstream with most players guaranteeing basic ethical standards, with a super-
ethical niche sitting on top.' But though sustainable shopping is becoming big business, we
shouldn't pop the organic champagne corks just yet.

4 For a start, we should be wary of the claims being made. Irish rocker Bono, ever the self-appointed
spokesperson for charitable causes, recently pontificated that: 'Shopping is politics. You vote every 20
time you spend money.' The view that you can spend your way to a sustainable world is echoed in
much of the ethical shopping sector's marketing. New Consumer, which purports to be the 'ultimate
ethical lifestyle magazine', enthuses that: 'creating a world that works for everyone has never been
easier. It lies in your simple shopping decisions and lifestyle habits!' Steady on now. It would be
great if this were true; but it isn't. 25

5 In fact, what ethical consumerism can accomplish is limited in many different ways. Of course no-
one wants to undermine the hard work, dedication and real progress of the many pioneers who
have made consuming with a conscience possible. What they have achieved is amazing. But if we
do not face up to the limitations of a consumer-driven approach to solving the world's problems,
openly debate the contradictions and shortcomings that are becoming increasingly clear, and 30
refocus our attentions on collective political action, we risk heading down a very dangerous
diversion that takes us away from the route towards genuine global justice.

6 The problem with the concept of 'ethical consumerism' is that it's something of an oxymoron. The
dictionary definition of 'consume' is 'to destroy by or like fire or disease: to cause to vanish'. A
consumer is 'a person who squanders, destroys, or uses up'. So we may be trying to do it in an 35
‘ethical' way (what's 'ethical' is of course subjective, but let's not even go there right now) but often
we are still engaged in a destructive activity. And consumerism is indeed destroying the planet. The
fatal flaw in treating consumer-led growth as the main indicator of economic success in
industrialized countries is that it assumes infinite growth is possible, and doesn't take into account
environmental and social limits. As a result, we are already well into the red, ecologically. The oil, 40
water, land, soil, clean air and mineral resources we depend upon are under severe pressure or
actively running out. It would take more than five planets to sustain the world's current population at
US consumption levels.

7 We need to change the entire structure of our exploitative, wasteful, resource-intensive economy;
and that includes buying much less stuff. Of course, purchasing more sustainable versions of the 45
things we actually need has to form part of the solution. No-one's arguing against low-energy
lightbulbs.

8 But so much of the ethical consumption boom focuses on luxury goods: fair trade roses grown in
3
huge hothouses next to Kenya's Lake Naivasha, sucking up precious water resources and then
being air-freighted to Northern supermarkets; pointless gadgets such as solar-powered cappuccino 50
whisks; silver cufflinks handmade in Mexico, screaming 'gilt without the guilt!'. Their main impact is
to make the shopper feel good – 'I’m doing something for the planet!' - without having to change
their lifestyle one bit, while the companies laugh all the way to the bank.

9 In fact, the rapid conversion of big business to all things ethical is not just about exploiting a
lucrative new market and making efficiency savings - it is also a self-preservation strategy. As the 55
science of climate change and evidence of shocking corporate practices in the Majority World have
become undeniable, the writing is slowly materializing on the boardroom wall. How to avoid being
broken up, regulated, eco-taxed, boycotted? Be one step ahead of the game and show you're
doing the right thing without the need for governments to resort to any extreme, potentially profit-
curbing measures. 60

10 One way is for companies to do what is known as 'choice editing' which is the new industry
buzzword. Quite simply, unethical options are removed from the market, 'edited out' by the
company, reducing consumer choice in pursuit of the greater good. How do you stop people buying
energy intensive incandescent lightbulbs? Just don't offer them as an option to consumers. It's as
simple as that. The problem is that this approach relies on the company to really do what is most 65
ethical, which from time to time will inevitably contradict what will make them money. So it's
possible that the best option for the environment would be not to buy a particular item from Marks &
Spencer at all, but to buy it second-hand, or maybe borrow it, or even - are you sitting down for
this? - to go without it completely. Given that the company exists to sell stuff, it's hard to imagine
'don't buy this' appearing as one of the edited options. 70

11 The voluntary nature of 'responsible' business is another severe limitation. It turns out that the
boom in organics, far from boosting small-scale sustainable farming around the world, is
industrializing the sector, squeezing the small farmers out and watering down organic standards.
Fair trade is increasingly driven, not by the needs of poor producers, but the demands of big
business. 'When fair trade cotton came on the market, you couldn't get the stuff,' reveals Paul 75
Monaghan, head of ethics and sustainable development at the Co-operative Group. 'Marks &
Spencer went out and bought the whole lot. When fair trade roses came out, Sainsbury's got them.
We were all fighting over the roses.'

12 So as a means to change the world, the ethical consumerist approach is a blunt and imprecise tool.
It is most effective when used collectively and strategically. Fair trade would not have got into the 80
public consciousness - and the supermarkets - without dedicated campaigning by thousands of
people in their local communities. But if we give ethical consumers too much power, if we believe
that the moral issues are black and white, if we get seduced by the idea that the market will
respond to our ethical and environmental concerns, adapt accordingly and somehow the woes of
the world will be solved, then we are making a huge mistake. 85

13 It is certainly a big mistake not to face up to the scale of change that's required. Surviving the
multiple impending catastrophes that our throwaway lifestyles have triggered will involve a seismic
shift in the way we live our lives. We must move away from limitless consumer-driven growth and
towards a sustainable, low-carbon model that meets everyone's needs through more connected
communities rather than gleaming shopping malls. Sometimes our most ethical shopping choice 90
will be to buy nothing; to embrace the idea that less can be more. But this is the one message that
is not coming through clearly - from NGOs, governments, business and the media. And this
particular eco-bullet is one we now have to bite.

Adapted from Jess Worth, Ethical Shopping, New Internationalist


INNOVA JUNIOR COLLEGE
JC 2 PRELIMINARY EXAMINATION 2
in preparation for General Certificate of Education Advanced Level
Higher 1

CANDIDATE
NAME

CLASS INDEX NUMBER

GENERAL PAPER 8806/02


Paper 2 2 September 2010
1 hour 30 minutes
Candidates answer on the Question Paper.
Additional Materials: 1 Insert

READ THESE INSTRUCTIONS FIRST

Write your name and class on all the work you hand in.
Write in dark blue or black pen on both sides of the paper.
Do not use staples, paper clips, highlighters, glue or correction fluid.

Answer all questions.


The Insert contains the passage for comprehension.
Note that 15 marks out of 50 will be awarded for your use of language.

At the end of the examination, fasten all your work securely together.
The number of marks is given in the brackets [ ] at the end of each question or part
question.

For Examiner’s Use

Content /35

Language /15

Total /50

This document consists of 7 printed pages and 1 blank page.

Innova Junior College [Turn over]


2
Read the passage in the Insert and then answer all the questions. Note that up to fifteen For
Examiner’s
marks will be given for the quality and accuracy of your use of English throughout this Paper. Use

NOTE: When a question asks for an answer IN YOUR OWN WORDS AS FAR AS
POSSIBLE and you select the appropriate material from the passage for your answer, you
must still use your own words to express it. Little credit can be given to answers which only
copy words or phrases from the passage.

1 What is meant by ‘tipping point’ (line 11)? Why does the author say we ‘seemed to have
reached a tipping point’?

……………………………………………………………………………………………………….

…………………………………………………………………………………………………........

…………………………………………………………………………………………………….[2]

2 ‘It would be great if this were true; but it isn't.’ (lines 24 - 25).

What is the author regretting in this sentence?

……………………………………………………………………………………………………….

..………………………………………………………………………………………………......[1]

3 What ‘fatal flaw’ (line 38) is the author referring to in paragraph 6 and what are its
consequences?

Use your own words as far as possible.

……………………………………………………………………………………………………….

……………………………………………………………………………………………………….

……………………………………………………………………………………………………….

…………………………………………………………………………………………………….[3]

4 ‘No one’s arguing against low-energy lightbulbs’ (lines 46 - 47).

What point is the author making in this sentence?

……………………………………………………………………………………………………….

…………………………………………………………………………………………………….[1]
3

5 ‘are you sitting down for this?’ (lines 68 - 69). For


Examiner’s
Use
What is the author suggesting about consumers in this question?

…..…………………………………………………………………………………………………...

…………………………………………………………………………………………………….[1]

6 Why is ‘responsible’ in inverted commas in line 71?

…..…………………………………………………………………………………………………...

…………………………………………………………………………………………………….[1]

7 In paragraph 11, what are the undesirable consequences of the growth of the organics
industry?

……………………………………………………………………………………………………….

……………………………………………………………………………………………………….

…………………………………………………………………………………………………….[2]

8 In paragraph 12, according to the author, when does the ethical consumerist approach
work best?

Use your own words as far as possible.

…………………………………………………………………………………………………….....

…………………………………………………………………………………………………….[1]

9 Explain what the author means by ‘this particular eco-bullet is one we now have to bite’
(lines 92 - 93).

……………………………………………………………………………………………………….

……………………………………………………………………………………………………….

…………………………………………………………………………………………………….[2]
4

10 Give the meaning of the following words as they are used in the passages. Use one For
Examiner’s
word or a short phrase. Use

a rage (line 1) ……………………………………………………………….. [1]

b defensively (line 15) ……………………………………………………………….. [1]

c pontificated (line 20) ……………………………………………………………….. [1]

d purports (line 22) ……………………………………………………………….. [1]

e seismic (line 87) ……………………………………………………………….. [1]

[Total: 5]
5
11 Using material from paragraphs 8 to 11, summarise why and how companies turn to For
Examiner’s
ethical business and the limitations of such business. Use

Write your summary in no more than 120 words, not counting the opening words which
are printed below. Use your own words as far as possible.

Companies turn to ethical business because……….………..………….……………………..

……………………………………………………………………………………………………….

……………………………………………………………………………………………………….

……………………………………………………………………………………………………….

……………………………………………………………………………………………………….

……………………………………………………………………………………………………….

……………………………………………………………………………………………………….

……………………………………………………………………………………………………….

……………………………………………………………………………………………………….

……………………………………………………………………………………………………….

……………………………………………………………………………………………………….

……………………………………………………………………………………………………….

……………………………………………………………………………………………………….

……………………………………………………………………………………………………….

……………………………………………………………………………………………………….

……………………………………………………………………………………………………….

……………………………………………………………………………………………………….

………………………………………………………………………………………………………

………………………………………………………………………………………………………

……………………………………………………………………………………………………….

……………………………………………………………………………………………………….

…………………………………………………………………………………………………….[8]
6
12 In this article, Jess Worth discusses the limitations of ethical consumerism. For
Examiner’s
Use
To what extent do you agree or disagree with her views? How far do you think your
society practises ethical consumerism?

………………………………………………………………………………………………………

………………………………………………………………………………………………………

………………………………………………………………………………………………………

………………………………………………………………………………………………………

………………………………………………………………………………………………………

………………………………………………………………………………………………………

………………………………………………………………………………………………………

………………………………………………………………………………………………………

………………………………………………………………………………………………………

………………………………………………………………………………………………………

………………………………………………………………………………………………………

………………………………………………………………………………………………………

………………………………………………………………………………………………………

………………………………………………………………………………………………………

………………………………………………………………………………………………………

………………………………………………………………………………………………………

………………………………………………………………………………………………………

………………………………………………………………………………………………………

………………………………………………………………………………………………………

………………………………………………………………………………………………………

………………………………………………………………………………………………………

………………………………………………………………………………………………………

………………………………………………………………………………………………………

………………………………………………………………………………………………………
7
For
Examiner’s
Use
………………………………………………………………………………………………………

………………………………………………………………………………………………………

………………………………………………………………………………………………………

………………………………………………………………………………………………………

………………………………………………………………………………………………………

………………………………………………………………………………………………………

………………………………………………………………………………………………………

………………………………………………………………………………………………………

………………………………………………………………………………………………………

………………………………………………………………………………………………………

………………………………………………………………………………………………………

………………………………………………………………………………………………………

………………………………………………………………………………………………………

………………………………………………………………………………………………………

………………………………………………………………………………………………………

………………………………………………………………………………………………………

………………………………………………………………………………………………………

………………………………………………………………………………………………………

………………………………………………………………………………………………………

………………………………………………………………………………………………………

………………………………………………………………………………………………………

………………………………………………………………………………………………………

………………………………………………………………………………………………………

………………………………………………………………………………………………………

…………………………………………………………………………………………………….[8]
8

BLANK PAGE
Innova Junior College
JC 2 Preliminary Examination 2
General Paper, Paper 2
Suggested Answer Scheme

From paragraph 3

1) What is meant by ‘tipping point’ (line 11)? Why does the author say we ‘seemed
to have reached a tipping point’? [2m]

From the passage Suggested answer

But we seem to have reached a It means that consumption habits are about to
tipping point. change significantly all over the world.

OR

‘Tipping point’ means when a significant change


takes place leading to a new and irreversible
development. / point of no turning back or point of
no return. [1] (Award 1m even if context is missing)

Although „ethical' sales still only He says so because even companies think that in
account for a tiny part of the time to come, there will be an increase in demand
global economy, for environmentally friendly products, [1/2]
analysts and companies firmly
believe the future of retail will
be green,

and are rebranding and and are thus changing their image and company
repositioning themselves philosophy to fit in [1/2]
accordingly.

From paragraph 4

2) ‘It would be great if this were true; but it isn't.’ (lines 24-25)
What is the author regretting in this sentence? [1m]

From the Passage Suggested Answer

you can spend your way to a He regrets that the belief held by many that
sustainable world consumerism could be an ethical way to save the
environment is actually false. [1]
OR
'creating a world that works for
everyone has never been
easier. It lies in your simple
shopping decisions and lifestyle
habits!'

..but it isn‟t (true).

From paragraph 6

3) What ‘fatal flaw’ is the author referring to in paragraph 6 and what are its
consequences?
Use your own words as far as possible. [3m]

From the Passage Suggested Answer

The fatal flaw in treating When the economic development in developed


consumer-led growth as the nations is measured chiefly in terms of people‟s
main indicator of economic spending [1/2], it supposes/takes for
success in industrialized granted limitless expansion can happen/take
countries is that it assumes place [1/2],
infinite growth is possible,

and doesn't take into account and does not take into consideration the extent that
environmental and social nature and human needs allow. [1/2]
limits.

Consequences Consequences
The oil, water, land, soil, clean The natural resources/endowments OR assets
air and mineral resources supplied by nature
we depend upon are that we rely on [1/2]

under severe pressure or are over-exploited or depleting fast. [1/2]


actively running out. (1/2 m even if they give either one)

It would take more than five Our present usage of natural resources far
planets to sustain the world's exceeds/is very much more than what the earth
current population at US can supply. [1/2]
consumption levels.
From paragraph 7

4) ‘No one’s arguing against low-energy lightbulbs’ (lines 46 - 47)


What point is the author making in this sentence? [1m]

From the Passage Suggested Answer

change the entire structure of While the author thinks that we should consume
our exploitative, wasteful, less, he concedes that [bonus 1/2]
resource-intensive economy;…
it is clear that we need to
purchasing more sustainable consume less energy/ be more protective of our
versions of the things we energy needs [1/2]
actually need
OR

that buying environmentally friendly versions of


essential products [1/2]
has to form part of the
must also be a way to save the earth. [1/2]
solution.

From paragraph 10

5) ‘are you sitting down for this?’ (lines 68-69). What is the author suggesting
about consumers in this question? [1m]
So it's possible that the best The writer thinks that consumers are too entrenched
option for the environment in their shopping habits [1/2]
would be not to buy a particular and may not be able to stop purchasing unnecessary
item from Marks & Spencer at products entirely. [1/2]
all, but to buy it second-hand, or
maybe borrow it, or even - are OR
you sitting down for this? - to
go without it completely. They will be shocked/taken aback to hear [1/2] the
suggestion to stop purchasing unnecessary products
entirely. [1/2]

OR

They will find the suggestion to stop purchasing


unnecessary products entirely [1/2]
inconceivable/preposterous/unbelievable. [1/2]
6) Why is ‘responsible’ in inverted commas in line 71? [1m]

From the Passage Suggested Answer

The voluntary nature of The inverted commas show that the word „responsible‟
'responsible' business is is used ironically OR the writer has reservations
another severe limitation. about/doubts the validity of the claim [1/2]

as the problems that come with such business


suggest that they are not so ethical after all. [1/2]

7) In paragraph 11, what are the undesirable consequences of the growth of the
organics industry? [2m]

It turns out that the boom in Instead of encouraging limited-sized,


organics, far from environmentally friendly agriculture globally, [1/2]
boosting small-scale
sustainable farming
around the world,

is industrializing the sector, the growth of the organics industry is becoming too
capital intensive/ requires too much machinery
[1/2],

squeezing the small farmers causing small producers to lose business and [1/2]
out and

watering down organic compromising the quality of organic products. [1/2]


standards.

From paragraph 12

8) In paragraph 12, according to the author, when does the ethical consumerist
approach work best?
Use your own words as far as possible. [1m]

From the Passage Suggested Answer


It is most effective when used It works best when large numbers of a
collectively group/society take up the cause together [1/2]

and strategically. with a clear plan of action/objective. [1/2]

From paragraph 13

9) Explain what the author means by ‘this particular eco-bullet is one we now have
to bite.’ (lines 92-93) [2m]

From the passage Suggested answer

Sometimes our most ethical We need to exercise responsible/prudent


shopping choice will be to buy consumerism /reduce our consumption as the
nothing; to embrace the idea public [1/2]
that less can be more.

But this is the one message But the voluntary citizen groups/non-profit agencies,
that is not coming through political leaders ,private companies and the news
clearly - from NGOs, agencies [1/2 for any 2] are not explicitly
governments, business and the promoting/advocating/ encouraging such
media. behaviour [1/2]

And this particular eco-bullet Hence we now have to face the


is one we now have to bite. inevitable/impending environmental/ecological
disaster [1/2] stoically/bravely/with fortitude.
[1/2m]

OR

Since political leaders, non-profit agencies,


companies and the mass media [1/2] are not explicitly
promoting [1/2] responsible consumerism amongst the
people [1/2], we now have to face the inevitable
environmental disaster [1/2] bravely. [1/2]

MAX 2m to be awarded

10) Vocab:

1m ½m 0m

Rage NOUN (line 1) - Trend that is widely - Trend - Anger


popular/fashionable - Popular - Fury
'Ethical' shopping is - Widespread - Violence
all the rage these enthusiasm - Buzz
days. - Fad/craze/fashion - Frenzy
- Talk of the town

Defensively - Protectively - Cautiously - Aggressively


ADVERB (line 15) - Carefully in order to - Carefully - Protected
safeguard/protect
'The big companies one‟s interest
have moved into the - Carefully in order to
ethical market protect against risks
defensively.

Pontificated VERB - Preached - Lectured - Mentioned


(line 20) - Lectured in a - Spoke
pompous manner - Talked
Irish rocker Bono, - Spoke/expressed - Expressed
ever the self- judgement in a - Argued
appointed pompous/dogmatic - Stated
spokesperson for manner
charitable causes,
recently pontificated
that: 'Shopping is
politics. You vote
every time you
spend money.'

Purports VERB (line - Claims/professes/ - Claims - Says


22) asserts/declares in - Professes - Asserts
an apparently false - Self-proclaims - Declares
New Consumer, manner
which purports to be
the 'ultimate ethical - Alleges/
lifestyle magazine', claims/professes/as
enthuses that: serts/declares in a
'creating a world manner that is not
that works for credible/believable
everyone has never
been easier. - Falsely gives the
appearance of
being something

Seismic - Massive - Major - Any literal


ADJECTIVE (line - Of enormous - Profound earthquake
87) proportions - Far-reaching references
- Very extensive and - Extensive - Very strong
Surviving the profound - Great - Groundbreaking
multiple impending - Having highly - Dramatic
catastrophes that significant - Significant
our throwaway consequences - Huge
lifestyles have - Enormous - Very big
triggered will involve proportions - Impactful
a seismic shift in the - Drastic - Important
way we live our - Tremendous
lives. - Gigantic
- Earthshaking
- Very important

11) Summary:
Using materials from paragraphs 8-11, summarise why and how companies turn
to ethical business and the limitations of such business. [8m]

Companies turn to ethical business because…

No Points Paraphrased Marks


Para 8
1 But so much of the ethical consumption  Companies concentrate on ½
boom focuses on luxury goods: (line 48) expensive/branded products
(how)

2 fair trade roses grown in huge hothouses  Even though producing these ½
next to Kenya's Lake Naivasha, (line 49) goods take up land space
(limit)
sucking up precious water resources… ½
(line 49-50)  and valuable water (limit)

pointless gadgets such as solar-powered


cappuccino whisks… (line 50-51)  and create useless devices ½
(limit) Max
1m
(any 2 points for max 1m)

3 Their main impact is to make the shopper  as it makes consumers guilt- ½


feel good – 'I‟m doing something for the free (how)
planet!' – (line 51-52)

without having to change their lifestyle  while not requiring/needing ½


one bit, (line 52-53) them to modify/alter their
spending habits (how)

while the companies laugh all the way to  which benefits ½


the bank. (line 53) businesses/reaps profits for
businesses (limit)

Para 9
4 rapid conversion of big business to all  Companies turn to ethical
things ethical is not just about exploiting a business not only to take ½
lucrative new market and (line 54-55) advantage of profitable
fresh demand, (why)
making efficiency savings – (line 55)  and become less wasteful ½
(why)

it is also a self-preservation strategy. (line  but also to protect their self- ½


55) interest (why)

5 to avoid being broken up, regulated,  To evade being divided, 1


eco-taxed, boycotted? (line 57-58) controlled, taxed and
shunned, (why)

Any 2 (divided, controlled, taxed


or shunned)

Or

Without the need for


governments to resort to any
extreme, potentially profit-
curbing measures
6 Be one step ahead of the game and (line  they must/have to pre-empt ½
58) possible restrictions (how)

or

 anticipate future problems

 and demonstrate that they ½


show you're doing the right thing (line are taking ethical actions
58-59) (how)

Para 10
7 One way is for companies to do what is  Companies can choose not ½
known as 'choice editing'… unethical to sell non-environmentally-
options are removed from the market, friendly/morally wrong
'edited out' by the company, (line 61-63) products (how)

reducing consumer choice in pursuit of


the greater good…. Just don't offer them  Thus decreasing buyers’ ½
as an option to consumers. It's as simple options for the sake of a
as that. (line 63-65) wider benefit/of the
environment

OR

 thus increasing the option


for more ethical alternatives
(how)

8 The problem is that this approach relies on  Not selling products ½


the company to really do what is most unavoidably goes against
ethical which from time to time will the profit motive (limit)
inevitably contradict what will make
them money. (line 65-66)
OR
OR

Given that the company exists to sell  And (but) the role of the
stuff, (line 69) business is to make profits
(limit)

So it's possible that the best option for  To save the natural
the environment would be not to buy a surrounding/nature we should ½
particular item from Marks & Spencer at avoid consumerism.
all, but to buy it second-hand, or maybe
borrow it, or even - are you sitting down for OR
this? - to go without it completely. (line
66-69)  The needs of nature are
incompatible with
businesses (limit)

OR OR

it's hard to imagine 'don't buy this'  They are unlikely to employ
appearing as one of the edited options. this method/use to stop
(line 69-70) anyone from purchasing
(limit)

Max 1m

Para 11 (All limitations)


9 The voluntary nature of 'responsible'  Ethical business is not ½
business is another severe limitation. It required/ mandatory/ or
turns out that the boom in organics, (line merely optional
71-72)

far from boosting small-scale  It does not encourage limited- ½


sustainable farming around the world sized, environmentally friendly
(line 72) agriculture globally

10 is industrializing the sector … (line 73)  But it is becoming too capital ½


intensive/involves intensive
machinery

squeezing the small farmers out and  Smaller producers are ½


(line 73) muscled out of the
market/lose business

watering down organic standards. (line  and the quality of organic ½


73) products are compromised

11 Fair trade is increasingly driven, not by  Fair trade does not suit/ ½
the needs of poor producers, (line 74) resolve the plight of
struggling farmers
½
but the demands of big business. (line  But is led by the needs of
74-75) large corporations.

12 'M&S went out and bought the whole lot.  Large companies can ½
When fair trade roses came out, monopolise the supply
Sainsbury's got them. (line 76-77)
We were all fighting over the roses.' (line  And limit its (supply) ½
78) availability/ increase
competition

12) AQ:
In this article, Jess Worth discusses the limitations of ethical consumerism.

To what extent do you agree or disagree with his views? How far do you think
your society practises ethical consumerism? [8m]

To what extent do you agree or disagree with his views?

From the passage Possible Points of Agreement Possible Points of


Disagreement
„But if we do not face up If people try too hard to find fault
to the limitations of a with ethical consumerism, it may
consumer-driven deter companies from wanting to
approach to solving the be socially responsible so as not
world‟s problems, openly to face any backlash from the
debate the contradictions public. This may set us back even
and shortcomings that further in our “route towards
are becoming genuine global justice”.
increasingly
clear……takes us away
from the route towards
genuine global justice.
„(line 28-32)

„We need to change the


entire structure of our
exploitative, wasteful,
resource-intensive Disagree. Resources that are
economy; and that required to sustain the
includes buying much environment will be reduced if
less stuff.‟ consumers do not buy these
No-one's arguing against items. Companies will not have
low-energy lightbulbs.‟ the incentive to go into ethical
(line 44-47) businesses since there are no
profit margins. They can then
contribute to unethical businesses.

„The problem with the Ethical consumerism can be a


concept of 'ethical logical statement. It can support
consumerism' is that it's environmentally friendly practices
something of an such as growing coffee in a
oxymoron‟ (line 33) responsible manner. Eg: The
Body Shop, Starbucks

„ethical consumption The luxury business is a


boom focuses on luxury competitive one and this is one of
goods‟(line 48) the marketing strategies. High net
worth consumers, given their
resources, are often targeted to
get them to do something for the
environment. However, luxury
brands also have a reputation for
exploiting workers, such as low
wages, exposure to harmful
chemicals (Eg: Gucci). Luxury =
not necessity

„the rapid conversion of Harmful industries such as those


big business to all things in the petrochemical industries
ethical… a self- have always been an avid
preservation strategy‟ supporter of environmental
(line 54-55) causes. (Eg: Shell, BP, tobacco
companies)
„unethical options are If it means consumers end up
removed from the buying more environmentally
market, …reducing friendly products (even if it is
consumer choice in because they have no other
pursuit of the greater choice), „forced responsible
good… Just don't offer shopping‟ may be worth the price
them as an option to of reduction in consumer choice
consumers. It's as simple (for the good of the environment).
as that.‟ (line 62-65) This is particularly true with goods
which have become more a
necessity than a luxury item such
as light bulbs.
„What ethical Leaving the „ethical decision‟ to Granted there is a tenuous line
consumerism can companies may indeed not be that separates companies‟ desire
accomplish is limited in good. Apart from the issue of a to be socially responsible and to
many different ways.‟ conflict of interest („doing what is bring in profits, there is no denying
(line 26) ethical‟ and company bottomline) that ethical consumerism can
which the writer raises, there is the accomplish some good in the
„The problem is that this issue of the lack of responsibility world however limited such good
approach relies on the and awareness on the consumers‟ may be.
company to really do part.
what is most ethical,
which from time to time Consumerism is ultimately driven
will inevitably contradict market demand and profit
what will make them margins. It is difficult for
money.‟ (line 65-66) companies to balance their
altruistic motives and profit
expectations. Hence, ethical
consumerism can hit an altruistic
ceiling, limiting the possible good
that it can actually achieve.

„So it's possible that the This is indeed the ideal form of
best option for the ethical consumerism. Although
environment would be clearly it is not applicable to all
not to buy a particular goods, if people embrace this
item from Marks & idea, it would mean less strain on
Spencer at all, but to buy the environment through reduced
it second-hand, or production and use of natural
maybe borrow it, or even resources and reduced waste
- are you sitting down for products.
this? - to go without it
completely.‟ (line 66-69)

„Fair trade is increasingly Beyond the problem the writer Fairtrade makes a major
driven, not by the needs raises, Fairtrade has developed as difference to millions of
of poor producers, but a badge system that people can developing-world farmers and
the demands of big identify, buy the product and workers. The Fairtrade Labelling
business. (line 74-75) abrogate responsibility. Just Organisation says that, in 2004,
because one product carries the producers got an extra $100m in
Fairtrade mark doesn't mean that revenue as a result of Fairtrade –
the company has a responsible 10% of the value of Fairtrade retail
social ethos. People feel good sales that year. But it‟s not just
purchasing brands like The Body about prices – close co-operation
Shop, Ben & Jerry‟s and De with producers and the assurance
Beers. of long-term contracts encourages
community development in areas
such as education and health. In
short, there are poor farmers who
benefit from Fairtrade still and this
alone may make this venture
worthwhile. It is not perfect but it
should not be dismissed as
useless either.
„as a means to change Ethical consumerism as it is
the world, the ethical practised now is indeed an
consumerist approach is ineffective and inaccurate way of
a blunt and imprecise dealing with our environmental
tool.‟ (line 79) problems.
„Surviving the multiple It will indeed require a very big Possible for ethical consumerism
impending catastrophes shift in lifestyle for most of us, to make a difference. With
that our throwaway particularly in modern affluent conscious buying comes
lifestyles have triggered society where the throwaway heightened awareness of
will involve a seismic culture is well-entrenched eg. environmental, human rights and
shift in the way we live societies with excessive shopping animal rights issues. Shopping is
our lives. (line 86-88) cultures like Singapore and Dubai. a common pastime of
It can also be a matter of survival Singaporeans which makes being
for societies such as the United environmentally conscious a much
States where 50% of the economy easier task for Singaporeans to do
is made up consumer spending. on a daily basis.

How far do you think your society practises ethical consumerism?

From the passage How far does your society practise EC?
“The problem with the A minority of Singaporeans practice ethical consumerism by supporting
concept of 'ethical these products but the movement is backed by strong and persuasive
consumerism' is that it's marketing strategies from companies like The Body Shop.
something of an
oxymoron” (line 33)

We need to change the Singapore is a rich country and many citizens are engaged in wasteful
entire structure of our activities such as excessive purchases, evident in the viability of the
exploitative, wasteful, many shopping malls. There is also emphasis on packaging on top of
resource-intensive quality purchases. Given ethical purchases can sometimes cost more,
economy; and that (eg: Body wash from The Body Shop VS Lux soap, energy saving air-
includes buying much conditioners) Ethical consumerism is probably the last thing on
less stuff. Singaporeans‟ practical minds.
No-one's arguing against
low-energy lightbulbs.‟
(line 44-47)

„ethical consumption Singaporeans do buy into this marketing strategy. The high net worth
boom focuses on luxury consumers could use this to get into the limelight or gain respect from
goods‟ (line 48) their peers.

the rapid conversion of In Singapore, businesses have used ethical consumerism as marketing
big business to all things strategies to capture market share. (Eg: HSBC, the environmentally
ethical… a self- friendly bank to differentiate itself from the competitors)
preservation strategy‟
(line 54-55)
„So it's possible that the Second hand culture not well-established in Singapore (except for cars)
best option for the largely due to the availability and affordability of new products. Easy
environment would be instalment schemes are used by businesses to attract customers to buy
not to buy a particular new products as well. (Courts, Harvey Norman, interest-free
item from Marks & instalments, credit cards etc.)
Spencer at all, but to buy
it second-hand, or maybe
borrow it, or even - are
you sitting down for this?
- to go without it
completely.‟ (line 66-69)
Organics (para 11) While the organic market is getting bigger with greater awareness
among Singaporeans, not everyone is turning to organic products
because they are much more expensive than the non-organic variety
and thus affordable only to the more affluent. For the majority of
Singaporeans, price seems to be the biggest determining factor in
making consumer choices.

Singaporeans like the idea but are unable to embrace ethical


consumerism entirely due to such restrictions. Variety is limited which is
perhaps due to the small market hence creating a vicious cycle.
„Sometimes our most 1. Media influence – advertisements
ethical shopping choice 2. Singaporeans love for sales
will be to buy nothing; to 3. Great Singapore Sale & IT Fair
embrace the idea that 4. „Endorsement‟ by the governmentt – good for the economy
less can be more. But Amidst all this, it is very difficult for Singaporeans to „embrace the
this is the one message idea that less is more‟. They are more used to the idea of good
that is not coming bargain buys where the slogan is „the more you buy, the more you
through clearly - from save.‟
NGOs, governments,
business and the media.‟ Even the choice of buying environmentally friendly products may
(line 90-93) not be available to many Singaporeans due to the high cost and the
government is not doing much to improve the situation e.g. hybrid
cars.

There is token effort on the part of businesses to support ethical


consumerism such as encouraging customers to bring their own
reusable bags or charging a small amount for plastic bags. But in
this „city of convenience‟, even the reusable bag culture has not
caught on among many Singaporeans although some do try to
make the effort to embrace this initiative.
JURONG JUNIOR COLLEGE
JC 2 Preliminary Examination 2010

CANDIDATE NAME

GP TUTOR’S NAME CLASS

General Certificate of Education

GENERAL PAPER 8806/1


Paper 1 19 August 2010

Additional Materials: Answer Paper 1 hour 30 minutes

READ THESE INSTRUCTIONS FIRST

Write your name, civics class and GP tutor’s name on all the work you hand in.
Write in dark blue or black pen on both sides of the paper.
Do not use paper clips, highlighters, glue or correction fluid.

Answer one question.


Note that 20 marks out of 50 will be awarded for your use of language.

At the end of the examination, fasten all your work securely together.
All questions in this paper carry equal marks.

EXAMINER’S USE

Content / 30 Comments:

Language / 20

Total / 50

This document consists of 2 printed pages.

© JURONG JUNIOR COLLEGE 2010 8806/01/PRELIM/10 [Turn over


Answer one question from this Paper

Answers should be between 500 and 800 words in length.

1. How important is the role of advertising in today’s economy?

2. ‘History holds little value for the suppressed.’ Is this a fair comment?

3. What is man without religion?

4. To what extent has water become the new basis of conflict in the modern
world?

5. ‘Prosperity makes friends, adversity tries them.’ Discuss.

6. Is education truly rewarding?

7. Should there still be limits on artistic expression?

8. Is it possible to achieve a balance between liberty and security?

9. Assess the impact of the mass media on the culture of your society.

10. ‘Celebrities and politics do not mix.’ What is your view?

11. Is it true that modern life breeds aggressiveness?

12. ‘Eco-tourism is merely a means of making easy profit.’ Do you agree?

© JURONG JUNIOR COLLEGE 2010 8806/01/PRELIM/10 [Turn over


JURONG JUNIOR COLLEGE
JC 2 Preliminary Examination 2010

CANDIDATE NAME

GP TUTOR’S NAME Class

General Certificate of Education

GENERAL PAPER 8806/2


Paper 2 19 August 2010

INSERT 1 hour 30 minutes

READ THESE INSTRUCTIONS FIRST

This insert contains the passage for Paper 2.

This document consists of 4 printed pages and 1 blank page.

[Turn over

© JURONG JUNIOR COLLEGE 2010 8806/02/INSERT/PRELIM/10


2

BLANK PAGE

© JURONG JUNIOR COLLEGE 2010 8806/02/INSERT/PRELIM/10


3
JOANN RODGERS writes about addiction.

1 Our addiction theories and policies are woefully outdated. Research shows that there
are no demon drugs. Nor are addicts innately defective. Nature has supplied us all with
the ability to become hooked—and we all engage in addictive behaviors to some
degree.

2 Millions of Americans are apparently "hooked," not only on heroin, 5


morphine, amphetamines, tranquilizers, and cocaine, but also nicotine, caffeine, sugar,
steroids, work, theft, gambling, exercise, and even love and sex. The War on Drugs
alone is older than the century. In the early 1990s, the United States spent $45 billion
waging it, with no end in sight, despite every kind of addiction treatment from
psychosurgery, psychoanalysis, psychedelics, and self-help to acupuncture, group 10
confrontation, family therapy, hypnosis, meditation, education and tough love.

3 There seems no end to our dependencies, their bewildering intractability, the glib
explanations for their causes and even more glib solutions. The news, however, is
that brain, mind, and behavior specialists are re-thinking the whole notion of addiction.
With help from neuroscience, molecular biology, pharmacology, psychology, 15
and genetics, they're challenging their own hardcore assumptions and popular
"certainties" and finding surprisingly common characteristics among addictions.

4 They're using new imaging techniques to see how addiction looks and feels and where
cravings "live" in the brain and mind. They're concluding that things are far from
hopeless and they are rapidly replacing conjecture with facts. 20

5 For example, scientists have learned that every animal, from the ancient hagfish to
reptiles, rodents, and humans, share the same basic pleasure and "reward" circuits in
the brain, circuits that all turn on when in contact with addictive substances or during
pleasurable acts such as eating. One conclusion from this evidence is that addictive
behaviors are normal, a natural part of our "wiring." If they weren't, or if they were rare, 25
nature would not have let the capacity to be addicted evolve, survive, and stick around
in every living creature. Everyone engages in addictive behaviors to some extent
because such things as eating, drinking, and sex are essential to survival and highly
reinforcing. We get immediate gratification from them and find them very hard to give
up. The inescapable fact is that nature gave us the ability to become hooked because 30
the brain has evolved a reward system, just as it has a pain system. The fact that some
things may accidentally or inadvertently trigger that system is somewhat beside the
point.

6 What we now call "addictions," in this sense, are cases of a good and useful
phenomenon taken hostage, with terrible social and medical consequences. Moreover, 35
that insight is leading to the identification of specific areas of the brain that link feelings
and behavior to reward circuits. In the case of addictive drugs, we know that areas of
the brain involved in memory and learning and with the most ancient part of our brain,
the emotional brain, are the most interesting.

7 The new concept of addiction is in sharp contrast to the conventional, frustrating, and 40
some would say cynical view that everything causes addiction. Ask 10 Americans what
addiction is and what causes it and you might get at least 10 answers. Some will insist
addiction is a failure of morality or a spiritual weakness, a sin and a crime by people
who won't take responsibility for their behavior. If addicts want to self-destruct, let them.
It's their fault; they choose to abuse. 45

© JURONG JUNIOR COLLEGE 2010 8806/02/INSERT/PRELIM/10


4
8 For the teetotaler and politicians, it's a self-control problem; for sociologists, poverty; for
educators, ignorance. Ask some psychiatrists or psychologists and you're told
that personality traits, temperament, and "character" are at the root of addictive
"personalities." Social-learning and cognitive-behavior theorists will tell you it's a case of
conditioned response and intended or unintended reinforcement of inappropriate 50
behaviors. The biologically oriented will say it's all in the genes and heredity;
anthropologists that it's culturally determined. And Dan Quayle will blame it on the
breakdown of family values.

9 The most popular "theory," however, is that addictive behaviors are diseases. In this
view, an addict, like a cancer patient or a diabetic, either has it or does not have it. 55
Popularized by Alcoholics Anonymous, the disease theory holds that addictions are
irreversible, constitutional, and altogether abnormal and that the only appropriate
treatment is total avoidance of the alcohol or other substance, lifelong abstinence, and
constant vigilance.

10 The problem with all of these theories and models is that they lead to control measures 60
doomed to failure by mixing up the process of addiction with its impact. Worse, from the
scientific standpoint, they don't hold up to the tests of observation, time, and consistent
utility. They don't explain much and they don't account for a lot.

11 While all addictions display common properties, the proportions of those factors vary
widely. And certainly not all addictions have the same effect on the quality of our lives 65
or capacity to be dangerous. Everyday bad habits, compulsions, dependencies, and
cravings have something in common with heroin and cocaine addiction, in terms of their
mechanisms and triggers. But what about people who are Type A personalities; who
eat chocolate every day; who, like Microsoft's Bill Gates, focus almost pathologically on
work; who feel compelled to expose themselves in public, seek thrills like racecar 70
driving and fire fighting, or obsess constantly over hand washing, hair twirling, or
playing video games. They have — from the standpoint of what their behavior actually
means to themselves and others — very little in common with heroin and crack addicts.

12 Or consider two of the more fascinating candidates for addiction—sex and love. In the
case of love, the reactions involve chemicals such as endorphins, a naturally 75
occurring hormone linked to male and female bonding. After a while, though, this effect
diminishes as the brain's receptor sites for these chemicals become overloaded and
thus desensitized. Tolerance occurs; attachment wanes and sets up the mind for
separation, so that the "addicted" man or woman is ready to pursue the high elsewhere.
In this scenario, divorce or adultery becomes the equivalent of drug-seeking behavior, 80
addicts craving for the high. The fact that most people stay married is "a triumph of
culture over nature," much the way, perhaps, non-addiction is.

13 If addictions have characteristics in common, so do addicts, the experts say. They have
particular vulnerabilities or susceptibilities, opportunity to have contact with the
substance or activity that will addict them, and a risk of relapse no matter how 85
successfully they are treated. They tend to be risk takers and thrill seekers and expect
to have a positive reaction to their substance of abuse before they use it.

14 Addicts have distinct preferences for one substance over another and for how they use
the substance of abuse. They have problems with self-regulation and impulse control,
tend to use drugs as a substitute for coping strategies in dealing with both stress and 90
their everyday lives in general, and don't seek "escape" so much as a way to manage
their lives. Finally, addicts tend to have higher-than-normal capacity for such drugs.
Alcoholics, for example, often can drink friends "under the table" and appear somewhat
normal, even drive (not safely) on doses of alcohol that would put most people
to sleep or kill them. 95

© JURONG JUNIOR COLLEGE 2010 8806/02/INSERT/PRELIM/10


5
15 The biological, psychological, and social process by which addictions occur also have
common pathways, but with complicated loops and detours. All addictions appear now
to have roots in genetic susceptibilities and biological traits. But like all human and
animal behaviors, including eating, sleeping, and learning, addictive behavior takes a lot
of handling. The end product is a bit like Mozart's talent: If he'd never come in contact 100
with a piano or with music, it's unlikely he would have expressed his musical gifts.

© JURONG JUNIOR COLLEGE 2010 8806/02/INSERT/PRELIM/10


JURONG JUNIOR COLLEGE
JC 2 Preliminary Examination 2010

CANDIDATE NAME

GP TUTOR’S NAME CLASS

General Certificate of Education

GENERAL PAPER 8806/2


Paper 2 19 August 2010

Candidates answer on the Question Paper. 1 hour 30 minutes

Additional Materials: 1 Insert

READ THESE INSTRUCTIONS FIRST

Write your name, civics class and GP tutor’s name on all the work you hand in.
Write in dark blue or black pen on both sides of the paper.
Do not use paper clips, highlighters, glue or correction fluid.

Answer all questions.


The Insert contains the passages for comprehension.
Note that 15 marks out of 50 will be awarded for your use of language.

At the end of the examination, fasten all your work securely together.
The number of marks is given in brackets [ ] at the end of each question or part question.

EXAMINER’S USE

Content / 35 Comments:

Language / 15

Total / 50

This document consists of 7 printed pages and 1 blank page.

© JURONG JUNIOR COLLEGE 2010 8806/02/PRELIM/10 [Turn over


2

Read the passages in the Insert and then answer all the questions. Note that up to fifteen For
Examiner’s
marks will be given for the quality and accuracy of your use of English throughout this Paper. Use

NOTE: When a question asks for an answer IN YOUR OWN WORDS AS FAR AS POSSIBLE
and you select the appropriate material from the passage for your answer, you must
still use your own words to express it. Little credit can be given to answers which only
copy words or phrases from the passages.

1 Explain in your own words as far as possible how addiction theories and policies have
become ‘woefully outdated’ (line 1).

……………………………………………………………………………………………………........

………………………………………………………………………………………………………….

……………………………………………………………………………………………………. [2]

2 From paragraph 2, explain in your own words as far as possible Americans’ efforts in
dealing with addiction.

………………………………………………………………………………………………………….

………………………………………………………………………………………………………….

……………………………………………………………………………………………………….[2]

3 Why is the word “certainties” (line 17) in quotation marks?

…………………………………………………………………………………….……………………

………………………………………………………………………………………….………………

………………………………………………………………………………………….………………

……………………………………………………………………………………………………... [2]

4 Identify the similarity and difference between ‘people who are Type A personalities’ (line
68) and ‘heroin and crack addicts’ (line 73). Use your own words as far as possible.

…………………………………………………………………………………….……………………

………………………………………………………………………………………….………………

………………………………………………………………………………………….………………

……………………………………………………………………………………………………... [2]

© JURONG JUNIOR COLLEGE 2010 8806/02/PRELIM/10 [Turn over


3

5 Explain what the author means by staying married is "a triumph of culture over nature” For
Examiner’s
(lines 81 – 82)? Use

……………………………………………………………………………………………………........

………………………………………………………………………………………………………….

……………………………………………………………………………………………………. [2]

6 ‘…even drive (not safely) …’ (line 94)

Why does the author place parenthesis around ‘not safely’?

……………………………………………………………………………………………………........

………………………………………………………………………………………………………….

………………………………………………………………………………………….………………

……………………………………………………………………………………………………. [2]

7 What point is the author making in the last sentence (lines 100 – 101)?

…………………………………………………………………………………….……………………

………………………………………………………………………………………….………………

………………………………………………………………………………………….………………

……………………………………………………………………………………………………... [2]

© JURONG JUNIOR COLLEGE 2010 8806/02/PRELIM/10 [Turn over


4

8 Using material from paragraphs 5 to 9, summarise how the scientific discoveries about the For
Examiner’s
nature of addiction differ from other existing theories. Write your answer in no more than Use
120 words, not counting the opening words which are printed below. Use your own words
as far as possible.

One of the proposed theories……………………………………………………………………….

…………………………………………………………………………………………………………

……………………………………………………………………………………………………….…

………………………………………………………………………………….………………………

……………………………………………………………………………………….…………………

…………………………………………………………………………………………….……………

………………………………………………………………………………………………….………

……………………………………………………………………………………………………….…

……………….…………………………………………………………………………………………

…………………….……………………………………………………………………………………

………………………….………………………………………………………………………………

……………………………….…………………………………………………………………………

…………………………………….……………………………………………………………………

………………………………………….………………………………………………………………

……………………………………………….…………………………………………………………

…………………………………………………….……………………………………………………

………………………………………………………….………………………………………………

……………………………………………………………………………….…………………………

…………………………………………………………………………………….……………………

………………………………………………………………………………………….………………

……………………………………………………………………………………………………… [8]

© JURONG JUNIOR COLLEGE 2010 8806/02/PRELIM/10 [Turn over


5

9 Give the meaning of the following words as they are used in the passage. Use one word or For
Examiner’s
a short phrase. Use

(a) bewildering (line 12) ……………...…………………………………...………………....................

(b) hardcore (line 16) ………………………………………………………………………………........

(c) conjecture (line 20) …………………………………………………………………………………..

(d) account (line 63) …………………………………………………...…………………………..........

(e) compulsions (line 66) ……………………………………………………………………………......

[5]

© JURONG JUNIOR COLLEGE 2010 8806/02/PRELIM/10 [Turn over


6

10 Joann Rodgers argues strongly that the theories and models of addiction ‘lead to control For
Examiner’s
measures doomed to failure’ in modern society. How far do you agree with her views on Use
addiction and its control measures?

Illustrate your answer by referring to the ways in which you and your society regard
addiction and develop control measures for it.

……………………………………………………………….…………………………………………

…………………………………………………………………….……………………………………

………………………………………………………………………….………………………………

……………………………………………………………………………….…………………………

…………………………………………………………………………………….……………………

………………………………………………………………………………………….………………

……………………………………………………………………………………………….…………

…………………………………………………………………………………………………….……

…………………………………………………………………………….……………………………

………………………………………………………………………………….………………………

……………………………………………………………………………………….…………………

…………………………………………………………………………………………….……………

………………………………………………………………………………………………….………

………………………………………………………………………….………………………………

………………………………………………………………………….………………………………

……………………………………………………………………………….…………………………

………………………………………………………………………….………………………………

……………………………………………………………………………….…………………………

…………………………………………………………………………………….……………………

……………………………………………………………………………….…………………………

…………………………………………………………………………………….……………………

…………………………………………………………………………………….……………………

……………………………………………………………………………….…………………………

…………………………………………………………………………….……………………………

………………………………………………………………………….………………………………
© JURONG JUNIOR COLLEGE 2010 8806/02/PRELIM/10 [Turn over
7
For
Examiner’s
……………………………………………………………………………….………………………… Use

…………………………………………………………………………….……………………………

………………………………………………………………………………….………………………

……………………………………………………………………………………….…………………

…………………………………………………………………………………….……………………

………………………………………………………………………………………….………………

…………………………………………………………………………………….……………………

…….……………………………………………………………………………………………………

………….………………………………………………………………………………………………

……………….…………………………………………………………………………………………

…………………….……………………………………………………………………………………

………………………….………………………………………………………………………………

……………………………….…………………………………………………………………………

…………………………………….……………………………………………………………………

………………………………………….………………………………………………………………

……………………………………………….…………………………………………………………

…………………………………………………….……………………………………………………

………………………………………………………….………………………………………………

……………………………………………………………….…………………………………………

…………………………………………………………………….……………………………………

…………………………………………………….……………………………………………………

………………………………………………………….………………………………………………

……………………………………………………………….…………………………………………

…………………………………………………………………….……………………………………

…………………………………………………….……………………………………………………

………………………………………………………….………………………………………………

……………………………………………………………….…………………………………………

..……………………………………………………………………..………………………………[8]

© JURONG JUNIOR COLLEGE 2010 8806/02/PRELIM/10 [Turn over


8

BLANK PAGE

© JURONG JUNIOR COLLEGE 2010 8806/02/PRELIM/10 [Turn over


1

GP JC2 PRELIMINARY EXAMINATION 2010


ANSWER SCHEME

1 Explain in your own words as far as possible how addiction theories and policies have become
‗woefully outdated‘ (line 1). [2m]

Text Inference / Paraphrase


Our addiction theories and Through experiments / studies, (½m)
policies are woefully outdated.
Research shows that there are we have realized that our previous understanding / existing
no demon drugs. Nor are addicts theories about addiction are invalid / not applicable. (½m)
innately defective.
Instead, we now have a better / new understanding of the
Nature has supplied us all with causes of addiction. (½m)
the ability to become hooked—
and we all engage in addictive However, this would suggest that our current methods to
behaviors to some degree. address addiction are obsolete and inadequate to deal with the
problem effectively. (½m)

2 From paragraph 2, explain in your own words as far as possible Americans‘ efforts in dealing
with addiction. [2m]

Text Inference / Paraphrase


The War on Drugs alone is older It has been on-going/through many years (½m)
than the century.

In the early 1990s, the United Vast amount of resources / money utilized (½m) (Billions/millions
States spent $45 billion waging NOT ALLOWED)
it,

with no end in sight, Yet we cannot observe any visible result (½m) (Any other
interpretations that address how the problem is still not resolved
will be accepted, adjectives used only to describe attitudes
towards the situation will not be awarded the mark)
despite every kind of addiction
treatment from Even though we have tried all possible forms of
psychosurgery, psychoanalysis… cure/management/means of healing (½m)

3 Why is the word ―certainties‖ (line 17) in quotation marks? [2m]

Text Inference / Paraphrase


(i)
The author questions / doubts the suitability of the word (1m)

OR

The author does not intend the word in its original meaning. (1m)

(ii)
―Certainties‖ refer to indisputable facts. (½m) The fact that
experts are challenging these facts suggest that they are actually
debatable or arguable. (½m)

(i) & (ii) can be awarded separately

© JURONG JUNIOR COLLEGE 2010 8806/02/ANSWER SCHEME/PRELIM/10


2

4 Identify the similarity and difference between ‗people who are Type A personalities‘ (line 68) and
‗heroin and crack addicts‘ (line 73). Use your own words as far as possible. [2m]

Text Inference / Paraphrase


something in common … Similarity:
mechanisms and triggers  Means/methods and (½m)
 catalyst/stimulus/impetus (½m)
not all addictions have the same
effect on the quality of our lives OR
or capacity to be dangerous  Way / process by which one becomes addicted (1m)

what their behavior actually Difference:


means to themselves and  differing impacts on our lives (½m)
others … very little in common  potential to result in harm to others (½m)
 the significance of the addict‘s action (½m)

(Any 2 of the 3 points on difference = 1 mark)

5 Explain what the author means by staying married is "a triumph of culture over nature‖ (lines 81 – 82)?
[2m]

Text Inference / Paraphrase


a triumph (1m) of culture over The author means that staying married is an indication that our
nature (1) social rules / regulations / conventions are able / successful in
suppressing our tendencies to stray.

6 ‗…even drive (not safely) …‘ (line 94)

Why does the author place parenthesis around ‗not safely‘? [2m]

Text Inference / Paraphrase


Finally, addicts tend to have Note: Parentheses de-emphasise the content within, and are
higher-than-normal capacity for meant to supplement information, not take away emphasis
such drugs. Alcoholics, for from the main text.
example, often can drink friends
"under the table" and appear She wants to qualify / explain her opinion / view / trying to be
somewhat normal, even drive tongue-in-cheek / That is an aside (1m)
(not safely) on doses of alcohol
that would put most people that though many would claim that addicts are still able to
to sleep or kill them. function under alcoholic influence (½m), this ability will be
hampered somewhat / addicts are not immune to alcohol (½m).

7 What point is the author making in the last sentence (lines 100 – 101)? [2m]

Text Inference / Paraphrase


The end product is a bit like The author‘s point is that we all possess the propensity /
Mozart's talent: If he'd never inclination to be addicted (1m) and it only requires a trigger /
come in contact with a piano or catalyst / opportunity to draw it out (1m).
with music, it's unlikely he
would have expressed his OR
musical gifts.
Addictive behaviour can be positively harnessed/ channelled so
that an obsession can be turned into talent (1m), like how Mozart
might initially have been addicted to piano or music. But as a
result of this addiction, he ended up with a great flair for music
(1m).

© JURONG JUNIOR COLLEGE 2010 8806/02/ANSWER SCHEME/PRELIM/10


3

8 Using material from paragraphs 5 to 9, summarise how the scientific discoveries about the nature of
addiction differ from other existing theories. Write your answer in no more than 120 words, not
counting the opening words which are printed below. Use your own words as far as possible. [8m]

One of the proposed theories…

From the passage Answer Mark


A Some will insist addiction is a failure involves people who are adamant that such 1
of morality or a spiritual weakness, patterns reflects a loss of values (½m) or
feebleness in our soul / religion (½m),
B A sin and a crime by people who won't take And is a deliberate moral violation (½m) of the 1
responsibility for their behavior. principle of accountability for one‘s actions.
(½m)
C For the teetotaler and politicians, it's a self- For government officials and those who Any 2
control problem; or sociologists, poverty; for abstain from liquor, addiction is a case of self- of 3
educators, ignorance. discipline; scholars of human society blame points
indigence; teachers, the lack of knowledge. = 1m
D Ask some psychiatrists or psychologists and Psychiatrists and psychologists attribute such 1
you're told that personality traits, obsessive behaviour to the emotional and
temperament, and "character" are at the social qualities of an individual.
root of addictive "personalities."
E Social-learning and cognitive-behavior Other researchers may just put it down as a 1
theorists will tell you it's a case of conditioned habit, whether deliberate or not, reaffirmed by
response and intended or unintended the outcome of our actions.
reinforcement of inappropriate behaviors.
F The biologically oriented will say it's all in the Some theories explain addiction as part of a 1
genes and heredity; anthropologists that it's genetic code of human behaviour while
culturally determined. anthropologists propose that it is dependent on
the environment we live in. Students should
attempt to change form of word for ‘genes’.
G And Dan Quayle will blame it on the And another theory may point a finger at the 1
breakdown of family values. erosion of kinship ties.
H The most popular "theory," however, is that The theory that has the widest acceptance is 1
addictive behaviors are diseases. that they are medical
conditions/ailments/illnesses. (DO NOT
ACCEPT bacteria, viruses, germs, cancers.)
I the disease theory holds that addictions are that are inherent, deviant and cannot be 1
irreversible, constitutional, and altogether treated. The feasible/suitable solution is to
abnormal and that the only appropriate abstain from objects of gratification and to be
treatment is total avoidance of the alcohol or alert for possible symptoms.
other substance, lifelong abstinence, and
constant vigilance.
J Scientists have learned that every animal, Scientists however / in contrast have 1
from the ancient hagfish to reptiles, rodents, discovered that all animals have similar
and humans, share the same basic pleasure fundamental cognitive sense of gratification. It
and "reward" circuits in the brain, circuits is triggered when we are hooked on something
that all turn on when in contact with or engaged in enjoyable behaviour/
addictive substances or during pleasurable indulgences.
acts such as eating.
K One conclusion from this evidence is that This shows that such obsessive activity 1
addictive behaviors are normal, a natural part patterns are a constituent / component /
of our "wiring." element of our genetic make-up/ biological
blueprint.
L If they weren't, or if they were rare, nature Otherwise, they would have been eradicated/ 1
would not have let the capacity to be removed/ purged from within us.

© JURONG JUNIOR COLLEGE 2010 8806/02/ANSWER SCHEME/PRELIM/10


4

addicted evolve, survive, and stick around OR


in every living creature. These tendencies would have died out through
natural selection/Darwinian processes.
OR
These tendencies would have removed via
adaptation.
M Everyone engages in addictive behaviors to Humans indulge in such obsessive patterns as 1
some extent because such things as eating, they reaffirm what we do and are crucial to our
drinking, and sex are essential to survival existence.
and highly reinforcing.
N We get immediate gratification from them They also give us instant satisfaction which 1
and find them very hard to give up. makes it even more difficult to renounce
O The inescapable fact is that nature gave us the because of the pleasure we recognise from 1
ability to become hooked because the brain our cognitive mechanism.
has evolved a reward system, just as it has a
pain system.
P What we now call ―addictions,‖ are cases of a Scientific discoveries show that addiction can 1
good and useful phenomenon taken be favourable and advantageous. However,
hostage, with terrible social and medical when taken to the extreme, this would have
consequences. severely impact our health and community.
Q Moreover, that insight is leading to the This demonstrates that there may be a 1
identification of specific areas of the brain correlation between our emotions and actions
that link feelings and behavior to reward and what we get in return.
R The new concept of addiction is in sharp The novel idea of obsessive behavior patterns 1
contrast to the conventional, frustrating, is clearly distinctive from the commonly
and some would say cynical view that accepted, dissatisfying and even pessimistic
everything causes addiction. perspective that anything can result in
addiction.

NOTE: Markers are to use their discretion when marking. However, these are some guidelines
 Coherence is essential. A word for word substitution is not encouraged.
 Keywords have to be accurately paraphrased before the full mark is given.
 If the meaning of the point is not in doubt, candidate can be awarded full marks.
 A maximum of 5 marks will be awarded for answers which only focus on 1 aspect of the question
requirement.
 For questions with compounded mark allocation, if students manage a partial, paraphrased
interpretation of the points, they may be awarded ½ marks for their answers. ALL LIFTED
ANSWERS WILL BE GIVEN O MARK.
 Mark to reward.

© JURONG JUNIOR COLLEGE 2010 8806/02/ANSWER SCHEME/PRELIM/10


5

9 Give the meaning of the following words as they are used in the passage. Use one word or a short
phrase. [5m]

1 mark ½ mark 0 mark


(a) bewildering Baffling, confounding, confusing, Astounding,
(line 12) inexplicable, puzzling, perplexing, astonishing, complex,
mind-boggling,
mystifying,
stupefying,
surprising, shocking
(b) hardcore Dogmatic, entrenched, fixed, long- Firm, strong Concrete, extreme,
(line 16) standing, resolute, rigid, steadfast,
uncompromising, unwavering,
unfaltering

(c) conjecture Guesswork, surmise, assumption, Imagination,


(line 20) presumption, speculation, inference
hypothesis, postulation, supposition,
(opinion/conclusion formed based on
incomplete information), theory
(d) account (line Answer for, give reasons for, To be responsible for
63) provide a rationale for, show
grounds for, justify

(e) compulsions Extreme/Irresistible/strong/uncontr Preoccupations, Drive, desires, ,


(line 66) ollable + urges, fixations, impulses, urges tendencies
obsessions (to behave in a certain
way)

© JURONG JUNIOR COLLEGE 2010 8806/02/ANSWER SCHEME/PRELIM/10


6

10 Joann Rodgers argues strongly that the theories and models of addiction ‗lead to control measures
doomed to failure’ in modern society. How far do you agree with her views on addiction and its control
measures?

Illustrate your answer by referring to the ways in which you and your society regard addiction and
develop control measures for it.

To pass, students need to discuss:

R1:
 Evaluation of the success or failure of the control measures
 Evaluation should be linked to the theories on addiction (causes and processes)

R2:
 Context of the discussion should revolve around you and your society
 Presence of personal opinion (demonstration of personal views regarding society‘s control measures
to treat addiction)

References from the passage Interpretation of References


1) (Para 1) Our addiction theories and policies ( Ref 1 & 2) Rodgers suggests that we have
are woefully outdated. Research shows that wrongly interpreted the addiction as an unnatural
there are no demon drugs. Nor are addicts phenomenon, resulting in us focusing on the
innately defective. Nature has supplied us all wrong factors in trying to address the issue.
with the ability to become hooked.

2) (Para 2) In the early 1990s, the United States


spent $45 billion waging it, with no end in sight,
despite every kind of addiction treatment from
psychosurgery, psychoanalysis, psychedelics,
and self-help to acupuncture, group
confrontation,
family therapy, hypnosis, meditation, education
and tough love.

4) (Para 4) They're concluding that things are far (Ref 4 & 5) Addiction is an intrinsic part of our
from hopeless. biology. What is now construed as addiction is
actually an extreme form of this intrinsic
(Para 5) One conclusion from this evidence is behaviour.
that addictive behaviors are normal, a natural
part of our "wiring." If they weren't, or if they
were rare, nature would not have let the capacity
to be addicted evolve, survive, and stick around
in every living creature.

5) (Para 6) What we now call "addictions," in this


sense, are cases of a good and useful
phenomenon taken hostage, with terrible social
and medical consequences.

© JURONG JUNIOR COLLEGE 2010 8806/02/ANSWER SCHEME/PRELIM/10


7

6) (Para 7) The new concept of addiction is in (Ref 6 – 8) The latest understanding of addiction
sharp contrast to the conventional, frustrating, contradicts the conventional reasons such as
and some would say cynical view that everything trained responses, genetics, culturally
causes addiction. determination, economic deprivation, the lack of
education, personality defects and even social
7) (Para 8) For the teetotaler and politicians, it's disintegration. The dominant understanding,
a self-control problem; for sociologists, poverty; however, is that addiction is an inherent medical
for educators, ignorance. Ask some psychiatrists condition.
or psychologists and you're told that
personality traits, temperament, and "character"
are at the root of addictive "personalities."
Social-learning and cognitive-behavior theorists
will tell you it's a case of conditioned response
and intended or unintended reinforcement of
inappropriate behaviors. The biologically
oriented will say it's all in the genes and heredity;
anthropologists that it's culturally determined.
And Dan Quayle will blame it on the breakdown
of family values.

8) (Para 9) The most popular "theory," however,


is that addictive behaviors are diseases. In this
view, an addict, like a cancer patient or a
diabetic, either has it or does not have it.
9) (Para 10) The problem with all of these (Ref 9) Rodgers argues that the abovementioned
theories and models is that they lead to control theories will result in ineffective outcomes in our
measures doomed to failure by mixing up the battle against addiction as they mistake the
process of addiction with its impact. Worse, from symptoms of addiction for its root causes.
the scientific standpoint, they don't hold up to the They also lack reliable empirical evidence to back
tests of observation, time, and consistent utility. it up and hence don‘t provide any useful insight
They don't explain much and they don't account and solutions.
for a lot.
10) (Para 11) While all addictions display (Ref 10) Addictions have differing degrees of
common properties, the proportions of those severity despite some of their shared traits.
factors vary widely. And certainly not all
addictions have the same effect on the quality of
our lives or capacity to be dangerous.
11) (Para 13) They have particular vulnerabilities (Ref 11) What someone will be addicted to is
or susceptibilities, opportunity to have contact defined by their unique weaknesses and the
with the substance or activity that will addict preconceived notions of their environmental
them, and a risk of relapse no matter how exposure to the addictive substance.
successfully they are treated. They tend to be
risk takers and thrill seekers and expect to have
a positive reaction to their substance of abuse
before they use it.
12) (Para 15) All addictions appear now to have (Ref 12) Addiction is an intrinsic part of our
roots in genetic susceptibilities and biological biology. However, it needs careful management to
traits. But like all human and animal behaviors, prevent it from becoming a socially debilitating
including eating, sleeping, and learning, condition.
addictive behavior takes a lot of handling.

© JURONG JUNIOR COLLEGE 2010 8806/02/ANSWER SCHEME/PRELIM/10


8
Answers In Agreement with Rodgers

A) Sample Argument

Rodgers suggests that we have wrongly interpreted addiction as an unnatural phenomenon, resulting in us
focusing on the wrong factors in trying to address the issue. I agree that the theories we have for addiction
are ‗woefully outdated‘ (Para 1), and a revolutionary approach to the problem has to be adopted in tackling
the issue. This can begin with the acknowledgement that ‗addictive behaviors are normal (and) a natural
part of our "wiring‖ (Para 5) and require careful management to prevent them from becoming socially
debilitating conditions.

In my society, I feel that we have adopted the wrong measures to deal with some of the addictions
observed here. One instance would be the addiction to gambling, which, while having been present in our
society in the form of the various national lotteries, has been brought into greater prominence with the
establishment of the casinos as a permanent fixture in our tourism industry. The government‘s simplistic
belief that gambling is a problem that can be dealt with using basic social safeguards such as casino
entrance fees and exclusions disregards the view that these merely deal with the symptoms and not with
the triggers of gambling, which would be the presence/existence of the casinos themselves. I share
Rodgers view that the higher the exposure to the activity, the greater the chance of addiction. The
presence of the casinos merely multiplies the average Singaporean‘s ‗opportunit[ies] to have contact with
the substance or activity that will addict‘ (Para 13). I do acknowledge, however, that the Singapore
government faces a great dilemma in this case. Do we give up the chance to advance our economy, or
allow the issue of gambling to ‗flourish‘?

B) Other Possible Approaches in Agreement with Rodgers

Candidates can:

 Explore Singapore-specific cultural and social perspectives regarding addictive behaviour, with
reference to our still-conservative society. This can be linked with new research which suggests
that addictive behaviour is not confined to that which has social consequences. (Reference from
Para 11)

 Address both ‗minor‘ and less impactful addictions besides those that are recognised as ‗severe
and dangerous‘ and examine the ways in which Singapore society perpetuates these addictions
rather than develop control measures to them.

 Comment on the specific implications of our symptomatic approaches to dealing with the range of
common addictions present in Singapore, and suggest more effective measures for tackling these
issues. Candidates could also consider the long-term impacts of continuing our existing
approaches on society.

 Pick up on Rodgers‘ assessment in Paragraph 15 that addictive behaviour can be positively


harnessed/channelled so that an obsession can be turned into talent/gift, like how Mozart might
initially have been addicted to piano or music. But as a result of this addiction, he ended up with
musical gifts.

© JURONG JUNIOR COLLEGE 2010 8806/02/ANSWER SCHEME/PRELIM/10


9
Answers In Disagreement with Rodgers

A) Sample Argument

While Rodgers explains at length what in her opinion is truly ―addiction‖ which calls for ―a lot of handling‖
different from the ―control measures‖ currently in place, I have my reservations.

Rodgers observes that current theories and models of addiction lead to control measures that are doomed
to failure. She goes to great lengths to criticize these models and explain why the control measures that
ensue will not guarantee success. What Rodgers subsequently does is however, to propose simply that
―addictive behavior takes a lot of handling‖. Her proposal is vague and offers no explicit instructions or
directions on how exactly addictive behavior can be addressed and resolved. Her observation that current
theories ―don‘t explain much and don‘t account for a lot‖ ironically applies to her own proposed solution to
addiction. As such, I find it hard to concur with her views on addiction and its control measures for the very
measures she criticizes have at least detailed and concrete means of resolving the issue of addiction,
some of which boast even track records of success. Overall, what Rodgers sets out to achieve ends up
being reminiscent of the same ―glib explanations‖ and ―glib solutions‖ that she is challenging.

B) Other Possible Approaches in Disagreement with Rodgers

Candidates can:

 Discuss how control measures are not outdated and ineffective as they are still relevant and work
for Singapore quite well.
 Explore how Singapore deals with certain types of addiction by removing all opportunities of
exposure completely (e.g. heavy penalties for drug trafficking, etc.).
 Preventive measures also prove to be quite effective in dealing with some types of addiction (like
gambling).
 Rodgers‘ definition of addiction is too broad, as it overlaps with many other functions & habits
necessary for survival. Necessities are unfairly likened to addictions (Reference from Para 5).
 Evaluate how it is an uphill task to tackle addictions at its root: the ‗biological‘ wiring and innate
quality of addiction. Understanding the root causes of addiction does not help us deal more
effectively with the myriad forms of addiction and the factors that perpetuate them (Reference from
Para 5).
 The only pragmatic way to deal with such issues would be to address and alleviate the varied and
endless manifestations and consequences of such conditions (Reference from Para 11).

© JURONG JUNIOR COLLEGE 2010 8806/02/ANSWER SCHEME/PRELIM/10


GP Tutor:

Candidate Name Class Admission No

MILLENNIA INSTITUTE

Preliminary Examination 2 2010


Pre-University 3
GENERAL PAPER 8806/01
Paper 1 3 September 2010
Friday 1 hour 30 minutes

QUESTION PAPER
Additional Materials: Writing Paper

READ THESE INSTRUCTIONS FIRST

Write your name, class, admission number and tutor’s name in the spaces provided at the top of
this page and on all the work you hand in.
Write in dark blue or black pen on both sides of the paper.
Do not use staples, paper clips, highlighters, glue or correction fluid.

Answer one question.


All questions in this paper carry equal marks.
At the end of the examination, fasten all your work securely together.

(Note that 20 marks out of 50 will be awarded for your use of language.)

For Examiner’s Use

Content /30

Language /20

Total /50

This question paper consists of 2 printed pages.


[Turn over
2

PAPER 1

Answer one question from this Paper.


Answers should be between 500 and 800 words in length.

1 Humour is becoming increasingly important in today’s world. How far do you agree
with this view?

2 ‘Singaporeans are intolerant of differences.’ To what extent do you agree with this
claim?

3 ‘We can no longer afford to use violence to resolve conflicts in today’s world.’ How far
do you agree with this claim?

4 How far does a State have the right to intervene in the decisions people make about
their lives?

5 Is a world without racial or religious conflict an impossible dream?

6 To what extent is there room for failure in your society?

7 A life of choice presents more problems than benefits. What do you think?

8 Is air travel really desirable?

9 Far too little attention is paid to the pleasures of reading in this electronic age.
Discuss.

10 Modern technology does not owe ecology an apology. To what extent is this true?

11 The leaders of today are all talk and no action. How far do you agree?

12 Can lifelong marriages still be sustained in your society today?

©2010 Millennia Institute 8806/01/PRELIM2/PU3


GP Tutor:

Candidate Name Class Admission No

MILLENNIA INSTITUTE

Preliminary Examination 2 2010


Pre-University 3
GENERAL PAPER 8806/02
Paper 2 3 September 2010
Friday 1 hour 30 minutes

INSERT

READ THESE INSTRUCTIONS FIRST

Write your name, class, admission number and tutor’s name in the spaces provided at
the top of this page.

This insert contains the passage for Paper 2.

This insert consists of 4 printed pages.


[Turn over
©2010 Millennia Institute 8806/02/PU3/Prelim2
2

Passage 1: C.A Mace writes…

1 “No man ever acts from a single motive”, wrote the author Dostoevsky, and this
simple truth should never be far from the minds of those who interest
themselves in the problem of incentives; for the failure to appreciate the
complexity of human motivation is, perhaps, the outstanding weakness of
every system of thought on this. The mistake of the world-wise, who like to say 5
that ‘the only effective incentive is the pay packet’, is not so much that they
overlook other sources of motivation as that they fail to observe the complexity
of this motive itself.

2 We all love money, but we love it most for what it enables us to do. To some it
may mean chiefly pleasure and amusement, to others it means greater 10
security, or a better chance for one’s children, or greater opportunity for
promoting a project for reforming the world. The pay packet theory is not a bad
one to start from, but it is apt to stifle thought precisely at the point where
thought should begin. We begin to understand men’s motives only when we
know how they spend their money and how they spend their time. He performs 15
these tasks from what he is apt to describe as a sense of love, or a sense of
responsibility for getting something done. Forces such as these must surely be
assigned some function in a balanced system of incentives.

3 For centuries, many believed the strongest incentive for anyone to do


something would be love. The appeal to Mankind’s love for one another seems 20
strong because it appeals to the fundamental core of human emotions. It can
be said that love is THE fundamental emotion of any rational human being (and
even for those who lack their complete mental faculties). Hence, by appealing
to this fundamental emotion, it can be said that a person has no choice but to
respond in a positive fashion because a person wants to be loved and to give 25
love. Nothing, I would argue, motivates a person more to go the extra mile than
tugging at their heartstrings. Indeed, it is because motivation through love
targets the basic, innate nature of Man, that many have considered it to be the
strongest, if not only, viable form of motivation. Today, many management
gurus have recommended modern day leaders to motivate their followers by 30
showing more care and concern for them.

4 If love is for someone you know, then what about complete strangers? Altruism
is the answer as it causes a person to work for a cause which benefits total
strangers. Altruistic behaviour is an unknown force, whose power to motivate
people to do extraordinary deeds is only second to love. One cannot explain 35
why a person would do something to benefit another for no pay nor reward,
except to give thanks, and to accept whatever benefits with gratitude. There
are, in human nature, forces that impel a man to perform tasks that he finds
unpleasant and which he is under no obligation or compulsion to perform. If he
does perform these tasks without any form of reward, then it would be apt to 40
describe this motivation as done out of obligation to a greater good. The daily
sacrifices of policemen or firemen are beyond price. Altruism is indeed a
mysterious force that motivates people to do great things for next to a pittance.

5 If all else fails, the last motivation for a person would be the sense of duty. The
need to achieve or complete something assigned to you as your responsibility 45
is a primal force of human nature. However, the fact that anyone can be

©2010 Millennia Institute 8806/02/PU3/Prelim2


3
surprised at this behaviour is a reflection of the traditional popular psychology
in terms of which we and they have been taught to think about such things. It is
the job that ‘just has to be done’, and we derive satisfaction, not so much
perhaps from doing it but from having got it done. Common experience 50
suggests that almost any man will do almost any job of work if the work
contributes to his self-respect and to a measure of recognition from his peers.

(adapted from Satisfaction in Work, Professor C.A Mace)

©2010 Millennia Institute 8806/02/PU3/Prelim2


4

Passage 2: Samuel Gregg writes…


1 If there was a phrase that aptly sums up the spirit of the age that we are living
in, it would be ‘Show me the money!’ from the movie Jerry Maguire. Let us face
it – we live for money, our actions lubricated by money, and (with not much
exaggeration) even die for money. Man does not, for one minute, do things
willingly because this is not reality. Our own experience shows us that Man, if 5
given a choice, never assumes responsibility and accountability without some
form of gain. The early social architects of modern society, after World War II,
sincerely believed in the “innate” altruistic nature of Man. They started many
social experiments – the Peace Corps being the figurehead, in the name of
promoting goodwill and self-sacrifice for the less fortunate. If only these 10
experiments had worked…

2 We live in the age of a Commercial Society. Commercial Society rejects this


vision of Man as well as the means to promote the utopian view of society. The
understanding of humans that pervades Commercial Society is one of realism.
It does not assume that human beings can be motivated by any other means 15
except that of financial gain. Many of the Commercial Society’s legal and
economic structures are thus predicated on a decidedly non-altruistic
understanding of humans and their world. We assume that there will always be
some people who will unreasonably decide to find some reasons not to fulfill
their obligations. Hence, the system that characterises the modern Commercial 20
Society from previous societies in motivating people, is that, the Commercial
Society requires exchanges into which people enter in the pursuit of their own
interest.

3 Commercial Society does not therefore attempt to interfere or eliminate human


frailties. Instead, it holds onto the fact that (in fact, celebrates) there is nothing 25
unnatural about self-interest. To try to interfere or eliminate it would be to go
against the grain of a universal and powerful human instinct. It is not an
abstract principle grounded without a standing in reality. Those who followed
Commercial Society’s early developments noted its ability to align human
weakness and self-regard with society’s overall progression towards a more 30
prosperous state of well-being. Adam Smith’s reference to the “invisible hand”
perplexes some, but it is simply a metaphor for the idea in which through
allowing people to pursue their self-interest, unintended but beneficial social
consequences for others will follow. As individuals pursue profit, unintentionally
or intentionally, it adds to the sum total of societal well-being, plus the fact that 35
people are motivated to get things done.

4 Granted, the pursuit of self-interest is the most controversial moral-cultural


conundrum facing societies today. However, the “ruthlessness” and “depravity”,
often portrayed by critics of Commercial Society, rarely account for the fact that
individuals are able to self-reflect and be critical of their behaviour. And, if I 40
might suggest, the criticisms of Commercial Societies are not unique to
Commercial Societies only. Perhaps, we should start recognising people for
who they are, not what they should be.

(adapted from The Commercial Society, Samuel Gregg)

©2010 Millennia Institute 8806/02/PU3/Prelim2


5

BLANK PAGE

©2010 Millennia Institute 8806/02/PU3/Prelim2


GP Tutor:

Candidate Name Class Admission No

MILLENNIA INSTITUTE

Preliminary Examination 2 2010


Pre-University 3
GENERAL PAPER 8806/02
Paper 2 3 September 2010
Friday 1 hour 30 minutes

ANSWER BOOKLET
Additional Materials: 1 insert

READ THESE INSTRUCTIONS FIRST

Write your name, class, admission number and tutor’s name in the spaces provided
at the top of this page, and on all the work you hand in.
Write in dark blue or black pen.
Do not use staples, paper clips, highlighters, glue or correction fluid.

Answer all questions.


The Insert contains the passage for comprehension.
(Note that 15 marks out of 50 will be awarded for your use of language.)

At the end of the examination, fasten all your work securely together.
The number of marks is given in brackets [ ] at the end of each question or part
question.

For Examiner’s Use

Content /35

Language /15

Total /50

This question paper consists of 7 printed pages and 1 Insert.


[Turn over

©2010 Millennia Institute 8806/02/PU3/Prelim2


2

Read the passage in the Insert and then answer all the questions which follow For
Examiner’s
below. Note that up to fifteen marks will be given for the quality and accuracy of Use
your use of English throughout this paper.

NOTE: When a question asks for an answer IN YOUR OWN WORDS AS FAR
AS POSSIBLE and you select the appropriate material from the passage for your
answer, you must still use your own words to express it. Little credit can be given
to answers which only copy words or phrases from the passage.

From Passage 1

1 From paragraph 1, what according to the author is the ‘mistake of the world-
wise’? Answer in your own words as far as possible. (line 5)

…………………………………………………………………………………………

………………………………………………………………………………………[1]

2 Using your own words as far as possible, give any three reasons from
paragraph 2 as to why ‘we all love money’ (line 9), and support your answers
with examples. Label your answers 1, 2 and 3.

…………………………………………………………………………………………

…………………………………………………………………………………………

…………………………………………………………………………………………

…………………………………………………………………………………………

…………………………………………………………………………………………

………………………………………………………………………………………[3]

©2010 Millennia Institute 8806/02/PU3/Prelim 2


3

3 Using material from paragraphs 3 to 5 of Passage 1 (lines 19-52), summarise For


Examiner’s
the author’s reasons for people’s motivation. Use

Write your summary in no more than 120 words, not counting the opening
words which are printed below. Use your own words as far possible.

People are motivated to help others out because……………………………….

…………………………………………………………………………………………

………..………………………………………………………………………………..

………………..……………..…………………………………………………………

…………………………………………………………………………………………

………………………………..…..……………………………………………………

……………………………………………..…………………………………………..

……………………………...……………………………………………………........

..……………………………………………………………………..………………...

…………………………………………………………………………………………

…………………..………………………………………………………………....….

.……………………………...………………………………………………………...

.………………………………………..……………………………………………….

………………………………………………..………………………………………..

………………………………………………………..………………………………..

………………………………………………………………..………………………..

………………………………………………………………………..………………..

.………………………………………………………………………………...………

..………………………………………………………………………………………..

…….…….……………………………………………………………………………..

…….…………….………………………………………………………………… [8]

©2010 Millennia Institute 8806/02/PU3/Prelim 2


4

From Passage 2 For


Examiner’s
Use
4 From paragraph 1, why does the author use parentheses? What does he
intend to convey to us? (lines 3 – 4)?

………………………………………………………………………………….........

........................................................................................................................

..................................................................................................................................

.............................................................................................................[2]

5 What is the author’s attitude towards the ‘early social architects’ (line 7) and
why did he have such an attitude?

.........................................................................................................................

.........................................................................................................................

.........................................................................................................................

.....................................................................................................................[2]

6 From paragraph 2, why does the author describe the ‘understanding of


humans’ as one of ‘realism’ (line14)? Answer in your own words as far as
possible.

...........................................................................................................................

...........................................................................................................................

...........................................................................................................................

.......................................................................................................................[2]

7 From paragraph 3, why does the author say we should not ‘interfere or
eliminate human frailties’ (lines 24 -25)? Answer in your own words as far
as possible.

…………………………………………………………..…………………………….

…………………………………………………………………………………………

………………………………………………………………………………………..

……………………………………………………………………………………...[2]

©2010 Millennia Institute 8806/02/PU3/Prelim 2


5

For
Examiner’s
8 What is the irony in ‘allowing people to pursue their self interest’ (line 33)? Use

…………………………………………………………..…………………………….

…………………………………………………………………………………………

………………………………………………………………………………………..

……………………………………………………………………………………...[2]

9 Give the meaning of the following words as they are used in the passages.
Write your answer in one word or a short phrase.

(a) stifle (Passage 1 line 13) …………..……………………………………..

(b) innate (Passage 1 line 28) ………………………………………………….

(c) figurehead (Passage 2 line 9) …………………………………….. ……..

(d) pervades (Passage 2 line 14) ……………………………………............

(e) predicated (Passage 2 line 17) ……………………………………………

[5]

©2010 Millennia Institute 8806/02/PU3/Prelim 2


6

For
Examiner’s
10 C.A. Mace suggests that "almost any man will do almost any job of work if Use
the work contributes to his self-respect and to a measure of recognition from
his peers," while Gregg asserts that society should "start recognising people
for who they are, not what they should be" when it comes to motivating
people.

With which of the two authors are you more in sympathy with? Whose views
are the most applicable to your society? Explain your answers with reference
to the authors' views.

…..……………………………………………………………………………………

…………..……………………………………………………………………………

…………………..……………………………………………………………………

…………………………..……………………………………………………………

………………………………….………………………………………………….…

…………………………………………..……………………………………………

…………………………………………………..……………………………………

……………………………………………………….....………………………….…

…………………………………………………………………..……………………

…………………………………………………………………………..……………

…………………………………………………………………………………..……

………………………………………………………………………………………..

…..……………………………………………………………………………………

…………..……………………………………………………………………………

…………………..……………………………………………………………………

…………………………..……………………………………………………………

…………………………………..……………………………………………………

…………………………………………..……………………………………………

…………………………………………………..……………………………………

…………………………………………………………..……………………………

©2010 Millennia Institute 8806/02/PU3/Prelim 2


7

…………………………………………………………………..…………………… For
Examiner’s
Use
…………………………………………………………………………..……………

………………………………………………………………………………………...

…………………………………………………………………………………………

…………………………………………………………………………………….......

...........................................................................................................................

...........................................................................................................................

...........................................................................................................................

...........................................................................................................................

...........................................................................................................................

...........................................................................................................................

...........................................................................................................................

...........................................................................................................……….…

.………………………………………………………………………….....................

...........................................................................................................................

...........................................................................................................................

..................................................................................................………………...

……………………………………………………………………….........................

...........................................................................................................................

...........................................................................................................................

..............................................................................................……………………

………………………………………………………………….................................

...........................................................................................................................

.....................................................................................………………………….

…………………………………………………………………………………….. [8]

©2010 Millennia Institute 8806/02/PU3/Prelim 2


8

BLANK PAGE

©2010 Millennia Institute 8806/02/PU3/Prelim 2


MILLENNIA INSTITUTE
GENERAL PAPER
2010 PRE-UNIVERSITY 3
PRELIMINARY EXAMINATION 2
PAPER 2 ANSWER SCHEME

1. From Paragraph 1, what according to the author is the „mistake of the world-wise‟? (line 5)
Answer in your own words as far as possible. [1]

From the passage Suggested paraphrase


…who like to say that ‘the only effective Do not realise that using money as a means to
incentive is the pay packet’, …is that they fail encourage people (1/2) to do something is not
to observe the complexity of the motive itself. as simple as it seems. (1/2)

2. Using your own words as far as possible, give any three reasons from Paragraph 2 for why
„we all love money‟, and support your answers with examples. Label your answers 1, 2 and 3.
(line 9) [3]

1st part of the answer


From the passage Suggested paraphrase + suggested examplle
To some it may mean chiefly pleasure and It allows us to spend on entertainment, e.g.
amusement spending on electronic goods and services,
which cost money.

To others it means greater security It allows us to pay for goods and services
aimed at protecting ourselves and keeping us
safe, e.g. buying of house alarm systems.

(students can also explain this in terms of using


money to provide for their financial security, e.g.
buying insurance, etc)
Or a better chance for one’s children It betters our children’s education or job
prospects, because with money, parents can
afford enrichment classes or pay for an overseas
education.

Or greater opportunity for promoting a project It provides one with the resources to be an
for reforming the world activist and initiate change in the world, e.g.
to tackle global warming or alleviate poverty.

1
3. Summary Question
Using materials from paragraphs 3 to 5 of Passage 1 (lines 19 - 52), summarise the author‟s reasons for people‟s
motivations. Write your summary in no more than 120 words, not counting the opening words which are printed
below. Use your own words as far possible.
People are motivated to help others out because…

Lifted Paraphrased
Paragraph 3:
The appeal to Mankind’s love for one another 1. …care and concern (1/2)
seems strong because it appeals to the
fundamental core of human emotions. 2. forms the basis of all emotions/ is
intrinsic/innate. (1/2).
It can be said that love is THE fundamental
emotion of any rational human being (and 3. This applies to those who can reason (1/2)
even for those who lack their complete
mental faculties). 4. as well as those who cannot (1/2)

Hence, by appealing to this fundamental 5. People want to have this affection (1/2)
emotion, it can be said that a person has no
choice but to respond in a positive fashion 6. and to share it (1/2).
because a person wants to be loved and to
give love.
Paragraph 4:
If love is for someone you know, then what 7. People are also motivated to help those they
about complete strangers? do not know (1/2)

One cannot explain why a person would do 8. without reimbursement (1/2),


something to benefit another for no pay nor
reward, except to give thanks, and to accept 9. but just to receive thanks / to be appreciated
whatever benefits with gratitude. (1/2). [can lift „thanks‟ and „gratitude‟]

There are, in human nature, forces that impel 10. They will do work that is unfavourable (1/2)
a man to perform tasks that he finds
unpleasant and which he is under no 11. even though it is not their responsibility (1/2)
obligation or compulsion to perform. …

If he does perform these tasks without any form 12. …due to their desire to benefit society (1/2).
of reward, then it would be apt to describe this
motivation as done out of obligation to a
greater good.
Paragraph 5:
The need to achieve or complete something 13. People will also help others because to finish a
assigned to you as your responsibility is a task
primal force of human nature.
14. is an inborn/ innate component (1/2)
It is the job that „just has to be done‟, and we
derive satisfaction, not so much perhaps from 15. We feel happy (1/2)
doing it but from having got it done.
16. not because of the process of completion (1/2)
Common experience suggests that almost any
man will do almost any job of work if the 17. but the completion itself (1/2).
work contributes to his self-respect and to a
measure of recognition from his peers. 18. He will also do it if others acknowledge the work
done (1/2),

19 . and if it raises one‟s self-esteem (1/2),


10 points, 19 phrases, 9.5 marks
2
4. From Paragraph 1, why did the author use parentheses? What does he intend to convey to
us? (lines 3-4) (2 marks)

From the passage Suggested answer


Let us face it – we live for money, our actions are The author uses parentheses to give us his
lubricated by money, and (with not much opinion/ to emphasise (1)
exaggeration) even die for money.
that there are people who will sacrifice their
lives for money. (1)

[if student just brings out the idea of


‘sacrifice’ without mentioning the sacrificing
of lives, award ½ mark.]

5. What was the author attitude towards the „early social architects‟ (line 7) and why did he have
such an attitude? [2]

From the passage Suggested answer


The early social architects of modern society The writer is critical/ disapproving/ skeptical/
…sincerely believed in the “innate” altruistic doubtful/ cynical. (1)
nature of Man.. they started many social
experiments… If only these experiments had He believes they are naïve/ unrealistic for
worked. thinking that people act out of goodwill and
self-sacrifice (1/2) because the reality is the
social experiments did not work. (1/2)

6. From paragraph 2, why does the author describe the „understanding humans‟ as on of the
„realism‟? (line 14-15) Answer in your own words as far as possible. [2]

From the passage Suggested answer


It does not assume that human beings can be This is because such an understanding is
motivated by any other means except that of based on a pragmatic approach/ it is based
financial gain… there will always be some on what people’s behaviour actually is (1),
people who will unreasonably decide to find
some reasons not to fulfill their obligations which is that people are motivated only by
monetary benefit (1) and there are people who
will actually avoid being responsible. (1)

3
7. Why does the author say we should not „interfere or eliminate human frailties‟ (line 24-25)?
Answer in your own words as far as possible. [2]
From the passage Suggested answer
…there is nothing unnatural about self There is nothing out of the ordinary (1/2)about
interest… to try to interfere or eliminate it would wanting to do things that benefit oneself.
be to go against the grain of a universal and This desire is strongly (1/2) inherent/
powerful human instinct. naturally occurring (1/2) in everyone/
humans.(1/2)

8. What is the irony in „allowing people to pursue their self interest‟ (line33 )? [2]

From the passage Suggested answer


Though allowing people to pursue their self- We expect that when people act only for their
interest, unintended but beneficial social own benefit (1/2), others will suffer (1/2)
consequences for others will follow. …it adds to However, the reality is that it contributes to
the sum total of societal well-being, plus the everyone being more well-off (1/2), and it
fact that people are motivated to get things increases people’s desire to get things done
done. (1/2).

[irony must be shown for any mark to be


awarded.]

9. Vocabulary

a. stifle – forcefully stop, repress; prevent (1/2), hinder (1/2), impede (1/2)
b. innate or primal- naturally occurring, Intrinsic, Inherent
c. figurehead – person in high position who has no real power
d. pervades – spreads widely
e. predicated – based on, built upon, built (1/2), based (1/2)

10. C.A. Mace suggests that "almost any man will do almost any job of work if the work
contributes to his self-respect and to a measure of recognition from his peers," while Gregg
asserts that society should "start recognising people for who they are, not what they should
be" when it comes to motivating people. With which of the two authors are you more in
sympathy? Whose views are the most applicable to your society? Explain your answers with
references to the authors' views. [8]

Suggested approach:

In order to come up with a reasonable response, students are encouraged to tackle R2 before
R1; it is more logical to suggest whose views are most applicable to Singapore before one can
decide who you are more in sympathy with.

Both authors offer their views of society that are directly opposite each other. However, both
are reasonable views in some sense. Mace‟s view of Man‟s motivation is one where we have
to recognise the underlying motivations of humans, apart from the pay packet, whereas Gregg
is certain that Man is motivated through money only. Mace‟s view of society is certainly more
idealistic that Gregg, but Gregg is closer to reality. In order to come up with an excellent
4
answer, students need to recognise that we should move closer to Mace‟s view, if we need to
build a more gracious society in Singapore. This has to be done without sacrificing the need
for progress in Singapore.

Suggested arguments:

Point Mace believes, in paragraph 3, that “the strongest incentive for anyone to do
something would be love”.

Elaboration For Mace, love is a fundamental emotion that everyone will have no choice but
respond to. For this reason, motivating people through love will be easier and
more effective.

Evaluation Certainly, Singapore has tried to adopt this thinking. The fundamental building
block for a stable nation is a stable family, and our leaders have tried to
encourage the building of a strong family by emphasising the familial bonds
that exist between family members. We have extended this by emphasising the
love of the nation, as the nation is made up of many family units. Hence, rather
than creating a more mercenary „love‟ of the family and nation via money, we
have tried to inculcate the love of family and nation via education and
campaigns.
The view to motivate people by love is certainly more long-lasting, than purely
through money. We certainly do not want to create a generation of people who
will only defend the country because of what they can get out of the country.
However, this view seems rather anachronistic with modern world view, as
young Singaporeans today are more individualistic and more mobile due to the
high levels of education. Thus, while we can appreciate Mace‟s view, it is
certainly too idealistic for modern day Singapore.

Example A recent study conducted by Institute of Public Policy Studies in 2009 showed
that, while 2/3 of Singaporeans remain loyal to the country, this sense of loyalty
is being threatened by the influx of foreign migrants, and that the majority will
not sacrifice anything for the well-being of the nation.

Link Back Hence, Mace‟s view is certainly good for the long-term viability of Singapore.
But, to purely depend on the inculcation of love is certainly not enough today.

Point Mace believes, in paragraph 5, that it is possible to motivate people through a


sense of duty.

Elaboration For Mace, he believes that “the need to achieve or complete something
assigned to you as your responsibility is a primal force of human nature.”

Evaluation While we can appreciate his view, Mace is definitely too idealistic to believe
that people today still possess the sense of duty. It would be definitely good for
Singapore, if people can do what is needed to be done because it has to be
done. However, the fact is that not many people will voluntarily do things out of
duty because Singaporeans seldom like to stand out from the norm. Worse
still, Singaporeans seem to only do things because there is a threat of
punishment or a promise of reward. Granted that there are people who do
perform out of duty, the fact is that these are the minority.

Example The various fines we have implemented to promote good social behaviour bear
witness to the fact that Singaporeans generally will not do what is their rightful
duty unless being threatened by punishment.

Link Back Hence, Mace‟s view is certainly too idealistic for modern day Singapore.

5
Point Gregg states in paragraph 3 that, in a commercial society, “to try to interfere or
eliminate it would be go against the grain of a universal and powerful human
instinct.”
He believes that “as individuals pursue profit, unintentionally or intentionally, it
Elaboration
adds to the sum total of societal well-being, plus the fact that people are
motivated to get things done.”
Evaluation We do not argue with Gregg‟s view that the basic human instinct of selfishness
drives Man to do the things he does for his own sake. The „what is it in for me‟
attitude is very much evident in Singapore society, to the point that even the
government adopts very much a „stick and carrot‟ approach to encourage
people to adopt certain policies. The idea is that people reacts well to
economic incentives. Hence, if our leaders can encourage Singaporeans via
some form of self-gains, the collective action of many Singaporeans will
ultimately result in something positive for Singapore.
However, while we can appreciate Gregg‟s view, the fact remains that his view
is a rather cynical, materialistic view of society that should not be encouraged
in Singapore. If people are purely motivated by monetary gains (no matter how
basic and primal the feeling is), sooner or later, the government will have to
increase the size of the incentive in order to get Singaporeans to do
something. Also, Singaporeans will come to expect the government to hand
out rewards for doing things that they should be doing willingly in the first
place. This will result in a blackmail scenario.

Example Various social campaigns that rewards people for being civic minded, like the
Traffic Police‟s safe driving campaigns which rewards motorists for safe driving
habits, should not even be happening in the first place. The fact that it does
serves to show how Singaporeans have regressed.

Link Back Hence, while Gregg‟s view is an accurate description of modern day societies
like Singapore, it is something that we should not encourage in the long-run as
it will undermine the civic-mindedness of Singaporeans.

6
MERIDIAN JUNIOR COLLEGE
Preliminary Examination
Higher 1

___________________________________________________________________

H1 General Paper 8806/01


rd
Paper 1 3 September 2010

1 hour 30 minutes
Additional Materials: Writing Paper
___________________________________________________________________
READ THESE INSTRUCTIONS FIRST

Write your full name, civics group and GP tutor’s name on all the work you hand in.
Write in dark blue or black pen on both sides of the paper.

Answer one question.


Note that 20 marks out of 50 will be awarded for your use of language.

Write the question number on all the work you hand in.

At the end of the examination, fasten your work securely together. You do not need to submit this
Question Paper with your Answer Script.
All questions carry equal marks.

___________________________________________________________________
This document consists of 2 printed pages.
[Turn over]
PAPER 1

Answer one question from this Paper.


Answers should be between 500 and 800 words in length

1. Is it important to love what you do?

2. “Business and ethics can never coexist.” Do you agree?

3. Can war ever be a good thing?

4. To what extent is the media playing a destructive role in society today?

5. Consider the view that too much faith is placed in paper qualifications.

6. Is clean energy for all a realistic aim?

7. If we speak well, does it matter how well we write?

8. Does technology divide or unite the world?

9. How tolerant is your society of diversity?

10. "Knowledge of the past hampers more than it helps." Do you agree?

11. Considering the rapid rate of change in the world, should we still hold on to traditions?

12. Can high medical costs ever be justified?


1 Formatted: Font: 10 pt

This Iinsert consists of 2 printed pages. Formatted: Font: 10 pt


Formatted: Font: 10 pt
Formatted: Top: 1", Bottom: 1", Header
Passage 1: Mark Twain comments about on the deterioration of the art of lying. distance from edge: 0.49", Footer distance
from edge: 0.49"
Formatted: Font: 11 pt, Not Bold
Formatted: Body Text, Left
1 Observe, I do not mean to suggest that the custom of lying has suffered any decay or interruption
– no, for the Lie, as a Virtue, aA Principle, is eternal; the Lie, as a recreation, a solace, a refuge in Formatted: Justified
time of need, man's best and surest friend, is immortal, and cannot perish from the earth. My Formatted: Right
complaint simply concerns the decay of the art of lying. No high-minded man, no man of right Formatted Table
feeling, can contemplate the lumbering and slovenly lying of the present day without grieving to 5
see a noble art so prostituted. Formatted: Font: Not Bold, Font color: Auto
Formatted: Right
2 No fact is more firmly established than that lying is a necessity of our circumstances – the Formatted: Right
deduction that it is then a Virtue goes without saying. No virtue can reach its highest usefulness
without careful and diligent cultivation,cultivation; therefore, it goes without saying that this one
ought to be taught in the public schools, even in the newspapers. What chance has the ignorant 10
uncultivated liar against the educated expert? What chance have I against a lawyer? Judicious
lying is what the world needs. I sometimes think it were even better and safer not to lie at all than
to lie injudiciously. An awkward, unscientific lie is often as ineffectual as the truth.
Formatted: Right
3 Note that venerable proverb: Children and fools always speak the truth. The deduction about Formatted: Right
adults and wise persons is plain. Francis Parkman, the historian, says, "The principle of truth may 15
itself be carried into an absurdity." It is strong language, but true. None of us could live with a
habitual truth-teller; but thank goodness none of us has to. A habitual truth-teller is simply an
impossible creature; he does not exist; he never has existed. Of course there are people who
think they never lie, but it is not so, and this ignorance is one of the very things that shame our so-
called civilization. Everybody lies – every day; every hour; awake; asleep; in his dreams; in his joy; 20
in his mourning; if he keeps his tongue still, his hands, his feet, his eyes, his attitude, will convey
deception – and purposely. Even in sermons, but that is a platitude.
Formatted: Right
4 I think that courteous lying is a sweet and loving art, and should be cultivated. What I bemoan is Formatted: Right
the growing prevalence of the brutal truth. Let us do what we can to eradicate it. An injurious truth
has no merit over an injurious lie. Neither should ever be uttered. An injurious lie is an 25
uncommendable thing; and so, also, and in the same degree, is an injurious truth – a fact that is
recognized by the law of libel.
Formatted: Right
5 Among other common lies, we have the silent lie – the deception which one conveys by simply Formatted: Right
keeping still and concealing the truth. Many obstinate truth-mongers indulge in this dissipation,
imagining that if they speak no lie, they lie not at all. 30
Formatted: Right
6 Lying is universal – we all do it. Therefore, the wise thing is for us to diligently train ourselves to lie Formatted: Right
thoughtfully, judiciously; to lie for others' advantage, and not our own; to lie healingly, charitably,
humanely, not cruelly, hurtfully, maliciously; to lie gracefully and graciously, not awkwardly and
clumsily; to lie firmly, frankly, squarely, with head erect, not haltingly, tortuously, with hesitatingly,
as being if ashamed of our high calling. 35
Formatted: Right
Formatted: Right
Formatted: Right
Formatted: Tab stops: Not at 4.9"
2 Formatted: Font: 10 pt

Formatted: Justified
3 Formatted: Font: 10 pt

Passage 2: Clive Thompson talks aboutpoints out how why it is increasingly difficult to lie.

1 The Internet makes us more truthful. Wasn’t cyberspace supposed to be the scary zone where Formatted: Font color: Auto
you could not trust anyone? Back when the Internet first came about, pundits worried that the Formatted: Right
digital age would open the floodgates of deception. Since anyone could hide behind an Formatted Table
anonymous Hotmail address or chat-room moniker, Net users, we were warned, would be free to
lie with impunity. Parents panicked and frantically cordoned off cyberspace from their children, 5
under the assumption that anyone lurking out there in the ether was a creep until provend
otherwise. And to a certain extent, the fear seemed justified. According to Psych 101, we are more Formatted: Font color: Auto
likely to lie to people when there is distance between us – and you cannot get much more distant
than a cyber-chat buddy in Siberia who calls himself 0minous-1.
Formatted: Right
2 Why were those fears unfounded? What is it about online life that makes us more truthful? It is 10 Formatted: Font color: Auto
simple: We are worried about being caught. In ''real'' life, after all, it is actually pretty easy to get Formatted: Right
away with spin. If you tell a lie to someone at a cocktail party or on the phone, you can always
backtrack later and claim you said no such thing.
Formatted: Right
3 On the Internet, though, your words often come back to haunt you. The digital age is tough on its Formatted: Right
liars, as a seemingly endless parade of executives are learning to their chagrin. Today's titans of 15 Formatted: Font color: Auto
industry are laid low not by ruthless competitors but by prosecutors gleefully waving transcripts of
old e-mail, filled with suggestions of subterfuge. Even Microsoft was tripped up by old e-mail
messages, and you would figure its employees would know better. Indeed, the axiom that
machines never forget is built into the very format of e-mail – consider that many e-mail
programmes automatically ''quote'' your words when someone replies to your message. Every 20
time I finish an e-mail message, I pause for a few seconds to reread it before I hit ''send''.
Formatted: Right
4 Still, it is not only the fear of electronic exposure that drives us to tell the truth. There is something Formatted: Right
about the apparent harmlessness of the Internet that encourages us to bare our souls, often in Formatted: Font color: Auto
rather outrageous ways. Psychologists have noticed for years that going online seems to have a
catalytic effect on people's personalities. The most quiet and reserved people may become 25
deranged loudmouths when they sit behind the keyboard, staying up until dawn and conducting
angry debates on discussion boards with total strangers. You can usually spot the newbies in any
discussion group because they a're the ones WRITING IN ALL CAPS – they are tripped out on
the Internet's heady combination of geographic distance and pseudo-invisibility.
Formatted: Right
5 Heated arguments are far more common in online discussion boards than in comparable face-to- 30 Formatted: Right
face communication. When people communicate online with invisible listeners, they are more Formatted: Font color: Auto
likely to offer up personal details about themselves without any prompting. The psychologically
comforting effect of the Net makes people willing to talk about anything – ‘disinhibited’ as the
mental-health profession would say. The Net was supposed to be a military tool. Instead, it has
become a vast arena for collective therapy – for a mass outpouring of what we are thinking and 35
feeling. Stripped of our bodies, it seems, we become creatures of pure opinion.
Formatted: Right
6 As more and more of our daily life moves online, we could find ourselves living in an increasingly Formatted: Right
honest world, or at least one in which lies have ever more serious consequences. With its Formatted: Font color: Auto
unforgiving machine memory, the Internet might turn out to be the unlikely conscience of the
world. 40
Formatted: Right
Formatted: Right
4 Formatted: Font: 10 pt

Formatted: Right
Formatted: Font: 11 pt
Formatted: Body Text
Candidate Name Civics Group General Paper Tutor

MERIDIAN JUNIOR COLLEGE


Preliminary Examination
Higher 1

_________________________________________________________________________

H1 General Paper 8806/02


rd
Paper 2 3 September 2010

1 hour 30 minutes
Candidates answer on the Question Paper

Additional Materials: 1 Insert


_________________________________________________________________________
READ THESE INSTRUCTIONS FIRST

Write your full name, civics group and GP tutor’s name in the spaces at the top of this page.
Write in dark blue or black ink on both sides of the paper.

Answer all questions.


The Insert contains the passages for comprehension.
(Note that 15 marks out of 50 will be awarded for your use of language.)

At the end of the examination, submit your Question Paper. You do not need to submit the Insert.
The number of marks is given in brackets [ ] at the end of each question or part question.

For Examiner’s Use

Content
/35

Language
/15

TOTAL
/50

_________________________________________________________________________
This document consists of 7 printed pages and 1 blank page.
2

BLANK PAGE
3

PAPER 2 (50 Marks)

Read the passage in the insert and answer all the questions which follow below. Note that
up to fifteen marks will be given for the quality and accuracy of your use of English
throughout this Paper.

Note: When a question asks for an answer IN YOUR OWN WORDS AS FAR AS POSSIBLE
and you select the appropriate material from the passage for your answer, you must still use
your own words to express it. Little credit can be given to answers which only copy words or
phrases from the passage.

From Passage 1

1. Explain the author’s argument in paragraph 1. Use your own words as far as possible.

[2]

2. Explain what the author implies about the Lie by calling it (a) a ‘Virtue’ (line 2), and (b) a
‘refuge in time of need’ (lines 2-3).
(a)

(b)

[2]

3. Explain in your own words as far as possible why the author proposes that lying
should be ‘taught in schools’ (line 10).

[3]
4

4. ‘What chance has the ignorant uncultivated liar against the educated expert?’ (lines 10-
11)
How does the author illustrate the concept of the ‘educated expert’?

[1]

5. ‘Children and fools always speak the truth.’ (line 14)


What deduction can be made from this?

[1]

6. Explain what the author means by ‘an injurious truth has no merit over an injurious lie’
(lines 24-25). Use your own words as far as possible.

[2]

7. Explain in your own words as far as possible why the author believes that people who
‘speak no lie’ (line 30) are still lying.

[1]

From Passage 2

8. Why does the author say that the fear of cyberspace ‘seemed justified’ (line 7)? Use
your own words as far as possible.

[1]
5

9. ‘Even Microsoft was tripped up by old email messages, and you would figure its
employees would know better.’ (lines 17-18)
Explain the irony evident in this statement.

[1]

10. Using material from paragraphs 2 to 5, summarise Clive Thompson’s arguments for why
the Internet is driving people to tell the truth.
Write your summary in no more than 120 words, not counting the opening words which
are printed below. Use your own words as far as possible.

The Internet drives people to tell the truth as

[8]
6

Questions on both passages

11. Give the meaning of the following words as they are used in the passages. You may
write your answer in one word or a short phrase.

From Passage 1

(a) prostituted (line 6)

(b) bemoan (line 23)

(c) eradicate (line 24)

From Passage 2

(d) titans (line 15)

(e) axiom (line 18)


[5]

12. Mark Twain believes that lying is an art which needs to be cultivated, while Clive
Thompson says that lying is now made more difficult by the Internet.
How far do you agree with the views of each author? Are these views applicable to
interactions in modern society? In your answer, develop some of the points made by the
authors, and give your own views and some account of the experiences which have
helped you to form them.
7
8

[8]

– End of Paper –
Meridian Junior College
General Paper JC2 Preliminary Examination (2010)
Paper 2 Suggested Answer Scheme

PASSAGE 1

(1) Explain the author’s argument in paragraph 1. Use your own words as far as possible. [2]
From Passage (lifts and contextual clues) Acceptable Answers
custom of lying…is eternal… is immortal… He shows that people have always lied/lying is an
(line 3) age old practice (tradition) ... [1]

decay of the art of lying…noble art so … but not everyone may be skilled at lying/ know
prostituted … (line 6) how to lie well/lying skills are deteriorating. [1]

(2) Explain what the author implies about the Lie by calling it (a) a ‘Virtue’ (line 2), and (b) a
‘refuge in time of need’ (line 3)? [2]
From Passage (lifts and contextual clues) Acceptable Answers
a) Virtue (line 2) It is a desirable/positive/good way of
behaving/should be encouraged/its practice
produces benefits. (1)
b) Refuge in time of need (line 2) * Not “skill”

It protects/shields/offers a solution (1/2) when we are


in trouble/enables people to get out of trouble. (1/2)
* Not “hideaway”
* Context must be correct.

(3) Explain in your own words as far as possible why the author proposes that lying should be
taught in schools (line 10). [3]
From Passage (lifts and contextual clues) Acceptable Answers
“…lying is a necessity of our There are situations where lying is essential. (1/2)
circumstances…a Virtue…”? (line 7)

No virtue can reach its highest usefulness Lying can only become most beneficial ... (1/2)
(line 8)

without careful and diligent cultivation (line ... if people are coached/trained to do so ... (1/2)
9)
... in a thorough and conscientious manner ... (1/2)
* Not “taught” (lifted from question and thus not an
explanation)

... so that people will not lose out to those very


“chance… against the educated expert” (line adept/skilled at lying. (1/2)
10)
Society could do with people who know how to lie
Judicious lying is what the world needs (line when necessary/when practical/when astute
11) to/sensibly. (1/2)
* Accept “with good judgement”

1
(4) ‘What chance has the ignorant uncultivated liar against the educated expert?’ (lines 10-11)
How does the author illustrate the concept of the ‘educated expert’? [1]
From Passage (lifts and contextual clues) Acceptable Answers
What chance have I against a lawyer (line 11) The author implies that lawyers are expert liars. (1)

(5) ‘Children and fools always speak the truth.’ (line 14) What deduction can be made from
this? [1]
From Passage (lifts and contextual clues) Acceptable Answers
Children and fools always speak the truth. The Adults and wise persons (or infer “any astute adult”)
deduction about adults and wise persons is do not always speak the truth/will lie (1).
plain.

(6) Explain what the author means by ‘an injurious truth has no merit over an injurious lie’.
(lines 24-25) Use your own words as far as possible. [2]
Acceptable Answers
A truth that hurts/causes harm ... (1)

... is no better than / has the same impact as (1) a deleterious/harmful lie. [Still touching on “injurious”.]
* Context must be correct.

(7) Explain in your own words as far as possible why the author believes that people who
‘speak no lie’ (line 30) are still lying. [1]
From Passage (lifts and contextual clues) Acceptable Answers
by simply keeping still and concealing the By not saying anything, one is still hiding the truth
truth… (line 29) ... (1/2)

…the silent lie – the deception which one ... and is misleading/ encouraging / tricking others
conveys … (line 28) to believe in something untrue (1/2)
* Accept lifts: Lie / Truth

PASSAGE 2

(8) Why does the author say that the fear of cyberspace ‘seemed justified’ (line 7)? Use your
own words as far as possible. [1]
From Passage (lifts and contextual clues) Acceptable Answers
Any 1 (1)
According to Psych 101, we are more likely to People are (psychologically) more inclined to lie
lie to people when there is distance between
- when far apart/when the person they are
us… (line 7)
interacting with is in a faraway location.
* Marker’s discretion w.r.t. “more likely”
…floodgates of deception (clue) (line 3)
- if nobody knows who you really are
…anyone could hide behind an anonymous * Marker’s discretion w.r.t. “more likely”
Hotmail address… (line 4) - if people do not know/cannot verify who they are
interacting with. (1)
* Does not require “more likely”

2
(9) ‘Even Microsoft was tripped up by old email messages, and you would figure its
employees would know better.’ (lines 17–18) Explain the irony evident in this statement. [1]
From Passage (lifts and contextual clues) Acceptable Answers
…and you would figure its employees would There programming experts who are supposed to
know better.‟ know how to cover their electronic trail, ...

„Even Microsoft was tripped up by old email ... and yet they were caught by digital evidence that
messages… they failed to destroy/had inadvertently created.
* General/alternate answer (marker’s discretion)
* Two-part answer has to convey the irony (no ½)

(11) Give the meaning of the following words as they are used in the passages. You may write
your answer in one word or a short phrase. [5]
Vocabulary 1 1/2 0
(a) No high-minded man, no  Widely used in an  Wrongly used  Widely used
man of right feeling, can unworthy manner [No indication of harm
contemplate the lumbering and  Used in an to nobility of lying]
slovenly lying of the present unworthy manner
day without grieving to see a  Widely used by
noble art so prostituted. people with no skill
(Passage 1, line 6)  Abused

(b) I think that courteous lying is  Express sorrow or  Object to


a sweet and loving art, and dissatisfaction
should be cultivated. What I  Complain about
bemoan is the growing
prevalence of the brutal truth.
(Passage 1, line 23)

(c) What I bemoan is the  Completely remove  Remove 


growing prevalence of the  Totally remove
brutal truth. Let us do what we
can to eradicate it.
(Passage 1, line 24)

(d) Today's titans of industry  Very important  Heroes


are laid low not by ruthless people
competitors but by prosecutors  Extremely
gleefully waving transcripts of successful people
old e-mail, filled with  Very powerful
suggestions of subterfuge. people
(Passage 2, line 15)  leaders

(e) Indeed, the axiom that  A statement or idea  Saying 


machines never forget is built which people
into the very format of e-mail – accept as true
consider that many e-mail  Widely accepted
programmes automatically saying
''quote'' your words when
someone replies to your
message.
(Passage 2, line 18)
* Students are not penalised for giving two answers, as long as the second answer does not contradict the first, and the
student is not giving the second answer as an alternative to the first (i.e. A or B).

3
(10) Using material from paragraphs 2 to 5, summarise Clive Thompson’s arguments for why
the Internet is driving people to tell the truth. Write your summary in no more than 120 words,
not counting the opening words which are printed below. Use your own words as far as
possible. [8]
The Internet drives people to tell the truth as…
Lifted Paraphrased
1 We are worried about being caught. (line 11) We feel uneasy at the thought of being
OR apprehended / found out. [1]
…fear of electronic exposure (line 22)

2 On the Internet, though, your words often Records of e-mails may serve as evidence. [1]
come back to haunt you. … transcripts of old
e-mail (lines 14 – 17)

3 The digital age is tough on its liars (line 14) The electronic era comes down hard/ is harsh
with people who lie ... [1]

4 …machines never forget is built into the very ... as this is built into the configuration of the e-
format of e-mail – consider that many e-mail mail system where previous e-mails are
programs automatically ''quote'' your words referenced to routinely[1]
when someone replies to your message (lines OR
19 – 20) permanent nature of e-mail system [1]

5 There is something about the apparent The Internet, which appears unable to hurt us /
harmlessness of the Internet… (line 23) benign,. . [1]

6 tripped out on ... due to the thrilling/ intoxicating effect of [1]

7 the Internet's heady combination of ... being far away… [1]


geographic distance… (line 29)

8 and pseudo-invisibility. (line 29) and hidden from the others / anonymity [1]
OR
When people communicate online with We are less hesitant/ reluctant to provide more
invisible listeners, they are more likely to private information of ourselves when we
offer up personal details about themselves converse with people we cannot see. [1]
without any prompting. (line 31)

9 The psychologically comforting effect of the The Internet has a calming/ soothing effect
Net makes people willing to talk about that makes us feel less reserved or self-
anything – „disinhibited‟ (line 32) conscious ... [1]

10 a vast arena for collective therapy – for a … and is a capacious/limitless platform/ forum
mass outpouring of what we are thinking and where we can express our thoughts and
feeling (line 35) emotions together, … [1]

11 Inferred ... and where, without our physical form, we


Stripped of our bodies, it seems, we become are free to pass judgements/ express ideas [1]
creatures of pure opinion. (line 36)

* Different word forms = L (½)

4
(12) Mark Twain believes that lying is an art which needs to be cultivated, while Clive
Thompson says that lying is now made more difficult by the Internet. How far do you agree
with the views of each author? Are these views applicable to interactions in modern society?
In your answer, develop some of the points made by the authors, and give your own views
and some account of the experiences which have helped you to form them. [8]

General Comments:
R1: How far do you agree with the views of each author?
R2: Are these views applicable to interactions in modern society?
Students must not forget: “develop some of the points made by the authors, and give your own views and
some account of the experiences which have helped you to form them.” Students could make use of
material that has been used for summary (Cambridge advice).
Some points can be argued both ways but students should not contradict their own points.

Suggested responses using Passage 1 (Twain)


Quotes Agree / Applicable Disagree / Inapplicable
(Reasons & Examples) (Reasons & Examples)
 for the Lie, as a Virtue, a  Lying is a practice that is not  Lying is indeed eternal and
Principle, is eternal; the Lie, as only omnipresent but also a unavoidable but it should
a recreation, a solace, a refuge norm. neither be taken as recreation
in time of need, man's best and  The act of lying should be nor a solace.
surest friend, is immortal, and encouraged as it can be  Interactions are only
cannot perish from the earth (l beneficial and protect us when meaningful and effective if the
2 – 3). we face trouble. basis of the communication is
 Interactions in modern society trust.
 Lying is universal – we all do it
are diverse in the people you  Personal or business
(l 31).
interact with, the topic of interactions alike require
 None of us could live with a conversations, the setting of honesty and truth, which is
habitual truth-teller; but thank the interaction. Often, contradicted by the act of lying.
goodness none of us has to. A interactions with strangers are
habitual truth-teller is simply an unavoidable.
impossible creature; he does  In unpleasant encounters and
not exist; he never has existed interactions, lying in the form of
(16 – 18). white lies, convenient excuses,
or false information can relieve
us of awkward situations and
even put us out of harm‟s way.
E.g. “White lies” that one tells
to avoid hurting people‟s
feelings.

 No fact is more firmly  We are faced with various  Not necessarily a Virtue as
established than that lying is a circumstances in our daily suggested by Twain.
necessity of our circumstances interactions.  It can also simply be calculated
– the deduction that it is then a  A lie told in pursuit of another or even malicious behaviour.
Virtue goes without saying (l7 - virtue e.g., compassion is not  Interactions in modern society
8). wrong. are complex and lying can be a
 Lying is not immoral when it is means to manipulate
a step toward, the process of someone.
becoming the best persons we E.g. Frequent cases of fraud
can be. and misinformation.

 No virtue can reach its highest  “Parents should not be  One of the important outcomes
usefulness without careful and alarmed if their child tells a lie. of education is strength of
diligent cultivation, therefore, it Almost all children lie. Those character, with honesty and
goes without saying that this who have better cognitive integrity being two of the
one ought to be taught in the development lie better because desired values schools would
schools, even in the they can cover up their tracks,” strive to inculcate in students.

5
Quotes Agree / Applicable Disagree / Inapplicable
(Reasons & Examples) (Reasons & Examples)
newspapers (l8 – 10). said Dr Kang Lee, director of  Lying contradicts the virtue of
the Institute of Child Study at honesty and integrity. This is
 Note that venerable proverb: Toronto University who carried still generally frowned upon by
Children and fools always out the research on 1,200 most societies.
speak the truth (l14). children aged two to 16 years  Not a value that we can see
old. being taught in Singapore
 A majority of the volunteers public schools.
told lies but it is the children  Schools aside, employers do
with better cognitive abilities also consider honesty and
who can tell the best lies. integrity as important qualities
 Lying involves multiple brain they seek in prospective
processes, such as integrating employees.
sources of information and
manipulating the data to their
advantage.
It is linked to the development
of brain regions that allow
“executive functioning” and use
higher order thinking and
reasoning.

 What I bemoan is the growing  According to utilitarian ethics,  People often poorly estimate
prevalence of the brutal truth. the only test necessary for the consequences of their
Let us do what we can to judging the morality of a lie is actions or specifically
eradicate it. An injurious truth balancing the benefits and undervalue or ignore the
has no merit over an injurious harms of its consequences. harmful consequences to
lie. Neither should ever be  Acts of lying in our modern society (e.g., mistrust) that
uttered. An injurious lie is an interactions, are morally their lies cause.
uncommendable thing; and so, acceptable when the resulting  As trust declines, cynicism
also, and in the same degree, consequences maximize spreads, and our overall quality
is an injurious truth (l23 – 26). benefit or minimize harm. A lie, of life drops. In addition,
therefore, is not always suggesting that people may lie
immoral; in fact, when lying is in pursuit of the greater good
necessary to maximize benefit can lead to a "slippery slope,"
or minimize harm, it may be where the line between
immoral not to lie. cleverly calculated moral
justifications and empty
excuses for selfish behavior is
exceedingly thin.
 Sliding down the slope
eventually kindles morally
bankrupt statements
 There is potentially great cost
in tolerating lies for vague or
subjective reasons, including
lies in honor of "the greater
good."

6
Suggested responses using Passage 2 (Thompson)
Quotes Agree / Applicable Disagree / Inapplicable
(Reasons & Examples) (Reasons & Examples)
 The Internet makes us more  As data on the Internet is  The fact that important
truthful. (l 36) difficult to destroy due to the communication cues are lost
way the Internet is structured, when interacting via the
 We are worried about being damaging evidence can be Internet means that people
caught. (l 46) dredged up even a long time have more opportunities to lie.
 As more and more of our daily after.  Besides, many people use the
life moves online, we could find  Also, many people are now Internet to fulfill some of their
ourselves living in an aware that records of online escapist fantasies or to play
increasingly honest world (l 74 activities are kept. around with their identity.
– 75)  All these make people less  In addition, we are not as
likely to lie. acutely aware of the people
Opposite point:
E.g. E-mails that can be whom we are interacting with,
 …we are more likely to lie to submitted in court as evidence. and we may have the
people when there is distance impression that our paths are
between us (l42 – 43) less likely to cross in real life.
E.g. Avatars used for online
gaming / social networking
sites.

 There is something about the  The Internet gives us a false  Not necessarily disinhibition as
Internet that encourages us to impression of privacy, since we suggested by Thompson.
bare our souls (l 58) often log in when we are in a  It can also simply be
relatively private space. calculated, attention-seeking
 It is just that they are more
behaviour.
comfortable talking about it –  There is actually a wide
 People are now more aware of
„disinhibited‟ (l 70) audience but we are unaware
the potential to gain a brief
of it and mistakenly think that
shot at fame/infamy, and plan
our views disappear into the
their actions accordingly. Some
“ether” (line 6).
even do it for money.
 As a result, we are more willing E.g. Exhibitionist behaviour of
to share views that we will bloggers / people on youtube,
normally keep secret. etc.
E.g. „Elite blogger‟, Wee Shu
Min‟s comments on ordinary
Singaporeans, etc.
 Heated arguments are far Possible reasons include: Online discussions may in fact be
more common in online  People get the impression that less damaging than face-to-face
discussion boards than in it is harder for others to track discussions as:
comparable face-to-face them down and un-mask their  People have the option of
communications. (l 65 – 66) true identity. As a result, they walking away from their
may seize the chance to speak computer or leaving the
out, or in the cases of some, discussion forum the moment
speak irresponsibly. things start to get too hostile
for their liking.
 The ability to bring together
 Physical blows cannot be
more views / perspectives
exchanged through online
increases the likelihood of
discussion boards.
people finding a point that they
 The moderator of the
find particularly objectionable.
discussion board can shut off a
 It is harder to monitor the particularly offensive member
people who post on a almost immediately, whereas
discussion board. Some in real life, things are more
unsavoury characters may join complicated.
the discussion board and
create trouble.
E.g. Searing critique of
government policies by
netizens from China via
popular web portals like
7
Quotes Agree / Applicable Disagree / Inapplicable
(Reasons & Examples) (Reasons & Examples)
Sina.com.
 a vast arena for collective  People tend to join online  Not everyone is active online.
therapy – for a mass groups, and as space and Many people, in fact, just lurk
outpouring of what we are distance are no longer factors, around and observe what is
thinking and feeling (l 71 – 72) the size of these groups can going on.
increase exponentially.
 People who desire attention
can make use of this outreach.
 People who need advice /
feedback can also obtain an
unprecedented number of
views.
E.g. Facebook and Twitter.
 the Internet might turn out to  Self-policing on the Internet –
be the unlikely conscience of Netizens take fellow members
the world. (l76 – 77) to task when certain norms /
rules are breached.
OR
 The Internet allows people to
make their experiences known  Not all experiences are able to
without having to go through capture the public‟s
elaborate bureaucratic imagination.
procedures.  Plenty of lapses are reported
 Once a particular experience but go unnoticed as they are
becomes widely known and too commonplace.
followed, the authorities are
placed in a position where they
must react and their reactions
are closely monitored and
critiqued by a large group of
people.
E.g. Singaporean netizens
highlighting misbehaviour /
lapses / injustices on STOMP.
 As Abraham Lincoln observed,
“With public sentiment, nothing
can fail. Without it, nothing can
succeed.”

8
Candidate Name:

Registration Number: Tutor’s Code:

NATIONAL JUNIOR COLLEGE


SH2 Preliminary Examination
GENERAL PAPER 8806/01
Paper 1 2nd September 2010
1 hour 30 minutes
Additional Materials: Answer Paper

READ THESE INSTRUCTIONS FIRST

Write your full name, registration number and tutor’s code on all the work you hand in.
Write in dark blue or black pen on both sides of the paper.
Do not use staples, paper clips, highlighters, glue or correction fluid.

Answer one question.


Note that 20 marks out of 50 will be awarded for your use of language.

At the end of the examination, fasten all your work securely together.
All questions in this paper carry equal marks.

© National Junior College 2010


 
2

PAPER 1

Answer one question.

Answers should be between 500 and 800 words in length.

1 To what extent is there a need for global cooperation in today’s world?

2 How far has technology helped us to achieve equality?

3 How important is it for a city to retain a sense of its history?

4 ‘Logic is more valuable than the imagination.’ Do you agree?

5 Should we treat our celebrities any differently from any other person?

6 Investing in education has never been as important as it is now. Do you agree?

7 How worthwhile is it to fund scientific research when not everyone will benefit?

8 ‘Powerful branding now dictates consumer choice.’ Do you agree?

9 ‘The benefits of migration are over-rated.’ Do you agree?

10 How successful has your country been in promoting the arts?

11 ‘Entertaining, but fundamentally useless.’ Discuss this with reference to either film or
music in your country.

12 Would you agree that apathy is the greatest obstacle to alleviating environmental
problems?

© National Junior College 2010


Candidate Name:

Registration Number: Tutor’s Code:

NATIONAL JUNIOR COLLEGE


SH2 Preliminary Examination
GENERAL PAPER 8806/02
Paper 2 2nd September 2010
1 hour 30 minutes
INSERT

READ THESE INSTRUCTIONS FIRST

This Insert contains the passages for comprehension.

This document consists of 3 printed pages.


Intelligence, Technology & the Mind

Passage 1: Steven Pinker writes about the mind.

1 New forms of media have always caused panic: the printing press, newspapers,
paperbacks and television were all once denounced as threats to consumers’
brainpower and moral fibre. This is also true of electronic technology. PowerPoint, we
are told, is reducing potentially intelligent discussion to bullet points. Search engines
lower our intelligence because we now skim on the surface of knowledge and social 5
networking platforms are shrinking our attention spans.

2 For a reality check today, take for instance, the state of science, which demands high
levels of brainwork and is measured by clear benchmarks of discovery. These days,
scientists are never far from their e-mail, rarely touch paper and cannot lecture without
PowerPoint. If electronic media were hazardous to intelligence, the quality of science 10
would be plummeting. Yet discoveries are multiplying like fruit flies, and progress is
dizzying. Other activities in the life of the mind, like philosophy, history and cultural
criticism, are likewise flourishing.

3 Casual critics of new media use science itself to press their case, citing research that
shows how “experience can change the brain.” However, experience does not 15
fundamentally change the basic capabilities of the brain. Speed-reading programmes
have long claimed to do just that, but the verdict delivered by brain experts is that these
claims are total lies as they encourage reading without deep understanding. Genuine
multitasking, too, is clearly a fiction, as evidenced by the familiar sight of cars undulating
between lanes as the driver negotiates business deals on his mobile phone. The fact 20
remains that we cannot change the way the brain operates.

4 The panic surrounding the so-called effects of consuming electronic media is


unfounded. The same critics write as if the brain takes on the qualities of whatever
information it consumes; this is an ill-informed extension of the well-known saying, “you
are what you eat.” As with primitive peoples who believe that eating fierce animals will 25
make them fierce, they assume that reading bullet points and Twitter postings turns
your thoughts into unconnected, incoherent statements.

5 Yes, the constant arrival of information packets can be distracting or addictive,


especially to people with attention deficit disorder. But distraction is not a new
phenomenon. The solution is not to bemoan technology but to develop strategies of 30
self-control, as we do with every other temptation in life. Turn off e-mail or Twitter when
you work, put away your Blackberry during meals and set designated times for
activities.

6 And to encourage intellectual depth, don’t blame PowerPoint or Google. Habits of deep
reflection, thorough research and rigorous reasoning do not come naturally to people. 35
They must be acquired at school, and maintained with constant analysis, criticism and
debate. They are not granted by sleeping on encyclopaedias, nor are they taken away
by efficient access to information on the Internet.

7 The new media thrive for a reason. Knowledge is increasing exponentially; human
brainpower and waking hours are not. Clearly, the Internet and information technologies 40
are helping us manage and search all the information available to us, which we
conveniently retrieve from Twitter, e-books and online encyclopedias. Far from making
us stupid, these technologies are the only things that will keep us smart.

Selectively edited from The New York Times, 10 June 2010


Professor Steven Pinker, ‘Mind Over Mass Media’

  2
Passage 2: Carl Zimmer writes about the ‘extended mind’.

1 Our minds are under attack. Text messages make us illiterate. Blogs make us coarse,
YouTube makes us shallow. Author Nicholas Carr has argued that the Internet is
damaging our brains, robbing us of our memories and deep thoughts. “As we come to
rely on computers to mediate our understanding of the world,” he wrote, “it is our own
intelligence that flattens into artificial intelligence.” 5

2 More significantly, the ominous warnings feed on a popular misconception of how the
mind works. We tend to think of the mind as separated from the world; we imagine
information trickling into our senses and reaching our isolated brains. As plausible as
this picture may seem, it does not explain a lot of recent scientific research. In fact, the
mind appears to be adapted for reaching out from our heads and making the world, 10
including our machines, an extension of itself. This concept of the extended mind was
first raised in 1998, by two philosophers, who believe that the mind is a system made up
of the brain plus parts of its environment. The mind appears to be adapted for reaching
out and making the world, including our machines, an extension of itself.

3 Essentially, some argue that there is no difference between the information that we 15
store in our minds and that which is written into our notebooks, saved on our laptops or
mobile phones – these are part of our ‘extended mind’. Hence, it follows that we all
have minds that extend out into our environments, rather than being entirely self-
contained.

4 Our memory holds a great deal of information. But the extended mind moves swiftly 20
between outside and inside sources, showing little regard for where its information
comes from. However, it does more than take in information, of course! It also makes
decisions and sends out commands—and those commands certainly don’t stay inside
the mind. Our hands and eyes constantly send signals to the brain, and that feedback
alters the signals coming back out. What’s even more remarkable about our brains is 25
that they actually search for new things to make part of this feedback system. The
eagerness with which the brain merges with tools has made it possible to create some
stunning mind-machine interfaces. The U.S. Navy has developed a flight suit which
links the pilot’s body to the helicopter’s mechanisms – this technology will allow pilots to
learn to fly blindfolded or to carry out complex manoeuvres. The helicopter becomes, in 30
effect, part of the pilot’s body, linked back to his mind.

5 The extended mind will change how we judge what’s good and bad about today’s mind-
altering technologies. There’s nothing unnatural about relying on the Internet for
information. After all, we are constantly consulting the world around us like a kind of
visual Wikipedia. Nor is there anything bad about our brains’ being altered by these new 35
technologies, any more than there is something bad about a monkey’s brain changing
as it learns how to play with a rake.

6 Neuroscientists will soon be able to offer fresh ways to enhance our brains, whether
with drugs or with implants. To say that these are immoral because they defile our true
selves—our isolated, distinct minds—is to ignore biology. Our minds already extend out 40
into the environment, and the changes we make to the environment already alter our
minds. There’s no point in trying to take apart the connections between the inside and
the outside of the mind. Instead we ought to focus on managing and improving those
connections. We need more powerful ways to filter the information we get online, so
that we don’t get a mass case of distractibility. Some people fear that trying to fine-tune 45
the brain-Internet connection is an impossible task. But if we’ve learned anything it’s not
to underestimate the mind’s ability to adapt to the changing world.

Selectively edited from Discover Magazine, 15 January 2009


Carl Zimmer, ‘How Google Is Making Us Smarter’
 

  3
National Junior College GP Preliminary Exam 2010

NJC Prelim 2010 Paper 2 Answer Scheme

Questions from Passage 1

1 According to paragraph 1, why do new forms of media cause ‗panic‘ (line 1)? Use
your own words as far as possible. [2]
Lift Suggested Answer
1. ‗threats to consumers’ brainpower… 1 mark
[they are seen as] attacking/ dangerous
to people/ shoppers/ our intellect/
intelligence/

intellectual/ cognitive/ mental + capacity/


functions/ ability
reference
0 marks for affect our memory/ ability to
focus/ concentration/ attention
2. …and moral fibre’. 1 mark
And sense of right and wrong/ ability to
choose the right thing/ sense of justice/
VALUES/ righteousness/ virtues

2 What does the phrase ‗progress is dizzying‘ (lines 11 - 12) suggest about the effect
of technology? [1]
Lift Suggested Answer
‗dizzying‘ It is so rapid (that it is hard to keep track
of it) OR it is confusing OR it is
overwhelming
National Junior College GP Preliminary Exam 2010

3 Identify and explain two phrases that emphasise the idea that experience cannot
alter the ‗basic capabilities‘ (line 16) of the brain. [2]
Lift (only these 4 to be entertained and no Suggested Answer
others)
‗long claimed‘ / ‘claimed’ Use of ‗claimed‘ - unproven assertions/
assumptions
OR
―long claimed‖ –assumed to be true but
not proven to be so
‗total lies‘ Total –absolute word/ indicates how the
belief is ―completely‖ wrong.
OR
Lies –shows the untruthfulness of the
critics‘ claim
‗clearly a fiction‘ ―clearly‖ is a strong word that
emphasises/ highlights how the critics‘
idea is not true/ obviously made up
OR
―clearly‖ adds to the meaning of ―fiction‖
to doubly emphasis that the situation is
obviously made up and does not exist
OR
―fiction‖ is a word that shows how what is
proposed by the critics is not true
‗the fact remains‘ ―fact‖ implies that the idea is
unchallenged truth/ proven to be the
case/ indisputable
OR
―remains‖ indicates that the idea is to stay
unchanged/ has withstood the test of time
as it has been unchanged over time and
will stay unchanged.
National Junior College GP Preliminary Exam 2010

4 What is the author‘s attitude towards ‗the same critics‘ (line 23)? Use your own words as far as
possible. [1]

Lift Suggested Answer


‗the same critics write as if the brain… The author expresses a critical/ derisive/ cynical/
ill-informed extension of the well-known scornful attitude towards them, given that they have
saying… As with primitive peoples‘ such simple ideas about the media.

Not accepted:
- The author disagrees with the critics (too vague)
- Negative attitude (too vague)
- Sceptical/doubtful (inaccurate)
- Sarcastic (this refers to tone, not attitude)
- Dislike/ discontent/ dissatisfied/ displeased/ intolerant
(inaccurate)
- Frustrated/annoyed (inaccurate)

5 What does the author mean by ‗They are not granted by sleeping on encyclopaedias‘ (line 37)?
Use your own words as far as possible. [3]
Award 3 marks for 4 accurately expressed ideas, 2 marks for 3 ideas and 1 mark for 2 ideas.
Lift Suggested Answer
1. Habits of deep reflection, thorough research and Practices/ traits/ the custom/ routines
rigorous reasoning of/ regular effort at higher order thinking.

Not accepted: skill/ability/technique


2. do not come naturally to people Are not innate/ in-born/ should not be
taken for granted/ are not spontaneous.

Not accepted: do not come about instantly

3. must be acquired at school, and maintained They have to be i) learnt/ require


education/ picked up at institutions of
learning and ii) need practise/ refining/
sustaining

4. They are not granted by sleeping on And not by being lazy/ using slip-shod and
encyclopaedias, quick methods.

Acceptable interpretation: ―not


merely/simply by absorbing information‖

Sample answer: These practices to cultivate higher order thinking skills (1) cannot be formed
without effort (4); instead, one must recognise that they are not innate (2), hence they have to be
learnt through education, and conscientiously sustained through practice (3).
National Junior College GP Preliminary Exam 2010

Questions from Passage 2

6 In paragraph 1, the author illustrates the idea of our minds being ‗under attack‘ (line 1). What
are two ways by which he does this? Use your own words as far as possible. [2]

Lift Suggested Answer


‗text messages… blogs… YouTube…‘ He uses examples of how technology causes
harm to our minds.
The Internet He cites everyday examples such as text
messages… to argue that they are harmful our
intelligence.
Author Nicholas Carr By quoting an acclaimed author to support and
strengthen this opinion/ to give credibility.

Text messages make us illiterate. Blogs (Language use)


make us coarse, YouTube makes us shallow. Parallel phrasing as emphasis to indicate the
direct, undesirable consequences that are
inflicted upon human beings, as a result of these
forms of technology.

Internet is damaging our brains, robbing (Language use)


us…flattens into artificial intelligence. The Internet is personified. OR

The use of highly negative verbs justify that our


minds are ―under attack‖ because the Internet
severely incapacitates the way our minds
function.
National Junior College GP Preliminary Exam 2010

7 Using material from paragraphs 4 to 6 summarise Carl Zimmer‘s observations about how the extended mind functions, how it will affect
technology and us and how we should respond to it.
Write your summary in no more than 120 words, not counting the opening words, which are printed below. Use your own words as far as
possible.

According to Carl Zimmer, the extended mind… [8]


Lift Suggested Answer
How the extended mind works:
1 the extended mind moves swiftly between outside and inside Transcends the boundaries of the mind/
sources travels/switches(odd expression) between internal and external pools
OR of information
Our minds already extend out to the environment, (line 40) reaches out to the external world
2 Little regard for where its information comes from. Doesn‘t discriminate/ doesn‘t pick and choose/ no limits to where the
facts come from/collecting information from everything/without caring
about the origins of the data.
(accept information)
3 also make(s) decisions and It makes choices
4 send(s) out commands Propagating signals and processes signals that are sent back/passing
(―Our hands and eyes… …signals coming back out‖ line 24 is the down instructions/directs instructions outwards/relays instructions/how
example) to carry out the actions

5 *to actually search for new things to make part of this feedback Looks for novel/ fresh information to integrate/ internalize/ include
system *credit answers that show that the extended mind looks for information
(this point is ‗4a‘ in the standardization scripts) to include/ grow the ‗information bank‘
How it affects technology
6 The eagerness… merges with tools It readily/ enthusiastically/willingly joins/ melds/ with
technology/anticipation to integrate with these technologies
(must be a recognition of enthusiasm to meld with technology)
7 (it will be) possible to create some stunning mind-machine So we can manufacture/ make amazing mind-machine systems/
interface programmes [bond them]
OR (must have amazing or marvelous mind-machine link)
…[machines become] linked back to [the] mind
8 will change how we judge what’s good and bad about today‘s It will alter how we decide what technology is beneficial or
mind-altering technologies detrimental/influences our principles/alter our moral values
How should we respond
9 [we will realize that] there‘s nothing unnatural about relying on the And we will realize dependency on the Internet for knowledge is not

5
National Junior College GP Preliminary Exam 2010

Internet for information. bad/ it is fine and normal to make use of them
10 Nor is there anything bad about our brains‘ being altered by these Or that our brains are being changed/ tweaked/ refined/make us open
new technologies, to our mind being enhanced/improved/changed drastically
11 [we will see] There‘s no point in trying to take apart the We should relinquish attempts to sever the link between the external
connections between the inside and the outside of the mind. and internal mind/environment an the mind/we are inextricably
connected to our environment
12 [Instead] we ought to focus on managing and improving those And work on manipulating/ caring for/ operating and enhancing/
connections augmenting/ making better/empowering this bond/ link/ attachment
(odd expression) concerted effort made to cope with and develop this
interdependence (this is point 11)
13 we need to develop] more powerful ways to filter the information Create stronger/ effective methods of discernment/ sorting/
we get online, sifting/sieving information/find methods to properly process this
information
14 so that we don‘t get a mass case of distractibility so we won‘t be sidetracked
15 not to underestimate the mind‘s ability to adapt to the changing We learn not to belittle/ undervalue the mind‘s ability to accommodate/
world. acclimatize/minds are strong enough to cope with the external world.
(accept ‗mind‘s ability)

Points Marks
11 and more 8
9-10 7
8 6
6-7 5
4-5 4
3 3
2 2
1 1

6
National Junior College GP Preliminary Exam 2010

8 Explain in your own words why the writer refers to Wikipedia in line 35. [1]
Lift Suggested Answer
‗we are constantly consulting the world around Just as Wikipedia provides a wealth of
us like a kind of visual Wikipedia‘ information for us/
Just as we seek opinions and get information
from other people
So do the our environment and observations we
make

9 Why does the writer place the words ‗our isolated, distinct minds‘ (line 40) in between two
dashes? [1]
Lift Suggested Answer
To say that these are immoral because they To show that he means the opposite/
defile our true selves—our isolated, distinct That he does not agree/
minds—is to ignore biology To emphasize that our minds are not
independent and unique

Accept any answer that shows that the writer


does not agree with the words in between the
two dashes.

Accepted answers:
 The writer wants to point out that the
words in between the dashes refer to the
public‘s general misconception.
 The writer wants to highlight how flawed
this view is.
 He wants to imply that the phrase is
simply an assumption by his opponents
that isn‘t valid.

Answers which were close but rejected [0m]


 This is the supposed idea that most
people believe in. [Does not quite show
that the writer disagrees with the idea in
the brackets. Needs further inference. ]

7
National Junior College GP Preliminary Exam 2010

Questions from both passages

10 1 mark 1/2 mark 0 marks


(a) flourishing Blooming, burgeoning, Expanding, Luxuriant, rich, rapidly
(Passage 1, line 13) in full swing, thriving, successful, doing very improving, growing
developing rapidly, well, developing/ well, growing, growing
prospering, increasing developing extensively, growing at
in popularity, booming, successfully, a very healthy rate,
proliferating, becoming increasingly progressing, making
developing in common good progress,
abundance present in large
abundance, becoming
greater in numbers, at
its peak, fruitful,
prevailing in numbers,
alive, enjoying great
attention
(b) incoherent Unintelligible, Disconnected, not Wandering,
(Passage 1, line 27) inarticulate connected/ lack of breathless, rambling,
Muddled, Impossible interconnectedness, illogical, irrelevant,
to understand, not discordant, disjointed loosely linked,
making sense, Difficult to understand, inconsistent, lack of
incomprehensible unclear, unrelated, organisation/ flow,
distorted, fragmented,
has no meaning, no
logical flow, without
any linkages, inability
to verbalise
eloquently, not
coordinated, not
continuous, not in
accordance
(c) access (Passage Admittance, entry, Avenue, ways, road, Attain, acquisition,
1, line 38) ability to tap into/ gateway, passage, connection, methods,
retrieve, availability entrance, ability to means, usage, reach,
approach use, allowance,
exposure, rights
(d) exponentially In a way that affects disproportionately Alarmingly, very fast,
(Passage 1, line 39) multiple areas/ Unevenly, drastically,
unknown variables/ increase at a
In way that is not compounding rate,
constant/ even dramatically, rapidly,
More than extremely, infinitely,
proportionately/ at an sudden and steep
increasing rate increment
(e) fine-tune (Passage Make minor To improve, refine, Change/ alter,
2, line 45) adjustments to narrow down to sharpen, polish, focus,
achieve stability, to perfection (narrow redefine, to further
make perfect, down is incorrect), develop, adjust
Optimize, refine, modify, adjust properly, sift out the
perfect/ bring to best, manage, correct,
perfection, make narrow down, control
precise

8
National Junior College GP Preliminary Exam 2010

Application Question – 11

Both writers discuss the impact that technology is having on human intelligence.

How persuasive do you find their arguments? In your assessment, how far does technology assist in making your generation smarter? [8]

Examiner‘s comments

To obtain a top mark, candidates must:


 Give systematic and close reference to the passages
 Have well-developed and substantiated arguments, showing the connection between technology and intelligence
 Provide current (and varied) examples that pertain to ‗your generation‘
 Demonstrate the ability to provide in-depth and insightful analysis about the impact and uses of technology, and its implications
 Organize their answer in a thoughtful and effective manner

Possible Arguments

Passage 1
Possible reference/ argument Analysis – is it persuasive?
‗Powerpoint … is reducing potentially *many candidates do not place this quote in context, often writing as if the author himself had made this
intelligent discussion to bullet points.‘ argument. Pinker is trying to debunk these notions so the analysis should follow appropriately.
OR YES – (the claims are unfounded). Simplistic causal relation established (similar to the argument made in
Pinker is critical of these claims lines 24-25). These are presentation tools that engage the reader/ audience. ‗Analysis, criticism‘ (line 36)
are still critical aspects to successful learning/ teaching. Research shows that 60% of people are visual
learners. 80 – 90% of Americans now go to university, up from 70% in the 1990s. The technology boom
does not seem to have adversely affected us.
NO – (technology does have an adverse effect). Pinker tries to debunk these claims by pointing to the
‗dizzying‘ progress of science today. Comparatively – experts in their field of knowledge, like scientists, do
not represent the masses. Surveys of secondary school students in Singapore showed that less than 60%
of them understand the information that they have gathered through online research.
‗develop strategies of self-control‘ YES – Pinker suggests that the key is managing this technology, not eschewing it. Agreed, managing such
‗new‘ technology is much like how the world adapted to the advent of the television, Internet etc. Many
schools today have support programs for youths to deal with cyber-addiction.
NO – Pinker‘s suggestion is too simplistic. Internet-addiction purportedly afflicts 1 in 8 Americans today, as
the Center for Online and Net Addiction reports and cases are increasingly hard to identify as Internet and
related technological usage usually occurs in the confines of the home/ office/
‗these technologies are the only things YES – The information boom today necessitates that we harness technology to keep updated. Every
that will keep us smart‘ minute, 24 hours of video footage is uploaded to YouTube; while a lot of it will never be seen by the vast
population of netizens, this just shows what volume of information is available today. Twitter, Facebook are
not just social networking platforms but also useful ways for us to keep abreast of current affairs. Good

9
National Junior College GP Preliminary Exam 2010

candidates may wish to comment that the claim that technology is the ‘only’ way to stay smart by Pinker is
an exaggeration but not entirely unfounded.
NO – A gross exaggeration Pinker‘s part (use of ‗only‘) as presentation tools and social networking
platforms and SMS may allow us greater access to information and grow our knowledge base, but they do
not fundamentally make us more critical or insightful by their mere existence. The volume of information
also necessitates that we are more discerning of information online – possible examples include,
Censorship Issues, Multiple Perspectives on certain issues

Passage 2
Possible reference/ argument Analysis – is it persuasive?

‗There‘s nothing unnatural about YES – Internet penetration rates are as high as 90% (Norway) and 50% for the Top 30 countries for Internet
relying on the Internet for information‘‗ Penetration (approx for the Worldwide Top 30 2010). Internet is the norm and hence we should embrace it.
OR Students can give examples of IT use in schools today, especially Singapore. A popular example is
‗We need more powerful ways to filter Crescent Girls‘ Secondary School. However, the access to information doesn‘t bring with it better
the information we get online‘ understanding or critical thinking skills, which is why educational institutions around the world are rallying for
boosting students critical thinking skills.
NO – While it is normal these days to rely on the Internet for research, the reality is that a lot of this research
is not carefully done nor critically presented. There have been more cases of plagiarism occurring in schools
and universities. In May this year, Scholastic (a publishing company specializing in educational material)
found itself at the heart of a controversy when a young lady passed off the work of another amateur artist as
her own in a competition for young artists. The artwork went on to win 2 Gold awards. While the matter was
eventually resolved, it did cast the spotlight on the flimsy terms of use in Scholastic‘s Terms of Use
regarding copyright. Clearly, we need much more than methods to filter information, which may not be
accurate, but also to tighten legal measures and netizens understanding of intellectual property.

Passage 1 and 2
Possible reference/ argument Does technology make your generation smarter?
‗PowerPoint … reducing potentially YES – A study by California State University suggests that ‗Gen Y‘ is rewired to multitask better than their
intelligent discussion to bullet points‘ Gen X counterparts. Many people can indeed not multi-task well and at the same time, a whole generation
OR ‗Genuine multitasking… is clearly of youth fundamentally better this way. Technology today is also built around the principle of multi-tasking.
a fiction‘ The oft-cited iPhone, for example, now features an operating system which (in addition to its many other
OR ‗knowledge is increasing appealing features) allows the user to multi-task.
exponentially‘ NO – Platforms like Twitter are inhibiting user‘s ability to express themselves clearly, which is an important
aspect of intelligence. Recently it was found that fewer and fewer young people in Hong Kong are able to
express themselves in written Chinese/ Mandarin because of their reliance on Hanyu Pinyin (a romanized
system of Chinese).

10
NANYANG JUNIOR COLLEGE

JC2 Preliminary Examination 2010

GENERAL PAPER 8806/1

PAPER 1

Friday 3 September 2010

TIME 1 hour 30 minutes

INSTRUCTIONS TO CANDIDATES

Write your name, class and GP Tutor’s name on your answer scripts.

Answer ONE question from Paper 1.

INFORMATION FOR CANDIDATES

In Paper 1, all questions carry equal marks.

[Note that 20 marks out of 50 will be awarded for the use of language.]

This question paper consists of 2 printed pages.


Answer one question from this Paper.

Answers should be between 500 and 800 words in length.

1. Technological advancements always bring about social equality. What is


your view?

2. Movies today are an accurate reflection of the realities in today’s world. To


what extent do you agree?

3. Is violence the only way to end terrorism?

4. To what extent have the Arts in Singapore become commercialised?

5. We are too pragmatic for our own good. Is this true of young people in
your society?

6. Do traditions still have a place in modern Singapore?

7. Is sustainable development a realistic aim in today’s world?

8. There is no longer a clear cultural distinction between the East and the
West. Do you agree?

9. Mind your own business. Is this good advice?

10. Modern society is still intolerant of the different. Is this really the case?

11. To what extent do rules and regulations make our lives better?

12. It is not easy to be a parent today. Comment.

2
NANYANG JUNIOR COLLEGE

JC2 PRELIMINARY EXAMINATION 2010

8806/2
GENERAL PAPER
PAPER 2: Friday 3 September 2010

INSERT 1 hour 30 minutes

READ THESE INSTRUCTIONS FIRST

This insert contains the passages for Paper 2.

______________________________________________________________

This document consists of 4 printed pages.


2

Passage 1 : Martin Jacques writes…

1 No one likes to admit they are racist or bear prejudices. Nor do they even like to
be open and honest when they witness racist behaviour. Look at the Big Brother1
housemates: apart from Shilpa, not one has been prepared to call it by its name
(though Jermaine Jackson, black of course, patently knows and understands). The
fact that hardly anyone is ever prepared to admit to racist behaviour is perhaps a 5
sort of strength: it speaks to the fact that racism is socially inadmissible. But it is
also testimony to profound weakness, a measure of how little distance we have
travelled as a society when it comes to understanding racism. For if the truth be
told, we are a society that is dripping in racism.

2 This is not in the least surprising. For the best part of two centuries, we British 10
ruled the waves, controlled two-fifths of the planet, and believed it was our
responsibility to bring civilisation to those who allegedly lacked it. There is now a
belief that all that is long gone, dead and buried, history forgotten in a tsunami of
amnesia about our past. But these attitudes live on in new forms, constantly
reproduced in each and every white citizen of this country. 15

3 We are not alone in our racism, of course. Every race exhibits racism towards
those whom they believe to be inferior: India is no exception, nor is China, nor is
Africa. What makes Britain - and whites - special in this regard is that we have
been top of the global pile for so long, inflicted our brand of racism on so many,
and have no idea what it is like to be discriminated against for our colour. When it 20
comes to our own racism, we shuffle our feet, fall silent, become incoherent and
pretend it is not true: we don't get it.

4 Racism always exists cheek by jowl with, inside and alongside culture and class.
As a rule it is inseparable from them. That is why, for example, food, language and
names assume such importance in racial prejudice. And that has certainly been 25
the case in Big Brother. Food is a signifier of difference: so are names, so is
language. So Jade and her sidekicks homed in on Shilpa's cooking and choice of
food, made fun of her name and refused to learn it. And with food came the
suggestion that Shilpa's hygiene left something to be desired, that she was
unclean (she had touched the food, it was claimed, and "you don't know where her 30
hands have been"). In other words, not only was she different, but she came from
an inferior civilisation. Her colour too - the most obvious manifestation of racial
difference - was tangentially drawn into the equation through the comment about
make-up and the Indian desire to be white.

5 Of course, class is central. Race always comes with class. Jade's reaction to 35
Shilpa has been shaped by her own class background, her racism articulated
within that context. The fact that Jade is hardly blessed with great intellectual gifts,
that her conversation is littered with profanities, that her behaviour rarely rises
above the crude, lacking any kind of subtlety, and that her status as a former
winner of Big Brother is her only claim to be where she is, makes it easy for the 40
middle class to dismiss her racism as that of a crude, ill-educated, white working-
class young woman, and that the middle classes would, it goes without saying,
never behave in that way.

1
Big Brother is a reality television show in which the participants live together in a large house, isolated from the outside world, but
continuously watched by television cameras. Jade Goody and Shilpa Shetty were among the contestants who were involved in the
controversy relating to the display of racist behaviour during the series.
3

6 Of course, they would, and do: but they practise it in a genteel middle-class kind of
way. Just as Jade's racism has a class intonation, so does theirs - the asides, the 45
put-downs, and the rest. Indeed, in some ways they are more ignorant - while
masquerading as so worldly - because in general they have far less contact with
ethnic minorities, unless they employ them as subordinates and/or domestics.
They live in different areas, work in different places, and send their children to
overwhelmingly white schools. 50

7 Almost from the outset, Big Brother's racism has had a new and novel dimension.
Because Gordon Brown was in India at the time, and was asked about it during his
trip, the issue immediately acquired an international dimension. In an earlier era, of
course, this would have been dismissed as of no consequence: the natives could
safely be ignored. But no longer. We saw this just a year ago in relation to the 55
Danish cartoons and their ridicule of Islam. Europe used to ignore what the former
colonial world felt. There was no feedback loop. But such was the reaction in the
Islamic world that it could not be ignored. That, though, was in the context of the
Muslim world which, in global terms, remains weak and marginalized.

Passage 2 : Brendan O’Neill writes…

1 There have been some disgusting expressions of prejudice in British public life
over the past few days. Foul-mouthed insults have been hurled at a defenceless
woman just because of where she comes from, how she speaks and what she
supposedly represents. Worse, an entire community has been branded as vile,
amoral and corrupt on the basis of this individual woman's flaws and faults. Yes, 5
liberal commentators' assaults on Jade Goody over the past week have been
obscene.

2 The great irony of the "Jade v Shilpa" debacle is that in the same breath that
commentators denounce Jade and her sidekicks Jo O'Meara and Danielle Lloyd
for being prejudiced and ignorant, they express their own ignorant prejudices 10
about entire swathes of people. Jade's idiotic utterance of the phrase "Shilpa
Poppadom" has, unbelievably, been held up as an indictment of the entire white
working class, who have been described as fat, thick, ugly and vile. You almost
get the impression that some people were waiting with bated breath for a moment
like this to arrive, such was the speed with which they unleashed their torrents of 15
abuse against the "underclass". At last, they seemed to think to themselves, we
can attack those ignoramuses while posing as tolerant opponents of prejudice.

3 The snobbish contempt for a whole class of people spread across the
broadsheets, tabloids and blogosphere. Many in the commentariat have written
about Jade the Bermondsey girl (or "slag"), Jo the Essex girl and Danielle the 20
Scouser as the real foreigners, with strange and alien habits, and Shilpa as the
embodiment of their own civilised and dignified values. This is classic snobbery.
During the British colonial era in India, old white-haired colonialists and their racist
wives would often comment on the good behaviour and jolly good sportsmanship
of "the Indians", who were seen as cool and calm and subservient, unlike the 25
4

rowdy, ungrateful and feckless underclass at home, the real "foreign body". There
are echoes of these old contemptuous attitudes today in the elevation of Shilpa
into a token Brit, someone who represents good old-fashioned British middle-class
values of politeness and sophistication, in contrast to the thick white slags from our
own slums who only shame us in the eyes of the world. 30

4 In the fallout from "Jade v Shilpa", it has become clear that the meaning of the
word "racist" has mutated in recent years. Accusations of "racism" are no longer
about indicting someone for their views on ethnic minorities but rather have
become a snobbish judgment on their lack of breeding and etiquette. "Racist" has
become code for "underclass" - uneducated, uncouth, thick, fat, "not one of us". 35
Snobs might once have said "That person lacks breeding"; today they say "She's a
racist". That is another galling element of the Big Brother scandal: the very elite
which, historically, has been responsible for racism and for sustaining and
inflaming racist attitudes - political leaders, the police, the media - have now
redefined racism as something that springs from the most powerless section of 40
society, from the Jade Goodies of this world. Racism is transformed from a top-
down social force into a well of ignorance bubbling over in Bermondsey and other
run-down areas.

5 There is a bitter irony here. Traditionally, racism has not only been about attitudes
towards blacks and Asians. Much racist ideology has its origins in elites' views of 45
the underclass in earlier eras. The underclass were viewed as a "race apart",
inferior to "normal people" in terms of intelligence and breeding. How remarkable
that today, under the cover of attacking racist attitudes, many still cling to that old,
foul view of the "underclass", which is itself racial in origin.
NANYANG JUNIOR COLLEGE
JC 2 PRELIMINARY EXAMINATION 2010

Name: _______________________ CT: _____ GP Tutor: ______________________

GENERAL PAPER 8806/2


PAPER 2: Friday 3 September 2010

1 hour 30 minutes
Candidates answer on the Question Paper.
No Additional Materials are required.

ANSWER BOOKLET

READ THESE INSTRUCTIONS FIRST

Write your Name, Class, and GP Tutor’s Name on all the work you hand in.
Write in dark blue or black pen in the spaces provided on the Question Paper.
Do not use staples, paper clips, highlighters, glue or correction fluid.

Answer all questions.


At the end of the examination, fasten all your work securely together.
The number of marks is given in brackets [ ] at the end of each question or part question.
The insert contains the passage for comprehension.
(Note that 15 marks out of 50 will be awarded for your use of language.)

For Examiner’s Use

Content /35

Language /15

Total / 50

________________________________________________________________

This document consists of 8 printed pages.


2

Read the passages in the Insert and then answer all the questions. Note that up to fifteen
marks will be given for the quality and accuracy of your use of English throughout this
Paper.

NOTE: When a question asks for an answer IN YOUR OWN WORDS AS FAR AS
POSSIBLE and you select the appropriate material from the passage for your answer, you
must still use your own words to express it. Little credit can be given to answers which only
copy words or phrases from the passages.

From Passage 1 For


Examiner’s
Use
1 a) Why does the author place the phrase ‘though Jermaine Jackson, black of course,
patently knows and understands’ (line 4) in brackets or parenthesis?

___________________________________________________________________________

____________________________________________________________________

___________________________________________________________________[1]

1 b) Explain the author’s use of the phrase ‘of course’ (line 4).

____________________________________________________________________

____________________________________________________________________[1]

2 a) ‘The fact that hardly anyone is ever prepared to admit to racist behaviour is
perhaps a sort of strength …it is also testimony to profound weakness’ (lines 4-7).

Explain the paradox in the quotation.

____________________________________________________________________

____________________________________________________________________

____________________________________________________________________

___________________________________________________________________ [2]

2 b) What does ‘perhaps’ (line 5) say about his conclusion?

____________________________________________________________________

____________________________________________________________________[1]
3

3 ‘…we are a society that is dripping in racism.’ For


Explain why the author uses the word ‘dripping’ (line 9). Examiner’s
Use

_____________________________________________________________________

____________________________________________________________________[1]

4 What does the expression ‘…believed it was our responsibility to bring civilization
to those who allegedly lacked it’ (lines 11 – 12) tell you about the author’s opinion
of the British conquerors and their justification?

___________________________________________________________________

___________________________________________________________________

___________________________________________________________________

__________________________________________________________________ [2]

5 Using material from paragraphs 4 to 6 of Passage 1, summarise the ways in


which racism exhibits itself and the reasons for its manifestations.

Write your summary in no more than 120 words, not counting the opening words
which are printed below. Use your own words as far as possible.

Racism exhibits itself through ___________________________________________

_____________________________________________________________________

_____________________________________________________________________

_____________________________________________________________________

_____________________________________________________________________

_____________________________________________________________________

_____________________________________________________________________

_____________________________________________________________________

_____________________________________________________________________

____________________________________________________________________

____________________________________________________________________
4

____________________________________________________________________
For
Examiner’s
____________________________________________________________________ Use

____________________________________________________________________

____________________________________________________________________

____________________________________________________________________

____________________________________________________________________

____________________________________________________________________

____________________________________________________________________

____________________________________________________________________

____________________________________________________________________

____________________________________________________________________

_____________________________________________________________________ [8]

6 In paragraph 7, why is it difficult for the Western world to ignore reactions from the
old colonial world? Use your own words as far as possible.

_____________________________________________________________________

_____________________________________________________________________

_____________________________________________________________________

_____________________________________________________________________[2]

7 What point does the author want to make when he says ‘That, though, was in the
context of the Muslim world which in global terms, remains weak and
marginalized’ (lines 58 – 59)?

____________________________________________________________________

____________________________________________________________________

_____________________________________________________________________[1]
5

From Passage 2 For


Examiner’s
8 Explain what makes the author conclude that ‘people were waiting with bated Use

breath for a moment like this to arrive’ (lines 14 – 15). Use your own words as
far as possible.

____________________________________________________________________

____________________________________________________________________

____________________________________________________________________

____________________________________________________________________[2]

9 ‘…the meaning of the word “racist” has mutated in recent years’ (lines 31 – 32).

What does it suggest about human nature?

____________________________________________________________________

___________________________________________________________________ _[1]

From both passages

10 Give the meaning of the following words as they are used in the passages.
Use one word or a short phrase.

a. patently (Passage 1,line 4) _______________________________________

b. masquerading (Passage 1, line 47) _________________________________

c. branded (Passage 2, line 4) ______________________________________

d. token (Passage 2, line 28) ________________________________________

e. cover (Passage 2, line 48) ________________________________________

[5]
6

For
11. Jacques argues that racism is exhibited by every race and O’Neill argues that Examiner’s
racism can take on different forms. To what extent has racism affected your Use

country? How are these experiences similar to, or different from those discussed
in both passages?

___________________________________________________________________

___________________________________________________________________

___________________________________________________________________

___________________________________________________________________

___________________________________________________________________

___________________________________________________________________

___________________________________________________________________

___________________________________________________________________

___________________________________________________________________

___________________________________________________________________

___________________________________________________________________

___________________________________________________________________

___________________________________________________________________

___________________________________________________________________

___________________________________________________________________

___________________________________________________________________

___________________________________________________________________

___________________________________________________________________

___________________________________________________________________

___________________________________________________________________

___________________________________________________________________

___________________________________________________________________

___________________________________________________________________
7

For
___________________________________________________________________ Examiner’s
Use
___________________________________________________________________

___________________________________________________________________

___________________________________________________________________

___________________________________________________________________

___________________________________________________________________

___________________________________________________________________

___________________________________________________________________

___________________________________________________________________

___________________________________________________________________

___________________________________________________________________

___________________________________________________________________

___________________________________________________________________

___________________________________________________________________

___________________________________________________________________

___________________________________________________________________

___________________________________________________________________

___________________________________________________________________

___________________________________________________________________

____________________________________________________________________

____________________________________________________________________

____________________________________________________________________

____________________________________________________________________

____________________________________________________________________

____________________________________________________________________
8

____________________________________________________________________
For
Examiner’s
____________________________________________________________________ Use

____________________________________________________________________

___________________________________________________________________

___________________________________________________________________

___________________________________________________________________

___________________________________________________________________

___________________________________________________________________

___________________________________________________________________

___________________________________________________________________

___________________________________________________________________

___________________________________________________________________

___________________________________________________________________

___________________________________________________________________

___________________________________________________________________

___________________________________________________________________

___________________________________________________________________

___________________________________________________________________

___________________________________________________________________

___________________________________________________________________

___________________________________________________________________

___________________________________________________________________

___________________________________________________________________

____________________________________________________________________ [8]

- End of Paper -
Answer Scheme (NYJC Preliminary Exam 2010)

Passage 1

Para 1
1. a) Why does the author place the phrase „though Jermaine Jackson, black of course, patently
knows and understands‟ (line 4) in brackets or parenthesis? [1]

b) Explain the author‟s use of the phrase „of course‟ (line 4). [1]

Lift Inferred
a Note the functions of parenthesis Racist behaviour against blacks is so obvious
that the information is almost non-
essential.(1m)
OR
To emphasise that racism is an obvious fact,
as experienced by Blacks, yet no one wants to
acknowledge it. (1m)
b. „Black of course…‟ To emphasise that the colour of one‟s skin
(being black) would surely be the reason for
being the target of racist behaviour. (1m)

2. „The fact that hardly anyone is ever prepared to admit to racist behaviour is perhaps a sort of
strength …it is also testimony to profound weakness‟ (lines 4-7).
a) Explain the paradox in the quotation. [2m]

Lift Inferred
a „The fact that hardly anyone is ever The very fact that no one wants to say they are
prepared to admit to racist behaviour is racist reflects the common belief that racism
perhaps a sort of strength: it speaks to goes against societal norms and this is a
the fact that racism is socially positive quality. (1)
inadmissible. Yet/on the other hand this denial also suggests
It is also testimony to profound weakness, the little progress society has made/reluctance
a measure of how little distance we to comprehend the problem of racism. (1)
have travelled as a society when it
comes to understanding racism.’

Bonus: Contradiction – Strength – a positive quality, yet it is misleading/deceptive because this


denial, in reality, is a lack of progress made in understanding the problem racism.

b) What does „perhaps‟ (line 5) say about his conclusion? [1m]

Lift Inferred
b. His conclusion is debatable/ controversial.(1m)

Bonus: If students are able to identify the author‟s intention to appear to accommodate the other
side of the argument when he actually believes in the opposite. (1)

3. „…we are a society that is dripping in racism.‟


Explain why the author uses the word „dripping‟ (line 9). [1]

Lift Inferred
„…we are a society that is dripping in racism‟ To show that there is excessive/too much
racism in our society.(1m)

Para 2
4. What does the expression „…believed it was our responsibility to bring civilization to those who
allegedly lacked it‟ (lines 11-12) tell you about the author‟s opinion of the British conquerors
and their justification? [2]
Lift Inferred
1. Author‟s opinion : keyword „allegedly‟ That they are arrogant/presumptuous (1m)

2. The justification The so-called noble reason of wanting to bring


civilization to the conquered was only an
excuse for their greed and ambition.(1m)

5. Using material from paragraphs 4 to 6 of Passage 1, summarise the ways in which racism
exhibits itself and the reasons for its manifestations. Write your summary in no more than 120
words, not counting the opening words which are printed below. Use your own words as far as
possible.[8]

Racism exhibits itself through…

Lift Paraphrase
Ways in which racism exhibits itself
1. Food, what we eat/diet (1/2m)
2. language speak/ (1/2m)
3. and names) and call ourselves/names (1/2m)

Note:
(Accept lift for “names”)
Any 2 out of 3 will merit 1m
„Culture‟ considered as a lift.
‘Way of life’ will merit 1m
4. …assume such importance in racial prejudice All of which play a crucial role in race
(l 25) discrimination (1m)
5. Why They are distinguishing
Food is a signifier of difference: so are characteristics/traits of culture/our way
names…language (I 26-27) of life. (1m)
6. … she came from an inferior civilisation. (l And they reflect a society’s level of
31-32) development/advancement.(1m)
7. Ways Skin tone/complexion is also a clear
Her colour too…obvious manifestation of indication/exhibition/mark of racial
racial difference. (I 32-33) difference. (1m)
8. Why Undoubtedly, one’s social
Class is central (l 35) stratum/hierarchy/rank is
…white working-class young woman (l 41-42) key/fundamental to racism(1m)
9. As a rule, it is inseparable (l 24) because race and one’s social
Race always comes with class (l 35) stratum/hierarchy/rank are
indivisible/co-exist. (1m)
10. For the working class, The working class exhibits racism
conversation littered with profanities… (l through vulgar speech/language and
38) actions, (1m)
behaviour rarely rises above the crude,
lacking any kind of subtlety… (l 38-39)
…crude (l 41)
11. …ill-educated (l 41) a result of poor upbringing. (1m)
12. For the middle class, The middle class
practise it in a genteel middle-class kind of displays/exhibits/shows it in a
way (l 44-45) sophisticated way through refined
the asides, the put-downs and the rest (l 45- linguistic/verbal cues/signals. (1m)
46)
13. …ignorant…have far less contact with Their racism is caused by a lack of
ethnic minorities (l 46-48) knowledge and interaction with
smaller racial groups. (1m)
14 They live in different areas, work in This is evident in the
different places, and send their children to deliberate/conscious practice of
overwhelmingly white schools. (l 49-50) segregation/ segregation by choice/ a
conscious decision to segregate from
other ethnic groups. (1m)

Racism exhibits itself through what we eat, speak and call ourselves, all of which play a crucial
role in race discrimination. They are the distinguishing characteristics of our way of life and they
reflect a society‟s level of development. Skin tone is also a clear indication of racial difference.
Undoubtedly, one‟s social stratum is fundamental to racism because race and social hierarchy are
indivisible. The working class exhibits racism through vulgar speech, a result of poor upbringing.
The middle class displays it in a sophisticated way through refined linguistic cues. Their racism is
caused by a lack of knowledge and interaction with smaller racial groups. This is evident in the
conscious practice of segregation.(110 words)

Para 7
6. In paragraph 7, why is it difficult for the Western world to ignore reactions from the old colonial
world? Use your own words as far as possible.[2]

Lift Paraphrase/Inference
In an earlier era…dismissed as of no Disregarding the reactions from the old colonial
consequence: the natives could safely be world would bring about dire/negative/serious
ignored. But no longer. consequences/impact (1m) (as witnessed in
We saw this just a year ago in relation to the the furore over the Danish cartoons and their
Danish cartoons and their ridicule of Islam. ridicule of Islam)
Europe used to ignore what the former colonial
world felt. There was no feedback loop. because the Western world would receive
criticisms/disapproval for doing so(1m)

7. What point does the author want to make when he says „That, though, was in the context of the
Muslim world which in global terms, remains weak and marginalized‟ (lines 58 -59)? [1]

Inferred
But such was the reaction in the Islamic world That if it involved a stronger and a more unified
that it could not be ignored. That, though... group, the impact/reaction towards such sensitive
issues as racism may be more serious and
widespread (1m)

Passage 2
Para 2
8. Explain what makes the author conclude that „people were waiting with bated breath for a
moment like this to arrive‟ (lines 14 – 15)? Use your own words as far as possible.[2]

Lift Paraphrase
…such was the speed with which they He concluded thus as he observed how
unleashed their torrents of abuse against quickly/rapidly (1m) they let loose/let go of control
the “underclass”. (1m) their pent up/ overwhelming/a barrage of
criticism/ condemnation/insults of the
„underclass‟. (1m)

Any 2 out of 3

Para 4
9. „…the meaning of the word “racist” has mutated in recent years‟ (lines 31-32).
What does it suggest about human nature? [1]

Lift Inferred
…it has become clear that the meaning of the Prejudice is inherent (1m) as it is always directed
word "racist" has mutated in recent years. at weakest part of society.
Accusations of “racism”…have become a (There is only a change of views about the
snobbish judgement on their lack of minorities.)
breeding and etiquette…„racist has become
a code for underclass’
…the very elite…have now redefined racism
as something that springs from the most
powerless section of society
Racism is transformed…a well of ignorance
bubbling over in Bermondsey and other run-
down areas

10. Vocabulary [5]

1 mark ½ mark 0 mark


a patently(P1,l 4) Evidently/obviously
Jermaine
Jackson, black of
course, patently
knows and
understands
b masquerading(P Pretending, putting on
1, l 47) a front
…they are more
ignorant – while
masquerading
as so worldly…
c Branded(P2, l 4) Labelled/marked
An entire
community has
been branded as
vile…
d token(P2, l 28) A symbol of/indication
elevation of of/ representation
Shilpa into a
token Brit
e cover(P2, l 48) Pretext/appearance
under the cover
of attacking racist
attitudes

11. Jacques argues that racism is exhibited by every race and O‟Neill argues that racism can take
on different forms. To what extent has racism affected your country? How are these
experiences similar to or different from those discussed in both passages.

R1: Assess the extent to which racism has affected your country.
R2: Compare the similarities and/or differences between the ideas discussed in both passages and
your country‟s experiences.
PIONEER JUNIOR COLLEGE
JC 2 PRELIMINARY EXAMINATION

GENERAL PAPER 8806/01


Higher 1

Paper 1 3 September 2010


1 hour 30 minutes

Additional Materials: Answer Paper

READ THESE INSTRUCTIONS FIRST

Write your name, CT group and GP Tutor’s name on all the work you hand in.
Write in dark blue or black pen on both sides of the paper.
Do not use staples, paper clips, highlighters, glue or correction fluid.

Answer one question.


Note that 20 marks out of 50 will be awarded for your use of language.

At the end of the examination, fasten all your work securely together.
All questions in this paper carry equal marks.

This document consists of 2 printed pages.

PJC 2010

[Turn over]
PAPER 1

Answer one question from this Paper.

Answers should be between 500 and 800 words in length.

1. ‘Technology will increasingly allow people to work from home.’


Is this desirable?

2. Do children really have rights in society?

3. ‘Our health is so important, yet we pay so little attention to it.’


Do you agree?

4. ‘A sense of humour is a valuable asset.’ Comment.

5. ‘A vibrant global city.’ Is this an apt description of your country today?

6. Consider the view that the study of another language is not necessary when
English is so widely used today.

7. To what extent does the media dictate our way of life?

8. Is there any value in popular culture?

9. Are monuments and memorials a waste of a country’s budget?

10. Are the concerns about international migration justified?

11. ‘Environmental awareness is changing the lifestyle of the people in your


country.’ To what extent do you agree with this view?

12 Is a world without nuclear weapons an impossible dream?

2
Passage 1

1 Why are some places prone to war? Iraq has deeply confused how people think about
twenty-first-century war. The war in Iraq is not a guide to the future; it is a rerun of a
phase in world history that is essentially over. Iraq started with an international invasion.
So did the two world wars, Napoleon’s wars, the Crimean War and the other great set
pieces of military history. In the twenty-first century, international invasions are going 5
to be infrequent. The wars that will fill our television screens this century will be civil
wars, not international ones. Of course, there were civil wars in the nineteenth century
as well as invasions, but even the civil wars of that era are a hopeless guide to what
warfare will be like. The major civil war of the nineteenth century was the American
Civil War where one alliance of states fought against another alliance of states, each 10
with its own recognised territory, government and army. It’s history.

2 Future civil wars will take the form of a government pitted against a private extra-legal
military grouping. They will variously be called rebels, terrorists, freedom fighters or
gangsters, but their essential characteristic will be the same. These wars will also be a
throwback, but to a different period of history: the time before nation-states cohered. 15

3 To rephrase the question, why are some countries more at risk of civil war than others?
If we could answer the question, we might be able to do something about it: some of the
factors that elevate the risk of civil war might be things that could readily be put right.

4 The economy matters. Low-income countries are significantly more at risk: poor is
dangerous. It is not just the level of income: it is also the rate of growth. Given the level 20
of income, societies that are growing at a faster per capita are significantly less at risk of
violent conflict than societies that are stagnant or in decline. Economic development is
peace-promoting. The statistics of the world post-1960 are supported by the deep
historical evidence of the societies of early history. Economic development is a key
remedy to violence: jobs, education, hope, a sense of having something to lose, and 25
more effective state security services, all of which contribute something.

5 Besides the economy, a country’s history also increases risks. The aspect of a country’s
history that most commonly excites interest when it comes to explaining a civil war is
its colonial experience. Understandably, many people in developed countries find it
convenient to emphasise the guilt of their own societies, and equally, many people in 30
developing countries want to avoid any impression that the violence of their societies is
a consequence of characteristics within those societies. But blaming colonialism for
civil war is a costly illusion because it detracts from the focus on its real causes, which
are often things that can still be changed. It may make people feel better, but it inhibits
action. 35

6 The cost of violence is enormous. Even if it is to lead to healthy change, we need to ask
whether the eventual benefits are worth these massive costs. But the final tragedy of
civil war is that it leads not to improvement but to deterioration.

Adapted from ‘Wars, Guns and Votes’ by Paul Collier

2
Passage 2

1 Everyone knows the central philosophical question which the human race has been
unable to answer: “Why is there violence rather than non-violence?” When once asked
if non-violent resistance was a form of “direct action”, Gandhi replied: “... It is the only
form.” It was the “greatest force...” Gandhi believed non-violence could be put into
practice at every level of human experience. Non-violence for him was not just a 5
political tactic but an exercise in spirituality and a way of life.

2 It is revealing that in a world where there are calamities such as terrorism, poverty,
illiteracy and fanaticism, history can still be made out of choices. The choice of non-
violence is ours. We live in a world of “overlapping destinies” where the fates of
cultures are heavily intertwined. It is no longer a world of closed communities where 10
tyrannical orders or religious traditions represented the sole layers of historical
legitimacy. Never in the history of the human race has non-violence been so crucial.
Only the most barbaric and despotic regimes, however, have attempted to prevent their
subjects to think and to practise non-violence.

3 Non-violence has recently evolved from a simple tactic of resistance to a 15


cosmopolitical1 aim based on international application of the principles of democracy.
Over the past three decades, global terrorism, violation of human rights and
environmental degradation have caused repercussions highlighting the concern for
global politics of non-violence. These can best be dealt with at the global level. Global
politics of non-violence, thus, is the task not only of governments but also of civil 20
societies, and inter-governmental, non-governmental and transnational organisations.
Most importantly, the international community has the moral obligation and duty to
intervene in countries if they slide into lawlessness and can’t protect citizens from
violations of human rights. Only a non-violent society can work its way up to creating
the institutions ripe for development and lead to inter-cultural and inter-religious 25
harmony.

4 We are living today in an era where social, cultural and political spheres are void of
spirituality. But Gandhi’s non-violence still offers us an ideal that may be upheld.
Gandhi remains the prophetic voice of the 21st century and his non-violence urges us to
continue struggling on behalf of what we view as just and right. At a time when 30
mankind is confronted with clashes of national interest, religious fundamentalism and
ethnic and racial prejudice, non-violence can be a well-trusted means of laying the
groundwork of new cosmopolitics. Though many continue to believe that non-violence
is an ineffective instrument against dictatorship and genocide, in the last several decades
many democratic initiatives, which were premised on non-violent militancy and an 35
affirmation of human rights and helped build a global civil society on solid ethical
foundations, could be associated with a kind of neo-Gandhian quest for peace and
justice.

1
cosmopolitical – a structure that encompasses a diversity of political systems

3
Passage 2

5 It would be folly to expect non-violence to become effective and durable, while the
majority still thinks of politics in terms of the use of violence. It is true, as Karl Jaspers 40
affirms: “In morality, moral conviction is decisive; in politics, it is success.” But it is
also true there is no long-term success in politics in the absence of morality. That is to
say, political events bring moral responsibilities, and in turn ethical views place their
imprint on political decisions. Politics without ethics is pure exercise of power. It is only
in relation to ethics that politics can be elevated as a public virtue. Terrible crimes have 45
been committed by political practice that tried to teach and impose moral behaviour.
Spiritualising politics, as Gandhi understood, is not about moralising it, but an effort to
redefine it in terms of civic responsibility in an explicit public sphere. Politics is the
morally conscientious and socially responsible exercise of civic roles. Non-violence is
the key to this. Violence is normally seen as a means to an end. Very few people desire a 50
world with war, crime and terrorism. But  in  non-violent  politics,  non-violence is  the 
means  and  non-violence  is  the  goal.

6 The time has come for humanity to renew its commitment politically, economically and
culturally to  the  wisdom  of  non-violence.

Adapted from ‘Non-violence in the 21st century: A matter of choice’ by


Ramin Jahanbegloo

4
PIONEER JUNIOR COLLEGE
JC2 PRELIMINARY EXAMINATION

GENERAL PAPER 8806/2


Higher 1

Paper 2 3 September 2010


1 hour 30 minutes

ANSWER BOOKLET

Candidate’s Name: ______________________________ CT Group: _____________

Index Number: _________________________________ GP Tutor: ______________

READ THESE INSTRUCTIONS FIRST

Write your name, CT group and GP tutor’s name on every sheet of paper that you hand in.
Write in dark blue or black pen on both sides of the paper.
Do not use staples, paper clips, highlighters, glue or correction fluid.

Answer all questions in the Answer Booklet.


Note that 15 marks out of 50 will be awarded for your use of language.

The number of marks is given in the brackets [ ] at the end of each question or each part
question.

For Examiner’s Use

Content /35

Language /15

TOTAL /50

This document consists of 6 printed pages and 1 cover page.

© PJC 2010
[Turn over]
Read the passages in the insert and then answer all the questions which follow below. For
Note that up to fifteen marks will be given for the quality and accuracy of your use of examiner's
English throughout this paper. use

NOTE: When a question asks for an answer IN YOUR OWN WORDS AS FAR AS
POSSIBLE and you select the appropriate material from the passages for your
answer, you must still use your own words to express it. Little credit can be
given to answers which only copy words or phrases from the passages.

Questions on Passage 1

1. Explain what the author means by “The wars that will fill our television screens this
century will be civil wars …” (lines 6-7).

………………………………………………………………………………………………………………………..

…………………………………………………………………………………………………………………....... [1]

2. What does the author intend you to understand by the statement “It’s history.” at the
end of the first paragraph?

……………………………………………………………………………………………………………............

………………………………………………………………………………………………………………………. [1]

3. “They will variously be called rebels, terrorists, freedom fighters or gangsters, but their
essential characteristic will be the same.” (lines 13-14)

What essential characteristic is the author thinking of here?

……………………………………………………………………………………………………………………… [1]

4. Explain how, according to paragraph 2, future civil wars will differ from those of the
nineteenth century. Use your own words as far as possible.

……………………………………………………………………………………………………………………..

……………………………………………………………………………………………………………………..

……………………………………………………………………………………………………………………..

…………………………………………………………………………………………………………………….. [2]

5. In paragraph 4, why does the author say that the “… poor is dangerous” (lines 19-20)?

………………………………………………………………………………………………………………………

……………………………………………………………………………………………………………………… [1]

2
6. Using your own words as far as possible, explain the danger of “… blaming For
colonialism for civil war…” (lines 32-33). examiner's
use
………………………………………………………………………………………………………………………

………………………………………………………………………………………………………………………

………………………………………………………………………………………………………………………

……………………………………………………………………………………………………………………… [2]

Questions on Passage 2

7. From paragraph 1, explain how the writer thinks we can be advantaged by Gandhi’s
approach of non-violence.

………………………………………………………………………………………………………………………

………………………………………………………………………………………………………………………

……………………………………………………………………………………………………………………… [2]

8. Explain why you would disagree with the writer’s use of quotation marks round
“overlapping destinies” (line 9).

………………………………………………………………………………………………………………………

……………………………………………………………………………………………………………………… [2]

9. The writer recognises that many people may doubt the effectiveness of Gandhi’s
approach of non-violence. Identify two words from paragraph four and explain how
each of them reflects this scepticism.

…………………………………………………………………………………………………………………….

…………………………………………………………………………………………………………………….

…………………………………………………………………………………………………………………….

……………………………………………………………………………………………………………………. [2]

3
10. Summarise the writer’s rationale for using non-violence as an effective strategy in For
building civil societies. examiner's
use
Using material from paragraphs 4 and 5, write your summary in no more than 120
words, not counting the opening words which are printed below. Use your own
words as far as possible.

Non-violence is effective in building civil societies because …………………………………

……………………………………………………………………………………………………………………..

……………………………………………………………………………………………………………………..

……………………………………………………………………………………………………………………..

……………………………………………………………………………………………………………………..

……………………………………………………………………………………………………………………..

……………………………………………………………………………………………………………………..

……………………………………………………………………………………………………………………..

……………………………………………………………………………………………………………………..

……………………………………………………………………………………………………………………..

……………………………………………………………………………………………………………………..

……………………………………………………………………………………………………………………..

……………………………………………………………………………………………………………………..

……………………………………………………………………………………………………………………..

……………………………………………………………………………………………………………………..

……………………………………………………………………………………………………………………..

……………………………………………………………………………………………………………………..

……………………………………………………………………………………………………………………..

……………………………………………………………………………………………………………………..

……………………………………………………………………………………………………………………..

……………………………………………………………………………………………………………………..

……………………………………………………………………………………………………………………..

…………………………………………………………………………………………………………………….. [8]

4
For
Questions on Passages 1 and 2 examiner's
use
11. Give the meaning of the following words as they are used in Passage 1 and Passage 2.
You may write your answer in one word or a short phrase.

From Passage 1:

(i) elevate (line 18) ………………………………………………………………………………………..

(ii) understandably (line 29) …………………………………………………………………………….

(iii) detracts (line 33) ……………………………………………………………………………………….

From Passage 2:

(iv) despotic (line 13) ………………………………………………………………………………………

(v) repercussions (line 18) .…………………………………………………………………………... .. [5]

12.Both passages deal with the importance of promoting peace to help build a global
civil society.

How far do you agree with the writers’ views? Using material from both passages,
discuss how your view has been challenged or confirmed by these passages. You
must also rely on your own ideas and knowledge of today’s world.

……………………………………………………………………………………………………………………

……………………………………………………………………………………………………………………

……………………………………………………………………………………………………………………

……………………………………………………………………………………………………………………

……………………………………………………………………………………………………………………

……………………………………………………………………………………………………………………

……………………………………………………………………………………………………………………

……………………………………………………………………………………………………………………

……………………………………………………………………………………………………………………

……………………………………………………………………………………………………………………

……………………………………………………………………………………………………………………

……………………………………………………………………………………………………………………

5
For
…………………………………………………………………………………………………………………… examiner's
use
……………………………………………………………………………………………………………………

……………………………………………………………………………………………………………………

……………………………………………………………………………………………………………………

……………………………………………………………………………………………………………………

……………………………………………………………………………………………………………………

……………………………………………………………………………………………………………………

……………………………………………………………………………………………………………………

……………………………………………………………………………………………………………………

……………………………………………………………………………………………………………………

……………………………………………………………………………………………………………………

……………………………………………………………………………………………………………………

……………………………………………………………………………………………………………………

……………………………………………………………………………………………………………………

……………………………………………………………………………………………………………………

……………………………………………………………………………………………………………………

……………………………………………………………………………………………………………………

……………………………………………………………………………………………………………………

……………………………………………………………………………………………………………………

……………………………………………………………………………………………………………………

……………………………………………………………………………………………………………………

……………………………………………………………………………………………………………………

……………………………………………………………………………………………………………………

……………………………………………………………………………………………………………………

……………………………………………………………………………………………………………………

……………………………………………………………………………………………………………………

……………………………………………………………………………………………………………………

6
For
…………………………………………………………………………………………………………………… examiner's
use
……………………………………………………………………………………………………………………

……………………………………………………………………………………………………………………

……………………………………………………………………………………………………………………

……………………………………………………………………………………………………………………

……………………………………………………………………………………………………………………

……………………………………………………………………………………………………………………

……………………………………………………………………………………………………………………

……………………………………………………………………………………………………………………

……………………………………………………………………………………………………………………

……………………………………………………………………………………………………………………

……………………………………………………………………………………………………………………

……………………………………………………………………………………………………………………

……………………………………………………………………………………………………………………

……………………………………………………………………………………………………………………

……………………………………………………………………………………………………………………

……………………………………………………………………………………………………………………

……………………………………………………………………………………………………………………

……………………………………………………………………………………………………………………

……………………………………………………………………………………………………………………

……………………………………………………………………………………………………………………

……………………………………………………………………………………………………………………

……………………………………………………………………………………………………………………

……………………………………………………………………………………………………………………

…………………………………………………………………………………………………………………… [8]

End Of Paper

7
8
Blank Page

9
10
PJC
JC2 Preliminary Examination 2010
GP (Paper 2)
Suggested Answers

Questions on Passage 1
1. Explain what the author means by “The wars that will fill our television screens this
century will be civil wars …” (lines 6-7). [1]

Line Lifted Inferred


None  Wars should they occur will be internal
in nature/ internal strife/conflict(1/2m)
 When they occur, they will be given
media coverage (1/2)

(can lift “civil wars”= 1/2m)

2. What does the author intend you to understand by the statement “It‟s history” at the end of
the first paragraph? [1]

Line Lifted Inferred


5-6 In the twenty-first century, There will no longer (1/2)
international invasions are going
to be infrequent. ..(L 11) …It‟s
history
be wars where a country or a group of countries
6-7 The wars … this century will each with its own land, lawful authority and
be… not international ones. military forces, fight one another. (1/2m)

10-11 …each with its own recognised


territory, government and army.

OR

The conflict( between different countries/


international wars (allow lift)=1/2m), each
with its own land, lawful authority and military
forces, (1/2) would be a thing of the past. (1/2m)

3. “They will variously be called rebels, terrorists, freedom fighters, or gangsters, but their
essential characteristic will be the same.” (lines 13-14)

What essential characteristic is the author thinking of here? [1]


Line Lifted Inferred
12-13 … pitted against a private extra-legal  Government opposing/ anti-government
military grouping. (1m)
4. Explain how, according to paragraph 2, future civil wars will differ from those of the
nineteenth century. Use your own words as far as possible. [2]

Line Lifted Paraphrased


12 - a government pitted against a  In the 21st century, violent
13 private extra-legal military unrest/disturbances/ fighting will be
grouping. between a group of people/lawful
authority governing a country(1/2) and a
secret military force/ armed forces not
regulated or sanctioned by law (1/2m)

15 but to a different period of  In the 21st century, fighting will take place
history: the time before nation- between different groups/states within a
states cohered. country(1/2) because of differences/ lack
of cooperation between them.(1/2m).

*Award 1m for lack of cooperation even when


„fighting will take place between different
groups/states within a country‟ is not stated.

5 . In paragraph 4, why does the author say that the “… poor is dangerous” (lines 19-20)? [1]

Line Lifted Inferred


21-22 societies that are growing at a  When the poor cannot afford things due
faster per capita are significantly to low salaries/ lack of economic
less at risk of violent conflict growth/ for a better life/ suffer from low
than societies that are stagnant or income (1/2),
in decline.

24-26 Economic development is a key


remedy to violence: jobs,
education, hope, a sense of
having something to lose, and
more effective state security
services, all of which contribute
something.

 they try to secure them by fighting others


/ going to war /civil war/ causing violent
conflicts. (1/2)

6. Using your own words as far as possible, explain the danger of “… blaming colonialism for civil
war…” (line 32 -33). [2]

Line Lifted Paraphrased


33 … costly illusion  Disadvantaged/ taking a huge risk /
disastrous / detrimental (1/2)
…it detracts
 as they are deluded /gives them a false
sense of security/assurance that others are
responsible for their plight (1/2)
from the focus on its real causes
 When they do not address the actual
35 it inhibits action reasons for their problem / they do not
have to do anything about the situation
/By ignoring/neglecting the actual cause
of the conflict (1/2)
34-35 which are often things that can
still be changed  which can be altered/controlled/ they
avoid adopting the necessary measures
which can prevent conflicts.(1/2)

Questions on Passage 2

7. From paragraph 1, explain how the writer thinks we can be advantaged by Gandhi’s
approach of non-violence. [2]

Line Lifted Paraphrased or inferred


4-6 Gandhi believed non-violence  Non-aggressive methods/ an ethos of
could be peaceful ways

put into practice at every level of  Can be implemented in all aspects of our
human experience. life/ as our culture/ widespread (1m)
and a way of life.

Non-violence for him was not  serves not only to help us settle political
just a political tactic differences (1m) political strategy/
government technique (1/2m)
but spirituality
 but also to make us more morally upright
/inner peace/ inner calm/ haven for the
soul/ go beyond worldly existence/ go
beyond material/ physical world/ mental
and emotional well-being (1)

calm/peace (1/2m)

anything to do with religion/god (0)


happiness/ inner happiness (0)
Any 2=2m

8. Explain why you would disagree with the writer‟s use of quotation marks round “overlapping
destinies” (line 9). [2]

Line Lifted Inferred


9 -12 We live in a world of  The use of quotation marks would usually
“overlapping destinies” where indicate a contradiction (1/2m)
the fates of cultures are heavily
intertwined.  however, there is none here. (1/2m) ,
It is no longer a world of closed
communities where tyrannical  The “overlapping destinies” are explained
orders or religious traditions as “ no longer a world of closed
represented the sole layers of communities” / interconnected, (1/2)
historical legitimacy
 and war in one country would affect
others hence, they should not resort to
brutal force /fight to settle differences or
achieve certain objectives (1/2m)

9. The writer recognises that many people may doubt the effectiveness of Gandhi‟s
approach of non-violence. Identify two words from paragraph four and explain how
each of them reflects this scepticism. [2]

Line Lifted Paraphrased /Inferred


Award ‘0’if words are identified without
explanation given.
28 But Gandhi‟s non-violence still  „Ideal‟ refers to something perfect
offers us an ideal that may be (1/2)and so the practice of non-violence
upheld. may not be achievable (1/2m)
 „may‟ or „could‟ implies uncertainty/
doubt/ probability (1/2) that non-violence
will work.(1/2)

„prophetic ‟= correctly saying what will


happen in the future (0)

10. Summarise the writer’s rationale for using non-violence as a strategy to achieve peace
and justice.
Using material from paragraphs 4 to 5, write your summary in no more than 120 words,
not counting the opening words which are printed below. Use your own words as far
as possible. [8]

Non-violence is effective in building civil societies because ……………………..

Line Lifted Paraphrased or inferred


1 27-28 living today in an era where  most people today who being secular
social, cultural and political in their thinking/ atheists/ do not
spheres are void of spirituality have religious/moral values to
govern/ guide the way/ lead their
lives may turn to aggression to settle
their differences (1)

2 28 offers us an ideal …  provides us a vision/ noble goal to


aspire towards
3 29-30 his non-violence urges us to  Gandhi‟s approach of non-violence
continue struggling on behalf encourages us to persevere (1/2)and
of what we view as right and uphold what we perceive as correct/
just. fair (1/2)

4 30-32 At a time when mankind is  Currently there is much tension arising


confronted with clashes of from conflicting beliefs in a society/
national interest, religious strict following of basic/ orthodox
fundamentalisms and ethnic religious teachings
and racial prejudices,  and cultural
 and racial discrimination (1m for any 2
factors)

5 32-33 non-violence can be a well-  Non-violence can be a very reliable


trusted means of laying the instrument to build the foundation/basis
groundwork of a new of a global political structure (1)
cosmopolitics.
6 34-35 in the last several decades  Lately numerous new
many democratic initiatives, policies/measures to enforce equal
rights/equality for everyone (1/2)
which were premised on non- were based on non-aggressive
violent militancy defence tactics/strategies (1/2)
7 36-37 helped build global civil  Non-violence has helped to promote
society on solid ethical morally upright(1/2) international
foundations communities (1/2) OR
[Repetition]…affirmation of
human rights or uphold human rights (1/2)

8 41-42 it is also true there is no long-  Non-violence is based on correct


term success in politics in the values that would ensure long term
absence of morality. political stability(1)
9 42-45 That is to say, political events  if political duties are conducted
bring moral responsibilities, and observing these correct / acceptable
in turn ethical views place their standards / with accountability (1/2)
imprint on political decisions.  then, the system of governing a state
It is only in relation with ethics can be glorified/ given
that politics can be elevated as a credit/recognised as something
public virtue worthy/ a value/ benefit/
advantage (1/2)

10 45-46 Terrible crimes have been  If allowed, violence can be misused to
committed by political practice impose law and order (1/2) and this
that tried to teach and impose itself is a violation of the law(1/2)
moral behaviour.
11 47-50 Spiritualising politics, …an effort  Political leaders have a moral
to redefine it in terms of civic duty/obligation to influence their people
responsibility in an explicit following accepted standards of
public sphere. behaviour (1/2) on what is right or
Politics is the morally wrong and this can be done without the
conscientious and socially use of brutal force. (1/2)
responsible exercise of civic
roles.
Non-violence is the key to this.
12 50-51 Very few people desire a world  Many people do not like fighting,
with war, crime and terrorism. violation of the law and terrorist
activities. (1) [Any 2 points]
(8 points = 8m)
11.Give the meaning of the following words as they are used in the passage. Write your
answer in one word or a short phrase.[5]

Do not award 1/2m

From passage A 1 mark 0 mark


1 Line 18:  increase  accelerate
… some of the factors that  increase greatly  exacerbate
elevate the risk of civil
war…  heighten  rise
 magnify  worsen
 raise  enhance
 promote  accentuate
 further  make higher
 escalate
 builds up
 intensify
 inflate
2 Line 29:  reasonably  knowingly
Understandably, many  as expected  as known
people in developed
countries find it convenient  justifiably  obviously
to emphasise guilt …  plausibly  considering the view
 sensibly  expectedly
 rationally  being able to comprehend
 logically  undeniably
 true enough  it should be known
 a fact known to many  using common sense
 that which is known  being able to fathom
 visibly
 comprehensibly
 generally
 accepted by most people
 common consensus
 widely claimed as right
 without doubt
3 Line 33  shift attention from  take away
… it detracts from the  shifts  derails
focus on its real causes …
 brings the attention
away
 diverges
 deviates
 digresses
 distracts
 diverts
 sidetracks
 draws away
 drift
 moves away
 steers away
 turns away
 strays away
 brings the attention
away

4 Passage B  dictatorial  authoritative


Line 13:  autocratic
Only the …. despotic
regimes … prevent their  authoritarian
subjects to think and to  repressive
practise non-violence  oppressive
 draconian
 totalitarian
 dogmatic

5 Line 18:  consequences  setback


… violation of human  effects;  undesired/unintended
rights and environmental
degradation have caused
undesirable/negative/ effects
repercussions highlighting bad effects  problems
the concern for global /consequences/impact  drawback
politics of non-violence.  aftermath
 outcomes
 ramifications
 results
 after effects
 negative implications

*Do not penalize for wrong form of tense and spelling. Penalise spelling under language.
For answers with “and”, both components must be correct to get 1 m.
Q11. Both passages deal with the importance of promoting peace to help build a global
civil society.

How far do you agree with the writers‟ views? Using material from both
passages, discuss how your view has been challenged or confirmed by these
passages. You must also rely on your own ideas and knowledge of today‟s world.
(8m)

Requirements:
1) Refer to points raised by both writers in passages 1 and 2 and identify key
ideas which support the view that peace is crucial in helping to
develop/create a civil society in the world.
2) Make your stand and evaluate the extent of how these points influence you to
accept or reject the writers‟ views.
3) Go beyond the texts and substantiate your stand by giving examples in today‟s
world which show the importance of promoting peace to help build a global
civil society.

Possible points from Passage 1 Evaluate the extent of how important the promotion of
peace is in helping to build a global civil society
View has been challenged View has been confirmed
i.e. Disagree with the i.e. Agree with the writers’
writers’ views views
Para 2 „Future civil wars…extralegal The fact that there are still One of the causes of
military grouping. They will variously be acts of terrorism in today‟s terrorism is hatred and
called rebels, terrorists, freedom world shows that radicalization by non-
fighters…before nation-states cohered’. promoting peace does not fundamentalists, hence
help to build a global civil violence often is not a good
(A civil society believes in co-operation society. Despite of global solution to curb terrorism as
and peace.) efforts to curb terrorism, it can result in more
there are new occurrences violence.
of terrorism in today‟s E.g The Iraq War
world. Thus, violence is
needed to rid these To achieve global civility,
terrorists. peaceful methods such
global co-operation , political
E.g. 1st May 2010 New diplomacy and negotiation
York City, Times Square could be adopted instead.
was evacuated after the The advent of globalisation
discovery of a car bomb. brings about more
interconnectedness amongst
E.g. Up to the middle of nations, hence all the more
2010, there were quite a necessary for them to guard
number of suicide each other‟s stability.
bombings in Iran, Pakistan,

1
Greece and Turkey. E.g. Singapore has
spearheaded the development
and sharing of best practices
for countering terrorism
including terrorist finance.
With the support of New
Scotland Yard‟s National
Terrorist Financial
Investigative Unit, the
International Centre for
Political Violence &
Terrorism Research
(ICPVTR) has built its
capacity in Singapore and
overseas to train and support
countries that need counter
terrorist finance capacity.
World-Check specialists
joined ICPVTR instructors in
conducting and sponsoring
ICPVTR counter terrorist
finance training in a number
of countries in Southeast
Asia.

Para 4 „Economic development is peace- Global trade does not 'Globalisation' refers to 'a
promoting‟. always foster peace and process of removing
Para 5 „ Economic development is a key hence hinders the building government-imposed
remedy to violence…a sense of having of a global civil society.
restrictions on movements
something to lose…’ This happens when
developed countries between countries in order to
(As we continue through the 21st engage in unfair trade create an "open",
century, the principles of civil society practices and have "borderless" world economy'
must move beyond the nation-state to protectionistic policies (Scholte 2000: 16) and
include a global political economy of toward the developing global trade is one outcome
relations and groups. Everyone is to be countries. of globalisation.
viewed as one another's neighbour; we
are mutually responsible for everyone's E.g. In 2006, America's Although global trade does
opportunity to experience a satisfying manufacturing sector not guarantee peace, it does
life. Our global economy, having produced about $2.7
strengthen peace by raising
trillion in goods. China's
brought us together, pushes us to the cost of war to
manufacturing sector
develop appropriate political and governments and citizens. As
produced about 8.74
social systems through which nations become more
trillion yuan in goods, or
participation, authority, and

2
responsibility can be just, equitable, about $1.124 trillion at the integrated through expanding
and non-violent--responding with prevailing exchange rate of markets, they have more to
concern for the disenfranchised, 7.77 yuan to the U.S. lose should trade be
marginalized, and impoverished.) dollar.
disrupted. Hence, with
The impact of China's vested interests(e.g. creation
cheap labor costs has of jobs and education
managed to lower opportunities), no nation
consumer prices would want to destabilise
worldwide, particularly for one another.
"such diverse items as
E.g. In 2007, Indian firms,
televisions, toys, T-shirts,
led by Hindalco and Tata
kitchen appliances, athletic Steel, have bought some 34
shoes and power tools." foreign companies for a
combined $10.7 billion.
However, the good news Indian IT-services companies
for China manufacturing such as Infosys, Tata
has led to "massive layoffs Consultancy Services and
throughout the world, as Wipro are putting the fear of
factories either were shut God into the old guard,
down or moved to China to including Accenture and
be competitive. The even mighty IBM. Big Blue
Economic Policy Institute, sold its personal-computer
a liberal-leaning think tank business to a Chinese
in Washington, D.C., said multinational, Lenovo, which
the United States is losing is now starting to get its act
more than 2,000 factory together.
jobs a month because of
the shift of work to China."

International aid can


sometimes pose more
problems and hinder the
promotion of peace so
crucial in the development
of a global civil society.

E.g. Food aid can


sometimes hurt starving
populations, by keeping
poor nations intertwined
with the economy of the
donor country (namely the
US)
The US is the largest food
donor. The aid is given in

3
kind or sold at subsidised
rates to recipient
governments. Under its
current regulations, at least
75% of food aid has to be
grown, procured and
bought in the US and
handled and shipped by US
firms. When shipping costs
rise, so does the price for
food aid. To keep costs in
check, sometimes less food
is delivered.
Selling cheaper food aid
below market prices in
recipient nations undercuts
local farmers‟ market
share, resulting in them
becoming poor.

Para 7 „The cost of violence is Indiscriminate bombing is The cost of violence is


enormous…that it leads not to improvement a strategy often used by enormous and there are many
but to deterioration.‟ terrorists to undermine the past evidences of such
will and morale of the destruction. Non-violence is
(A civil society upholds the belief that whole community. the foundation of a global
there should be love, hope and peace Therefore, violence is civil society.
rather than hatred and violence.) needed to rid these
terrorists. E.g. World Wars I & II

E.g. The use of force and E.g. Some 400,000 ethnic


violence against terrorism Uzbeks remained displaced
has been demonstrated on both sides of the
periodically. U.S. military Kyrgyzstan-Uzbekistan
action against the Taliban border following a series of
in Afghanistan is an attacks in June 2010, killing
example of the use of force at least 251 and possibly as
against terrorism. many as 2,000.

4
Possible points from Passage 2 Evaluate the extent of how important the promotion of peace is in
helping to build a global civil society
View has been challenged i.e. Disagree with View has been
the writers’ views confirmed i.e. Agree
with the writers’ views
Para 3 „Global politics of non- The 1990s saw the UN refocus its attention International aid is a
violence…also of civil society, on genocide and ethnic cleansing. The Civil remedy for the global
and inter-governmental, non- War in Rwanda and the breakup of income- divide
governmental and Yugoslavia both were occasions of problems and thus help
transnational organisations.’ widespread atrocities and ethnic violence. build a global civil
The UN launched its peacekeeping mission in society.
Para 4 …’non-violence is an October 1993, but its mandate did not allow E.g. Oxfam
ineffective instrument against for military force protection in the International which is a
dictatorships and genocide…‟ investigation of cease-fire violations or confederation of 13
securing humanitarian aid. organisations working
(A civil society is inclusive and with over 3,000
involves all sectors of a society partners in more than
with each having a stake in it. ) E.g. The United Nations and its member states 100 countries to find
failed Rwanda "in deplorable ways" in 1994, lasting solutions to
ignoring evidence of planned genocide and poverty and injustice.[
abandoning Rwandans in need of protection,
E.g. The UN is
according to a report released today.
involved in supporting
http://www.un.org/News/dh/latest/rwanda.htm
development, e.g. by
(UN Newservice, 15 Dec 1999).
the formulation of the
The independent report, commissioned by the
Millennium
then UN Secretary-General Kofi Annan, says
Development Goals.
the UN peacekeeping operation in Rwanda
The UN Development
was "doomed from the start" by an inadequate
Programme (UNDP) is
mandate and destroyed by the Security
the largest multilateral
Council's refusal to strengthen it once killings
source of grant
began."The fundamental failure was the lack
technical assistance in
of resources and political commitment
the world.
devoted to developments in Rwanda and to
Organizations like the
the United Nations presence there," the report
World Health
said. "The manner in which the troops left,
Organization (WHO),
including attempts to pretend to the refugees
UNAIDS, and The
that they were not in fact leaving, was
Global Fund to Fight
disgraceful," the report added (Nicole
AIDS, Tuberculosis and
Winfield, Associated Press, 16 Dec).
Malaria are leading
institutions in the battle
against diseases around
the world, especially in
poor countries.

5
Para 3 „ Only a non-violent Promoting social solidarity in practice means Social cohesion
society can work its way up to confronting corruption involves tolerance and
creating the institutions ripe for and trying to ensure democratic governance, unity in a diversified
development and lead to inter- power sharing, and the equitable world. This is important
cultural and inter-religious distribution of resources among all members in promoting peace and
harmony.’ of society. therefore in building a
The threat of the use force i.e. the use of global civil society.
Para 4 „…an affirmation of military can prevent conflict amongst Some countries are still
human rights and helped build different factions of society. fraught with
global civil society on solid incompetent political
ethical foundations…‟ E.g. Ethnic riots in Urumqi, western Xinjiang leaders and are unable
province between the majority Han and to establish civil
(Social cohesion is crucial in a minority Uighurs on 5 July 2009 witnessed societies; therefore
civil society.) the use of the police to help control further there is a need for some
uprisings. international
organisations to help to
overcome this problem.

E.g. The pursuit of


human rights was a
central reason for
creating the UN. World
War II atrocities and
genocide led to a ready
consensus that the new
organization must work
to prevent any similar
tragedies in the future.
The UN and its
agencies are central in
upholding and
implementing the
principles enshrined in
the Universal
Declaration of Human
Rights. A case in point
is support by the UN for
countries in transition to
democracy. Technical
assistance in providing
free and fair elections,
improving judicial
structures, drafting
constitutions, training
human rights officials,
and transforming armed

6
movements into
political parties have
contributed
significantly to
democratization
worldwide. The UN has
helped run elections in
countries with little or
no democratic history,
including recently in
Afghanistan.

Para 4 „…non-violence is an Violence such as coup d‟etat is sometimes A civil society resorts
ineffective instrument against necessary in order to bring about civility in to peaceful methods
dictatorships and genocide‟ societies. such as civil obedience*
A failed state often has a government that is in solving tensions and
(A civil society resorts to unable to exercise authority (law and order; can be effective in
peaceful methods in conflict taxation etc.) over most of the country. overthrowing an
resolution.) This failure can be manifested by wide gap autocratic leader.
between rich and poor, endemic corruption,
rising child mortality rate, etc. *Civil disobedience is a
Hence violence is inevitable to prevent these public, non-violent and
states from being a threat (economic, conscientious breach of
political, social etc) to other nation(s) as they law undertaken with the
are unable to exercise their sovereignty over aim of bringing about a
their political boundaries. change in laws or
government policies.
E.g. Liberia; Sierre Leone; Ivory Coast;
Solomon Islands; Ex-Yugoslavia; Congo; E.g. In 1985 under
Taleban and post-Taleban Afghanistan. mounting pressure,
Marcos called for a
. snap presidential
election and a million
signatures were quickly
amassed. When Marcos
claimed victory in the
contest , a surge of
“people power”
signalled that his
authority and credibility
had drained away. His
resort to widespread

7
fraud and intimidation
had been as brazen as
the killing of Benigno
Aquino. Hundreds of
thousands took to the
streets, staging a civil
disobedience campaign
and boycotting
businesses owned by
Marcos supporters. Key
army figures declared in
favour of Aquino,
ending a dictatorial rule
under Marcos .

Band A (6-8 marks) Band B (3-5 marks) Band C (1-2 marks)

Points are taken from both Examines points from both Examines points from only one
passages; there are systematic passages and explanation passage.
references to the both texts with shows an adequate level of Answers the question without
regard to the importance of understanding of how important systematic reference to ideas in
promoting peace to help build a the promotion of peace is in the texts.
global civil society. helping to build a global civil
society.
May include some minor
misinterpretation.

Evaluates critically and Evaluates relevant arguments to Merely summarises the ideas of
convincingly the extent of how some extent but discussion is the writers without critically
important the promotion of superficial. Explanation is often commenting on selected points.
peace is in helping to build a limited or not linked to the main Little or no attempt made to
global civil society. arguments well. justify the evaluation.

Brings in apt and effective Examples cited may be clichéd, Lack examples or give examples
examples, going beyond the common or not truly effective. without link to today’s world.
texts, to support the evaluation.

Shows a high degree of Coherence and organisation are Inconsistency in the argument.
coherence and organisation— not as sharp or systematic as Coherence is in question—
good paragraphing, apt use of the top band. disorganised lacking a clear
connectors and has clarity of direction of answer.
expression, showing a logical
flow of argument.

8
RAFFLES INSTITUTION
2010 PRELIMINARY EXAMINATION

GENERAL PAPER
PAPER 1

Thursday 2 September 2010 1 hour 30 mins

RAFFLES INSTITUTION RAFFLES INSTITUTION RAFFLES INSTITUTION RAFFLES INSTITUTION RAFFLES INSTITUTION RAFFLES INSTITUTION
RAFFLES INSTITUTION RAFFLES INSTITUTION RAFFLES INSTITUTION RAFFLES INSTITUTION RAFFLES INSTITUTION RAFFLES INSTITUTION
RAFFLES INSTITUTION RAFFLES INSTITUTION RAFFLES INSTITUTION RAFFLES INSTITUTION RAFFLES INSTITUTION RAFFLES INSTITUTION
RAFFLES INSTITUTION RAFFLES INSTITUTION RAFFLES INSTITUTION RAFFLES INSTITUTION RAFFLES INSTITUTION RAFFLES INSTITUTION
RAFFLES INSTITUTION RAFFLES INSTITUTION RAFFLES INSTITUTION RAFFLES INSTITUTION RAFFLES INSTITUTION RAFFLES INSTITUTION

INSTRUCTIONS TO CANDIDATES
Answer one question from Paper 1.

Head each sheet of writing paper with your name, CT group and the name of your
GP tutor.

INFORMATION FOR CANDIDATES

There will be two separate sessions for the GP examination. Your answer to Paper 1
will be collected after 1 hour and 30 minutes. After a break, you will be issued with
Paper 2, which is also a 1-hour-and-30-minute paper.

Note that 20 marks out of 50 will be awarded for your use of language in Paper 1.

This question paper consists of 2 printed pages, including this one.

© RAFFLES INSTITUTION 2010 Page 1


Paper 1

Answer one question from this Paper.

Answers should be between 500 and 800 words in length.

1. How far is modern consumerism a threat to the environment?

2. ‘Inequality is a fact of life.’ To what extent should we accept this?

3. Is there still a place for public libraries in your society?

4. Consider the significance of silence.

5. ‘The media works best when it gives the masses exactly what they want.’
Discuss.

6. Is world government a good idea?

7. ‘Scientific decisions should be left to scientists.’ To what extent do you agree?

8. ‘The best way to make children good is to make them happy’ (Oscar Wilde).
What is your view?

9. ‘Not my problem!’ Is this a good attitude to have?

10. Is creativity all about breaking rules?

11. ‘Our obsession with speed is getting us nowhere fast.’ Do you agree?

12. To what extent should government influence education? Discuss with reference
to your country.

© RAFFLES INSTITUTION 2010 Page 2


© RAFFLES INSTITUTION 2010

RAFFLES INSTITUTION
2010 PRELIMINARY EXAMINATION

GENERAL PAPER
PAPER 2

Thursday 2 September 2010 1 hour 30 mins

RAFFLES INSTITUTION RAFFLES INSTITUTION RAFFLES INSTITUTION RAFFLES INSTITUTION RAFFLES INSTITUTION RAFFLES INSTITUTION
RAFFLES INSTITUTION RAFFLES INSTITUTION RAFFLES INSTITUTION RAFFLES INSTITUTION RAFFLES INSTITUTION RAFFLES INSTITUTION
RAFFLES INSTITUTION RAFFLES INSTITUTION RAFFLES INSTITUTION RAFFLES INSTITUTION RAFFLES INSTITUTION RAFFLES INSTITUTION
RAFFLES INSTITUTION RAFFLES INSTITUTION RAFFLES INSTITUTION RAFFLES INSTITUTION RAFFLES INSTITUTION RAFFLES INSTITUTION
RAFFLES INSTITUTION RAFFLES INSTITUTION RAFFLES INSTITUTION RAFFLES INSTITUTION RAFFLES INSTITUTION RAFFLES INSTITUTION

INSERT

INSTRUCTIONS TO CANDIDATES
This insert contains the passages for Paper 2.

This question paper consists of 4 printed pages, including this one.


2
Passage 1

1 There aren't many ideas that unite former U.S. president George W. Bush and his successor,
Barack Obama. But one safe topic for conversation would be Internet freedom and the power of
technology to foment democratic revolutions. In mid-April Bush welcomed to his new think tank in
Texas six dissidents who used Web tools to oppose dictatorships, applauding them as examples
"of how the Internet can be effectively used to advance the freedom agenda." Obama, meanwhile, 5
has made Internet freedom a centerpiece of his foreign policy, and in a speech in Beijing late last
year hailed "access to information" as a "universal right."

2 This kind of talk taps into a wide vein of techno-utopianism that has been around since at least the
dawn of the Web. The Internet is disruptive by nature, rapidly overturning business models and
mores, so it was natural for tech-savvy foreign-policy thinkers to believe that dictatorships, too, 10
would fall with the click of a mouse. That, of course, didn't happen. In fact, quite the opposite is
true, say a growing number of cyberskeptics. Autocrats have "mastered the use of cyberspace for
propaganda," says Evgeny Morozov, one of the smartest and best-known cyberskeptics. Worse,
they've learned to mine online information, such as Facebook profiles, for intelligence purposes.
"The KGB used to torture to get ahold of this data," says Morozov. "Now it's all available online." In 15
short, say the cyberskeptics, the Internet will lead to the entrenchment of dictatorship, not its end.

3 But that is a shortsighted view, and one predicated on the trend line over the last few years, in
which autocracies appear to have gained the upper hand against democrats. If it seems as if they
have, it's because the hardliners are playing catch-up – they've finally recognized the existential
threat posed by the Internet. The color revolutions in former Soviet republics, the post-election 20
protests in Iran, the saffron revolt in Burma, and smaller-scale Chinese demonstrations against
pollution and corruption – all prominently featured the use of online tools and mobile phones to
organize protesters and project their message around the world.

4 The cyberskeptics are right that this is not a one-sided fight; Iran's Basij militiamen can use
Facebook, too. But the Internet represents "the largest increase in expressive capability in human 25
history," as the writer Clay Shirky puts it, and because of its open, decentralized architecture, it is
biased toward open, decentralized systems, i.e., democracies. For instance, the use of Twitter by
protesting youths in Moldova last year to create a flash mob in the capital city of Chisinau
illustrated just how powerful an organizing and communicating tool the Internet is, even when
limits are placed on it. And when dictators fight back against it, they're pushing against a wall of 30
water. "It's a cat-and-mouse game," says Daniel Calingaert, the deputy director of programs at
Freedom House – each new government restriction is met with an inventive workaround, which
prompts new restrictions. In Moldova the government blocked cell-phone reception in the square
where protesters had gathered. So the protesters simply walked a couple of blocks away to post
tweets, then returned to the square. Their tweets dominated the micromessaging service for days. 35

5 Of course, there is a logical end to any cyber cat-and-mouse game that goes on long enough.
During Burma's saffron rebellion in 2007, the junta maintained its heavy-handed Web censorship
tactics, blocking many foreign sites and e-mail programs, but protesters easily circumvented them
and managed to post photos and firsthand accounts of the regime's brutality, including a video of a
soldier shooting a Japanese reporter dead. On September 29, the junta decided it had had 40
enough and simply shut down the country's two Internet service providers. To the techno-utopians,
this was a splash of ice-cold water to the face, suggesting that the government in power virtually
always holds the trump card. But in one way the junta's extreme reaction actually revealed the
futility of its censorship. Their choice was a binary one: accept that the Web cannot be controlled,
or eliminate it altogether. Choosing the latter sets a nation on a path to becoming the next Hermit 45
Kingdom, a decision that almost every nation is unwilling to make.
3
6 The cyberskeptics also forget that the path toward democracy is a long one, and that the Internet
is, in many places, less than 10 years old. The late political scientist Samuel Huntington once
remarked that the wisest democratic reformers "tend to be leery of simple solutions and of
revolutions." Massive protests and toppled statues make for great TV, but most of the hard work 50
comes well before and lasts long after the putsch. It's during that long process – which academics
such as Huntington call "democratic consolidation" – that the Web's impact will be most felt.
Mobile banking and e-commerce are helping more people join the ranks of the middle class,
typically the first group to agitate for freedoms. Bloggers and tweeters are fulfilling the watchdog
role in places where the mainstream media is muzzled. Election monitoring can now be performed 55
by anyone, thanks to open-source platforms like Ushahidi, which facilitate anonymous reporting. In
other words, Presidents Bush and Obama are right to agree on this issue: the digital masses
trump the despots.

Barrett Sheridan, “The Internet Helps Build Democracies” (Newsweek, April 2010)

Passage 2

1 When Google announced earlier this year that it would shut down its China search engine after
hackers allegedly broke into Chinese human-rights advocates' Gmail accounts, the activists,
editorial boards, and commentators lauded the company. In doing so, they echoed an argument
made by many technophiles and politicians since the late 1990s: the Web, and new
communications technology in general, will open up closed societies and hasten the demise of 5
authoritarian regimes, with freedom-loving Internet companies leading the charge. Bill Clinton said
as much at the time. He told Chinese leaders that they stood "on the wrong side of history."

2 But the idea that the Internet will spark the decline of autocrats has been proven false. In the past
four years, Web penetration has grown in most authoritarian states, yet overall the number of free
societies worldwide has declined in that time period, according to annual reports by monitoring 10
organization Freedom House. Online activists in many places have much less freedom than they
did four years ago as well. In Vietnam the government has rounded up most of the leading online
activists and sentenced many to jail; in Thailand, a soft-authoritarian state, the government
recently arrested an editor of one of the most respected and vibrant online news sites. In China
the government has shuttered thousands of blogs and sites in the past year alone. 15

3 Authoritarian regimes have undermined the potential power of the Web in several ways. Like
China, many have developed highly sophisticated methods of monitoring and filtering Web sites.
These autocrats increasingly are learning from each other's filters: Vietnam has sent officials to
Beijing to study the "Great Firewall of China," while China has exported its technology and
Internet-control strategies to Saudi Arabia and Burma, among other countries. These governments 20
also use state-backed commentators to control online discourse and threaten political opponents.
China reportedly has some 250,000 state-backed commentators weighing in on popular forums,
and the Kremlin has its own funded "Web brigades," which attack liberals and praise Prime
Minister Vladimir Putin. And rather than just put portals in the hands of Western companies, savvy
autocrats are creating their own. China is launching state-backed versions of Twitter, YouTube, 25
and Facebook, all of which surely will not include content about controversial topics like the
Tiananmen crackdown or the Dalai Lama. The Kremlin reportedly may launch a government-
backed search engine as well, which will create headaches for Google in Russia as well.

4 Worse, much of the public in countries like China and Vietnam has no idea how much news and
information they are missing out on in their filtered Web universe. Since these states allow just 30
enough online freedom to perpetuate a fiction of a free Internet, while quietly blocking politically
hot sites, users often think they are seeing the same Internet as someone in the U.S. or Japan or
another free country.
4

5 In some ways the Internet and other new communications technology make it easier for
authoritarian regimes to monitor political activists. Relying on technology to organize can lead 35
activists to ignore old-fashioned on-the-ground political networking, critical for massing street
protests or getting bodies to a polling booth. And by creating their own personal Web pages or
Facebook pages, activists in countries like Iran are building the kinds of dossiers of information
about themselves that, in the old days, government security services had to work hard to piece
together. It also has become easier for the security services to follow dissidents, since they can 40
track groups of them online rather than having to infiltrate meetings of dissidents in people's
homes or in bars. When the Chinese government wanted to crack down on the creators of Charter
08, an online petition calling for the rule of law and greater freedom, it could find most of their
details online; many Iranian Green Movement protestors think Tehran used protest leaders'
Facebook pages and other online identifiers to find them and their friends after last summer's 45
antigovernment demonstrations.

6 Those supposedly beneficent Internet companies, dedicated to open networks and free
expression, also have turned out to be, well, companies – interested first and foremost in the
bottom line. Nokia and Siemens allegedly helped produce technology for Iran's state
telecommunications company that the Iranians then used for monitoring phones and other 50
communications. Yahoo helped Beijing find and arrest a Chinese journalist and prominent rights
activist. The largest Chinese Web sites, like local search-engine giant Baidu, employ teams of
Web monitors who look for and remove anything in their traffic banned by China's propaganda
department. And though Google may have made an ethically responsible choice in China (though
it had only a small slice of the search market in China anyway), in the days after its decision there 55
was a deafening silence from other big technology firms with China operations. "We have done
business in China for more than 20 years and we intend to continue our business there," a
Microsoft spokesman told the press after Google pulled out. Turns out, these companies didn't
want a free and uncensored Web that badly. And once the hype over Google's exit fades, the
companies will go back to business as usual. And the countries in which they operate will crack 60
down once again.

Joshua Kurlantzick, “The Web Doesn’t Spread Freedom” (Newsweek, April 2010)
© RAFFLES INSTITUTION 2010

Student‟s name CT group GP tutor‟s name

RAFFLES INSTITUTION
2010 PRELIMINARY EXAMINATION

GENERAL PAPER
PAPER 2

Thursday 2 September 2010 1 hour 30 mins

RAFFLES INSTITUTION RAFFLES INSTITUTION RAFFLES INSTITUTION RAFFLES INSTITUTION RAFFLES INSTITUTION RAFFLES INSTITUTION
RAFFLES INSTITUTION RAFFLES INSTITUTION RAFFLES INSTITUTION RAFFLES INSTITUTION RAFFLES INSTITUTION RAFFLES INSTITUTION
RAFFLES INSTITUTION RAFFLES INSTITUTION RAFFLES INSTITUTION RAFFLES INSTITUTION RAFFLES INSTITUTION RAFFLES INSTITUTION
RAFFLES INSTITUTION RAFFLES INSTITUTION RAFFLES INSTITUTION RAFFLES INSTITUTION RAFFLES INSTITUTION RAFFLES INSTITUTION
RAFFLES INSTITUTION RAFFLES INSTITUTION RAFFLES INSTITUTION RAFFLES INSTITUTION RAFFLES INSTITUTION RAFFLES INSTITUTION

Candidates answer on the question paper.


Additional material: 1 insert.

READ THESE INSTRUCTIONS FIRST


Write your name, CT group and GP tutor‟s name in the spaces at the top of this page.
Write in dark blue or black ink in the spaces provided on the question paper.
Do not use staples, paper clips, highlighters, glue or correction fluid.
Answer all questions.
The number of marks is given in brackets [ ] at the end of each question or part question.
The insert contains the passages for comprehension.
(Note that 15 marks out of 50 will be awarded for your use of language.)
For Examiner‟s Use

Content
/35

Language
/15

TOTAL /50

__________________________________________________________________________________
This question paper consists of 8 printed pages and an insert.
© RAFFLES INSTITUTION 2010

Read the passages in the Insert and then answer all the questions which follow below. Note that
up to fifteen marks will be given for the quality and accuracy of your use of English throughout
this Paper.

Note: When a question asks for an answer IN YOUR OWN WORDS AS FAR AS POSSIBLE and
you select the appropriate material from the passages for your answer, you must still use your
own words to express it. Little credit can be given to answers which only copy words or phrases
from the passages.

From Passage 1 For


Examiner’s
Use
1 What does the word „hailed‟ (line 7) suggest about Obama‟s response to Internet
freedom?

………………………………………………………………………………………………………

……………………………………………………………………………………………………[1]

2 What are the two opposing views found in paragraph 2? Use your own words as far as
possible.

………………………………………………………………………………………………………

………………………………………………………………………………………………………

………………………………………………………………………………………………………

………………………………………………………………………………………………………

……………………………………………………………………………………………………[2]

3 Why does the author call the cyberskeptics‟ concerns „shortsighted‟ (line 17)? Use your
own words as far as possible.

………………………………………………………………………………………………………

………………………………………………………………………………………………………

………………………………………………………………………………………………………

………………………………………………………………………………………………………

………………………………………………………………………………………………………

……………………………………………………………………………………………………[3]

2
© RAFFLES INSTITUTION 2010

4 „And when dictators fight back against it, they‟re pushing against a wall of water‟ (lines For
Examiner’s
30-31). Explain the metaphor. Use your own words as far as possible. Use

………………………………………………………………………………………………………

………………………………………………………………………………………………………

………………………………………………………………………………………………………

……………………………………………………………………………………………………[2]

5 Why does the author use capital letters in „Hermit Kingdom‟ (lines 45-46)?

………………………………………………………………………………………………………

………………………………………………………………………………………………………

……………………………………………………………………………………………………[1]

6 Give the meaning of the following words as they are used in the passage. You may write
your answer in one word or a short phrase.

From Passage 1:

(a) agenda (line 5) ………….……….………..………………………………………………….

(b) futility (line 44) ..………………………………….……………………………………….......

(c) leery (line 49) ..………………………………....……………………………………………..

(d) agitate (line 54) ..………………………………….……………………………………….....

From Passage 2:

(e) beneficent (line 47) ….………………………..…………..………………………………[5]

3
© RAFFLES INSTITUTION 2010

From Passage 2 For


Examiner’s
Use

7 „He told Chinese leaders that they stood “on the wrong side of history”‟ (line 7).
What was Bill Clinton implying about authoritarian regimes? Use your own words as far
as possible.

………………………………………………………………………………………………………

………………………………………………………………………………………………………

……………………………………………………………………………………………………[1]

8 What evidence is there that „the decline of autocrats has been proven false‟ (line 8)? Use
your own words as far as possible.

………………………………………………………………………………………………………

………………………………………………………………………………………………………

………………………………………………………………………………………………………

………………………………………………………………………………………………………

……………………………………………………………………………………………………[2]

9 What does the author imply when he uses the phrase „deafening silence‟ (line 56)? Use
your own words as far as possible.

………………………………………………………………………………………………..........

………………………………………………………………………………………………..........

………………………………………………………………………………………………..........

………………………………………………………………………………………………..........

……………………………………………………………………………………………………[2]

4
© RAFFLES INSTITUTION 2010

10 According to Kurlantzick, how do authoritarian governments manage the Web? Using For
Examiner’s
relevant material from paragraphs 3 to 5, summarise the ways in which they do so. Write Use
your summary in no more than 120 words, not counting the opening words which are
printed below. Use your own words as far as possible.

Authoritarian governments manage the Web……………..….……………………………….

………………………………………………………………………………………………………

………………………………………………………………………………………………………

………………………………………………………………………………………………………

………………………………………………………………………………………………………

………………………………………………………………………………………………………

………………………………………………………………………………………………………

………………………………………………………………………………………………………

………………………………………………………………………………………………………

………………………………………………………………………………………………………

………………………………………………………………………………………………………

………………………………………………………………………………………………………

………………………………………………………………………………………………………

………………………………………………………………………………………………………

………………………………………………………………………………………………………

………………………………………………………………………………………………………

………………………………………………………………………………………………………

………………………………………………………………………………………………………

………………………………………………………………………………………………………

………………………………………………………………………………………………………

……………………………………………………………………………………………………[8]

Number of words: ______________

5
© RAFFLES INSTITUTION 2010

11 Sheridan argues that the Internet plays a crucial role in building democracies, whereas For
Examiner’s
Kurlantzick is of the view that the Internet does not spread freedom. Use

How far do you agree with their views?

Support your answer by referring to what you have read in both passages and to the
situation in your society.

………………………………………………………………….……………………………………

……………………………………………………………………….………………………………

…………………………………………………………………………….………………………..

………………………………………………………………………………..…………………….

……………………………………………………………………………………………………….

……………………………………………………………………………………………………….

……………………………………………………………………………………………………….

……………………………………………………………………………………………………….

……………………………………………………………………………………………………….

……………………………………………………………………………………………………….

……………………………………………………………………………………………………….

……………………………………………………………………………………………………….

……………………………………………………………………………………………………….

……………………………………………………………………………………………………….

……………………………………………………………………………………………………….

……………………………………………………………………………………………………….

……………………………………………………………………………………………………….

……………………………………………………………………………………………………….

……………………………………………………………………………………………………….

……………………………………………………………………………………………………….

……………………………………………………………………………………………………….
6
© RAFFLES INSTITUTION 2010

For
………………………………………………………………………………………………………. Examiner’s
Use

……………………………………………………………………………………………………….

……………………………………………………………………………………………………….

.………………………………………………………………………………………………………

………………………….……………………………………………………………………………

……………………………………………………………………………………………………….

……………………………………………………………………………………………………….

……………………………………………………………………………………………………….

……………………………………………………………………………………………………….

……………………………………………………………………………………………………….

……………………………………………………………………………………………………….

……………………………………………………………………………………………………….

……………………………………………………………………………………………………….

……………………………………………………………………………………………………….

……………………………………………………………………………………………………….

……………………………………………………………………………………………………….

……………………………………………………………………………………………………….

……………………………………………………………………………………………………….

……………………………………………………………………………………………………….

……………………………………………………………………………………………………….

……………………………………………………………………………………………………….

……………………………………………………………………………………………………….

……………………………………………………………………………………………………….

……………………………………………………………………………………………………….
7
© RAFFLES INSTITUTION 2010

……………………………………………………………………………………………………….

………………………………………………………………………………………………………. For
Examiner’s
Use
……………………………………………………………………………………………………….

……………………………………………………………………………………………………….

.………………………………………………………………………………………………………

………………………….……………………………………………………………………………

……………………………………………………………………………………………………….

……………………………………………………………………………………………………….

……………………………………………………………………………………………………….

……………………………………………………………………………………………………….

……………………………………………………………………………………………………….

…………………………………………………………………………………………………….…

………………………………………………………………………………………………………

………………………………………………………………………………………………………

…….…………………………………………………………………………………………………

………….……………………………………………………………………………………………

……………………………………………….………………………………………………………

…………………………………………………….…………………………………………………

………………………………………………………….……………………………………………

………………………………………………………………………………………….……………

………………………………………………………………………………………….……………

………………………………………………………………………………………….……………

………………………………………………………………………………………………….. [8]

End of Paper

8
© RAFFLES INSTITUTION 2010

Raffles Institution
2010 Preliminary Examination
GP (Paper 2)

Questions from Passage 1

1. What does the word „hailed‟ (line 7) suggest about Obama‟s response to Internet freedom?
[1m]

Pt Lift Inference
a Obama…hailed „access to information‟ as a For a full mark, answer should address 1
(a) support/approval/advocacy (1/2) &
„universal right‟. (b) enthusiasm (1/2) for Internet
freedom
e.g. He welcomed it enthusiastically
0 marks – He ‘values…’, ‘regards as
important’, ‘has high regard for it’

2. What are the two opposing views found in paragraph 2? Use your own words as far as
possible. [2m]

Pt Lift Suggested Paraphrase


a Techno-utopianism … to believe that One view is that the Internet would be 1
dictatorships, too, would fall with the click able to overcome
of a mouse despotic/authoritarian
regimes/governments easily
b Quite the opposite is true, say… while the contrasting view is that the 1
cyberskeptics. In short… the Internet will Internet will perpetuate these
lead to the entrenchment of dictatorship, despotic/authoritarian
not its end. regimes/governments instead of
eliminating them.
Note:
Answer must be expressed as two distinct views for the full mark.
Maximum of 1 mark for answer that does not capture the contrast.
No marks if only one view is offered.
Do not accept examples in lieu of stating a particular view.
Allow lift of ‘the Internet’.
© RAFFLES INSTITUTION 2010

3. Why does the author call the cyberskeptics‟ concerns „shortsighted‟ (line 17)? Use your own
words as far as possible. [3m]

Pt Lift Suggested paraphrase


It is shortsighted because…
a predicated on the trend line over the … it is based only on a recent 1
last few years… development/pattern
b …autocracies appear to have gained … it only looks as if authoritarian regimes 1
the upper hand against democrats are stronger than democracies but…
Entire idea must be captured to get 1 mark
Answer does not have to specify ‘against
democrats’ to get 1 mark
c …the hardliners are playing catch-up …in reality, they are still lagging behind 1

4. „And when dictators fight back against it, they‟re pushing against a wall of water‟ (lines 30-31).
Explain the metaphor. Use your own words as far as possible. [2m]

Pt Metaphor Suggested explanation


a Pushing against a wall of water… Just as
(a) pushing against a wall of water is an 1
impossible/a pointless/meaningless/futile/
fruitless endeavour,
(b) restricting/opposing/censoring Internet use is 1
pointless/impossible/meaningless/futile/fruitless.

5. Why does the author use capital letters in „Hermit Kingdom‟ (lines 45-46)? [1m]

Pt Lift Inference
a a decision that almost every nation The capitals emphasize being ostracized / 1
is unwilling to make unpopular / shunned / left out of the global
community
Answer must capture both the idea of emphasis
and isolation.
Answer has to capture the idea that isolation is
imposed, and not by choice.
No ½ mark
© RAFFLES INSTITUTION 2010

6. Vocabulary [5m]
Word 1 mark ½ mark 0 marks
(a) agenda (line 5) motive plan list
noun cause idea outline
objective programme
‘…to advance the freedom purpose movement
agenda…’

(b) futility (line 44) ineffectiveness hopelessness senselessness


noun pointlessness weakness
uselessness failure
‘…revealed the futility of its
censorship.’
(c) leery (line 49) wary doubtful unsure
adjective suspicious disbelieving uncertain
apprehensive aware
‘…tend to be leery of simple skeptical ignorant
solutions…’
mistrustful disapproving
distrustful
(d) agitate (line 54) lobby push ask
verb campaign demand fight
advocate
‘…typically the first group to pressure
agitate for freedoms…’

(e) beneficent (Passage 2 magnanimous benevolent considerate


line 47) altruistic
generous
philanthropic
adjective
liberal
‘…supposedly beneficent helpful
Internet companies…’ kind
noble

Note: Do not penalize spelling, word form errors under content mark
© RAFFLES INSTITUTION 2010

Questions from Passage 2

7. „He told Chinese leaders that they stood “on the wrong side of history”‟ (line 7).
What was Bill Clinton implying about authoritarian regimes? Use your own words as far as
possible. [1m]

Pt Lift Suggested paraphrase


In the face of the Internet /
technological development,
[½ mark only if answer doesn’t
include idea of ‘technological
development’]

a The Web, and new communications technology in authoritarian/paternalistic 1


general, will open up closed societies (line 5) regimes/states will become
liberalized

OR OR

b and hasten the demise of authoritarian regimes and will fall 1


(lines 5-6).

OR OR

c …the Internet will spark the decline of autocrats authoritarian regimes will be 1
(line 8) rendered out-of-
date/obsolete/irrelevant.

8. What evidence is there that „the decline of autocrats has been proven false‟ (line 8)? Use
your own words as far as possible. [2m]

Pt Lift Suggested paraphrase


a …overall the number of free In general, there are fewer democratic/liberal 1
societies…has declined nations today
b Online activists in many places Internet-based campaigners/protesters/People 1
have much less freedom… who use the Web for their causes are facing far
more restriction
0 mark if answer cites examples in lieu of the
main idea
© RAFFLES INSTITUTION 2010

9. What does the author imply when he uses the phrase „deafening silence‟ (line 56)? Use your
own words as far as possible. [2m]

Lift Suggested paraphrase


… other big technology firms a) The writer implies that the lack of 1
with China operations. „We have support/response/comment (silence) makes a
done business in China for more strong statement (deafening),
than 20 years and we intend to
continue our business there,‟ b) that the well-established/major/significant 1
a Microsoft spokesman told the companies regard enterprise or profit is more
press after Google pulled out important to them than other considerations.
(line 56-58).
© RAFFLES INSTITUTION 2010

10. Summary [8m]


According to Kurlantzick, how do authoritarian governments manage the Web? Using relevant
material from paragraphs 3 to 5, summarise the ways in which they do so. Write your summary
in no more than 120 words, not counting the opening words which are printed below. Use your
own words as far as possible.

Authoritarian governments manage the Web…


Lift Paraphrase
A Many have developed highly sophisticated …to come up with extremely advanced
methods of monitoring and filtering Web ways (1/2) to keep an eye on/track and/or
sites (line 17) screen/sieve/censor online content/pages
(1/2)
B …are learning from each other’s filters (line They adopt/copy/exchange/share one
18) another‟s screening methods
Accept lift of ‘filters’
Do not accept ‘technology’ for ‘filters’ if it
is not linked to point A
C Use state-backed commentators… (line 21) Employ government-approved/-
sanctioned observers/reporters/officials…
Key idea: ‘state-backed’ (1 mark)
D … to control online discourse (line 21) … to dictate/influence/shape/restrict/
regulate online/Web discussions
E … and threaten political opponents (line 21) … and to intimidate enemies of the
state/antagonists
0 mark: ‘coerce’, ‘blackmail’
F …savvy autocrats create their own [portals] Astute/ clever dictators can
start/commission/have their own portals
(lines 24-25) /online gateways
Accept lift of ‘portals’
G …will not include content about [that allow them to] leave out
controversial topics (line 26) information/issues/subjects that are
contentious/potentially divisive
H perpetuate a fiction of a free Internet… (line Give/continue/sustain the illusion of
31) / users often think they are seeing the uncensored online content
same Internet as someone in…another free Accept lift of ‘Internet’
country (line 32) ½ mark if answer lifts ‘freedom’
I … while quietly blocking politically hot sites …even as they secretly restrict access to
(lines 31-32) politically contentious/popular Web pages
Accept lift of ‘politically’
© RAFFLES INSTITUTION 2010

J …monitor political activists (line 35) / Track/Spy/Keep an eye/Keep a watch on


follow dissidents (line 40)… protesters/opponents…
K …activists…are building…dossiers of [INFERENCE]…by getting details/data
information about themselves (line 38) about who they are
L … and track groups online rather than [INFERENCE] Observe/Trail these people
having to infiltrate meetings…in people’s virtually instead of physically
homes or in bars (lines 41-42) Answer must include the contrast between
virtual and physical tracking

Sample summary:
Authoritarian governments use the Web to come up with extremely advanced ways to keep an
eye on and screen online content. They copy one another‟s screening methods and employ
government-sanctioned observers to influence Web discussions and to intimidate enemies of
the state. Astute dictators can commission their own portals and leave out information that is
potentially divisive. They give the illusion of uncensored online content even as they secretly
restrict access to politically contentious sites.
First 8 points – 75 words when expressed succinctly
© RAFFLES INSTITUTION 2010

12. AQ

Sheridan argues that the Internet plays a crucial role in building democracies, whereas
Kurlantzick is of the view that the Internet does not spread freedom.

How far do you agree with their views?

Support your answer by referring to what you have read in both passages and to the situation in
your society. [8m]

For a reasonably competent discussion, candidates are expected to:

 Make reference to and show engagement with some key ideas in the passages with
evaluation/comment, and not just pick examples to agree or disagree with. For
example:
o Passage 1, Para 2: “dictatorships…would fall with the click of a mouse”
o Passage 1, Para 5: “futility of…censorship”
o Passage 2, Para 4: “fiction of a free Internet”
 There must be clear justification of their view/position, and the justification must be
backed by concrete examples from their society. For example, in the context of
Singapore:
o The influence of OB markers on Internet discussion/dialogue
o Self-censorship
o Selective blocking of sites as a symbolic gesture of Web regulation/control =
“light touch regulation” (e.g. pornographic/illegal drugs/fanatical religious sites)
o Government using the Web for outreach, feedback
o PAP embracing the Web out of necessity in spite of initial reservations [= “savvy
autocrats” @ Passage 2, Para 2]
o Monitoring of online activity resulting in arrests, punishment, etc (recent case of
self-radicalized NS man)
o Online petitions to change/challenge/influence policy (e.g. petition re: Mother
Tongue weightage in the PSLE)
 Students should be able to show nuanced application / critical evaluation of ideas from
the text to the situation/dynamics in their own country, instead of making
sweeping/unconvincing parallels. For example:
o In Singapore, although there is a lot of online activity, not much of it actually
translates actual activism, protests (vs. Thailand)
o In Singapore, the majority of online comments tend to gravitate towards socio-
economic issues rather than real political issues
 Students can frame their responses according to various domains – e.g. social /
political / economic
 NOTE: Students are NOT required to fully agree or fully disagree with either text, unless
they can present a convincing case for doing so
© RAFFLES INSTITUTION 2010

AQ – More possible points / examples / arguments

From Passage 1 (Barrett Sheridan, “The Internet Helps Build Democracies”)


From passage Possible points / arguments / examples
A. Autocrats have Government use of Internet: Students need to be
"mastered the use of careful about making sweeping accusations of
cyberspace for “propaganda – vs.
propaganda” (lines 12- genuine effort by government to engage the ground
13)
E.g.
 “Singapore Elections” page on Facebook (fans
from both the ruling People's Action Party (PAP)
as well as opposition political parties. Almost 300
people have signed up and they include Senior
Minister of State for National Development Grace
Fu, PAP Members of Parliament Lim Biow Chuan
and Zaqy Mohamad, as well as grassroots
organisations such as Aljunied GRC.)
 Ministers on Facebook (Deputy Prime Ministers
Wong Kan Seng and Teo Chee Hean, and Ministers
George Yeo, Yaacob Ibrahim, Vivian Balakrishnan
and Lim Hwee Hua)
 Central Singapore District Mayor Zainudin
Nordin, an MP for Bishan-Toa Payoh GRC, has
received requests from residents seeking jobs. "It
is an enabling tool that allows residents to
communicate with us easily, and see what we are
up to," he said.
 Senior Parliamentary Secretary and North East
District Mayor Teo Ser Luck said a key
advantage of Facebook is that it consolidates
information from a variety of people at a glance.
"It has helped me know better what people feel
about a range of issues and how they respond to
our work," said the Pasir Ris-Punggol GRC MP
who has his own Facebook site.
 Young PAP on Facebook
© RAFFLES INSTITUTION 2010

From Passage 1 (Barrett Sheridan, “The Internet Helps Build Democracies”)


From passage Possible points / arguments / examples
B the use of online tools Hardly any of this in Singapore.
and mobile phones to
organize protesters and Even the gay rights event at Speaker’s Corner in May
project their message 2010 was touted by organizers pinkdot.sg as “NOT a
around the world (lines protest, rally or demonstration but a simple
22-23) gathering of like-minded Singaporeans”
(http://www.pinkdot.sg/index-2010.swf). Of
course, the organizers were granted a permit to hold
the event – which hardly makes it a “protest”!
C just how powerful an No “wall of water” here?: Students can reflect that
organizing and even though many Singaporeans are Net-savvy,
communicating tool the many may not be involved in activism. In fact, even
Internet is, even when those who want more say in governance & policy
limits are placed on it. are not stepping up to get involved
And when dictators fight  “SINGAPOREANS want to have a greater say
back against it, they're and be more involved in the political process
pushing against a wall of and policymaking compared to 12 years ago,
water. (lines 29-31) according to a new study. Yet, the study by the
Institute of Policy Studies (IPS) found that when
it came to actually making their views on public
policy issues known to the Government, just 8
per cent actually did so…” (ST, 2 Aug 2010)

BUT there is the possibility/threat of the Net


enabling those who are self-radicalised – e.g. NS
man Muhammad Fadil Abdul Hamid arrested on
April 4, to be detained for two years under the
Internal Security Act (ST, 7 Jul 2010)
D heavy-handed Web The MDA officially adopts a “light-touch” approach
censorship tactics, to Internet regulation in Singapore [see Annex A at
blocking many foreign the end of this document].
sites and e-mail
programs (lines 37-38)
© RAFFLES INSTITUTION 2010

From Passage 1 (Barrett Sheridan, “The Internet Helps Build Democracies”)


From passage Possible points / arguments / examples
E the futility of its Students would be hard pressed to find good
censorship. Their choice evidence of the government clamping down on the
was a binary one: accept Internet. In fact, its stance is to increasingly loosen
that the Web cannot be control & move with the times, largely in
controlled, or eliminate it recognition of the fact that (a) it is hard to police
altogether. Choosing the the Net; (b) Singapore cannot afford to be “cyber-
latter sets a nation on a landlocked” if it seeks to continue developing
path to becoming the next (economically, socially, politically, etc.)
Hermit Kingdom, a
decision that almost every  The next GE is likely to see “podcasts and
nation is unwilling to vodcasts put out by political parties as part of
make (lines 44-46) their Internet election advertising” (ST, 10 Jan
2009)
 In terms of “engaging netizens”, Minister for
Information, Communications and the Arts Lee
Boon Yang signalled a "mindset change" in a
Government that had previously kept cyber
chatter at arm's length… The Government is
now "fully into e-engagement" (ST, 10 Jan
2009)
F the path toward Students will need to cite good examples of
democracy is a long one … “democratic consolidation” – i.e. political/activism
It's during that long sites that have good, sharp analysis (vs.
process – which “grumbling” blogs, chatrooms full of vitriol but little
academics such as substance, etc.)
Huntington call
"democratic Online political sites, e.g.:
consolidation" – that the (1) http://theonlinecitizen.com/
Web's impact will be most TOC began in December 2006 with a simple
felt. (lines 47; 51-52) aim in mind: telling the stories about
Singapore and Singaporeans that weren’t
being told in the mainstream press. Blog site
which endeavours to reflect the views and
opinions of ordinary Singaporeans
(2) http://thevoiddeck.org/
This website is about the goings-on of the
Internet regarding Singapore issues both
close and not so close to our heart.

Students may also wish to reflect on the


© RAFFLES INSTITUTION 2010

“roadbloacks” facing such “democratic


consolidation” that comes not from the government
but from the people themselves, e.g.:

 Lack of political maturity  grumbling, personal


attacks, unsubstantiated accusations/criticism,
etc. – e.g. attack against Young PAP member in
early 2010 (accusing him of using his grassroots
connections to drum up business for his events
management company)
 Singaporeans still trying to decide how much
they want the government to be “hands off” –
could be a case of wanting their cake & eating it
too? E.g. in the cases of Pastor Rony Tan making
insensitive remarks about Buddhism and 3
teenagers making online racist comments:
“…when it comes to remarks on race and religion,
the instinctive reflex is to summon the authorities.
Indeed, the irony of anti-establishment netizens
clamouring for the iron fist of the law did not pass
unnoticed” (ST, 6 Mar 2010).
© RAFFLES INSTITUTION 2010

From Passage 2 (Joshua Kurlantzick, “The Web Doesn’t Spread Freedom”)


From passage Possible points / arguments / examples
A Online activists in many Sharper students would reflect on whether
places have much less activism online may actually be problematic given
freedom than they did how public such activism may be, and not only
four years ago as well. because of government monitoring or control.
(lines 11-12)
Full-time national serviceman (NSF) Muhammad
Fadil Abdul Hamid was arrested on April 4 and will
be detained for two years under the Internal
Security Act (ISA).

B monitoring and filtering Students would be hard pressed to “prove” that


Web sites (line 17) arrests of activists (e.g. self-radicalized NS man
Muhammad Fadil Abdul Hamid) was due to
government monitoring/filtering of the Internet or
online “tracking” of activists. The reports in
newspapers tend to be silent about how such
radicals are tracked down, and whatever is
reported can be taken as facts that have surfaced
after arrests are made – e.g. “Fadil began zealously
surfing the Internet for jihadist propaganda and
videos while studying in a local polytechnic… He
later made contact online with a known radical,
Anwar Al-Awlaki, and with an Al-Qaeda recruiter…”
(ST, 7 Jul 2010)

However, there have been calls/recommendations


of proactive official measures that can be taken in
the fight against state threats, and students need to
reflect on why such measures may be necessary (for
national security, racial harmony, etc.), e.g.:
“religious and community institutions, government
agencies and think-tanks should work together to
identify best practices in building "theological and
ideological firewalls". Such firewalls would help
"immunise" the wider population against violent
extremist ideas…” (ST, 8 Jul 2010)
© RAFFLES INSTITUTION 2010

From Passage 2 (Joshua Kurlantzick, “The Web Doesn’t Spread Freedom”)


From passage Possible points / arguments / examples
C use state-backed Again, students would be hard pressed to “prove”
commentators to control that this is being practiced in Singapore. They must
online discourse and avoid simplistic arguments, e.g. that the presence of
threaten political official/government websites & portals points to
opponents (line 21) the attempt to control/curtail open discourse.

Students may also consider the conservative


mindset of many Singaporeans, who actually see
the Internet as a tool for propagating
shared/desired social practices & ideals. They need
to consider if many Singaporeans actually want
some control/policing of the Internet in order to
preserve social order, impart desired values, e.g.:

Current: “Muis [has] set up an online portal for


young people to query religious teachers.” (ST, 12
Jul 2010)

Proposed: “One new idea thrown up by a


community leader who does not want to be named
is that of a national council to prevent
radicalisation. He suggests it could work like
existing national councils on drug abuse and crime
prevention, which focus on raising awareness of
these issues.” (ST, 12 Jul 2010)
D And rather than just put E.g. http://www.gov.sg/ (S’pore govt portal) –
portals in the hands of Students need to reflect on whether such
Western companies, government portals are there for authoritarian
savvy autocrats are control or for improving transparency, providing
creating their own… information to citizens, allowing feedback channels,
government-backed etc. Again, they cannot merely label a website as an
search engine (lines 24- “autocratic” tool the moment it belongs to the
25; 27-28) government!
© RAFFLES INSTITUTION 2010

From Passage 2 (Joshua Kurlantzick, “The Web Doesn’t Spread Freedom”)


From passage Possible points / arguments / examples
E much of the public…has Here again, MDA’s “light-touch” approach means
no idea how much news that Singaporeans actually enjoy a great deal of
and information they are unfiltered web content.
missing out on in their
filtered Web universe /
users often think they are
seeing the same Internet
as someone in the U.S. or
Japan or another free
country (lines 29-30; 31-
32)
F political activists [are] Similar to the approach to point A above – students
creating their own need to reflect on how the public nature of the
personal Web pages or Internet makes online activism/protest particularly
Facebook pages, activists problematic, e.g.:
in countries like Iran are  “[Self-radicalized NS man Muhammad Fadil
building the kinds of Abdul Hamid] searched for bomb-making
dossiers of information information online, and posted a self-made
about themselves (lines video glorifying martyrdom and justifying
37-39) suicide bombing.” (ST, 7 Jul 2010)
 “[27-year-old Abdul Malik Mohammed
Ghazali] was arrested…after he posted
comments related to the Youth Olympic
Games (YOG) on a Facebook group…[saying]
it was time to "burn Vivian Balakrishnan and
the PAP" (ST, 26 Aug 2010)

Students could also reflect on how such online


posts could be “outed” by fellow concerned or
offended citizens, and comment on whether this
actually underlines the Internet’s power as a
tool of democracy, giving “people power” a
boost & allowing them to participate in active &
responsible citizenry
© RAFFLES INSTITUTION 2010

From Passage 2 (Joshua Kurlantzick, “The Web Doesn’t Spread Freedom”)


From passage Possible points / arguments / examples
G It also has become easier Students need to recognize that, for opposition
for the security services parties or “dissidents”, the Internet is a double-
to follow dissidents, since edged sword. While it offers them a fast, effective
they can track groups of way to reach a bigger audience, it also opens them
them online rather than up to scrutiny by the authorities.
having to infiltrate
meetings of dissidents in But again, students will not be able to “prove” that
people's homes or in bars the PAP has attempted/is attempting to “infiltrate”
(lines 40-42) any opposition/dissident group, whether online or
offline – they should avoid such baseless
accusations!

Some examples of opposition online presence:

Singapore Democratic Party: http://yoursdp.org/

Workers’ Party: http://wp.sg/ + Youth Wing:


http://wpyouth.sg
[On Twitter: http://twitter.com/wpsg/]
© RAFFLES INSTITUTION 2010

Annex A: http://www.mda.gov.sg/PUBLIC/MEDIACLASSIFICATION/Pages/Internet.aspx

Light-touch Regulation
 In regulating the Internet, MDA adopts a balanced and light-touch approach to ensure
that minimum standards are set for the responsible use of the Internet while offering
maximum flexibility for industry players to operate.
 MDA also encourages industry self-regulation and public education efforts to
complement its co-regulatory approach.

MDA’s Guidelines
 MDA has established an Internet regulatory framework to promote and facilitate the
growth of the Internet while at the same time safeguarding social values, and racial and
religious harmony.
 MDA's focus is with the provision of Internet content to the public, and one of its main
concerns is the ease of access to pornography on the Internet, especially by the young.
 MDA’s guidelines do not cover webpages operated by individuals and personal
communications such as email and instant messaging.

Cyber Wellness
 MDA recognises the need to educate the public on the positive as well as the hazardous
aspects of the Internet. MDA’s MediAction programme seeks to promote media literacy
and discerning use of the media. A key component of this programme is the promotion
of cyber wellness.
 Cyber wellness refers to the positive well-being of Internet users and a healthy cyber
culture for the Internet community. It involves an understanding of the risks of harmful
online behavior, an awareness of how to protect oneself and others from such behavior,
and recognition of the power of the Internet to affect oneself and the community at
large.
 The four core values underpinning the cyber wellness vision are:
· balanced lifestyle
· embracing the Net and inspiring others
· astuteness
· respect & responsibility

Internet Filtering Features and Software


 MDA has received feedback outlining the concerns of parents and schools about the
dangers of the Internet, especially its impact on the young.
 To address these concerns, the MDA has worked with the three Internet Service
Providers (ISPs) in Singapore to provide optional family access networks that parents
can subscribe to for their children. All the ISPs launched their family access networks in
1998.
 Meanwhile, stand-alone filtering software such as CyberPatrol and NetNanny are also
available on the market.
 With these services and such software, parents can be better equipped to help their
children access the Internet safely.
RIVER VALLEY HIGH SCHOOL
Year 6 Preliminary Examination

General Paper
PAPER 1
3RD SEPT 2010
FRIDAY

1 ½ HOURS
NAME

CLASS

INDEX NO.

QUESTION
INSTRUCTIONS TO CANDIDATES
NUMBER

DO NOT TURN OVER THIS PAGE UNTIL


INSTRUCTED. CONTENT
( 30 marks)
Write your name, class, index number and question
number clearly on all pages of your answer script,
including the cover page. LANGUAGE
(20 marks)
Write your answer legibly in dark blue or black pen.

Answer one question.


At the end of the paper, submit your answer script, the TOTAL
question paper and your essay plan to the invigilator. (50 marks)
(Note that 20 out of 50 marks will be awarded for
your use of language.)

[Turn over

RVIP 2010 Preliminary Examination Page 1 of 2 Year 6 General Paper


Answer ONE question from this Paper.
Your answer should be between 500 and 800 words in length.

1. “Olympism is the marriage of sport and culture.” (Juan Antonio Samaranch). Is this
still true?

2. What, in your opinion, is a purposeful life?

3. The arts today have been exploited for commercial ends. Discuss.

4. Do you agree that there is too little privacy in today’s digital age?

5. “Let the people think they govern and they will be governed.” (William Penn) How far
is this true in Singapore?

6. “Nothing that is worth knowing can be taught.” (Oscar Wilde). Discuss this with
reference to formal education in your country.

7. Envy brings progress. How far do you agree with this statement?

8. Tourism in Singapore is a mixed bag of blessings. Comment.

9. “The march to globalisation has also meant the marginalisation of women and girls.”
(Hilary Clinton) Is this an accurate statement?

10. Obedience is overrated and outdated. Discuss.

11. In today’s world, are we too pragmatic for our own good?

12. How successful has your society been in embracing the disadvantaged?

RVIP 2010 Preliminary Examination Page 2 of 2 Year 6 General Paper


1. ―Olympism is the marriage of sport and culture.‖ (Juan Antonio Samaranch).
Is this still true?

Question requirement/s: Show how this was true at some point, and evaluate
if this still true or if it no longer applies. If it no longer applies, why not?

The Olympic Movement is a social movement, attempting to ―contribute


to building a peaceful and better world‖ as stated in the Olympic Charter.

Key words/terms:
Olympism:
- Coined by the founder of the Modern Olympics Baron Pierre de Coubertin
(1887-1937)
- He wanted a universal philosophy that would have sports & physical activity at
its core
- Aimed first to promote the harmonious development of individuals to enhance
the society since a better society has to be made by better persons, and
finally to build a peaceful and better world as the world consists of many
societies.
- Therefore, the core of Olympism lies in a harmonious development of
individuals & the key to this is education
- Olympism relies heavily on sport as its main practical tool to reach noble
goals, thus forming a link between the social movement and sport.
- Olympism expects many great valuable things from sport and uses sport to
cultivate various fundamental social values and universal ethical principles.

Culture:
Students can discuss individual & social values such as:
- justice, fair play, solidarity, friendship, honour for talent, healthy life style, non-
discrimination, respect for those who do not win, equality for all
- Sports can also be a platform to achieve peace, international understanding,
promotion of democracy

Marriage:
- Blending together, positive combination/relationship

Arguments to support the statement i.e. that Olympism continues to be the


marriage of sport and culture
- Olympic athletes have always been and still continue to be role models
for young people esp. Aspiring athletes e.g. Michael Phelps, Ian Thorpe,
local YOG athletes such as Rainer Ng – their excellence in their chosen
sporting fields is held up for others (especially young people) to emulate
- Many athletes become ambassadors for charities/good causes (both sport &
non-sport related)
- Their perseverance, sacrifice, respect for tough training regimens etc are
held up as positive qualities for individuals and larger communities to aspire
towards – the notion that sports with its emphasis on physical conditioning
makes a person ―better‖ in terms of character – free of bias, someone who
respects hard work, justice etc
- Olympics & YOG continue to represent the pinnacle of sporting achievement

– a way out of social problems such as poverty, drugs for people in Africa,
South America etc.
For female athletes especially in countries where women do not enjoy equal
rights, sports meets such as the Olympics are a platform for proving their
abilities. Sports with its emphasis on fair play, respect for talent etc can provide a
platform for education e.g. sports can serve as a powerful rehabilitation
tool in prisons/reform centres/ schools => e.g. can be tied to civics & moral
education programmes to teach specific values, or Olympic museums/
displays can serve as tangible reinforcements of positive social values.

Note that this marriage of sports and culture is very idealised and might have
been true in earlier contexts such as Ancient Greece, or even Victorian England,
however in modern times, it might not hold true anymore or not as much

Arguments that might show that the statement is no longer true


- Sports don‘t seem to be a platform to reinforce positive social values any
more.
- The values associated with Olympism in the contemporary world have
changed, mostly in a negative way
- Increasing emphasis on the commercial rewards of sports esp at major
international sporting events such as the Olympics => chasing gold medals,
which in turn secures product endorsements & sponsorships => individual
glory at all cost rather than positive values of fair play and respect for talent
=> corporations & sports agents exist on their own apart from society =>
sports as a global business
- Increasing cases of doping, cheating, scandals in the world of sports =>
Bribery in international sporting arenas such as cricket, F1 racing & soccer =>
sports for profit & not national glory

The increasing politicisation of sports => Increasing incidence of ethnic


discrimination in sports e.g. African & Muslim players in French & German
national soccer teams have been humiliated and discriminated against & also
awarding the right to hold major sports meets to certain countries => choice of
Beijing for 2008 Olympics was initially condemned by US and other Western
countries => no sense of fair play or integrity

Increasing lack of professionalism among so-called sporting heroes (who


once upon a time would have been mindful that their personal conduct was
equally important as their sporting performance) => sex scandals, bribery &
doping scandals
Common mistakes
- Simply listing examples of poor sportsmanship
- Focusing only on Olympics (not so much the spirit of Olympism) and sports
- Ignoring ―culture‖ and its implications
- Little analysis and evaluation e.g. if more people cheat at high profile sporting
events like the Olympics, what does that imply? What might it reflect about
our values?
2. What, in your opinion, is a purposeful life?

Question requirements
Purposeful life => a life with meaning, direction, focus, has significance for the
person himself or for people around him

- Must set out criteria that define a purposeful life

a) Advocating a good/noble cause and/or


b) Helping others (charity work) who are disadvantaged or in need

=> Positive contribution to society


Egs:
- environmental activists like Jane Goodall, Wangari Maathai,
- social activists like Mohd Yunus & Bono who actively campaign to help poor
people (micro-financing, alleviating 3rd world debt),
- Warren Buffet & other billionaires like Bill Gates and Steve Jobs who have
promised to donate most of their fortunes to charitable causes/foundations to
help improve childrens‘ lives & education

=> These people do not just live self-absorbed lives; their work has tangible
impact on people‘s lives

Other examples:
- Mother Teresa
- Ms Braema Mathi (Singaporean activist who fights for equal opportunities for
women and for the rights of foreign domestic workers) &
- Mr Brenton Wong, Singaporean HIV patient & activist who fights for greater
acceptance of HIV infected people in Singapore

c) Public service
- going into civil service or politics to serve the people‘s needs & interests / fight
political injustice

Egs:
- the late Dr Goh Keng Swee,
- Minister Mentor Lee Kuan Yew
- Gandhi
- Nelson Mandela

=> people who fought for independence & national development


=> espoused a clear cause, willing to be jailed for it => walk the talk
=> created a better society for their people

d) Having a dream & working towards it


Egs:
Steve Jobs &
Bill Gates

 vision of making computing personal => their companies‘ personal computers


and other devices have become such an integral part of our daily lives – we
use them to work, communicate and entertain ourselves

Good scripts will be able to see that:

 ‗purposeful‘ is subjective and depends on the culture and society one lives in

 The notion of ‗purposeful‘ has changed/evolved over time

 achieving material wealth & a high status in life could give one a purpose to
live and work, but unless that wealth and status is put to use helping others,
one‘s life might not be as purposeful as those who help people or make a
difference.

Weak scripts:

 example-based essay
 anecdotal evidence – inability to draw on real-life, concrete examples of
people who exemplify various aspects of ‗purposeful‘
 vague concepts such as ―living according to morals‖ & ―seeking
progress/improvement‘ – no explanation of how a moral life is purposeful or
defining what sort of progress - missing link to question
3. The arts today have been exploited for commercial ends. Discuss.

Essay should define the key terms of ‗the arts‘, ‗exploited‘ and ‗commercial ends‘.

The discussion of whether the various collaborative efforts by government bodies


(MICA), agencies (NAC) and even advertising agencies should investigate
whether the outcome could constitute exploitation. The abuse of the arts for the
sake of profit is one of the possible interpretations.

Possible approach: the ideals of the arts have been compromised through cheap
commercialization that makes a mockery of the true art form. Tattoo artists who
offer to ink replicas of classic masterpieces, pop songs that integrate classical
pieces along with ballet classes that seek only to enrich their coffers are
examples.

Even when the true art form has not been brutalized, the performance arts have
been overly commercialized, with high ticket prices placing profitability ahead of
mass enjoyment.

Auction houses such as Sotheby‘s have also taken advantage of the rising
interest in art to profit from it, for the increasingly obscene amounts of cash
offered for art has made it a recession proof investment opportunity. The
outcome is that these highly priced pieces of art have been hidden away to be
unviewed at yet another auction, denying the masses the chance to view and
appreciate them.

Governments have also participated in the exploitation. The Esplanade was built
to draw in the tourist dollar, not just to create a venue to celebrate the arts.

Yet, the arts have to adapt to a new world, to new expectations and to new forms
of the media to remain relevant. The arts still represent the aesthetics, and
today‘s world offers opportunities to showcase the arts in all the different form
and also encourage adaptation, modification and hence regeneration of the arts.
4. Do you agree that there is too little privacy in today’s digital age?

Key words / phrases:


 privacy: the ability to keep secure information about oneself; to be free from
unsolicited contact & associations
 digital age: an era / society characterised by the manipulation of information (ie:
focus on Information & Communications Technology)
 too little: not enough

Main contention:
 We do not enjoy enough security / protection of our private lives / information
today, when ICT is so pervasive / intrusive

Main reason for low / mediocre marks:


 failure to correctly identify main contention
 many essays only discussed areas where privacy has been lost, and where
efforts have been made to boost security
 Answering a different Qn (see earlier), not focusing sufficiently on whether we
have ―too little‖ privacy, more interested in listing areas where it exists / doesn‘t
exist
 Inaccurate definition of digital age: extended to include technology in general, or
a really specific application that isn‘t clearly linked to ICT (eg biometrics) or the
effects of the mass media (ie: TV is not part of digital age)

Questions to consider:
 WHY is privacy seen as positive? (as Q implies) Are there situations in which too
much privacy leads to detrimental costs?
 WHO are the interested parties in making or refuting this claim? Is ICT the only
contributor to loss of privacy?

Making the case for “too little” privacy – establishing criteria:


 ―too little‖ implies negative implication
 So, argue that there are high, significant costs of privacy loss – at expense of
other important things (eg: national security)
 the inability of the average user / consumer to protect her information with current
levels of controls

Possible arguments

No! Privacy controls are adequate; users are incompetent / uneducated:


 Most privacy loss stems from user irresponsibility
- failure to educate self on privacy controls
- Failure of parents to guide children (sexting, social networking, etc)
- Failure to educate self on malicious possibilities like computer viruses, phishing,
etc

No! In fact we suffer from the effects of too much privacy:


 tools of the digital age fatally combine (apparent) anonymity with rapid and
permanent dissemination; tracing is possible but often too late. Costs?
- Child sex predators
- Cyber-stalking / bullying
- Misinformation b/c instant, democratic publication of info, hard to check source,
vested interests, info gets repeated rapidly
- Wikileaks (costs?)

Yes! Too little privacy for the average user:


 almost impossible to secure all personal information:
- Mandatory submission of personal details for participation in e-commerce, social
networking sites, etc
- constant tweaking of privacy controls on social networking sites
- lack of transparency abt sale & transfer of information (costs?)
5. ―Let the people think they govern and they will be governed‖ (William Penn).
How far is this true in Singapore?

Essay requires students to identify that the quotation refers to democracy and
how it provides people with an illusion that they participate in governance.
Because of that illusion, the population ends up docile and obedient.

Should discuss in relation to the social scene in Singapore (or another cty).

Consider: are people compliant b/c of democracy?

Good essays demonstrated:

 Awareness of the political scene, NMP scheme, opposition parties, policies,


OB markers, GRC, defamation suits and the international standing of
Singapore as a democracy.

 General response of the people, changing demographics, immigration,


materialism, economic goals, psychological profile etc.
6. Nothing that is worth knowing can be taught (Oscar Wilde). Discuss this with
reference to the formal education system of your country.
 Interpret the quote in the context of our edn system.

 Nothing => absolute

 Worth knowing => what is meaningful/significant enough/has deep long-


lasting value (up to students to define what is worth knowing)

 critical thinking & analytical skills

 creative thinking skills

 interpersonal skills

 time management skills

 crisis management skills

 digital literacy

 financial literacy

 home economics

 moral values (integrity, compassion, generosity)

These are the skills & traits worth knowing & having as they get one through daily
life, and ensure survival in life and in our jobs.
Formal education system => discuss curriculum (choice & range of subjects),
assessment (range & diversity of exams & tests) & methods of teaching
Statement is true
Our education system (from pri to pre-university) focuses largely on content
acquisition
Heavy emphasis on academics => theories, concepts, laws & theorems
Content is available not just from teachers but also from new media (constantly
evolving) => renders the knowledge taught in classrooms as obsolete
Ultimately pure content is of little practical use in the real world e.g. complex
Maths & Science are almost never used by us when we land jobs in areas such
as human resource, public relations & administrative work. Apart from doctors,
lawyers and engineers who will use the theories they learn, few of us will ever
need to use specific historical or geographical knowledge to get by in our jobs

There are more important & useful & valuable skills (abovementioned list)
Our system doesn‘t really teach the abovementioned valuable skills.
We have tried to provide a platform to help students experience/come into
contact with them but these tend to be token measures => not sustained
throughout all schools and over the entire system.
Project-based learning, courses/workshops in social & emotional learning,
financial literacy packages which are designed to be covered during civics and
moral ed lessons => token attempts => not actually teaching students
Statement is not true
Important knowledge & skills can be taught & our formal education system
recognises this and has taken steps to incorporate the abovementioned skills
For example, maths & science are extremely significant for the development of
our economy
Innovation that leads to increased productivity is seen as the most important way
to generate economic wealth for us
Maths and science underpin many innovations — basis for much of modern
society, from the gadgets people use to the ways in which people interact with
one another and the way in which they think.
Hence MOE invests in the teaching of maths and science (labs, teaching
software, Olympiads & various other competitions like Tan Kah Kee Young
Inventors, scholarships from DSTA….)
Subjects like Project Work & Science Practical Assessment have been
introduced to provide authentic platforms for students to acquire higher order
critical thinking skills, creative thinking
Assessment is not wholly pen & paper => coursework, independent study &
research projects are more challenging assessment forms that our system uses
to teach students the abovementioned skills
CCA programme => teach students resilience, tenacity, confidence and
perseverance, which prepare them to adapt and thrive in a rapidly changing
world
Community Involvement & Service Learning Programmes => nurtures our
students to become socially responsible and develops their sense of belonging
and commitment to our country. Students also learn the value of service. Service
Learning is an approach where CIP participants not only serve the community,
but also learn to identify with the needs of the community and to reflect on their
own experience in working with the community so that they have a better
understanding and appreciation of what it involves
National Education => aims to develop national cohesion, cultivate the instinct for
survival as a nation and instil confidence in our nation's future. It also
emphasises on cultivating a sense of belonging and emotional rootedness to
Singapore
Common mistakes
 Over-generalisations

 our education system does not cater to students‘ interests

 our education system produces students who do well in overseas


competitions – link?

 our society values important knowledge & skills

 specific types of knowledge & skills not discussed.


 merely asserting that maths and science are worth knowing

 hierarchy where maths and science are worth knowing but languages and
humanities are not so important => basis for this?

 Starting with a description of how Singapore has moved from a fishing


village to a modern city => so what? Or explaining education system as
merely ―pri , sec and JC‖.
7. Envy brings progress. How far do you agree with this statement?

Key words / phrases:


Envy: grudging admiration for and desire to achieve or possess something exhibited by /
belonging to another

Progress: growth/development in a positive sense

Main contention:
Our envious/jealous pangs give rise to/catalyse progress rather than impede/retard it

Main reason for low / mediocre marks:


 simplistic, inaccurate causal claims (ie, only stipulating envy as impetus, without
acknowledging role of other factors)
 little examination of HOW envy goads progress; mostly description of the
progress itself (ie became history/econs/SS essay)
 limited scope, focusing on personal level

Questions to consider:
 WHAT is the r‘ship b/t envy, fear, pride, jealousy, greed?
 WHAT aspects of progress: financial, moral/ethical, social…
 WHO would endorse this statement? WHO would repudiate it?
 WHO benefits from said progress? All always benefit equally?
 WHEN does envy cease being beneficial, if ever? WHY?

Possible arguments

AGREE! Envy encourages progress


 commercial/economic stimulus: almost the entire basis of advertising and the
manufacturing/industrial complex is the constant build-up of insecurity based on envy of
others‘ looks, material goods
 when one‘s economy is thus centred, economic progress, industrial productive
capacity is easily raised
 scientific/technological: The space race of the 1960s

AGREE! Envy  competition/rivalry  greater achievements


 personal level: students, colleagues, companies, nations…
 Prevents complacency, keeps us on our toes
 Progress through visible, tangible or measurable areas, eg results, monuments,
military buildup, etc

DISAGREE! Envy impedes future economic progress


 New knowledge-based economy, creative economy, call for intellectual sharing,
co-operation and openness, traits discouraged by the traditional cutthroat one-
upsmanship of industrial economies; envy has served SG well, but can only bring us so
far

DISAGREE! Envy impedes social progress


 this is related to the role of envy in retarding social progress: envy breeds
ungraciousness (fear of losing out)
 ppl can‘t compartmentalise lives (ie constant competition at work/school does not
automatically become benevolent charity or graciousness afterwards)

 Envy breeds fear of failure, loss of face  has historically led to excessively
nationalistic displays that have severely damaged progress (eg USSR‘s Potemkin
villages, China‘s equivalents, N Korea‘s…well, everything)

DISAGREE! Envy impedes environmental progress


 Envy as the driver of the manufacturing economies (keeping up with the
Joneses) creating and feeding material wants, constant upgrades  irresponsible
demands on the earth

Some closing thoughts:


 Envy can provide impetus for improvement but may be short term and bear too
heavy a cost (social, envr)
 aspects of progress that are intangible (graciousness, equality, etc) don‘t appear
to be motivated by envy, and in fact can‘t be duplicated by it
 Interesting that many religions frown on envious feelings, while corporations (and
perhaps nations) thrive on it – this indicates the nature of its mixed blessings
 the overwhelming and growing popularity of modern events like TED (spilling
over into TEDx) suggests that educated, skilled professionals are leaning
towards a more collegial, cooperative framework towards social and even
technological process.
8. Tourism in Singapore is a bag of mixed blessings. Comment.

 Mixed blessings => along with the many benefits that tourism has brought, it‘s
also created problems/concerns.

 Students need to be clear about the specific benefits & problems associated
with tourism

Economic Benefits

 generates revenue
o In 2007, more than 10 million tourists visited S‘pore which earned the
country around S$13 billion in tourism receipts.
o In fact, we aim to raise tourist arrivals to 17 million by 2015 with a view
to raise tourism revenue to S$30 billion.

Social Benefits
o revenue generated has allowed us to consistently maintain a high
standard of living which manifests itself in terms of cleanliness,
pollution-free environment despite large urban settlement and
industrialization, modern infrastructure and high-end transportation.

o Job creation in sectors such as food and beverage, entertainment,


budget air travel, retail & services.

o Entertainment, arts & cultural scene have both benefitted from the
tourist dollar
 more international productions & exhibitions (puts us on the
world map)
 more night-spots & world class events (F1, Singapore Biennale
etc)
 People living here benefit from these too, not just tourists

However

 Tourist dollar is not something all businesses experience


e.g. 57,000 tourists came for F1 over the past 2 years, spending about
S$260 million
BUT retailers in Chinatown & Suntec City have seen business drop by
some 60% - F1 cited as main cause despite current peak tourist season.

 Is the huge amount of money spent on events such as YOG worth it?
Couldn‘t that 387 million have been spent on improving domestic facilities?

National identity & cultural identity have been compromised in the bid to cater to
tourists
 Inauthentic cultural preservation – note the examples of Chinatown & Malay
Village) => artificial conservation and commercialisation of these places =>
They don‘t attract tourists in sufficient numbers => Malay Village has been a
commercial failure

 Historic areas/buildings torn down to make way for glitzy malls or modern
highways => good that tourists see we have first class transport or shopping but
the heritage and history lost is irreplaceable e.g. old National Library.

 New places of interest such as Integrated Resorts have stirred up controversy


& social debate about the evils of gambling

Common mistakes

- listing places & events that tourists visit & attend


- unsubstantiated speculation about how locals lose out to tourists in terms of
paying more for goods or being treated badly by service personnel
- assertions about how the IRs have ruined our lives
- tourists bring pollution
- no clear stand on the issue -> stems from an inability to analyse and evaluate
our current situation
9. “The march to globalisation has also meant the marginalisation of women
and girls.” Is this an accurate statement?

Key words / phrases:

 march to globalisation: the unstoppable, almost military advance towards


integration of societies, cultures, economies
 marginalisation: exclusion, rendered powerless

Main contention:
 Globalisation, for all the progress it brings, is also responsible for diminishing the
rights of females.

Main reasons for low / mediocre marks:


 failure to clearly, causally link the marginalisation of females specifically to
globalisation
 unexplained, unsubstantiated claims (eg globalisation  increased education for
girls)

Possible arguments

No! Globalisation has enabled women to participate more equitably in the


global economy
 Research strongly suggests that economically stable females strengthen overall
familial and social stability.
 Globalisation, most recently in the form of micro-financing, has enabled a greater
number of women to become successful small-time entrepreneurs, supporting
families, removing them from dependence
 Manufacturing economies, flourishing under globalisation  feminisation of the
global workforce

No! Globalisation has better educated women about their rights and offered
avenues to exercise them
 ICT has brought a greater global awareness of the plight of women experiencing
deprivation and sometimes violence. Platforms like Youtube and Twitter have
brought us the horrifying reality of their suffering, and international outrage has
brought some pressure to bear on the respective authorities.

Yes! Globalisation has encouraged, even necessitated the employment of


cheap labour
 More open economies mean sourcing for ever-cheaper labour and these are
inevitably women, esp for manufacturing industries, as they form, a flexible labour
pool that can be easily laid off (Philippines, Mexico, Chile, etc). Female children are
often the hardest hit.

Yes! Globalisation has encouraged polarisation of labour


 Although employed in greater numbers, women are mainly concentrated in low-
skilled, menial, and repetitive jobs and in the lower echelons of the industrial
hierarchy
- Corporations and govts have little incentive to provide education and training
opportunities. Tog. with employer bias, and limited mobility due to family
responsibilities, women remain these lower rungs of employment
- Eg in Pakistan, women in the textile industry are paid less than the men and are
forced to work overtime but not paid accordingly. They are often unregistered in
various factories and are not provided any additional facilities to which only the
permanent or registered employees are entitled.

Yes! Globalisation paves the way for the abuse of women


 globalisation‘s ―free flow‖ of goods, services, labour, is not truly an equitable
relationship. As rich/poor divides widen, domestic workers are increasingly in
demand, and correspondingly, so are cases of domestic worker exploitation
 on a related note, other migrant female workers, whose numbers are rising,
including farm workers and sex workers, are at high risk of indentured servitude
and trafficking

Closing thoughts:
 the influence of globalisation on long-lasting benefits to the roles and rights of
women, is more a reflection of wealth and cultural inequality than it is of gender
bias  it is not primarily globalisation that is to blame for marginalisation of
women and girls, but its priorities and bias mirror those extant in the globalising
culture/force
10. Obedience is overrated and outdated. Discuss.

Key words / phrases:


obedience: adherence, submission to a rule/law/norm
overrated: credited more than its due
outdated: irrelevant

Note: “and” operator, but don‘t need to fixate on it

Key contention:
Submission to law/authority/norms is irrelevant and not very beneficial in meeting today’s
needs. (Implied assumption that obedience WAS much valued/prized and brought
positive outcomes in the past)

Questions to consider:

- if indeed our views on the roles/contributions of obedience have changed, WHAT


factors have shaped the change/s?
- are these factors universal or subjective?
- WHO is paying obeisance? WHO is receiving it?

Main reasons for low / mediocre marks:


- too general, too hypothetical, lacking specific examples
- no past-present comparison
- limited scope – mostly on anecdotal, personal level

Possible arguments

Yes, obedience valued in past but not as much today

 Obedience was seen as a necessary and desirable quality in menial/blue-


collar workers / extremely class-restricted, hierarchical societies (explain
WHY desirable!)
- eg wage labourers, factory workers, feudal society, faith-centred communities
- H/w, blind adherence to rules does not serve us well in the new economy where
creativity and lateral thinking are crucial. (make ref to and explain how this belief
is acted upon in the edu system not just in SG but other devd countries)
- In addition, higher education and prevailing cultural norms in most developed
nations (though not all) encourage the notion and inevitability of class mobility
(social climbing). This is inextricably linked to meritocracy  many young ppl
especially find it deeply unappealing to be bound by rules / norms that cannot be
questioned / that they cannot find an acceptable, logical rationale for

 In religious societies, obedience seen as outward sign of the elect /


sociologically, an implicit sign of identity  impt for tribal identification,
unity
- no longer a useful form of primary identification for most, since many of us have
multiple, borderless, simultaneous identities (national, ethnic, religious, gender…)

No, obedience valued just as much today, if not more so


 Institutions spend a great deal of time and energy on inculcating obedience
– adherence to explicit rules as well as implicit social/cultural norms
- Schools are a prime example of this. The prevalence of standardised testing
reflects a certain reluctance to credit individuality. Even away form academic
grading, schools from as early as kindergarten emphasise socialisation skills,
which are all about obeying social cues and norms

 Ultimately, the unprecedented influence of the mass media has encouraged


new generations of conformity which have been further flattened by
globalisation’s culturally homogenous fist (conformity as the most prevalent
form of obedience to modern society)
11. In today‘s world, are we too pragmatic for our own good?

Essay should define what pragmatism is and what it entails.


Discussion should refer to current challenges / trends / developments.
Should discuss in relation to the proposal that there are negative outcomes to
being pragmatic.

Common mistakes:
 Poor definition of what being pragmatic entails:

Practical point of view or consideration/s


Pragmatic choices made usually have tangible outcomes

 Poor illustration of what ‗for our own good‘ is:

- Negative outcome
- Undesirable consequences

 Generally, a lack of understanding of current affairs / history / events


identified essays as flawed.

 Lack of knowledge of Singapore in terms of:


Legislation / decisions
Rationale for the decisions
Outcome of the decisions

 Decision to allow legalised gambling in Singapore – ‗integrated resorts‘ –


casinos to attract tourists…(pragmatic considerations)

…due to the economic benefits as well as the challenges posed by the


deterioting world economy...(today‘s world)

…outcome is that the problem of gambling addiction is created…(for our own


good)

Good essays were able to:

• Discuss the validity of pragmatism as a philosophy for decision-making

Contrast pragmatism with idealism, dreams and hopes

Demonstrate how particular advancements / achievements that were for


tangible outcomes turned out to have negative impact/s
12. How successful has your society been in embracing the disadvantaged?

Students need to judge or measure the outcomes of legislation, policies, public


campaigns and efforts by the government or the NGOs to help integrate the
disadvantaged into society.

Need to define ‗society‘ – must be sufficiently large

Should distinguish between tolerating and fully integrating

Define the criteria for being disadvantaged

Weak essays assumed that:


 Presence of legislation / strategy / organisation / scheme / awareness =
‗embracing‘

 Minority group = ‗disadvantaged‘

 Exaggeration: a society so perfect that every policy has worked so well that
all the disadvantaged have been taken care of. Err…. then why do they still
exist?

 Assumption: all the handicapped need money, all the old are poor and infirm,
all the poor can be helped simply because of subsidies in education…

 Mentally handicapped vs low academic ability vs low academic qualifications

Good essays were able to demonstrate:

 Critical differentiation of acceptance vs integration

 Separation of historical vs current outcomes


(ie, problems of integration in the past was solved so today the problems are
gone)

 Critical assessment of government policies: intended outcome vs actual


(eg Ethnic Integration Policy for HDB flats imposes a quota on each race for the
composition of each block of flats – on paper there is a mix but in reality there is
very little interaction, if at all)
 Skepticism and criticism instead of propagandist proclamations about the
wonderful outcomes of NGOs, government policies and grassroots
organisations, creating a Utopia that is reminiscent of Huxley‘s Brave New
World.
RIVER VALLEY HIGH SCHOOL
Year 6 Preliminary Examination

General Paper
PAPER 2
3RD SEPT 2010
FRIDAY

INSERT
NAME

CLASS

INDEX NO.

INSTRUCTIONS TO CANDIDATES

DO NOT OPEN THIS BOOKLET UNTIL INSTRUCTED.

RVIP 2010 Preliminary Examination INSERT Year 6 General Paper


Passage 1 A C Grayling writes…

1 Prince Jefri Bolkiah of Brunei had to auction his possessions after the collapse of his 1
business empire in 1998, leaving him with debts of three billion dollars. It is unclear why a
member of one of the world’s richest families chose to engage in business in the first place
(to describe the Brunei royal family as rich is like saying Mount Everest is a pebble), but he
did it anyway, perhaps for fun. When bankruptcy followed, he quarreled with his family, who 5
would not rescue him. In the end, he had to accept the sale of his possessions, and an
allowance from the Sultan of Brunei of a mere 300,000 dollars a month.

2 Hundreds flocked from all over the world when news of the prince’s auction spread, to buy a
piece of royalty and to slake their fascination with the idea of the mammoth luxury involved.
On sale were gold-plated toilet-brush holders, gigantic marble jacuzzis, several grand 10
pianos, dozens of huge television sets, and thousands of items of jewellery. An unkind
observer might say that what the purchasers and the prince thought of as luxurious was
what others would describe as ostentatious and excessive – invariably the way, so such an
observer might say, with those who have more money than taste.

3 Citizens of ancient Sybaris on the Gulf of Tarentum became a byword for luxury, so wealthy 15
and easeful was their life. They demanded their dinner invitations one year in advance so
that they could properly choose what to wear. As imperial Rome waxed fat on its conquest, it
followed the Sybarites’ example, wealthy citizens holding stupendous banquets at which
guests periodically quit their couches to vomit and make room for yet more food and wine.
Seneca criticised not just the excess of his contemporaries but the effect on their 20
sensibilities. In the third book of his Questions about Nature, he describes the way Roman
epicures liked to watch surmullets die on the dinner table before them, so that they could
enjoy the beautiful changes of colour displayed as the fish suffocated.

4 The idea of luxury as excess – and later, under Christianity, sin – was marked in the
distinction drawn by Roman thinkers between luxus, meaning abundance and pleasure, and 25
luxuria, meaning grossness. It is the latter that weakens minds as it does bodies, pulping the
tissues of thought as it softens and bloats flesh. Indeed, luxuria results from a failure to see
that refinements of sensuality are not the same as ostentation and excess, and are indeed
negated by them.

5 By sensuality is accurately meant the physical pleasure taken in colours and textures, tastes 30
and sounds – things that delight the senses, charm and heighten them, offer them the best
of what they are apt for. The eye enjoys light, tonalities, hues, shapes; the sense of touch
loves silk, skin, warmth and coolness, the roughness of sand and the gliding face of marble;
the ear loves harmonies and concords, melodies and rhythms, and the calmer sounds of
nature such as falling water, air stirring among leaves, birdsong. 35

6 When the pleasures of the other senses are added to gustation, the result is as writer
Christopher Isherwood describes in recognising that the little dishes and delicate
instruments of a Chinese banquet are like artists’ materials, as if the assembled company
were going to paint rather than eat. Among the dishes are various types of food present for
texture more than taste, such as water chestnut and bamboo. At informal dinners, the 40
Chinese are not so restrained; they relish instead the sight and aroma of rising steam,
glistening edibles, abundance – as luxus, not luxuria.

7 But the senses are only conduits, and their work would have little lasting value if it did not
serve something of greater significance – as materials of mind. The senses’ pleasure in
colour and melody is the mind’s pleasure in pictures and music, and they in turn most often 45
(though not invariably, because the mind enjoys abstraction too) prompt thought. And
thought is the greatest luxury of all. It fills immensity, as Blake said; and as Emerson said, it
sets you free.

Adapted from Luxury, The Reason of Things

RVIP 2010 Preliminary Examination INSERT Year 6 General Paper


Passage 2 Peter Singer writes…

1 Is the global financial crisis an opportunity to forge a new form of capitalism based on sound 1
values? French President Nicholas Sarkozy and former British Prime Minister Tony Blair
appear to think so. At a symposium in Paris entitled “New World, New Capitalism,” Sarkozy
described capitalism based on financial speculation as “an immoral system” that has
“perverted the logic of capitalism.” He argued that capitalism needs to find new moral values 5
and to accept a stronger role for governments. Blair called for a new financial order based
on “values other than the maximum short-term profit”.

2 It is surprising how readily politicians of all parties – even strong ideological defenders of the
unregulated market – accepted the idea that the state should bail out banks and insurance
companies when they get into trouble. With the exception of a small number of ideologically 10
committed defenders of free enterprise, few were willing to take the risks inherent in letting
major banks collapse. Many feared mass unemployment, a tidal wave of bankruptcies,
millions of families evicted from their homes, the social safety net strained to the breaking
point, and perhaps even riots and a resurgence of the political extremism that brought Hitler
to power in Germany during the depression of the 1930’s. 15

3 It is no accident that the “New World, New Capitalism” symposium was held in France. The
French have always been less likely to go into debt – when they pay with plastic, they tend
to use debit cards, drawing on funds they already have, rather than credit cards. Now they
see the current crisis as a vindication of the value of not spending money that you don’t
have. 20

4 That means, in many cases, less luxury spending – something that is hard to reconcile with
the image of France as the country of fashion, perfume, and champagne. But excess is out
of style, and there are reports of cutbacks in luxury goods everywhere. But does this mark
an enduring change in values, or just a temporary reduction, forced upon consumers by
investment losses and greater economic uncertainty? Could the crisis remind us that we buy 25
luxury items more because of the status they bring than their intrinsic value? Could it help us
to appreciate that many things are more central to our happiness than our ability to spend
money on fashion, expensive watches, and fine dining? Could it even make us more aware
of the needs of those who are living in real poverty and are far worse off than we will ever be
financial crisis or no financial crisis? 30

5 The danger is that the potential for a real change in values will be co-opted, as has
happened so often before, by those who see it as just another opportunity to make money.
The designer Nathalie Rykiel is reportedly planning to show the new Sonia Rykiel collection
not in the usual vast rented area, but in the smaller space of her own boutique. “It's a desire
for intimacy, to go back to values,” she told the International Herald Tribune. “We need to 35
return to a smaller scale, one that touches people. We will be saying, ‘Come to my house.
Look at and feel the clothes.’”

6 Ah yes, in a world in which ten million children die every year from avoidable, poverty-related
causes, and greenhouse-gas emissions threaten to create hundreds of millions of climate
refugees, we should be visiting Paris boutiques and feeling the clothes. If people were really 40
concerned about defensible moral values, they wouldn’t be buying designer clothes at all.
But what are the chances of Nathalie Rykiel – or the affluent elites of France, or Italy, or the
United States – adopting those values?

Adapted from Capitalism’s New Clothes, Project Syndicate

RVIP 2010 Preliminary Examination INSERT Year 6 General Paper


RIVER VALLEY HIGH SCHOOL
Year 6 Preliminary Examination

General Paper
PAPER 2
3RD SEPT 2010
FRIDAY

1 ½ HOURS
NAME

CLASS

INDEX NO.

INSTRUCTIONS TO CANDIDATES QUESTION MARKS

1 /2
DO NOT OPEN THIS BOOKLET UNTIL INSTRUCTED.
2(i) /2
Write your class, index number and name on all the work you 2(ii) /1
hand in.
3 /1
Write in dark blue or black ink.
4 /1
Do not use staples, paper clips, highlighters, glue or correction 5 /2
fluid.
6 /1

7 /2
This Question Paper has 8 printed pages, including this
cover page. Candidates are to answer on the Question 8 /8
Paper itself.
9 /2
Additional Materials: 1 Insert
10 /5
The number of marks is given in brackets [ ] at the end of 11 /8
each question or part question.
(Note that up to 15 marks will be awarded for your use of CONTENT / 35
language.)
LANGUAGE / 15
TOTAL / 50

RVIP 2010 Preliminary Examination Page 1 of 8 Year 6 General Paper


Read the passage and answer all the questions

Answer all questions IN YOUR OWN WORDS as far as possible. Little credit will be given
to answers which only copy words or phrases from the passage. Up to 15 marks will be
given for the quality and accuracy of your use of English throughout this Paper.

Questions on Passage 1

1. Why does the writer assert that “to describe the Brunei royal family as rich is like saying
Mount Everest is a pebble” (line 4)? [2]

2. (i) According to the author, how did some people react to the news of the auction by the
prince? [2]

(ii) What criticism is being levelled at this group of people and the prince? [1]

3. Explain what the author means by “…as imperial Rome waxed fat on its conquest”
(line 17). [1]

RVIP 2010 Preliminary Examination Page 2 of 8 Year 6 General Paper


4. What does the author convey by referring to the Roman epicures (lines 22 to 23)?
[1]

5. Why is the idea of “luxury as excess” (line 24) frowned upon by Christianity? [2]

6. What is the author’s point in ending the passage with Emerson’s words? [1]

Questions on Passage 2

7. Explain the difference between the old and the proposed new system of capitalism. [2]

RVIP 2010 Preliminary Examination Page 3 of 8 Year 6 General Paper


8. Using materials from paragraphs 3 to 5, summarise what Singer sees as the possible
positive and negative implications of this new form of capitalism.

Write your summary in no more than 120 words, not counting the opening words which
are printed below. Use your own words as far as possible. [8]

The author argues that capitalism…

( words)

RVIP 2010 Preliminary Examination Page 4 of 8 Year 6 General Paper


9. Explain the tone of the author in the first line of the last paragraph. [2]

From both passages

10. Give the meaning of the following words as they are used in the passage. Give only
ONE answer, in a single word or a short phrase. [5]

(a) stupendous (passage 1, line 18)

(b) relish (passage 1, line 41)

(c) conduits (passage 1, line 43)

(d) resurgence (passage 2, line 14)

(e) vindication (passage 2, line 19)

RVIP 2010 Preliminary Examination Page 5 of 8 Year 6 General Paper


11. Grayling shares his perspective on the true meaning of luxury while Singer reminds us
how our quest for luxury has derailed us from embracing sound values. With which of
the two authors are you most in sympathy? Do you regard the quest for luxury for you
and your society as broadly beneficial or harmful? [8]

RVIP 2010 Preliminary Examination Page 6 of 8 Year 6 General Paper


RVIP 2010 Preliminary Examination Page 7 of 8 Year 6 General Paper
R

EV

EX /8

END OF PAPER

Content: 35 m; Language: 15m

RVIP 2010 Preliminary Examination Page 8 of 8 Year 6 General Paper


RVIP 2010 Year 6 GP Preliminary Examination Paper 2
Based on A C Grayling’s Luxury & Peter Singer’s Capitalism’s New Clothes

Questions on Passage 1

1. Why does the writer assert that “to describe the Brunei royal family as rich is like saying
Mount Everest is a pebble” (line 4)? [2]

Lifted from the passage In your own words


It is unclear why a member of one of the Explanation of the metaphor
world‟s richest families chose to engage in
business in the first place (to describe the Just as Mt Everest is countless times larger
Brunei royal family as rich is like saying than a pebble (½ m), likewise the Brunei
Mount Everest is a pebble), but he did it royal family is many times more affluent than
anyway, perhaps for fun. When bankruptcy the average person‟s concept of wealth. (½
followed, he quarreled with his family, who m)
would not rescue him. In the end, he had to
accept the sale of his possessions, and an OR
allowance from the Sultan of Brunei of a
mere 300,000 dollars a month. It is a gross understatement to merely depict
the Brunei royal family as rich; it is like how
Mount Everest is compared to a mere
pebble. (1 m)

Inference (Why the writer asserts so):

He wants to emphasise the idea of


overabundance. (1m)

OR

He wants to show that it is difficult for the


average person to visualise such wealth.
(1m)

2 (i) According to the author, how did some people react to the news of the auction
by the prince? [2]

Lifted from the passage In your own words


Hundreds flocked from all over the world Many people rushed to the event (½ m) OR
when news of the prince‟s auction spread, to capitalised (½ m) on the prince‟s
buy a piece of royalty and to slake their misfortune…
fascination with the idea of the mammoth by purchasing (½ m) the auctioned goods,
luxury involved. On sale were gold-plated in order to satisfy their curiosity (½ m) about
toilet-brush holders, gigantic marble jacuzzis, such material indulgence. (½ m)
several grand pianos, dozens of huge
television sets, and thousands of items of
jewellery.

Note: Students are to be sensitised to the negative connotations associated with these
purchasers‟ reaction; hence, terms like „reacted enthusiastically‟ are not accepted as
an apt rephrasal.

RVIP 2010 Preliminary Examination Page 1 of 7 Year 6 General Paper


(ii) What criticism is being levelled at this group of people and the prince? [1]

Lifted from the passage In your own words


An unkind observer might say that what the They are criticised for splurging mindlessly
purchasers and the prince thought of as (½ m) and lacking in discernment / class.
luxurious was what others would describe as (½ m)
ostentatious and excessive – invariably
the way, so such an observer might say,
with those who have more money than
taste.

3. Explain what the author means by “…as imperial Rome waxed fat on its conquest” (line 17).
[1]
Lifted from the passage In your own words
As imperial Rome waxed fat on its He means that Rome leeched (½ m) on its
conquest, it followed the Sybarites‟ example, colonies (½ m) so as to enjoy the high
wealthy citizens holding stupendous standard of life in the material sense.
banquets at which guests periodically quit (½ m)
their couches to vomit and make room for yet
more food and wine. OR
He means that Rome enjoys a high standard
of living in the material sense (½ m) at the
expense of (½ m) its colonies / people they
have taken over
(½ m).

Note: Any 2 of the 3 points will give students the full 1 m for this question; they do not need to make
any reference to Sybaris in their answer as the question does not require them to compare both
societies.

4. What does the author convey by referring to the Roman epicures (lines 22 to 23)? [1]

Lifted from the passage In your own words


Seneca criticised not just the excess of his It is to highlight how humans can be rather
contemporaries but the effect on their depraved / senseless (½ m) in our quest for
sensibilities. In the third book of his luxury. (½ m)
Questions about Nature, he describes the
way Roman epicures liked to watch OR
surmullets die on the dinner table before
them, so that they could enjoy the Our own sense of right and wrong /
beautiful changes of colour displayed as compassion and empathy can be
the fish suffocated. compromised or dulled / we become
unfeeling as we are blinded by (½ m) the
quest to be indulgent. (½ m)

5. Why is the idea of “luxury as excess” (line 24) frowned upon by Christianity? [2]

Lifted from the passage In your own words


The idea of luxury as excess – and later, The idea of “luxury as excess” is seen as
under Christianity, sin – was marked in the obscene /disgusting (1m) as it undermines
distinction drawn by Roman thinkers between rational thought (½m), makes people give in
luxus, meaning abundance and pleasure,
to / be at the mercy of / overindulge in their
and luxuria, meaning grossness. It is the
latter that weakens minds as it does physical desires. (½ m)
bodies, pulping the tissues of thought as
it softens and bloats flesh.

RVIP 2010 Preliminary Examination Page 2 of 7 Year 6 General Paper


6. What is the author‟s point in ending the passage with Emerson‟s words? [1]

Lifted from the passage In your own words


But the senses are only conduits, and He is trying to highlight that thought liberates
their work would have little lasting value if (½ m) one from the constraints of one‟s
it did not serve something of greater senses OR our reliance on such senses (½
significance – as materials of mind. The
m).
senses‟ pleasure in colour and melody is the
mind‟s pleasure in pictures and music, and
OR
they in turn most often (though not invariably,
because the mind enjoys abstraction too)
Thought allows one to be unbounded OR
prompt thought. And thought is the greatest
luxury of all. It fills immensity, as Blake said; transcend (½ m) one‟s physical limitations
and as Emerson said, it sets you free. OR sensual pleasures (½ m).

OR

Senses are only a means to an end. [1/2]

Questions on Passage 2

7. Explain the difference between the old and the proposed new system of capitalism. [2]
Lifted from the passage In your own words
Is the global financial crisis an opportunity to The old system has a lower degree of state
forge a new form of capitalism based on involvement (1 m) while the proposed one
sound values? French President Nicholas allows more room for state intervention and
Sarkozy and former British Prime Minister support. (1 m)
Tony Blair appear to think so. At a
symposium in Paris entitled “New World, OR
New Capitalism,” Sarkozy described The previous system of capitalism is ethically
capitalism based on financial speculation skewed (1 m) while the new proposed one is
as “an immoral system” that has ethics-driven. (1 m)
“perverted the logic of capitalism.” He
argued that capitalism needs to find new OR
moral values and to accept a stronger role The previous model of capitalism is overly
for governments. Blair called for a new focused on quick earnings / returns (1 m),
financial order based on “values other than while the new one takes ethics into account.
the maximum short-term profit”. (1 m)

Note: Students have to pick out a contrasting pair of points to explain the clear distinction between
the old versus the newly proposed system of capitalism; they are not to lift “moral values” and “profits”
as these are key words which can be easily re-expressed.

RVIP 2010 Preliminary Examination Page 3 of 7 Year 6 General Paper


8. Using materials from paragraphs 3 to 5, summarise what Singer sees as the possible positive

and negative implications of this new form of capitalism.

Write your summary in no more than 120 words, not counting the opening words which

are printed below. Use your own words as far as possible. [8]

The author argues that capitalism…


Lifted from the passage In your own words
Positive implications
a) The French have always been less likely a) testifies / proves / attests to the importance
to go into debt – when they pay with plastic, of (½ m) spending within one‟s means. (½ m)
they tend to use debit cards, drawing on
funds they already have, rather than credit
cards. Now they see the current crisis as a
vindication of the value of not spending
money that you don’t have

b) excess is out of style, and there are b) It makes people spend more prudently
reports of cutbacks in luxury goods (1 m)
everywhere

c) does this mark an enduring change in c) This reflects a paradigm shift (½ m)


values towards wise spending (½ m)

d) crisis remind us that we buy luxury items d) as we look beyond the prestige of products
more because of the status they bring than (½ m) and acknowledge their real worth (½m)
because of their intrinsic value
e) appreciate that many things are more e) It makes people realise that there is more
central to our happiness than our ability to to a good life (½ m) than the pursuit of
spend money on fashion, expensive material fulfilment. (½ m)
watches, and fine dining

f) make us more aware of the needs of f) People may be more sympathetic or


those who are living in real poverty and are informed / have more empathy or heightened
far worse off than we will ever be, financial realisation (½ m) for the disadvantaged in
crisis or no financial crisis society / the poor who are suffering (½ m)

g) less luxury spending – something that is Negative implications


hard to reconcile with the image of France g) It may clash with the branding of a city
as the country of fashion, perfume, and (½ m) that is renowned for its pure
champagne. indulgence in ostentatious items OR
epitomises pure indulgence. (½ m)

h) just a temporary reduction, forced upon h) Thrift may just be a passing fad (½ m) due
consumers by investment losses and to unfavourable economic / financial
greater economic uncertainty conditions. (½ m)

i) The danger is that the potential for a real i) This new form of capitalism may be
change in values will be co-opted, as has capitalised / exploited (½ m) for further
happened so often before, by those who see income generation by businesses. (½ m)
it as just another opportunity to make
money

Note: Any 8 of the 9 points will give a maximum of 8 marks.

RVIP 2010 Preliminary Examination Page 4 of 7 Year 6 General Paper


9. Explain the tone of the author in the first line of the last paragraph. [2]
Lifted from the passage In your own words
Ah yes, in a world in which ten million He is sarcastic (1 m) in pointing out…
children die every year from avoidable,
poverty-related causes, and greenhouse-gas OR
emissions threaten to create hundreds of
millions of climate refugees, we should be
He is mocking / scorns (1 m) how…
visiting Paris boutiques and feeling the
clothes. AND

(i) designers are trying to get people to buy


more clothes when there are so many
problems that the world faces. (½ m)

OR

(ii) Designers / people in general are


apathetic / indifferent towards the world‟s
problems in relation to their own desire to
promote their business.(1m)

OR

(iii) how people are caught up with trivial


materialistic concerns (½ m) instead of caring
about the sufferings of the world at large
(½m)

OR

(iv) these people are not using their wealth to


alleviate the real / urgent problems of the
world (½ m), but are splurging mindlessly on
non-necessities / insignificant desires
instead. (½ m)

Note: Students have to describe the tone of the author (1 m) before explaining its significance (1 m);
Clear contrast must be shown in terms of their trivial indulgent concerns versus the bigger picture of the
world‟s problems. Students should not conclude that designers / people are ignorant about the pressing
issues of the world as this is a misinterpretation.

RVIP 2010 Preliminary Examination Page 5 of 7 Year 6 General Paper


From both passages

8) Give the meaning of the following words as they are used in the passage. [5]

Word 1m ½m 0m
(a) stupendous  surprisingly  astonishing  out-of-this-world
(passage 1, impressive  stunning  large
line 18) Adjective  breathtaking  colossal  phenomenal
 awe-inspiring  extraordinary
note: scale and impact  astounding  sensational
on the observer must  amazing  incredible
be conveyed  impressive  staggering
 outstanding

(b) relish  delight in  appreciate  look forward to


(passage 1,  revel in  adore
line 41) Verb  enjoy  cherish
 treasure
note: the experience  value
must be clearly  rejoice in
conveyed  has a penchant for

(c) conduits  channels  tools  agents


(passage 1,  means to an end  instruments
line 43) Noun  mediums

(d) resurgence  revival  rebirth


(passage 2,  re-emergence  resurrection
line 14) Noun  re-surfacing  renewal
 return
 restoration

(e) vindication  exoneration  proof  support


(passage 2,  justification  testament  substantiation
line 19) Noun  affirmation  defence
 proven right  excuse

RVIP 2010 Preliminary Examination Page 6 of 7 Year 6 General Paper


11. Grayling shares his perspective on the true meaning of luxury while Singer reminds us

how our quest for luxury has derailed us from embracing sound values. With which of

the two authors are you most in sympathy? Do you regard the quest for luxury for you

and your society as broadly beneficial or harmful? [8]

R1: With which of the two authors are you most in sympathy?

R2: Do you regard the quest for luxury for you and your society as broadly beneficial or
harmful?

 BOTH R1 and R2 MUST be addressed in order to pass the AQ (i.e. 4 m or more).

 R1: Students‟ response should draw references from BOTH passages (a minimum of 3 to 4
points in all) in order to examine and explain why they are in agreement with either author, by
providing sound analysis and evaluation of the arguments put forth by both authors. Sound
justification of their stand is crucial in order to get a decent pass (i.e. a mid to a high „B‟) for this
component of the AQ.

 R2: A crucial point to note for this AQ is how students should not equate the quest for luxury as
being materialistic only. The term „luxury‟ connotes the high end of the materialistic spectrum as it
means extreme comfort, largely in beautiful, exquisite and ostentatious settings. It can also refer
to an extremely extravagant attitude / lifestyle and outlook on life, which is not exactly the same
as merely „keeping up with the Joneses‟ via constant „upgrading‟.

 R2: Students‟ response should focus on examining how the quest for luxury has affected them
and their society at large. They should not be penning a personal response by citing whether they
buy branded goods and if they do, why they do so.

 R2: Strong responses would analyse the impact of this quest for luxury in various aspects of the
society (e.g. food, fashion, gadgets, recreation / entertainment, housing, other lifestyle-enhancing
products and services). They would be informed about the general ethos of the society with
regard to the quest for luxury by being aware that not everyone (can or choose to) embrace(s)
this approach towards life. Clear evidence of how this quest towards an ostentatious lifestyle has
affected various groups of people in the society should be provided. They must show awareness
that this quest has had its positive and negative implications, depending on a myriad of factors.

RVIP 2010 Preliminary Examination Page 7 of 7 Year 6 General Paper


ST. ANDREW’S JUNIOR COLLEGE
JC2 PRELIMINARY EXAMINATIONS

GENERAL PAPER 8806/01

Paper 1 Monday, 30 August 2010


1 hour 30 minutes

Additional Materials: Answer Paper

READ THESE INSTRUCTIONS FIRST

Write your name, civics group, index number and GP tutor’s name on all the
work you hand in.
Write in dark blue or black pen on both sides of the paper.
Do not use staples, paper clips, highlighters, glue or correction fluid.

Answer one question.


Note that 20 marks out of 50 will be awarded for your use of language.

At the end of the examination, fasten all your work securely together.
All questions in this paper carry equal marks.

This document consists of 2 printed pages.


2

Answer one question.

Answers should be between 500 and 800 words in length

1 To what extent can businesses strike a balance between corporate and social
goals?

2 How far can we depend on science to solve environmental problems?

3 ‘A country should not allow international opinions to affect how it deals with
domestic issues.’ Is this a fair comment?

4 ‘No modern city is truly unique.’ Do you agree?

5 Do you agree that there is too much control over the arts scene in Singapore?

6 Is it true that the social media has created more problems than benefits?

7 Should we still promote bilingualism in Singapore?

8 ‘Advertisements hinder us from making good decisions.’ Discuss.

9 ‘Spare the rod, spoil the child.’ Should physical punishment be used on
children?

10 To what extent is your society an inclusive one?

11 Examine the claim that sport has lost its meaning in today’s world.

12 ‘Achieving equality should be the aim of all governments.’ Do you agree?


In this article, Vigen Guroian writes about Teaching Virtues Through Fairy Tales.

1 Fairy tales and modern fantasy stories project fantastic other worlds; but they also pay close
attention to real moral ‘laws’ of character and virtue. By portraying wonderful and frightening
worlds in which ugly beasts are transformed into princes and evil persons are turned to stones
and good persons back to flesh, fairy tales remind us of moral truths whose ultimate claims to
normality and permanence we would not think of questioning. 5

2 The notion that fairy tales and fantasy stories stimulate and instruct the moral imagination of the
young is, of course, not new. The Victorians certainly held to that notion when they brought the
fairy tale into the nursery. In our day, we have seen a resurgence of interest in fairy tales.
Parents and teachers thirst for literary resources that they might use in nurturing the moral
imagination of children. They need, and are asking for direction in how to influence the moral 10
character of the young.

3 Yet religious and philosophical ethicists have not reflected a great deal on children as moral
learners nor written much on children's literature. Perhaps this is because, like so many others,
they have subscribed to the falsehood that children are at a pre-moral stage and that
socialisation rather than moral formation is more appropriate to their kind. But intuitively we 15
ought to know that it is not that simple.

4 The great fairy tales and fantasy stories capture the meaning of morality through vivid depictions
of struggles between good and evil where characters must make difficult choices between right
and wrong. The great stories avoid lecturing and supply the imagination with important symbolic
information about the shape of our world and appropriate responses to its inhabitants. 20

5 Moral living is about being responsive and responsible toward other people. And virtues are
those traits of character that enable persons to use their freedom in morally responsible ways.
Mere instruction in morality is not sufficient to nurture the virtues. It might even backfire,
especially when the presentation is heavily exhortative and the pupil's will is coerced. Instead, a
compelling vision of the goodness of goodness itself needs to be presented in a way that is 25
attractive and stirs the imagination. A good moral education addresses both the cognitive and
affective dimensions of human nature. Stories are an irreplaceable medium of this kind of moral
education. This is the education of character.

6 Moral character is an impression stamped upon the self. Character is defined by its orientation,
consistency, and constancy. Today we often equate freedom with morality and goodness. But 30
this is naive because freedom is transcendent and the precondition of choice itself. Depending
upon his character, an individual will be drawn toward either goodness or wickedness. Moral
and immoral behaviour is freedom enacted either for good or for ill.

7 The great fairy tales and children's fantasy stories attractively depict character and virtue. In
these stories the virtues glimmer as if in a looking glass, and wickedness and deception are 35
unmasked of their pretensions to goodness and truth. These stories make us face the
unvarnished truth about ourselves while compelling us to consider what kind of people we want
to be.

1
8 Much of what passes for moral education fails to nurture the moral imagination. Yet, only a
pedagogy that awakens and enlivens the moral imagination will persuade the child or the 40
student that courage is the ultimate test of good character, that honesty is essential for trust and
harmony among persons, and that humility and a magnanimous spirit are goods greater than
the prizes won by selfishness, pride, or the unscrupulous exercise of position and power.

9 The moral imagination is not a thing, not so much a faculty even, as the very process by which
the self makes metaphors out of images given by experience and then employs these 45
metaphors to find and suppose moral correspondences in experience. The moral imagination is
active, for well or ill, strongly or weakly, every moment of our lives, in our sleep as well as when
we are awake. But it needs nurture and proper exercise. Otherwise it will atrophy like a muscle
that is not used. The richness or the poverty of the moral imagination depends upon the
richness or the poverty of experience. When human beings are young and dependent upon 50
parents and others who assume custodial care for them, they are especially open to formation
through experiences provided by these persons. When we argue or discuss what kind of
education or recreation our children should have we are acknowledging these realities.

10 Unfortunately, more often than not, this society is failing to provide children with the kinds of
experience that nurture and build the moral imagination. One measure of the impoverishment of 55
the moral imagination in the rising generation is their inability to recognise, make, or use
metaphors. Students have gotten used to the idea that all they need to do is to look for the so-
called facts in a book. Facts are things whose meaning belongs to their use and whose use
requires relatively little interpretation. We are living in a culture in which metaphor is discarded
for the so-called facts. We train minds to catch these facts much as one breaks in a baseball 60
glove. Meanwhile, the imagination is neglected and is left unguarded and untrained.

11 Fairy tales and fantasy stories transport the reader into other worlds that are fresh with wonder,
surprise, and danger. They challenge the reader to make sense out of those other worlds, to
navigate his way through them, and to imagine himself in the place of the heroes and heroines
who populate those worlds. The safety and assurance of these imaginative adventures is that 65
risks can be taken without having to endure all of the consequences of failure; the joy is in
discovering how these risky adventures might eventuate in satisfactory and happy outcomes.
Yet the concept of self is also transformed. The images and metaphors in these stories stay with
the reader even after he has returned to the "real" world.

12 When the moral imagination is wakeful, the virtues come to life, filled with personal and 70
existential, as well as social, significance. The virtues need not be the dry and lifeless data of
moral theories or the ethical version of hygienic rules in health science classes; they can take on
a life that attracts and awakens the desire to own them for oneself. We need desperately to
adopt forms of moral pedagogy that are faithful to the ancient and true vocation of the teacher
— to make persons into mature and whole human beings, able to stand face to face with the 75
truth about themselves and others, and desiring to correct their faults and to emulate goodness
and truth wherever it is found. We need to take greater advantage of the power in stories to
humanise the young.

2
Read the passages and then answer all the questions which follow below. Note that For
up to fifteen marks will be given for the quality and accuracy of your use of English Examiner’s
throughout this paper. Use

Note: When a question asks for an answer IN YOUR OWN WORDS AS FAR AS
POSSIBLE and you select the appropriate material from the passage(s) for your
answer, you must still use your own words to express it. Little credit can be given to
answers which only copy words or phrases from the passage(s).

1. From paragraph 1, explain why the word ‘laws’ (line 2) is in quotation marks.

…………………………………………………………………………………………………

…………………………………………………………………………………………….. [2]

2. What does the word ‘resurgence’ (line 8) suggest about the change in interest in
fairy tales?

…………………………………………………………………………………………………

…………………………………………………………………………………………..… [1]

3. (a) What does the word ‘yet’ (line 12) suggest about the author’s attitude towards
the lack of children’s literature?

…………………………………………………………………………………………………

………………………………………………………………………………………..…… [1]

(b) Why does the author feel this way?

…………………………………………………………………………………………………

…………………………………………………………………………………………….. [1]

4. Explain why the author uses the word ‘intuitively’ (line 15).

…………………………………………………………………………………………………
………………………………………………………………………………………..…… [1]

1
5. From paragraph 5, explain why the author says that ‘mere instruction in morality is For
not sufficient to nurture the virtues’ (line 23). Use your own words as far as Examiner’s
possible. Use

…………………………………………………………………………………………………

……………………………………………………………………………………..……… [2]

6. ‘In these stories the virtues glimmer as if in a looking glass, and wickedness and
deception are unmasked of their pretensions to goodness and truth.’ (lines 35 – 36)

Explain the metaphor of the looking glass.

…………………………………………………………………………………………………

…………………………………………………………………………………………………

……………………………………………………………………………………..……… [2]

7. From paragraphs 4, 8, 9, and 11, summarise the benefits of using fairytales in


teaching morals and the difficulties in cultivating the moral imagination.

Write your summary in no more than 120 words, not counting the opening words
which are printed below. Use your own words as far as possible. [7]

One of the benefits …….…………………………….……………………………………

…………………………………………………………………………………………………

…………………………………………………………………………………………………

…………………………………………………………………………………………………

…………………………………………………………………………………………………

…………………………………………………………………………………………………

…………………………………………………………………………………………………

…………………………………………………………………………………………………

…………………………………………………………………………………………………

…………………………………………………………………………………………………

…………………………………………………………………………………………………

…………………………………………………………………………………………………
2
For
………………………………………………………………………………………………… Examiner’s
Use
…………………………………………………………………………………………………

…………………………………………………………………………………………………

………………………………………………………………………………………….……..

………………………………………………………………………………………….……..

8. Explain what the author means by ‘it will atrophy like a muscle that is not used.’
(lines 48 – 49). Use your own words as far as possible.

…………………………………………………………………………………………………

………………………………………………………………………………………..….… [2]

9. Which word in paragraph 10 shows the author’s skepticism towards information


found in books?

………………………………………………………………………………………..….… [1]

10. According to paragraph 10, explain how society can (a) ‘nurture’ (line 55), and
(b) cause the ‘impoverishment’ (line 55) of the moral imagination through books.
Use your own words as far as possible.

(a)……………………………………………………………………………………………

……………………………………………………………………………………………… [1]

(b)……………………………………………………………………………………………

……………………………………………………………………………………………… [1]

11. Give the meaning of the following words as they are used in the passage. Write
your answer in one word or a short phrase.

a) project (line 1) ………………………………………………………………...…... [1]

b) thirst (line 9) …………………………………………………………………..…… [1]

c) backfire (line 23) ……………………………………………………….…….....… [1]

d) unvarnished (line 37) ………………………………………….…………….....… [1]

e) lifeless (line 71) …….………………………………………….………………..… [1]

3
12. The author argues for the use of moral imagination in teaching morality. Do you For
agree with the author’s views on the use of the moral imagination in moral Examiner’s
education? How effectively can morals be taught today? [8] Use

…………………………………………………………………………………………………

…………………………………………………………………………………………………

…………………………………………………………………………………………………

…………………………………………………………………………………………………

…………………………………………………………………………………………………

…………………………………………………………………………………………………

…………………………………………………………………………………………………

…………………………………………………………………………………………………

…………………………………………………………………………………………………

…………………………………………………………………………………………………

…………………………………………………………………………………………………

…………………………………………………………………………………………………

…………………………………………………………………………………………………

…………………………………………………………………………………………………

…………………………………………………………………………………………………

…………………………………………………………………………………………………

…………………………………………………………………………………………………

…………………………………………………………………………………………………

…………………………………………………………………………………………………

…………………………………………………………………………………………………

…………………………………………………………………………………………………

…………………………………………………………………………………………………

…………………………………………………………………………………………………
4
………………………………………………………………………………………………… For
Examiner’s
………………………………………………………………………………………………… Use

…………………………………………………………………………………………………

…………………………………………………………………………………………………

…………………………………………………………………………………………………

…………………………………………………………………………………………………

…………………………………………………………………………………………………

…………………………………………………………………………………………………

…………………………………………………………………………………………………

…………………………………………………………………………………………………

…………………………………………………………………………………………………

…………………………………………………………………………………………………

…………………………………………………………………………………………………

…………………………………………………………………………………………………

…………………………………………………………………………………………………

…………………………………………………………………………………………………

…………………………………………………………………………………………………

…………………………………………………………………………………………………

…………………………………………………………………………………………………

…………………………………………………………………………………………………

…………………………………………………………………………………………………

…………………………………………………………………………………………………

…………………………………………………………………………………………………

…………………………………………………………………………………………………

…………………………………………………………………………………………………
5
For
………………………………………………………………………………………………… Examiner’s
Use
…………………………………………………………………………………………………

…………………………………………………………………………………………………

…………………………………………………………………………………………………

…………………………………………………………………………………………………

…………………………………………………………………………………………………

…………………………………………………………………………………………………

…………………………………………………………………………………………………

…………………………………………………………………………………………………

…………………………………………………………………………………………………

…………………………………………………………………………………………………

…………………………………………………………………………………………………

…………………………………………………………………………………………………

…………………………………………………………………………………………………

…………………………………………………………………………………………………

…………………………………………………………………………………………………

…………………………………………………………………………………………………

…………………………………………………………………………………………………

…………………………………………………………………………………………………

…………………………………………………………………………………………………

…………………………………………………………………………………………………

…………………………………………………………………………………………………

END OF PAPER

6
St Andrew’s Junior College
Preliminary Examinations 2010 – Paper 2

(Literary Devices—punctuation/ irony)


1. From paragraph 1, explain why the word „laws‟ (line 2) is in quotation marks. [2]

Lifted Inference
Fairy tales and modern fantasy stories Laws are usually concrete/ codified/ certain /
project fantastic other worlds; but they definite and enforceable/ rules and
also pay close attention to real moral regulations [1]
"laws" of character and virtue.
However, in this context these laws are not
policed yet accepted/ these moral laws are
not explicitly written down yet they are
implicitly accepted / acknowledged /
recognized/ deemed correct by society
norms/ guiding principles. [1]

(Inference)
2. What does the word „resurgence‟ (line 8) suggest about the change in interest in
fairy tales? [1]

Lifted Inference
Victorians certainly held to that notion A: The word suggests that interest in fairy
when they brought the fairy tale into the tales declined in the time between the
nursery. In our day, we have seen a Victorians and modern day
resurgence of interest in the fairy tale.
Do not accept: dismiss, not taken seriously

B: and there is now a rekindling/ increase/


reviving of interest

Note. Need both A and B to be awarded 1


mark. No ½ marks.

(Inference – Literary Devices—use of language)


3. (a) What does the word „yet‟ (line 12) suggest about the author‟s attitude towards
the lack of children‟s literature? [1]

1 mark 0 marks
surprise, astonishment, Critical, disdain, condemning, disgusting, upset, pessimistic,
disbelief, disapproving, a sense of pity, shock, discontented, not satisfied
disappointed, regretful,
puzzled, perplexed Note. Do not accept words which are too strong
(b) Why does the author feel this way? [1]
- The author expects/ hopes that the religious and philosophical ethicists
will act as the gatekeepers or moral issues in society/ write more literature on
children as moral learners/ meet the needs of parents and teachers

Note. Need to refer to a specialized group of people – e.g. those in authority,


academics, religious people, teachers, educators (anything that implies authority)

(Inference)
4. Explain why the author uses the word „intuitively‟ (line 15). [1]

Lifted Inference
Perhaps this is because, like so many A: To suggest that innately/ naturally,
others, they have subscribed to the
falsehood that children are at a pre- and
moral stage and that socialization
rather than moral formation is more B1: young children are not only suitable for
appropriate to their kind. But intuitively socialisation
we ought to know that it is not that
simple. or

B2: we would know that young children can


(NOT should) undergo moral education

Note. Both A and B1 or B2 must be present


for 1 mark.

(Literal direct)
5. From paragraph 5, explain why the author says that „mere instruction in morality
is not sufficient to nurture the virtues‟ (line 23). Use your own words as far as
possible. [2]

Lifted Paraphrased
Mere instruction in morality is not A: Instruction is not enough in
sufficient to nurture the virtues (line cultivating/enriching virtues [1/2]
24)

It might even backfire, especially when B: especially when it is didactic/ forced


the presentation is heavily exhortative upon students/ given no choice/ simply tell
and the pupil‟s will is coerced. (lines the person what to do [1/2]
24-25)

… needs to be presented in a way that C: captivating [1/2]


is attractive (lines 25-26)
or
or
fires their creative spirit [1/2]
… and stirs the imagination (line 26)
A good moral education addresses D: Emotional aspects need to be included
both the cognitive and affective [1/2]
dimensions of human nature. (line 25-
26)

(Literary Devices-- metaphor)


6. „In these stories, the virtues glimmer as if in a looking glass, and wickedness and
deception are unmasked of their pretensions to goodness and truth.‟ (lines 35 –
36)

Explain the metaphor of the looking glass. [2]

Lifted Inference
…the virtues glimmer A1: Just as the looking glass magnifies/enhances objects and
as if in a looking makes things clearer
glass, and wickedness
and deception are AND
unmasked of their
pretensions to A2: likewise, in fairytales/ fantasy stories, virtues will appear
goodness and truth more alluring/captivating and appealing.

or

A3: likewise, in fairytales/ fantasy stories, devious behaviour


will be shown/ exposed/ stripped of its concealments.

Note. Need A1 (looking glass) and either A2 or A3 (fairy


tales) for the full 2 marks. Do not award 1 mark.

B1: Just as the looking glass is a mirror which reflects its


object

AND

B2: likewise, in fairy tales/ fantasy stories, the virtues are also
shown/echoed very clearly

or

B3: likewise, in fairy tales/ fantasy stories, devious behaviour


are also shown/ made obvious/ revealed

Note. Need B1 (looking glass) and either B2 or B3 (fairy


tales) for the full 2 marks. Do not award 1 mark.

7. From paragraphs 4, 8, 9, and 11 summarise the benefits of using fairytales in


teaching morals and the difficulties in cultivating the moral imagination. [7]

Lifted Paraphrased Main idea


Benefits
A capture the meaning of grasp/ understand/ represent employs metaphors of
morality through vivid what is right and wrong good and evil to teach
descriptions of struggles through clear narrations of morality
between good and evil (lines stories of battles between the
17-18) virtuous and wicked

B avoid lecturing (line 19) prevent/ try not to talk down do not be too
prescriptive
C supply the imagination with Provide/ fuel one‟s not taking things at face
important symbolic imagination with icons/ value; make meaning
information about the images/ representations of the world
shape of our world… (line about the world‟s situation/
19-20) meaning

Do not accept: ideas, signs

D … appropriate responses to And how we should suitably Suitable reactions


its inhabitants (line 20) react/ respond to others

E Yet, only a pedagogy that Fairy tales aid in igniting/ awaken and enliven
awakens and enlivens the activating/ stimulating/
moral imagination will stirring the moral imagination.
persuade the child… (line
39-40) Do not accept: cultivate

F … into other worlds that are It provides novel/ new provides new insight/
fresh with wonder, surprise, insights/ ways of seeing the ways of seeing the
and danger (lines 62- 63) world. world

Note. Students should not


paraphrase “wonder,
surprise, and danger”.
These are examples.

G The safety and assurance able to try out different roles/ not threatening/ secure
of these imaginative experiment/ face threats/ to take risks
adventures is that risks can dangers in a secure/ non-
be taken (line 65-66) threatening environment
H without having to endure all do not have to bear/ live with/ can bear the outcomes
the consequences of experience the implications/ of not succeeding
failure (line 66) costs/ outcomes of not
succeeding

I The images and metaphors the lessons learnt will be lessons will live in the
in these stories stay with retained by/ continue to reader/ continue to be a
the reader even after he has influence/ impact the reader part of his self identity
returned to the “real” world.
(lines 68-69)

or or

yet the concept of self is the reader‟s self-identity will be


also transformed (line 68) changed

Difficulties
1 Much of what passes for *inferred* Quality of resources is
moral education fails to Existing/current moral education poor
nurture the moral curricula/ curriculums are
imagination (line 39) inadequate/ do not meet the
standard/ is substandard

2 But it needs nurture and requires appropriate effort/ effort is required/ needs
proper exercise. line 48) time/ energy/ use/ training constant use

3 Otherwise it will atrophy without which/ if not, it will if not, it will degenerate
like a muscle that is not degenerate/ be weakened/
used. (lines 48-49) debilitated/ wasted

Do not accept: stagnate

4 The richness or the poverty *inferred* inadequate/ limited


of the moral imagination Not all have the means/ exposure to the
depends upon the exposure/ ability to acquire world/circumstances
richness or the poverty of worldliness thorough diverse
experience (lines 49-50) exposure.

or

Reliant on the quality/ level/


abundance/ lack of breadth
and depth of exposure to
cultivate.

Note. Students need to paraphrase 5 points for Benefits and 2 points for
Difficulties to be awarded the full 7 marks. Points MUST BE properly signposted.
Wrong signposting will cost students 1 mark from their overall summary marks.
8. Explain what the author means by „it will atrophy like a muscle that is not used.‟
(lines 48 – 49). Use your own words as far as possible. [2]

Lifted Paraphrased
„it will atrophy like a muscle Moral imagination will reach a stage of being
that is not used.‟ weakened/ debilitated [1]

Do not accept: shrink, destroyed, stagnate, useless

when it falls into inactivity [1]

Note. Sentence structure must be verbatim to capture


the meaning. It is recommended that tutors teach
students that this is a simile question.

9. Which word in paragraph 10 shows the author‟s skepticism towards information


found in books? [1]

“So-called”

10. According to paragraph 10, explain how society can (a) „nurture‟ (line 55), and (b)
cause the „impoverishment‟ (line 55) of the moral imagination through books. Use
your own words as far as possible. [2]

Lift Inference
(a) ‘nurture’ A1: Society can enrich/ cultivate/ teach/
stimulate moral imagination [1/2]
One measure of the impoverishment
of the moral imagination in the rising A2: through the identification/ creation/
generation is their inability to application of metaphors [1/2]
recognise, make, or use
metaphors. (lines 56-58)

(b) ‘impoverishment’ B1: the moral imagination is being


deprived [1/2]
Students have gotten used to the
idea that all they need to do is to AND
look for the so-called facts in a book.
(lines 58-59)/ We are living in a B2: because students have become
culture in which metaphor is conditioned to simply/ only
discarded for the so-called facts. source/identify facts. [1/2]
(lines 60-61)
OR
… whose use requires relatively
little interpretation. (lines 59-60) B3: because facts require minimal
deciphering [1/2]

1 mark 0 marks
a) project (line 1) display, depict, represent, make up, demonstrate,
show, portray, showcase illustrate, create

b) thirst (line 9) yearn, craves too literal: hungry for, hunger,


seek
strong/ badly/ deep demand,
Note. Degree must need, desire too strong/ wrong connotation:
be strong. desperately/urgently

c) backfire (line 23) counterproductive,

A: have opposite effect, not


Note. This is a 2-part the intended effect
answer.
B: undesirable, negative

d) unvarnished (line plain, simple, not sugar not veiled, not exaggerated, not
37) coated reality, not hidden, refined, untainted, ugly,
unadulterated, real, not original, raw
modified, without glossing
over

e) lifeless (line 71) meaningless, lacks vibrancy, dead, listless, boring, dull,
lacks vigour, lacks force uninteresting, mundane, not
interactive

11. The author argues for the use of moral imagination in teaching morality. Do you
agree with the author‟s views on the use of moral imagination in moral education?
How effectively can morals be taught today? [8]

R1: Student must indicate whether they agree/ disagree with the author‟s views
R2: Pick at least 2 arguments from the passage
R3: Application - argue for whether morals can be taught effectively

Author’s point Agree/ disagree How effectively can morals be


taught today?
The great fairy tales Agree: Effective:
and fantasy stories - Children are engaged by the - Increasing literacy rate and
capture the meaning vivid descriptions provided in education level of parents, as
of morality through fairy tales. This stimulates well as increasing awareness
vivid descriptions of their imagination as the of what good parenting
struggles between scenes in the fairytale play entails, leads to parents
good and evil (lines out in their minds. knowing the importance of
17-18) - Such imagination is reading fairy tales to their
necessary for learning about children and explaining the
people and events they do moral concepts found in these
not directly experience and stories.
allows them to understand - Many child psychology books
abstract moral values. and parenting books in the
- Many fairy tales do contain market so parents are aware
moral values such as The of the importance of teaching
Ugly Duckling (to not be moral values from a young
easily let down and to look age. Such books usually
beyond the physical come with tips and strategies
appearance), Cinderella (to on how to teach morals, thus
be good-natured and kind to making it more effective.
those around you) and
Rapunzel (female purity). Ineffective:
- Parents are increasingly
Disagree: relying on maids and
- Children may simply be grandparents to raise their
captivated by what they read children. These caregivers
and not understand the might not be able to explain
deeper moral values depicted the morals present in the fairy
in the story. However, one tales to these children.
could argue that it is the job of - Children nowadays are more
parents to use fairytales as a interested in television shows
teaching resource. Parents and computer games rather
should follow up the story with than in fairy tales. They are
a discussion/ talk about it with used to being visually
their children so that they can stimulated, rather than
grasp the moral meaning of mentally stimulated – there
the story. seems to be no room for
- Fairy tales, especially those imagination anymore.
presented by Disney, actually - Morality as relative. A
promote many ideals that are prevalent notion that has
often superficial. E.g. Casting caught on in society that may
the ideal woman as fair, render the absolute
youthful, pure, angelic and dichotomy of an event as
precious. There is usually an „good‟ or „evil‟ meaningless.
emphasis on physical beauty - A generation that grows up
(Belle: her name means too fast – young children may
beauty and her looks have also be skeptical about these
got no parallel; Snow white is tales .e.g the tooth fairy/Santa
the fairest of them all) which Claus myth. Influenced by
causes readers to falsely materialism ….
equate beautiful with good.
The use of imagination in
imagining these “perfect”
princesses actually hinders
moral development of a child.

The great stories Agree: Effective:


avoid lecturing (line - The speed & availability at - In line with the rise of
19) which information reaches us consultative parenting…
or at which we can access - Brings to life… the
information makes for an entertainment aspect
impatient audience. Where addresses the problem of
society is used to instant short attention span
gratification of needs and - Morals can be effectively
desires, youth and children taught through this method
increasingly find it difficult to depending on the age of the
sit for long periods and listen child. Younger children and
to expositions on topical unassuming youth might be
issues. Stories provide an more receptive to this method
avenue for children and youth of teaching and even find it
to be inculcated with moral engaging and refreshing.
values as stories encapsulate
underlying values in concise, Ineffective:
subtle and interesting ways. - Youth, being more perceptive
- The speed & availability at and cynical about moral
which information passes education and the „imposition‟
through the eyes and minds of values on them might see
of readers are becoming through the intentions of such
shorter with the increasing lessons no matter the means
efficiency of technology. with which they are carried
Presentation of information out, put across or couched.
has to be attention grabbing - Youth do not like to be told
to catch readers‟ attentions what to do. They crave
amidst the vast sea of autonomy to make their own
information available. Stories decisions. The implicit way in
fulfill this aspect of which stories emulate values
information presentation on or denounce them allows
morality and prevent a top- youth and children to evaluate
down, didactic approach to their own principles and
teaching which will capture consider the consequences of
the imaginations of children their actions within a non-
especially of this age. threatening environment. This
might be a more appealing
Disagree: and effective way of teaching
- The use of stories will not be morals.
effective and will still end up
as „lecturing‟ in the eyes of It depends:
children if they have the - The teaching of morals
wrong mindset. Depending on depends very much on the
the age of children, youth educator imparting the
tend to be more cynical and knowledge and his rapport
skeptical of information with the students. Such an
passed on to them. Some educator will have students
may be able to see through receptive and open to
the use of stories for teaching everything he teaches
of morals and still find these regardless of the tools he
lessons instructional and employs.
exhortative and far from
engaging. Thus, while stories
have potential to avoid
lecturing and are probably the
best way to do so, students‟
receptiveness to the method
play a big role in how
powerful the use of the moral
imagination can be in
imparting of values.

… supply the Agree: Effective:


imagination with - Teaches children about the - Values promoted in the
important symbolic challenges they may face in stories are universal and
information about the life‟s journey. These tales appealing to all. Long lasting
shape of our world hide a wealth of information impact with real lessons as it
(line 19-20) beneath the surface. appeals to the senses.
(experiential learning)
- Fairy tales reflect the cultural
Disagree: settings and how the stories
- We can question the quality are still alive in these places.
and accuracy of this The importance and
representation of the “real relevance of the stories is a
world” in such tales. powerful experience.
- It is not “timeless” but reflects - E.g. Pied Piper – parenting
archaic views and values that and dealing with issues of
are dichotomous with the letting children go (grow up)
modern world.
Ineffective:
- Issues are outdated. Gender
bias perpetuating patriarchal
beliefs. Disney princesses.
- Too young to understand the
deeper meaning of these
tales.

Yet, only a pedagogy Agree: Effective/ ineffective:


that awakens and - Compacted wisdom in stories - Used wisely yet how many
enlivens the moral that yields more information (parents/teachers) can
imagination will with each reading. harness the potential of these
persuade the child… - Teaching morality often ends stories and link them to larger
(line 39-40) up being very “preachy”. The themes and issues. Are we
lesson will become not only trained in this aspect?
prescriptive, but could also Adequate training for pre-
sound judgmental. All these school teachers?
will not go down well for - Merely letting the child read
students, especially those in on his own may not be
their rebellious teenage effective if he is merely
years. This could be why reading it for entertainment.
students often complain how This is especially true for
Moral Education lessons are fantasy stories such as Harry
boring because they are Potter where children are
being talked down to. They more interested in the magic
would not pay attention and and incantations than the
could even be disruptive. moral lessons, if any, that
Lessons therefore must be Rowling may try to impart. An
made relevant and more adult – parent or teacher – is
interesting, and one way stir still required to explain the
the imagination of students to moral lessons explicitly for the
rouse their interest. This child who may simply be too
could probably be done more young to be discerning.
creatively through movies or
animation, things that they There is really no one best way to
students have an interest in. teach morals.
That way, they could be - Different children learn
hooked in to the lessons and differently, and those
even get involved in activities responsible for the moral
designed to complement the upbringing of children have
lesson. to employ the different
methods appropriately.
Disagree: Parents, for instance, could
- To “awaken and enliven the use socialisation by modeling
imagination” through telling the right behaviour for their
fairy tales may cause the children through their words,
child to think that it is all deeds and actions.
fictional stories only. At the Teachers could use more
end of the day, so-called real prescriptive ways, e.g. during
teaching is still required to Moral Education tutorials.
decipher for the child the Lessons, however, need not
morals – teaching points – be staid and boring.
behind the stories. Teachers could utilise
- creative way through music
(Youtube), movies (Avatar),
animation (Kung Fu Panda),
etc., to bring the lesson alive
in order to draw their
students in to the lessons to
be learned.
- All in all, a combination of
various methods of teaching
would be most effective.

The moral Agree: Effective:


imagination is active, - Our moral imagination is such - Trains the moral imagination.
for well or ill, strongly an essential / integral part our E.g. the site of Hamelin where
or weakly, every lives that it constantly helps the tale of the Pied Piper
moment of our lives, us make moral / ethical takes place may appear
in our sleep as well choices. different from how it is
as when we are - Creativity and play engage imagined in real life. This
awake (lines 46-48) the childlike energies that can reflects the ability of fairy
leave us in a state of tales to unleash the power of
depression if they are not imagination.
nurtured.
I subscribe to the author‟s view that
Disagree: teaching morals through moral
- But I would take issue with imagination is only as effective as
the author who claims that the richness / breadth of
even in our sleep, or in our experiences provided by parents
dreams, we continue to make and teachers. But in this day of the
moral choices. This is relative absence of parents at home
because morality must be a or the increased emphasis placed on
conscious, voluntary choice reality non-fiction in schools, the
which is very much under our effectiveness of imparting moral
personal control. values to young minds is often called
into question.

Fairy tales and Agree: Effective:


fantasy stories - Fairy tales can ignite the - Morals can be taught
transport the reader moral imagination by allowing because even though it can
into other worlds that a safe haven for people to be argued that morals is a
are fresh with roam in a fantasy world where matter of personal
wonder, surprise, there are no inhibitions and experience, it still requires the
and danger (lines 62- risks could be taken without proper guidance of adults and
63) fear of repercussions, e.g. moral authorities to guide
Harry Potter. This is in children in the proper way of
contrast to reality where interpreting images and
people are usually risk averse metaphors in a way that does
and tread safely. However, it not twist the fundamental
is through taking risk, failing meaning of the morals
and admitting one‟s mistakes imparted especially in this
that one picks up moral age associated with moral
lessons. decline.

Disagree: Ineffective:
- The writer is assuming that - Even though fairy tales do
the images and metaphors enrich the moral imagination,
found in fairy tales will be imparting morals is still more
understood and internalized effective when it is taught in
by the reader in the real world the case of children who will
and that the reader will not need guidance in
misinterpret the images and maneuvering through the vast
metaphors presented. He is children‟s literature made
being idealistic because it available by the explosion of
takes judicious exercising of the mass media today. They
these images and metaphors will need guidance on what
before the reader can be they read lest they apply the
considered as having information found in the fairy
internalized the moral values. tales erroneously such as
taking the wrong risks in real
life and be endangered in the
process.

Stories are unique to their settings Effective:


The images and reflecting the mystic and aura of the Long lasting impact – yet difficult to
metaphors in these places quantify/measure.
stories stay with the
reader even after he
has returned to the Ineffective
“real” world. (lines  Idealistic to think that we can
68-69) drastically change the self
and eradicate bad habits
or through invoking the moral
imagination. Societal forces
yet the concept of ( secularism, sci and
self is also technology pragmatism)
transformed (line may not be easily countered.
68)
Name Civics Group GP Tutor

SERANGOON JUNIOR COLLEGE


2010 JC2 PRELIMINARY EXAMINATION

GENERAL PAPER 8806/01


Paper 1

16 August 2010 1 hour 30 minutes

INSTRUCTIONS TO CANDIDATES
Write your name and CG number in the spaces provided on the question paper.

Answer one question.

Write your answer on the separate answer paper provided.

If you use more than one sheet of paper, fasten the sheets together.

INFORMATION FOR CANDIDATES FOR EXAMINER’S USE


All questions carry equal marks. Content /30

Language /20

TOTAL /50

1
PAPER 1

Answer one question from this paper.

Answers should be between 500 and 800 words in length.

1. Does technology lead to greater equality in society?

2. “The media's first priority is to entertain, not inform.” Is this the case in your
society today?

3. Should international bodies intervene in the affairs of nations?

4. How far can we satisfy the worldwide demand for food and still protect the
environment?

5. “There are no winners in war.” Do you agree?

6. “Democracy promises much but rarely delivers.” Comment.

7. “Failures have only themselves to blame.” Do you agree?

8. "Give me your tired, your poor…your wretched refuse…" Should developed


nations welcome immigrants in the same manner today?

9. “Professional sportsmen today are a mercenary lot.” To what extent do you


agree with this statement?

10. “Mankind has achieved nothing; we have only brought about catastrophe.”
Discuss.

11. Is there still a place for integrity in the modern world?

12. “Men are redundant figures in the family today.” How far do you agree?

2
SRJC Preliminary Examination 2010
Paper 2 Insert
This Insert consists of 2 printed pages.

Passage 1
Trevor Turner writes on the behaviorist and psycho-analytical understanding of punishment.

1 Punishment is the infliction of some kind of pain or loss upon a person for the transgression of a law
or command. There are various reasons for modern chastisement. These include the need for revenge;
the need for compensation for losses suffered; the need to consider the wider safety of society; and
some sort of re-instatement or even therapy for the offender. These four horsemen of the correctional
apocalypse vary throughout history as to their level of influence. It is well established in 5
industrialized societies that there is an inverse relationship between the numbers of people in prison
and the numbers in mental hospitals. Discharge people from the latter, as in the present era of
community care and prison, and numbers start to climb. Expand your ‘asylum’ system and prison
numbers tend to fall.

2 The two most powerful theories shaping punishment in the twentieth century are behaviourism and 10
psycho-analysis. The former suggests that behaviour is all and that inner feelings, fantasies, and
fixations are but metaphysical claptrap. Rats in cages can be punished - usually by electric shock - to
conform to certain behaviour patterns. Behaviour modification is frequently used in bringing up
children even in the most liberal societies, including measures of verbal or even physical admonition.
While there is a debate about the effects of corporal punishment and its excesses on the personalities 15
of young children, many parents still accept the use of some physical sanction to deter wayward
behaviour.

3 The more complex approach of Freud and the psycho-analysts gives a rather sexier reading. This
theory suggests that as we grow up we develop, initially, our basic instincts; then our control centre
for balancing these primitive drives with the realities of the outside world. Then, finally, a Superego. 20
This subconscious self derives from the control our parents exercised over us and acts as a kind of
conscience. This may sometimes involve the need for the pain of punishment to alleviate the chronic
guilt coming from our Superego. Thus, accepting punishment – and, of course, seeing other people
accept punishment – becomes part-and-parcel of our personal stability.

4 Freud seems to suggest that punishment is a natural part of the human state. We can enjoy it. We all 25
enjoy a good thrashing; whether it is the ‘thrashing’ of the opposition at a game of rugby or football,
or whether it is the thrashing we receive from an angry parent. In a fragmented, individualized world,
the desire for discipline in oneself and others becomes an increasingly influential undercurrent. Such
primordial instincts may even be seen as the basis for the new ‘tough-on-crime’ approach. The dark
army of medieval moralists, increasingly threatened by the outriders of post-Darwinian rationalism, is 30
becoming ever more fierce in its demands. Just as the new ideas of the Reformation generated the
heightened reactions of the Inquisition, with its’ burning of witches and hunting down of heretics, so
the heightened awareness, in the modern world, of crime and criminality leads to similar demands to
control and punish. The return to the death penalty in many American States reflects exactly this
primitive, sadistic urge. 35

5 It may be that punishment has yet another derivation. A particular feature of modern times is the
perception of violence on the rise, simply because a growing number of people no longer accept such
behaviour as normal. There is also the fact that we are more aware of what is going on. Whatever the
cause, an urgent feeling persists that ‘something must be done’ to quell this awful upsurge in
violence. The Superego within us has its desire to punish, and the ‘criminal’ is an obvious target. 40
Getting rid of a few degenerate human beings seems less troubling in an over-peopled world, where
life seems cheapened daily with public deaths from Bosnia to Somalia, or in awful air crashes or
natural disasters. Is this punishment for our own excessive breeding? As the crush gets noisier the
demand to get rid of the overt criminals becomes more insistent. Even adolescents and the
intellectually disabled are no longer immune from the death penalty in America. A fervour for moral 45
eugenics has taken over the administration of justice.
Adapted from the article “Naughty Naughty”

Passage 2
Michael Hwang writes on the purposes of punishment.

1 The old view of punishment holds that there is a necessary moral connection between wrongdoing
and punishment, termed ‘retributive justice’ which is based on the ‘eye for an eye’ principle. I prefer
an alternative view, which is that offenders are punished only for social reasons. This principle is best
expressed in the words of the Utilitarian philosopher, Jeremy Bentham, who wrote: All punishment is
in itself evil. It ought only to be admitted in as far as it promises to exclude some greater evil. 5

2 Retributive justice looks to the past when it seeks to punish the offender for what he has done, while
the utilitarian looks to the future to justify the imposition of punishment. The utilitarian justification
for punishment is not to take revenge on the offender for his wrongdoing but to prevent future
offences of a similar kind, whether by that offender or others. In short, the principle of deterrence
should underpin a rational policy of sentencing. The sentence should be determined by its effect upon 10
the person punished or by serving as a warning to others.

3 In addition, the penal process can have a certain educational effect, both on the offender as well as the
community at large in reinforcing the social values of the community as expressed through its
criminal laws. However, the educational process should only be regarded as a side effect of
punishment, and not as its primary justification. To that extent, I would therefore disagree with Lord 15
Denning who once famously said, “The ultimate justification of any punishment is not that it is a
deterrent but that it is the emphatic denunciation by the community of a crime.” The purpose of the
criminal law is not to enforce the moral standards of the community as such, but to protect the
community and individuals from tangible harm. Accordingly, punishment should not be based on
moral denunciation as its primary justification. 20

4 But utilitarianism does not provide a self-contained justification of punishment. If one carries
utilitarianism to its logical conclusion and makes deterrence the sole criterion for punishment, there
would be times when the easy way to abolish a socially undesirable practice would be to impose
extremely harsh penalties, such as to impose huge fines or even imprisonment for parking offences.
But the community would, rightly, reject such penalties because they would violate another principle 25
that is commonly accepted as a necessary ingredient of a rational sentencing policy, that is, the
principle of proportionality. That principle reflects the correct place where retribution ought to be
reflected in punishment – in the distribution of justice. The extent to which an offender ought to be
punished cannot be determined solely by the need to stamp out future repetitions of the same offence;
there is a moral limit to the law’s power to make an offender an example for others to fear. 30

5 Conversely, there may be occasions where an offence may result in no overt harm, but may attract
such moral outrage that, a failure to punish - or punish adequately - such an offence will lead to those
outraged to take physical action to vent their feelings. This is the basis for laws and punishment
against those who express views which give serious offence to religious or racial groups. Indeed, this
was the original justification for having laws at all, because, in the absence of government having a 35
system of law and order and punishment for violation of those laws, victims of wrongs committed
against them would have to resort to self-help to gain redress for the loss and suffering they had
sustained.
Adapted from the article “Crime and Punishment”
Name Civics Group GP Tutor

SERANGOON JUNIOR COLLEGE


JC2 Preliminary Exam 2010

GENERAL PAPER 8806/02

Paper 2

Monday 16 August 2010


Insert & Question Paper 1 hour 30 minutes

Candidates answer on the Question Paper.


Additional Materials: 1 Insert

READ THESE INSTRUCTIONS FIRST

Write your name and civics group on all the work you hand in.
Write in dark blue or black pen in the spaces provided on the
For Examiner’s Use
Question Paper.
Short Qn /14
Answer all questions.
At the end of the test, fasten all your work securely together. Vocab /5
The number of marks is given in brackets [ ] at the end of each
question or part question. Summary /8
The Insert contains the passages for comprehension.
(Note that 15 marks out of 50 will be awarded for your use of AQ /8
language.)
Content /35

Language /15

Total /50

This Question Paper consists of 7 printed pages and 1 blank page and 1 Insert.

[Turn over]
2

Read the passage in the insert and then answer all the questions which follow below. Note that up
to fifteen marks will be given for the quality and accuracy of your use of English throughout this
Paper.

Note: When a question asks for an answer IN YOUR OWN WORDS AS FAR AS POSSIBLE and
you select the appropriate material from the passage for your answer, you must still use
your own words to express it. Little credit can be given for answers which only copy
words or phrases from the passage.

Questions on Passage 1

1 What are the ‘four horsemen of the correctional apocalypse’ (line 4-5)? Use your own words
as far as possible.

.............................................................................................................................................................

.............................................................................................................................................................

.............................................................................................................................................................

.........................................................................................................................................................[2]

2 ‘...there is an inverse relationship between the numbers of people in prison and the numbers in
mental hospitals’ (line 6-7). What does the phrase suggest about criminals?

.............................................................................................................................................................

.........................................................................................................................................................[1]

3 (a) What does the author mean by the word ‘sexier’ (line 18)?

.............................................................................................................................................................

.........................................................................................................................................................[1]

(b) Why might the Freudian approach be the ‘sexier’ theory?

.............................................................................................................................................................

.........................................................................................................................................................[1]
3

4 “... whether it is the ‘thrashing’ of the opposition at a game of rugby or football...” (line 26).
Why does the author place the word ‘thrashing’ in inverted commas?

.............................................................................................................................................................

.............................................................................................................................................................

.........................................................................................................................................................[2]

5 What is the author’s attitude towards the ‘moralists’ (line 30)? Justify your answer.

.............................................................................................................................................................

.............................................................................................................................................................

.........................................................................................................................................................[2]

6 Explain the author’s intention in using the word ‘even’ (line 44)?

.............................................................................................................................................................

.............................................................................................................................................................

.............................................................................................................................................................

........................................................................................................................................................[2]

Questions on Passage 2

7 Suggest why the principle of proportionality is ‘commonly accepted as a necessary ingredient


of a rational sentencing policy’ (line 26)?

.............................................................................................................................................................

.............................................................................................................................................................

.............................................................................................................................................................

........................................................................................................................................................[2]

8 Explain, in your own words as far as possible, why there are laws “against those who
express views which give serious offence to religious or racial groups” (line 35).

.............................................................................................................................................................

.............................................................................................................................................................

.........................................................................................................................................................[1]
4

9 Using material from paragraphs 1-5 of the passage, summarise what the author has to say
about the different approaches of punishment, which he prefers, and the limitations of the
utilitarian approach.

Write your summary in no more than 120 words, not counting the opening words which are
printed below. Use your own words as far as possible.

One justification for punishment is …………………………………………………………………….

…………………………………………………………………………………………………………………

…………………………………………………………………………………………………………………

…………………………………………………………………………………………………………………

…………………………………………………………………………………………………………………

…………………………………………………………………………………………………………………

…………………………………………………………………………………………………………………

………………………………………………………………………………………………………………..

…………………………………………………………………………………………………………………

…………………………………………………………………………………………………………………

…………………………………………………………………………………………………………………

………………………………………………………………………………………………………………..

…………………………………………………………………………………………………………………

………………………………………………………………………………………………………………..

…………………………………………………………………………………………………………………

…………………………………………………………………………………………………………………

…………………………………………………………………………………………………………………

………………………………………………………………………………………………………………..

…………………………………………………………………………………………………………………

…………………………………………………………………………………………………………………

……………………………………………………………………………………………………………….[8]
5

10 Give the meaning of the following words as they are used in the passage.

Write your answer in one word or a short phrase.

Passage 1

a) admonition (line 14) ........................................................................................................ [1]

b) primordial (line 29) ........................................................................................................ [1]

c) quell (line 39) ........................................................................................................ [1]

Passage 2

d) stamp out (line 29) ............................................................................................................. [1]

e) conversely (line 31) ........................................................................................................ [1]

[Total: 5]
6

11 Turner explores the different reasons for punishment, while Hwang discusses the functions of
punishment.

To what extent are the authors’ views relevant to your society?

…………………………………………………………………………………………………………………

…………………………………………………………………………………………………………………

…………………………………………………………………………………………………………………

…………………………………………………………………………………………………………………

…………………………………………………………………………………………………………………

…………………………………………………………………………………………………………………

………………………………………………………………………………………………………………..

…………………………………………………………………………………………………………………

…………………………………………………………………………………………………………………

…………………………………………………………………………………………………………………

………………………………………………………………………………………………………………..

…………………………………………………………………………………………………………………

………………………………………………………………………………………………………………..

…………………………………………………………………………………………………………………

…………………………………………………………………………………………………………………

…………………………………………………………………………………………………………………

………………………………………………………………………………………………………………..

…………………………………………………………………………………………………………………

…………………………………………………………………………………………………………………

…………………………………………………………………………………………………………………

…………………………………………………………………………………………………………………

…………………………………………………………………………………………………………………

…………………………………………………………………………………………………………………

………………………………………………………………………………………………………………..

…………………………………………………………………………………………………………………

…………………………………………………………………………………………………………………
7

…………………………………………………………………………………………………………………

………………………………………………………………………………………………………………..

…………………………………………………………………………………………………………………

………………………………………………………………………………………………………………..

…………………………………………………………………………………………………………………

…………………………………………………………………………………………………………………

…………………………………………………………………………………………………………………

………………………………………………………………………………………………………………..

…………………………………………………………………………………………………………………

…………………………………………………………………………………………………………………

…………………………………………………………………………………………………………………

…………………………………………………………………………………………………………………

…………………………………………………………………………………………………………………

…………………………………………………………………………………………………………………

………………………………………………………………………………………………………………..

…………………………………………………………………………………………………………………

…………………………………………………………………………………………………………………

…………………………………………………………………………………………………………………

………………………………………………………………………………………………………………..

…………………………………………………………………………………………………………………

………………………………………………………………………………………………………………..

…………………………………………………………………………………………………………………

…………………………………………………………………………………………………………………

…………………………………………………………………………………………………………………

………………………………………………………………………………………………………………..

……………………………………………………………………………………………………………….[8]
8
BLANK
SRJC Preliminary Examination 2010
Paper 2 Answer Scheme

1. In your own words as far as possible, identify the „four horsemen of the correctional
apocalypse‟. (line 6-7) [2m]

Lifted
the need for revenge, a form of The desire for:
pleasure; the need for Retribution/ vengeance
compensation for losses Reparation/ receive payment for damages/ replacement for
suffered; the need to consider damages
the safety of society; some sort public security
of reinstatement or even therapy rehabilitation/ reintegration
for the offender
Award ½ mark per point

2 „...there is an inverse relationship between the numbers of people in prison and the
numbers in mental hospitals‟ (line 8-9). What does the phrase suggest about criminals?[1m]

Lifted
Inferred Most criminals are psychologically unsound
The relationship is an „inverse‟ Or
one... There is little difference between the insane and the criminal
Or
Criminals are insane and the insane are responsible for crime

3. (a) What does the writer mean by the word „sexier‟ (line 18)?
(b) Why might the Freudian approach be the „sexier‟ theory? [2m]

Lifted
The more complex approach... Less boring/ more interesting/ intriguing/ appealing [1]

(If the comparative word „more‟ is missing – 0m)

Psycho-analysis...inner feelings, The Freudian approach deals with the mind and emotions, while
fantasies, fixations etc... (line14- the behaviourist approach considers the actions of the individual.
15) [1]
OR
The Freudian approach is more sophisticated/ profound/ complex
than the behaviourist approach [1]
This subconscious self derives
from... (line 28) (To be awarded the mark, there MUST be some comparison with
the behaviourist approach.
Although it is not necessary for the student to explain the
behaviourist approach.
Just need to make mention of behaviourism)
4. “... whether it is the „thrashing‟ of the opposition at a game of rugby or football...” (line 26)
Why does the author place the word „thrashing‟ in inverted commas? [2m]

Lifted Inferred
“whether it is the „thrashing‟ of The author uses it to indicate that he is not referring to the
the opposition at a game of conventional understanding of the word, which means a
rugby or football, or whether it physical beating. (1m)
is the thrashing we receive
from an angry parent.” Instead the word „thrashing‟ refers to the huge psychological or
emotional effect of losing a game. (1m)
OR
„thrashing‟ refers to the idea of defeating one‟s opponent by a
huge margin (1m)

[Stating that it is a metaphor – ½ m]

5. What is the author‟s attitude towards the „moralists‟ (line 30)? Justify your answer. [2m]

Lifted
The dark army of medieval The author [is critical of]/ [strongly disapproves of] the moralists
moralists, increasingly [1]
threatened by the outriders of
post-Darwinian rationalism, is He uses the word „medieval‟/ „primitive‟ to describe the
becoming ever more fierce in moralists, which indicates their backwardness.
its demands. (line 43) Or
The return to the death He uses the word „dark‟, suggesting that the moralists are too
penalty in many American harsh in their approach. [1]
states reflects exactly this Or
primitive, sadistic urge. (line The author uses negative connotations [ ½ m]
48 – 49)
[Marks can be awarded for justification of the answer, even
when the attitude is not precise; if the attitude is wrongly
inferred, no mark will be given to the part.]

[If the words from the passage are not identified – ½ m]

6. Explain the author‟s intention in using the word „even‟ (line 44)? [2m]

Lifted Inferred
Even adolescents and „retarded Youths and the mentally challenged are generally
individuals‟ are no longer immune considered to be less capable of rational/sound
from the death penalty in America. decision/less capable of knowing the full consequence of
their action, and thus are supposed to be spared the death
penalty [1]

Therefore, the author is emphasizing his point that the


American society/our society has gone overboard in our
use of punishment to control crime/the punishment is
excessive.[1]

[If the student does not include „author intends to show‟/the


author is emphasizing the point - ½ m]
3

PASSAGE 2
7. Suggest why the principle of proportionality is „commonly accepted as a necessary
ingredient of a rational sentencing policy‟ (line 26)? [2m]

Lifted Inferred
That principle reflects the correct The principle ensures that the punishment we impose on a
place where retribution ought to be criminal fits seriousness of the crime. [1]
reflected in punishment – in the
distribution of justice. It is commonly accepted because it appeals to human
being‟s natural desire/instinct for fairness. [1]
OR
It appeals to most people‟s sense of fairness/justice. [1]

[If the answer does not include „naturally‟/ „appeals to most


people‟ – ½ m]

[Accept other reasonable answers]

*need to explain proportionality - punishment must meet


severity of crime.

8. Explain, in your own words as far as possible, why there are laws “against those that
express views which give serious offence to religious or racial groups” (line 35) [1m]

Lifted Paraphrase
an offence may result in no overt Such remarks may cause strong reactions among people.
harm, but may attract such (1/2m)
moral outrage that, a failure to
punish - or punish adequately - Thus if there are no laws to punish the culprits, those who
such an offence will lead to those are insulted will feel a strong/great sense of injustice ( ½ m)
outraged to take physical action
to vent their feelings. and may decide to take matters into their own hands and
(line 32-34) turn violent against those who have offended them ( ½ m)

Bonus: ½ m

9. Using material from paragraph 1-5, summarise what Michael Hwang has to say about the
different purposes of punishment, which he prefers, and the limitations of the utilitarian
approach. 120words. [8m]
Lifted Paraphrased
The traditional understanding of punishment is
The old view...„retributive justice‟ which is
a founded on retaliation against the perpetrator/ tit for
based on the „eye for an eye’ principle.
tat.
Hwang prefers the utilitarian approach, in which
I prefer an alternative view...punished only punishments are meted out because they serve to
b
for social reasons. benefit the community.

the utilitarian looks to the future to justify Punishment functions to caution against potential acts
the imposition of punishment (line 7) of crime.
The utilitarian justification for punishment OR
is...to prevent future offences of a similar Punishment discourages the rest of the society from
c
kind, whether by that offender or others engaging in transgressions.
(line 8-9) OR
principle of deterrence (line 9) Punishment functions to avert potential acts of
serving as a warning to others (line 11) transgressions/crimes.
The punishment imposed should be appropriate or
sentence should be determined by its effect
d suitable for the criminal, such that it deters him from
upon the person punished (line 10-11)
repeating the crime.

Moreover,
penal process... reinforcing the social values punishment strengthens the moral codes/ ethics/code
e
of the community (line 13-14) of behaviour/standards of society

However,
The educational process [of punishment]
the teaching of moral codes and/or reformative
should only be regarded as a side effect
f process should not be the sole/exclusive/main
of punishment, and not as its primary
reason/rationale for punishment.
justification (line 14-15)
Instead, punishment should serve to keep the society
and its occupants/people safe from
protect the community and individuals from
g *palpable/actual/real danger.
*tangible harm (line 19)
*Award ½m if student does not qualify the type of
harm.
utilitarianism does not provide a self-
Utilitarianism alone is inadequate as a reason for
h contained justification of punishment.
castigation (1m)
make deterrence the sole criterion for
i When prevention becomes the only purpose (½ m),
punishment harsh penalties
there would be times when the easy way to
abolish a socially undesirable practice would
sometimes the convenient approach to eliminate a
be to impose extremely harsh penalties
communally unacceptable behaviour might involve
j
enforcing exceedingly ruthless sentences.(1m)
...is commonly accepted as a necessary
ingredient of a rational sentencing policy,
that is, the principle of proportionality
The legal system faces ethical constraints in using
there is a moral limit to the law’s power to a criminal as a case in point to deter others.
k make an offender an example for others to Or
fear. There must be an ethical boundary which punishment
should not exceed.
On the other hand,
there may be occasions where an offence a punishment that does match the offence might be
may result in no overt harm, but may attract considered inadequate by those who are offended.
such moral outrage that, a failure to punish - (1/2m)
l
or punish adequately - such an offence will The strong sense of injustice felt might in turn cause
lead to those outraged to take physical these people to take things into their own hand,
action to vent their feelings leading to greater crime. (1/2m)

Give the meaning of the following words as they are used in the passage. You may write
your answer in one word or a short phrase.

1m ½m 0
admonition (line  chastisement  violence carried out to  reprimand
14)  punishment correct a mistake  rebuke
 disciplinary action  scolding
primordial (line  primal  ancient  prehistoric
29)  basic  inborn
 fundamental  innate
 primitive  natural
 existing from the
beginning of time
5

quell (line 39)  put down  limit  calm


 subdue  contain  pacify
 suppress  stop  ease
 curb  counter
 to put a stop to  mitigate

stamp out (line  eliminate  weed out  exclude


29)  wipe out  end  take out
 eradicate  abolish
 remove  exterminate
 put an end to  quell
 prevent

conversely (line  on the contrary  however  vice versa


31)  on the other hand  alternatively
 on the flip side
 in contrast

[5m]
6
SRJC 2010 Prelim Paper 2 AQ Answer Scheme

11. Turner discusses the different reasons for punishment, while Hwang discusses the functions of punishment

To what extent are the authors’ views relevant to your society? [8m]

Requirements
- Using arguments from the passage, students should explain the extent to which the author’s views are relevant to Singapore society and evaluate the
authors’ views in the context of Singapore

Possible approaches
- The authors’ views are relevant to Singapore society to a large/small extent.
- Students can address:
=> How the authors’ views are/ are not represented in Singapore.
=> How significant the authors’ views are to Singapore?
=> Why does Singapore practise the existing policies and explain this in relation to the authors’ views.

Passage 1

Reference Relevant Irrelevant


1. There are various reasons for modern Revenge implies that the wrongdoer is punished
chastisement. These include the need for only with the aim of satisfying the victim(s).
revenge... (line 2-3) While this may be true to some extent, it does
not adequately represent the rationale behind
From Passage 2: Singapore’s legal system.
‘retributive justice’ which is based on the ‘eye for an
eye’ principle (line 2-3)
It is obvious that revenge is not the sole reason
for punishment in Singapore, considering the
emphasis placed on deterrence (refer to point
10) and rehabilitation (refer to point 5).
Reference Relevant Irrelevant

2. There are various reasons for modern Although it is conceivable that people do feel
chastisement... a form of pleasure (line 2-3) some kind of pleasure to see wrong-doers
punished, this reason appears superfluous and
From Passage 2: redundant in the modern world.
psycho-analysts... This subconscious self derives
from the control our parents exercised over us and
It seems to appeal to our baser instincts and as
acts as a kind of conscience. This may sometimes
involve the need for the pain of punishment to modern, civilised humans, we should eschew
alleviate the chronic guilt coming from our such a motive for punishing people.
Superego. Thus, accepting punishment – and, of
course, seeing other people accept punishment – It is also strangely perverse to maintain that
becomes part-and-parcel of our personal stability. people enjoy corporal punishment being
(line 21-25) inflicted upon them, as if all of us are natural
masochists. This is highly unlikely to be the
Freud seems to suggest that punishment is a natural case especially when masochism may be
part of the human state. We can enjoy it. (line 25) classified as a neurosis.

3. There are various reasons for modern Punishment seems to offer scant compensation
chastisement... the need for compensation for losses where criminal acts are concerned. For instance,
suffered (line 2-4) with regards to capital punishment which is
practised in Singapore, executing a murderer
cannot bring the murdered person back to life
and hence cannot really compensate for the loss
suffered by the family of the victim.

4. There are various reasons for modern See point 5, 10 & 11


chastisement... the need to consider the wider safety
of society (line 2-4)

2
Reference Relevant Irrelevant

5. There are various reasons for modern The Singapore Prison Service uses the words Much of the rehabilitation programs are not
chastisement... re-instatement or even therapy for ‘Rehab, Renew, Restart’ to explain the work working, as shown by the relatively large
the offender (line 2-5) they do, highlighting the efforts placed in percentage of re-offenders.
looking after the offender’s psychological,
social, emotional and physical needs to ensure This high rate of recidivism- past offenders
re-integration into society. returning to prison- can be attributed to the
problems they face while in the criminal justice
The Singapore Prison Service launched the system (e.g stigma of being an ex-convict).
Yellow Ribbon Project to raise awareness of the Prisons also act as schools for crime. Research
stigma and labeling attached to being an ex- has shown that ex-convicts often leave prison
convict. This is meant to encourage society to more committed as criminals and with a higher
accept the ex-convicts and help them re- likelihood of continued criminal involvement.
integrate into the community.

6. ... in industrialized societies that there is an According to a study published in the American
inverse relationship between the numbers of people Journal of Public Health, about a quarter of
in prison and the numbers in mental hospitals. (line inmates in the US had a history of chronic
7-8) mental illnesses like schizophrenia, bipolar
illness and depression. This could similarly be
the case for Singapore.
Passage 2

Reference Relevant Irrelevant

7. utilitarian justification for punishment... to In Singapore, corporal punishment such as The principle of deterrence may have proven to be
prevent future offences of a similar kind, whether by caning is used to protect society through ineffective.
that offender or others (line 9-10) deterrence. It is hoped that the experience of
being caned would be sufficiently bad, that E.g In 1994, Michael Fay, an American teenager
the principle of deterrence should underpin a
future crime will be discouraged. was convicted of car vandalism and was caned 4
rational policy of sentencing. The sentence should
times in a Singapore prison.
be determined by its effect upon the person
punished or by serving as a warning to others (line
10-12) Similarly, out of a total of 32,800 criminals
convicted in 2000, it was found that 43,000 of them
were reoffenders. The shows that the principle of
deterrence is not effective in Singapore.

With regards to capital punishment in relation to


murder, it is also not a suitable deterrent. Research
has shown that murder is usually committed not
after a rational consideration of the act or its
consequences but during an outburst of emotion.
Under such irrational circumstances, one would not
expect the fear of capital punishment to be a
deterrent.

4
Reference Relevant Irrelevant

8. The purpose of the criminal law is not to enforce To protect Singapore society from drugs and
the moral standards of the community as such, but to prevent the human misery which drugs
only to protect the community and individuals from create, tough laws and penalties are meted
tangible harm (line18-20) out to drug traffickers.

E.g
Drug Traffickers who carry significant
amounts of harmful drugs in Singapore face
capital punishment.
This is meant to send a strong deterrent
signal to would-be traffickers.

9. ... make deterrence the sole criterion for See point made above. This did not serve as a deterrent to others as shown
punishment, there would be times when the easy by the vandalism committed by Oliver Fricker a
way to abolish a socially undesirable practice would Swiss, who was recently charged and sentenced to
be to impose extremely harsh penalties (line 23-25) 3 strokes of the cane.
Reference Relevant Irrelevant

10. ...the community would, rightly, reject such One area that breaches the principal of
penalties because they would violate another proportionality is the mandatory death penalty for
principle that is commonly accepted as a necessary drug trafficking. Detractors of this harsh law point
ingredient of a rational sentencing policy, that is, the out that the punishment seems to be manifestly
principle of proportionality. (line 26-28)
excessive and making it mandatory for judges to
impose such a penalty worsens an unjust law.

Making it mandatory sends a strong deterrence


signal to would-be offenders. But the nagging
question remains - what if there are extenuating
circumstances, as in the recent case of Yong Vui
Kong who committed the offence just after he
turned 18.

In some countries, vandalism is seen as a mere


misdemeanor. However, in Singapore those who
are charged with vandalism face corporal
punishment. The punishment does not fit the
severity of the crime.

It can also be observed that the Singapore


community does not feature prominently in official
decision making in the context of criminal justice.
This could be attributed to the possibility that
(a) Singaporeans view the punishments to be
proportionate to the crime and therefore fair
(b) there is official down playing of moral
discourse
(c) Singaporeans are willing to trade-in their
human rights for better crime control.

6
Reference Relevant Irrelevant

11. ... occasions where an offence may result in no Most Singaporeans will empathise with the
overt harm, but may attract such moral outrage that, view that those who pass disparaging
a failure to punish - or punish adequately - such an remarks concerning race or religion should
offence will lead to those outraged to take physical be dealt with by the law. Indeed,
action to vent their feelings. This is the basis for
transgressors have been charged for
laws and punishment against those who express
contravening either the Sedition or
views which give serious offence to religious or
racial groups. (line 33-36) Maintenance of Religious Harmony Acts.
As such, incidents such as the publication of
the Prophet Mohammad cartoons in certain
European newspapers some years ago would
never be allowed to occur here.

The drawback to this inhibition of free


speech however, is that there is little
dialogue about these important matters.
Civil disorder is pre-empted but the cost that
is paid is stunted minds that are apt to
indulge in self-censorship or to be overly
sensitive whenever such ideas are
mentioned.
TEMASEK JUNIOR COLLEGE
JC 2 PRELIMINARY EXAMINATION 2010

GENERAL PAPER 8806/01

Paper 1 13 Sep 2010


1hour 30 minutes

READ THESE INSTRUCTIONS FIRST

Write your Centre number, index number and name on all the work you hand in.
Write in dark blue or black pen on both sides of the paper.
Do not use staples, paper clips, highlighters, glue or correction fluid.

Answer one question.


Note that 20 marks out of 50 will be awarded for your use of language.

At the end of the examination, fasten all your work securely together.
All questions in this paper carry equal marks.

This paper consists of 2 printed pages.

[Turn over
Answer one question from this Paper.

Answers should be between 500 and 800 words in length.

1. Bad government results from too much government. Do you agree?

2. Globalization is a pandemic that is making the whole world sick. Is this a


fair comment?

3. In youth we learn; in age we understand. Comment.

4. Discuss the view that harsh punishment has no place in a civilized society.

5. “Science without religion is lame, religion without science is blind.” (Albert


Einstein) To what extent is this true?

6. Transnational corporations should pay more for environmental damage.


Discuss.

7. Consider the view that artists can only contribute to society if they are
given full freedom.

8. Celebrities show us the life we all yearn to live. Discuss.

9. “Success is the ability to go from one failure to another with no loss of


enthusiasm.” (Winston Churchill) Is this good advice for young people
today?

10. Is the road less travelled always the better path?

11. To what extent is education the solution to social ills today?

12. Ability may get you to the top, but it takes integrity to keep you there.
Comment.
TEMASEK JUNIOR COLLEGE
JC 2 PRELIMINARY EXAMINATION 2010

GENERAL PAPER 8806/02

PAPER 2

INSERT

13 Sep 2010

1 hour 30 minutes

INSTRUCTIONS TO CANDIDATES
This insert contains the passages for Paper 2.

This insert consists of 4 printed pages


[Turn over]
2

Passage A

1 Rubbishing cities is a popular sport. Not simply because of the garbage, graffiti,
pollution, congestion and crowds people complain about – there is something
about the very essence of cities and their inhabitants that offends too. Surveys
have shown that, while around three-quarters of Britain's town- and country-
dwellers are satisfied with their quality of life, only about 10% of urbanites are 5
happy. According to writer Julie Burchill, that is why you see so many of them on
the Brighton seafront every weekend – "thousands of Londoners set free for the
day, blinking and smiling with surprise at all this light and space, poor mole-
people above ground at last."

2 At the very least, life in cities should offer more variety and be more fulfilling than 10
a life spent scratching a bare living direct from the soil. It might even be more fun,
but as cities have severed the ties that once bound people firmly to the land, so
the links between urban and rural environments have become more important
than ever. The inhabitants of today's cities are more utterly dependent on the
services of nature than at any previous time in history. We tend to forget that, 15
while London, Paris, Venice, New York and numerous other cities sustain and
entertain millions of us, cities are monstrous parasites, consuming the resources
of regions vastly larger than themselves and giving very little back. In fact, though
cities today occupy only 2% of Earth's land surface, they consume more than
75% of its resources. The implications of that are powerfully illustrated by a 20
concept environmental scientists developed during the 1990s: the ecological
footprint.

3 The ecological footprints of many cities have been assessed and the results are
uniformly alarming. Vancouver, for instance, though rated highly in terms of the
quality of life its half a million residents enjoy, has an ecological footprint more 25
than 200 times the size of the city. The 29 largest cities of the Baltic Sea drainage
system appropriate the resources of an area 565 times larger than the land they
occupy.

4 Given the success of the evolutionary trajectory humanity pursued for the first few
million years – no other species has achieved such total dominance of the global 30
environment – cities are a complete contradiction. It is biology that drives
evolution and, from a biological point of view, cities are a seriously bad idea. The
dangers of disease multiply when people are crowded together, and our aversion
to squalor and unpleasant odours is a measure of the depth at which an innate
acknowledgement of those dangers is set in our evolutionary history. We are 35
social animals, true enough, but there are limits, and our hunting and gathering
ancestors probably had the numbers about right. They were nomadic, moving
around in groups of up to 40 or so, and never staying long enough in one place
for pathogens to build up to potentially deadly levels. In contrast, cities have been
– quite literally – the breeding grounds of disease. 40

5 Bacterial and viral diseases are the price humanity has paid to live in large and
densely populated cities. Virtually all the familiar infectious diseases have evolved
only since the advent of agriculture, permanent settlement and the growth of
cities. Most were transferred to humans from animals – especially domestic
3

animals. Measles, for instance, is akin to rinderpest in cattle; influenza came from 45
pigs; smallpox is related to cowpox. Humans share 296 diseases with domestic
animals. Thus, until comparatively recent times, cities had a well-earned
reputation for being unhealthy places. No wonder demographers and historians
write of the "urban graveyard effect".

6 However, the brutal fact is that, while one-third or more of city-dwellers in the 50
developing world live on or below the poverty line, only about one-third of the
rural population lives above it. A typical study of urbanisation in the developing
world concludes that despite appalling housing conditions, lack of fresh water and
services, minimal health care and few chances of finding a job, the urban poor
are on average "better off than their rural cousins, on almost every indicator of 55
social and economic well-being".

7 Better off? Well-being? Don't ask how the lives of these impoverished city-
dwellers compare with those of the 90% of British urbanites who are dissatisfied
with their quality of life. Only note that, for many millions of people, cities are the
solution, not the problem. 60

Adapted from “No City Limits” by John Reader,


The Guardian, 11 September 2004

Passage B

1 Those who say the city has had its day point to economic and technical changes
that seem to remove one of the most basic reasons for getting together in an
urban huddle. No longer do people have to gather round the agora 1 to do their
business. Information technology allows them to work wherever they want. Given
that they can also get a religious, sporting or cultural fix by turning on the 5
television, and do their shopping as well as their work on the Internet, why live in
a city? As Jefferson said, cities are “pestilential to the morals, the health, and the
liberties of man.” They are the sort of places where you get mugged.

2 Not so fast. Other changes suggest that the city is not doomed. Land is finite,
population is still expanding and the motor car's dominance may not last much 10
longer. With global warming and no economic alternative to scarce petrol, it may
not be feasible to go on living 20km away from everything – school, work,
babysitter, Starbucks.

3 In any event, other trends suggest that for every Timmy Willie, there is a Johnny
Town-Mouse. Many people like urban life and want to go on living in a city, 15
particularly the centre. Among them are the elderly, a growing share of the
population, who want easy access to transport, doctors, hospitals, cinemas and,
above all, family and friends. And the young are urban creatures, too. They like
the buzz of a city, the concentration of restaurants, clubs and other forms of
entertainment. And the better educated (and so the richer) are likely to find work 20
in the universities, hospitals and research centres that tend to cluster in cities.
The suburbs may be pleasant enough when parents are absorbed with work and

1
A place of assembly; a political and commercial space
4

children, but for the childless and the empty-nesters, the city has many merits.

4 Several academics take this view. Some, such as Richard Florida, of George
Mason University, see cities as natural homes for the “creative class”, whose 25
members are artists, designers, academics and so on. Others, such as Terry
Nichols Clark, of the University of Chicago, stress the pleasures of the city as a
reason to live there: entertainment, they say, can replace manufacturing in the
post-industrial city, providing both jobs and fun.

5 Others find further reasons for optimism. Bruce Katz, of the Brookings Institution 30
in Washington, argues that there is much more inventiveness at municipal and
state levels in America than at federal level. A city like Denver is exploiting its
power to tax to introduce a light-rail system. Private-sector investment is being
combined with government money for urban purposes much more widely and
effectively. Cities such as Chicago are now seen as central to environmental 35
improvements. All this means that public policy is becoming more city-centred.

6 At the same time, cities are becoming sexier in the popular imagination – literally,
in the case of “Sex and the City”, but more metaphorically through other television
shows like “Seinfeld” and “Friends”. The trendiness is not confined to New York.
For anyone on the way up, the city is the place to be. Some 60% of the jobs in 40
American cities fall into the “new economy” category, compared with about 40%
in the Sprawl-Mart suburbs. And once they have gotten to the top, the successful
do not always opt for wide-open spaces: the most densely populated borough in
Britain is London's smart Kensington and Chelsea.

7 But talk to many an inhabitant of today's big cities and you soon detect a rural 45
background, and often a slight wistfulness with it. Where do Chinese city-dwellers
go for their holidays? Back to where they, or their family, once came from. Where
do urban Africans get buried? In their villages. Even in highly urbanised Japan,
the farmer and his rice field maintain a special place in the mind of the Tokyo
sarariman. Cities may be the epitome of modernity, but they are inhabited by a 50
creature designed for a pre-agricultural existence. The supermarket is no
substitute for the steppes, plains and savannahs of the hunter-gatherer. The
office chair is no place for the descendants of Homo erectus. No wonder there is
a tension between habitat and inhabitant. But there is no going back to the
countryside now. 55

Adapted from “Et in Suburbia Ego”,


The Economist, 3 May 2007
Candidate’s Name CG GP Tutor’s Name

TEMASEK JUNIOR COLLEGE


JC 2 PRELIMINARY EXAMINATION 2010

GENERAL PAPER 8806/02

PAPER 2 13 Sep 2010


1 hour 30 minutes

Candidates answer on the Question Paper.


Additional Material: 1 Insert

READ THESE INSTRUCTIONS FIRST

Write your name, CG and GP tutor’s name in the spaces at the top of the page.
Write in dark blue or black pen.
Do not use staples, paper clips, highlighters, glue or correction fluid.

Answer all questions.


The Insert contains the passages for comprehension.
Note that 15 out of 50 marks will be awarded for your use of language.

The number of marks is given in brackets [ ] at the end of each question or part question.

For Examiner’s Use

Content /35

Language /15

Total /50

This document consists of 7 printed pages and 1 blank page.

[Turn over]
2

Read the passage in the insert and then answer all the questions which follow For
below. Note that up to fifteen marks will be given for the quality and accuracy of Examiner’s
Use
your use of English throughout this Paper.

NOTE: When a question asks for an answer IN YOUR OWN WORDS AS FAR
AS POSSIBLE and you select the appropriate material from the passages for
your answer, you must still use your own words to express it. Little credit can be
given to answers which only copy words or phrases from the passage.

From Passage A

1 “set free for the day, blinking and smiling with surprise at all this light and
space, poor mole-people above ground at last” (lines 7 – 9). What does this
imply about city life? Use your own words as far as possible.

…………………………………………………………………………………………

………………………….……………………………………………………………..

…………………………………………………………………………………………

…………………………………………………………………………………………

…………………………………………………………………………………………

……………………………………………………………………………………. [2]

2 In what way are cities “monstrous parasites” (line 17)? Use your own words
as far as possible.

…………………………………………………………………………………………

…………………………………………………………………………………………

…………………………………………………………………………………………

…………………………………………………………………………………………

…………………………………………………………………………………………

…………………………………………………………………………………………

…………………………………………………………………………………………

…………………………………………………………………………………….. [3]

[Turn over]
3

For
Examiner’s
Use
3 Explain, in your own words as far as possible, why “cities are a complete
contradiction” (line 31) to the success of human evolution.

……………………………………………………………………………..….……….

……….………………………………………………………………………………...

…………………………………………………………………………………………

…………………………………………………………………………………………

…………………………………………………………………………………….. [2]

4 What is meant by the phrase “urban graveyard effect” (line 49)? Use your
own words as far as possible.

………………………………………………………………………………………

………………………………………………………………………………………

…………………………………………………………………………………….[1]

5 Explain the irony of city living. Use material from lines 52 – 56 for your
answer.

………………………………………………………………………………………

………………………………………………………………………………………

………………………………………………………………………………………

………………………………………………………………………………………

…………………………………………………………………………………… [2]

From Passage B

6i Explain the meaning of “the city has had its day” (line 1).

…………………………………………………………………………………………

…………………………………………………………………………………………

………………………………………………………………………………………[1]

[Turn over]
4

For
6ii What are the reasons cited by those who say that the city “has had its day” Examiner’s
Use
(line 1)? Use your own words as far as possible.

............................................................................................................................

............................................................................................................................

............................................................................................................................

............................................................................................................................

........................................................................................................................[2]

7 What is the tone intended by the author when he included “Starbucks” (line 13)
in his list of “everything” (line 12)?

…………………………………………………………………………………………

…………………………………………………………………………………………

……………………………………………………………………………………… [1]

8 Why does the writer mention a series of examples in lines 46 to 48?


…………………………………………………………………………………………

…………………………………………………………………………………………

……………………………………………………………………………………….[1]

9 Give the meaning of the following words as they are used in the passage.
Write your answer in one word or a short phrase.

From Passage A

(a) severed (line 12) …………………..…………………………………........ [1]

(b) powerfully (line 20) ………...……………………………………………… [1]

(c) brutal (line 50) ……….…………………………………………………….. [1]

From Passage B

(d) absorbed (line 22) ………………………………………………..……….. [1]

(e) epitome (line 50) .………………………………………………………….. [1]

[Turn over]
5
For
Examiner’s
10 Using material from paragraphs 3 – 6, summarise the factors which indicate Use
that cities are not doomed (Passage B, line 9).

Write your summary in no more than 120 words, not counting the opening
words which are printed below. Use your own words as far as possible.

The city is not doomed because…………………………………………………....

………………………………………………………………………………………..

………………………………………………………………………………………..

………………………………………………………………………………….………

…………………………………………………………………………………………

…………………………………………………………………………………………

…………………………………………………………………………………………

…………………………………………………………………………………………

…………………………………………………………………………………………

…………………………………………………………………………………………

…………………………………………………………………………………………

…………………………………………………………………………………………

…………………………………………………………………………………………

…………………………………………………………………………………………

…………………………………………………………………………………………

…………………………………………………………………………………………

…………………………………………………………………………………………

…………………………………………………………………………………………

…………………………………………………………………………………………

…………………………………………………………………………………………

………………………………………………………………………………………[7]

[Turn over]
6

For
11 In Passage A, Reader argues that cities present a host of environmental and Examiner’s
health problems while in Passage B, the writer argues that “the city is not Use
doomed.” (line 9).

Are you more optimistic or pessimistic about city living?

In your answer, assess some of the issues raised by both writers and support
your views with examples drawn from your own observations and experiences.

.……….…………………..…………………………………………………………………

……………………………..………………………………………………..………………

……………………………….………………………………………………………...……

………………………………………..………………………………………………..……

…………………………………………..………………………………………………..…

………………………………………….....………………………….………………….…

…………………………………………………..………………………………………..…

……………………………………………………..……………………………………..…

………………………………………………………..…………………………………..…

………………………………………………………..…..……………………………..….

.………………………………………………………………..……………………………

……………………………………………………………………..…………………….…

………………………………………………………………………..…………………..…

…………………………………………………………………………..………………..…

……………………………………………………………………………..……………..…

……………………………………………………………………….……………………..

………………………………………………………………………………………………

…………..…………………………………………………………………….………….…

…………..……………………………………………………………………………..……

………………………………………………………………………………………………
7
For
Examiner’s
Use
…………..………………………………………………………………………………..…

………..…………………………………………………………………………………..…

…………..………………………………………………………………………………..…

……………..……………………………………………………………………………..…

………………..…………………………………………………………………………..…

………………………………………………………………...........................................

..................................................................................................................................

..................................................................................................................................

..................................................................................................................................

..................................................................................................................................

..................................................................................................................................

..................................................................................................................................

..................................................................................................................................

......................………………………………………………………………………………

…………….…...........................................................................................................

..................................................................................................................................

..................................................................................................................................

..................................................................................................................................

..................................................................................................................................

..................................................................................................................................

..................................................................................................................................

..................................................................................................................................

..................................................................................................................................

..................................................................................................................................

............................................................................................................................. [8]
8

BLANK PAGE
Prelims 2010: Proposed Questions and Answers

From Passage A

1. “Set free for the day, blinking and smiling with surprise at all this light and space,
poor mole-people above ground at last” (lines 7–9). What does this imply about
city life? Use your own words as far as possible. [2]

Lifted Answer Suggested Paraphrase


set free … surprise at all  City life appears to be stifling/ confining/ restrictive/
this light and space, poor claustrophic/ oppressive/ suffocating/ much like a
mole-people above ground prison/ confined cell/ overwhelming/ hectic and full of
at last (l.7-9) – to be stress/ pressurizing/ people feel imprisoned/ shackled/
inferred entrapped by the pressures of city life (1)
Hectic /harried ( ½ ); fast paced/ busy (0)

 The city is overcrowded, overly congested/ densely


populated/ too many people crammed/ packed/ trapped
in a small space (1)

 City life also seems to grey/ dull/ dreary/ gloomy/


dreary/ depressing (1) - due to the poor living
conditions/ dirt and grime/ pollution (explanation
optional)
Monotonous/ boring ( ½ ); burdened/ unpleasant (0)

Accept any 2 of the above answers.

2) In what way are cities “monstrous parasites” (line 17)? Use your own words as far as
possible. [3]

Lifted Answer Suggested Paraphrase


 consuming the  Both parasites and cities are small in size (1)
resources of regions  but just as parasites devour/ feed on their hosts
vastly larger than voraciously despite their size, cities utilize a massive
themselves, giving very amount of resources from other bigger regions (1)
little back (l.17-18) Lift: “consume/ consuming resources” (0)
 though cities today  Both also do this at the expense of the welfare of their
occupy only 2% of the hosts/ without benefiting their hosts ( 1 )
Earth‟s land surface, Or
they consume more  Just as the parasites deplete the nutrients of their
than 75% of its hosts, weakening them, cities contribute to the
resources (l.18-20) depletion of resources and caused environmental
 giving very little back damage to other regions
Cities contribute very little to environmental protection (0)

If students do not make clear step by step comparisons


between the qualities of parasites and cities, students will
only get half of the total marks allocated for each part.
1
3) Paragraph 4
Explain, in your own words as far as possible, why “cities are a complete
contradiction” (line 31) to the success of human evolution [2]

Lifted Answer Suggested Paraphrase


 Hunting and gathering ancestors  In the past humans did not reside in one place
had the numbers about right, long enough for diseases to spread, contributing
nomadic, never staying long to the success of human survival. (1)
enough in one place for  This is opposite to/ in conflict with the current
pathogens to build up to congested, dirty living conditions in the city which
potentially deadly levels (l.37- are conducive to the spread of epidemics (1).
39)
 but cities… the breeding If only one set of conditions (past or present) is
grounds of disease (l.39-40). given & mentions that the other is the opposite/
 Dangers of disease multiply contrasts with it is given, then answer = 1½ marks
when people are crowded e.g.
together… squalor & The over-populated/ over-crowded/ congested and
unpleasant odours (l.33-34) dirty conditions of the city which facilitate the easy
transmission of potentially fatal diseases ( 1 ) seem
to be in conflict with/ totally the opposite of the
conditions that allowed humans to thrive in the past
(½ )

4) What is meant by the phrase “urban graveyard effect” (line 49)? Use your own words
as far as possible. [1]

Lifted Answer Suggested Paraphrase


Bacterial and viral diseases are  It means living in cities can be hazardous to
the price humanity has paid to live health / results in many illnesses being
in large and densely populated transmitted easier to the dwellers ( ½ ), resulting
cities … share 296 diseases with in many deaths ( ½ )
animals (l.41-47) Deaths not mentioned = 0; greater risk of
transmission of diseases not mentioned also = 0

5) Explain the irony of city living. Use material from lines 52 – 56 for your answer. [2]

Lifted Answer Suggested Paraphrase


 despite appalling housing  Given the poor living conditions in the city, one
conditions, lack of fresh water would assume/ presuppose city dwellers to
and services, minimal health have a low quality of life. (1)
care and few chances of  Instead/ however, contrary to expectation,
finding a job they seem to enjoy a better standard of living
 the urban are on average and quality of life relative to those in rural areas.
“better off than their rural (1)
cousins, on almost every
indicator or social and → For 2nd part of the answer, if there is no
economic well-being comparison made between the urban and rural
poor, only ½ mark.
2
→ No marks will be awarded if the student gave
the reverse explanation i.e. “Instead of having a
better quality of life as expected, living conditions in
the city are poor.”

→ No marks will be given if only one part of the


answer is provided.

From Passage B

6i) Explain the meaning of “the city has had its day” (line 1). [1]

Suggested Answer
(inferred) - urban living is no longer popular / practical/ the city has outlived its
usefulness/ no longer serves its purpose/ no longer an ideal or good place to live in / has
lost its significance/ less attractive/ less appealing / not as desirable/ past its prime/ on
the decline(1)

→ City life is no longer needed/ necessary/ essential/ irrelevant/ obsolete/ no longer


useful/ beneficial/important/ significant (0)

6 ii) What are the reasons cited by those who say that the city “has had its day” (line 1)?
Use your own words as far as possible. [2]

Lifted Answer Suggested Paraphrase


 No longer get together in an  The city seems to have lost its function as a
urban huddle/ gather round the meeting place/hub for people to carry out
agora to do their business (l.3-4) transactions since technology now enables
 Info technology allows them to people to work and carry out their daily lives
work wherever they want – get anywhere, independently. (1)
their religious, sporting or
cultural fix by turning on the → Technology allows people to work and carry out
television, do their shopping… their daily activities (0) → does not explain why the
work on the internet (l.4-6) city has had its day.
 Pestilential to morals, the health
and liberties of man, sort of  The city is no longer an ideal place to live in as it
places you get mugged (l.7-8) is full of vices, crime and diseases (1)
→ Any two points = 1 mark
→ Morals/ moral values/ immoral/ unhealthy = 0
(lift)

7) What is the tone intended by the author when he included “Starbucks” (line 13) in his
list of “everything” (line 12)? [1]

Inferred - The author was taking a tongue-in-cheek / humorous / wry/ light hearted/
amused/ cheeky look at the essentials of modern life or
The writer‟s tone is one of resignation as trivialities are now seen as essentials
3
→ Even if the tone is right but the explanation is way off, making no sense (0)
→ Tries to be funny ( ½ ); mocking/ sarcastic (0)

8) Why does the writer mention a series of examples in lines 46 to 48? [1]

Lifted Answer Suggested Paraphrase


Where do Chinese city-dwellers go To emphasise/ highlight/ drive home the point/
for their holidays? Back to where reinforce his point
they, or their family, once came
from. Where do urban Africans get  our sense of loss and longing for the rural
buried? lifestyle
or
 our affinity for/ desire to re-connect with/ go
back to our “rural background”/ roots/ the
countryside (1)

→ ½ mark for author’s intention and ½ mark for an


explanation of his intention
→ ½ mark for “the writer wants to say/ show/
illustrate, express + explanation…”
→ 0 mark for “humans want to go back to…”
without a reference to the writer’s intention.
→ 0 mark for “go back to nature/ go back to their
homeland or home country”

9) Give the meaning of the following words as they are used in the passage.

Write your answers in one word or a short phrase. [5]

1 ½ 0
severed cut/cut off ended/ intentionally break/ break away/ break off/
(passage A, ended broken/ broken off
line 12) v forcibly/abruptly ended strained, disabled, disenabled,
disconnected/ weakened, destroyed,
forcibly/ abruptly completely damaged, amputated,
disconnected disconnected forcibly removed/ removed,
ripped, demolished, terminated,
detached completely
powerfully strongly, vividly clear/ clearly
(Passage A, with great impact/ persuasively evidently, obviously, patently,
line 20) adv impactfully convincingly substantially, forcefully,
have a significant and precisely, influentially, greatly,
widespread impact effectively, graphically,
significantly, saliently, cogently,
aptly, starkly, very well
brutal harsh, unvarnished, cruel, blunt crude, painful, cold, hard,
(Passage A, unpleasant, stark real but hard to shocking, serious, obvious
line 50) adj painful and hard to accept accept alarming, sad, real, critical,
4
very undesirable but true hard to accept
devastating but true very inconvenient, undeniable,
undisguised unfortunate, hurtful, aggressive
hard hitting, unforgiving nasty, bitter, unsparing, ugly,
sad but true
absorbed precoccupied, engrossed, occupied, concerned, involved
(Passage B, immersed, consumed by caught up, focused/ overly/ too focused/
line 22) v tied up obsessed/ fanatical
heavily/ fully/ too/ deeply/ overly concerned saturated, overwhelmed,
greatly/ completely/ overly concentrated on,
+ caught up/ involved in/ too/ totally/ overly busy,
tied up with/ occupied taken away,
fully filled with,
focused to the exclusion of overloaded, surrounded,
everything else/ totally indulged, chained, tires,
focused on completely glued
spending all of their time
deeply buried

epitome hallmark, embodiment, showing Typical/ main/ most significant


(Passage B, signature, quintessence, traits/qualities that feature
line 50) n (the) superlative characterize most apt characteristic
role model, strong symbol,
prime/ most triumphant/ icon/ iconic sign, representation, paragon,
perfect/ ultimate/ best/ distinguishing/ classic salient, record, highlight,
outstanding/ apt + symbol/ illustration example, high/ highest point/
representation/ model/ level/ form/ peak, pinnacle,
example/ illustration/ well representation apex, trademark, benchmark
symbol/ form (exp) crux/ core
defining aspect
model representation/ a good essence
example representation of essential factor
centre/ central
state of perfection defining aspect
the very representation of
symbolic representation top model
very essence

10) Using material from paragraphs 3 – 6, summarise the factors which indicate that
cities are not doomed.

Write your summary in no more than 120 words not counting the opening words which
are printed below. Use your own words as far as possible. [7m]

The city is not doomed because………………………………………………………………...

5
Lifted Answer Suggested Answer

 many people like urban life and want to A1  Many people still prefer/desire/ opt/
go on living in the city (l.15) choose to live in the city/ still a popular
choice for many people(1) {without the “still
prefer” ½ mark only}

 the elderly, a growing share of the A2  The city puts the elderly within easy reach
population, want easy access to of good infrastructure, services and
transport, doctors, hospitals, facilities. (1) {no mention the elderly = ½ }

 family and friends (l.16-18) A3  and loved/ close ones (1) { kin/ relatives only =
½ ; social circle only =½; companionship= 0}
 the young like the buzz of the city, the
concentration of restaurants, clubs and A4  The young enjoy the vibrant city life with its
other forms of entertainment (l.19-20)… diversity of leisure/ recreational activities.
pleasures of the city (l.27)…fun (l.29) (1) {no mention of the young = ½ ; entertainment =
lift = 0}
 for the childless and the empty-nesters, A5  The city thus offers many benefits
the city has many merits (l.22-23) especially to those with few or no family
commitments (1) {no mention of who = ½ }
 the better educated (and so the richer)
are likely to find work… (20-21)… B  More employment opportunities (1)
providing jobs (l.29) …some 60% of
American jobs in American cities fall
into the “ new economy” (l.40-41)

 cities as natural homes for the “creative  Cities have a rich cultural heritage/
class…artists, designers,”(l.25-26) C culturally vibrant/ rich in the arts and
culture (1)
 much more inventiveness at the
municipal and state level than at the  City policy makers have more innovative
D1
federal level… cities like Chicago are ideas to improve city living conditions (1)
now seen as central to environmental
improvements … public policy is
becoming more city-centred (l.31- 36)

 private sector investment …combined


with government money for urban  More private and public funds are
D2
purposes more widely and effectively channelled for these improvements (1)
(l.33-35)
 cities are becoming sexier in the  cities also have a hip/ cool image thanks to
E1
popular imagination…through television the media (1) {no mention of media = ½; trendy
shows… trendiness (l.37 – 39) to live in the city = lift = 0 }

 for anyone on their way up, the city is E2  it is the preferred/obvious choice for the
the place to be (l.40) upwardly mobile/ those seeking a better
life/ betterment/improvement (1)
6
11) In Passage A, Reader argues that cities present a host of environmental and health
problems while in Passage B, the writer argues that “the city is not doomed” (line 9).

Are you more optimistic or pessimistic about city living?

In your answer, assess some of the issues raised by both writers and support your
views with examples drawn from your own observations and experiences. [8]

Suggested approach:
R1: Are you more optimistic or pessimistic about city living?
1. Choose a view – optimistic (agree with writer from The Economist) OR
- pessimistic (agree with Reader)

2. Give reasons for your choice


*** While either is theoretically possible, it makes more sense to be optimistic.

R2a & R2b: Assess some of the arguments made by both writers.
1. Identify one important argument raised by Reader and the writer from The Economist
for discussion.
2. Evaluate the argument and provide reasons why you agree.

R3a & R3b: Support your views with examples drawn from your own
observations and experiences.
1. Substantiate your reasons with evidence i.e. provide relevant, concrete examples.
These examples need not be from your society.

Why I am more optimistic


Explanation and
Passage B Examples
Evaluation
1) …it may not be Stand 1) Moscow, Russia: one of the
feasible to go on living I am more optimistic about most precise subway in the world,
20km away from city living and thus agree carries over 8.2 million passengers
everything largely with the writer from on an average weekday.
(l.12-14) The Economist because city 2) Paris, France: the Paris Metro
… easy access to life does indeed provide a has one of the best coverage,
transport, doctors, host of benefits and average distance to a station from
hospitals, cinemas conveniences. any point is only about 500 meters.
and above all family 3) Atlanta, USA: parks, subway
and friends Explanation stations, museums, world‟s largest
(l.17-18) The amenities are located in aquarium, movie theatres, world‟s
close proximity, and this is largest drive-in, universities all
further enhanced by the located downtown.
efficient transportation
system which provides fast
and convenient access.
Health facilities and loved
ones are also nearby.

7
2) …like the buzz of a Explanation 1) Rome, Italy: one can be
city, the concentration City life is often vibrant and exposed to the cultural heritage
of restaurants, clubs there is a diversity of leisure through ancient monuments,
and other forms of activities which one can medieval and Renaissance
entertainment engage in to relax and buildings and fountains, museums.
(l. 19-20) unwind. Some cities are It addition, it is also renowned for
…pleasures of the also rich in cultural heritage. its many fine restaurants and
city… entertainment, cafes, a good nightlife, and lively
they say, can replace streets and squares. It is also an
manufacturing in the international center of fashion and
post-industrial city, the film industry.
providing both jobs 2) Turin, Italy: major cultural hub
and fun with excellent museums, elegant
(l. 27-29) shops, and good restaurants. One
can view baroque architecture,
historic palaces, cafes, artisan
workshops, and arcades.
3) Birmingham, UK: internationally
recognised as a leader in leisure,
entertainment, business, shopping,
and sport. Presence of modern
shopping arcades, diverse pubs,
clubs and restaurants and fine
museums, theatres and art
galleries. £500 million Bullring mall
in city centre.

3) …likely to find work Explanation 1) Austin, Texas, USA: job growth


in the universities, The city offers various job between 2004 and 2008 was
hospitals and opportunities, both for the 14.8%. Strong growth in service
research centres that highly educated and also for sectors like education and health,
tend to cluster in cities people seeking progress leisure and hospitality.
(l. 20-21) and who wish to experience
…for anyone on the upward social mobility.
way up, the city is the
place to be
(l. 40)
4) …private-sector Explanation 1) Surat, Bombay, India: in 1994,
investment is being Due to more sources, and a 40% of its 2.2 million people lived
combined with larger pool of funding, the in slums, without drainage and
government money for living conditions in cities are sanitation. There was outbreak of
urban purposes much now vastly improved plague following a flood, migrant
more widely and through the efforts of workers fled, thousands of
effectively. Cities... concerned policy makers. businesses closed. Only 40% of
now seen as central to the daily garbage was cleared.
environmental Now, 97% of the daily garbage is
improvements(l.33- cleared, morbidity fell by 65%
36) within two-and-a-half years.
Pledge to extend sanitation to

8
every corner of the city with piped
water, a result of two schemes
costing $90 million.
2) Naga, Philippine: leaped from
third-class city in 1988, in terms of
income and services, to first-class
status in 1990. Of 5500 squatter
families, 4668 now own land.
Number of concrete roads and
households with running water
doubled between 1988 and 1996,
and garbage collection efficiency
rose from 29% to 85%.
Explanation and
Passage A Examples
Evaluation
1) … cities are Evaluation 1) Linfen, China: soot-blackened
monstrous parasites, I must concede though, that city, air is filled with burning coal.
consuming the Reader does have some Automobile and industrial
resources of regions valid concerns about city emissions main source of coal and
vastly larger than living. particulates pollutants.
themselves and giving 2) La Oroya, Peru: 99% of children
very little back have blood levels that exceed
(l. 17-18) Explanation acceptable limits, their average
… the ecological Due to residential density, lead level was triple the WHO limit.
footprints of many transportation systems and 3) Los Angeles, California, USA:
cities have been electricity consumption, consumes 235438 MkWh (millions
assessed and the cities utilize a large amount of kilowatt hours).
results are uniformly of resources and contribute 4) Lexington, Kentucky, USA:
alarming greatly towards pollution, 3.455 per capita carbon emission
(l. 23-24) thus leaving a large carbon from transportation and residential
footprint behind. energy use (largest carbon
footprint).
2) … dangers of Explanation 1) Philadelphia, USA: yellow fever
disease multiply when City life is potentially epidemic killed as many as 5000
people are crowded hazardous to health as the people, roughly 10% of the
together close proximity of humans population, in 1793.
(l. 33)… breeding facilitates the transmission 2) Singapore: 144 of Singapore‟s
grounds of disease and rapid spread of 206 probable cases of SARS have
(l.40) diseases, some of which been linked to contact with only 5
…bacterial and viral could be fatal. individuals.
diseases are the price 3) Hong Kong: 2.7 secondary
humanity has paid to infections of SARS were generated
live in large and on average per case at the start of
densely populated the epidemic. Transmission rates
cities fell after efforts made to reduce
(l. 41-42)… urban population contact rates, among
graveyard effect (l.49) other measures.

9
Counter arguments
Explanation and
to ideas in passage Examples
Evaluation
A
1) … cities are Evaluation 1) Calgary, Canada: city with the
monstrous parasites, However, his concerns and best eco-ranking based on water
consuming the pessimism can be easily availability and drinkability, waste
resources of regions allayed and dismissed, removal, quality of sewage
vastly larger than especially if we take into systems, air pollution and traffic
themselves and giving consideration developments congestion.
very little back in today‟s world. Hence, I 2) Oakland, California, USA: 17%
(l. 17-19) firmly believe that city life of power produced from renewable
… the ecological should be viewed with energy. Renewable energy
footprints of many optimism instead of sources produce electricity with no
cities have been pessimism. global climate-changing
assessed and the greenhouse gas emissions or
results are uniformly Explanation regional air pollution that comes
alarming There is a greater from burning fossil fuels.
(l. 23-24) awareness of the carbon 3) Honolulu, USA: 1.356 per capita
footprint which cities are carbon emission from
leaving behind and hence, transportation and residential
there is increased and energy use (smallest carbon
sustained effort by footprint).
community and global 4) Melbourne, Australia: various
leaders in tackling this methods of improving public
issue. There is now the transport implemented, including
push to go-green and car free zones and entire streets.
certain cities are testament Programs for becoming carbon
to this success. This is neutral such as „Zero Carbon
manifested in sustainable Moreland‟.
cities, or eco-cities. These 5) Tianjin, China: China‟s
are cities designed with collaboration with Singapore, Sino-
consideration of Singapore Tianjin Eco-city.
environmental impact, Existing wetlands and biodiversity
inhabited by people will be preserved. Green spaces
dedicated to minimization of will be interspersed throughout
required inputs of energy, city. Significant part of water
water and food, and waste supply drawn from non-traditional
output of heat, air pollution. sources such as desalinated
A sustainable city can feed water. Integrated waste
itself with minimal reliance management, with particular
on the surrounding emphasis on reduction, reuse and
countryside, and power recycling of waste. Light-rail transit
itself with renewable system, supplemented by
sources of energy. The crux secondary network of trams and
of this is to create the buses, will be main mode of
smallest possible ecological transportation.
footprint, and to produce the 6) Freiburg, Germany: known for
lowest quantity of pollution its strong solar economy. All
possible, to efficiently use houses built to low energy

10
land; compost use \d consumption standard and whole
materials, recycle it or district designed to be car free.
convert waste-to-energy, 7) Clonburris, Dublin, Ireland: high
and thus the city‟s overall levels of energy efficiency,
contribution to climate mandatory renewable energy for
change will be minimal. heating and electricity, use of
recycled and sustainable building
materials, direct heating system for
distributing heat, provision of
allotments for growing food,
banning of tumble driers, provision
of natural drying areas.
2) … dangers of Explanation 1) With outbreak of swine flu in
disease multiply when Advanced medical science Mexico City, WHO raised its alert
people are crowded able to cure diseases. More level to four on its six-level scale.
together importantly though, is the Countries responded by taking
(l. 33) ability to prevent the measures such as issuing travel
…bacterial and viral outbreak of diseases health notice, banning imports,
diseases are the price through close monitoring setting up of screening centers,
humanity has paid to and sharing of information minimizing public contact, contact
live in large and on potential tracing.
densely populated outbreaks/pandemics
cities through governments and
(l. 41-42) global organizations such
as WHO.

11
TAMPINES JUNIOR COLLEGE
Preliminary Examination 2010

GENERAL PAPER 8806/01


Paper 1 30 August 2010

1 hour 30 minutes

Additional Materials: Writing Paper

READ THESE INSTRUCTIONS FIRST

Write your name, civics class and GP Tutor’s name on all the work you hand in.
Write in dark blue or black pen on both sides of the paper.
Do not use staples, paper clips, highlighters, glue or correction fluid.

Answer one question.


Note that 20 marks out of 50 will be awarded for your use of language.

At the end of the examination, fasten all your work securely.


All questions in this paper carry equal marks.

This document consists of 2 printed pages

© TPJC GP Unit 2010 [Turn over


2

Answer ONE question from this Paper

Answers should be between 500 and 800 words in length.

1 Examine the claim that the world is becoming too crowded.

2 Should Internet anonymity be a right or privilege?

3 To what extent are democratic ideals relevant to Asian countries?

4 Regrettably, the sole purpose of education today is to get our young people to
make a living. Discuss.

5 Young people today demand their rights but do not fulfill their responsibilities. Is
this true of Singapore youths?

6 ‘Science and ethics are like oil and water.’ Discuss.

7 ‘Win at all costs.’ To what extent has this attitude adversely affected society?

8 To what extent is the environment the responsibility of the government, not the
corporation?

9 ‘Not all who wander are lost.’ (J. R. R. Tolkien). Comment.

10 Can Singapore society be considered as liberal?

11 There is no better time than now to preserve a nation’s culture and heritage.
Do you agree?

12 The written word is more powerful than the spoken. What is your view?

TAMPINES JUNIOR COLLEGE H1 GENERAL PAPER 2010 PRELIMINARY EXAMINATION


TAMPINES JUNIOR COLLEGE
Preliminary Examination 2010

GENERAL PAPER 8806/02


Paper 2 30 August 2010
1 hour 30 minutes
INSERT

READ THESE INSTRUCTIONS FIRST

This insert contains the passages for Paper 2.

This insert consists of 4 printed pages including this page.


CONFORMITY

Passage 1: Marcus Emery thinks that conformity is bad.

1 The individual who sacrifices his individual way of life to conform to his neighbours’
lifestyle is lessening the quality of his life. ‘We are all in this together’ is the mantra of
the masses. Pop culture must reign in every man's heart and mind. It is the idea that
together we can be great, but separated... Our individual voices drowned by the
massive roar that is the world around us. We are all part of the well-oiled machine that 5
is the human race. Conformity then, crudely, is the willingness to put one's own
personal beliefs, values and morals aside in exchange for someone else's in order to
better fit in.

2 Realistically, the non-conformist doesn't really have an alternative, does he? As our
societies are organized fairly tightly – from wearing clothes that society does not agree 10
with, to choosing a lifestyle which is deemed deviant – it is sometimes too easy to
appear to be outside the normal behaviour of the community. And not to mention how
society reinforces the code through the use of sanctions. A microcosmic example of
sanctions may be found at the local child care centre. Little Suzy has performed each
activity of the day obediently and with a positive attitude. She earns a shiny gold star 15
next to her name on the board. Conversely, Little Christy threw a tantrum during snack
time and was sent to time-out.

3 Yet, advocates of the present consensus do not necessarily enamour the eyes and
mind of the non-conformist. After all, success, he would contend, is the result of
bending if not breaking the rules. Yes, he knows that many of the highly successful 20
cyberspace megabuckers are not college graduates. Yes, he knows that the public
education system is more driven by creative thought than common knowledge. Yes, he
remembers that those who succeed in school in his day were those who could and did
think for themselves. And yes, they were even given a grade on "Deportment". (That
means manners and obedience). 25

4 Indeed, study the life of successful people and you will see they always buck the odds.
Medical science said it was physically impossible for a person to run a mile in under
four minutes. People conformed to the word of the experts until Roger Bannister broke
this myth. Fundamentally, non-conformity coexists with risk and the greatest risk is not
taking a risk. How can we understand our potential without taking risks? During our 30
educational experiences, the average student has spent 35,000 hours listening to
someone else. Conforming is listening to the loud noises of the world when the
whispers of our hearts tell us the opposite!

5 And those are just some of the personal benefits. In 1859, the anti-slavery campaigner
and women's rights advocate John Stuart Mill wrote in On Liberty, ‘That so few now 35
dare to be eccentric, marks the chief danger of the time.’ A man who hated any kind of
tyranny, Mill particularly despised the sneering, curtain-twitching, self-elected arbiters of
social conformity. For him, they were tyrants responsible for enslaving the soul.
Instead, Mill saw nonconformity (so long as it harmed no one) as not only a matter of
personal freedom, but a boon to society. 40

6 But does being a non-conformist mean that one lives by no standards at all? Not really;

2
one can conform to the norms to the best of his ability. But to conform to the nonsense
of the moment, at a social and monetary cost that is both damaging and unnecessary,
is irresponsible. Non-conformists can act responsibly. Radical, no? Conformity is a
disadvantage but we opt to choose it because it simply is easier. We avoid the 45
displeasure of others but we don't exercise the courage to move forward.

Adapted from ‘Not I’ by Marcus Emery

Passage 2: Colin Barth takes a different view of conformity.

1 In the pursuit of individualism and individuality, it is sometimes easy to demonize


conformity. Even if this is avoided, it is easier to dismiss conformity as entirely
disagreeable, so that one is always in the position of wishing to argue for the unusual,
for the exception, simply because it is different. In most cases, this is a position
warranted in order to correct the heavy, combined weight of mediocrity that people 5
have become accustomed to endorse in the traditions of normalcy, convention,
expectation, and similarity.

2 But most criticism of conformity entirely misses what should be the real point. Most
criticism of conformity consists of little more than a contempt for what is conventional
merely because it is not different, without judging its possible merits (which might 10
explain how it came to be followed), or of criticizing those who follow the example of
others, on the assumption that they are feeble-minded as sheep. Reflex anti-
conformity is still too simple, too predictable. Always ‘rebelling against conformity’
becomes a habit, an inevitable conformity in itself.

3 If we consider conformity as simply following the path established by others without 15


deliberation, conformity would include every instance of following an established
convention which has not been consciously researched, examined, analysed,
evaluated ... Surely, if we stop to consider every convention we follow with every
action and every thought, we will get nothing done. Nor do we actually have the
choice. Our mental faculties are incapable of pausing to reflect on everything, 20
including the very elements of language, symbols and conceptual models we use to
formulate thoughts. So, at the extreme generality of what we might mean by
conformity, clearly it is innate, and it is necessary. Therefore, it is, first of all, important
to accept that if conformity were always such a hindrance to the fullest expression of
life, it would be puzzling to explain just why it is so ingrained in human behaviour. 25

4 If we are really honest in examining the value of conformity according to the standard
of what is life-advancing, we must admit that conforming can happen to be quite
advantageous for those who follow it, and not only for the rewards that have been built
up around it. Positive conformity might take the form of following already established
and quite popular life-advancing customs and ideological precedents in order to build 30
upon them. A blunt example is the nearly worldwide, if diversely expressed, bias
against murder (at least, outside of the allowed exceptions in war, honour-killings, and
so forth). Or, positive conformity might take the form of following less mainstream,
more unpopular life-advancing traditions simply because of a greater exposure to
them. Ultimately, like the flocking behaviour of birds or the herding behaviour of 35
grazing animals, the human urge to conform to what is perceived as established
cultural practice is an evolved protective advantage of sorts. Conforming to a norm

3
can protect against change which might be more detrimental than advantageous to
the group of individuals, quite possibly including the individual who would introduce it.

5 But, to the critics of conformity, the possibility that unquestioned tradition first became 40
established thanks to being truly advantageous is sadly still not enough of an
explanation for the repeated human preference for what is the same at the expense of
what is different – even when what is different might be entirely unhelpful and
disagreeable for all concerned, and even when the different is different to the extent of
ostracizing those who create such positive traditions, as though they are cancerous 45
cells in the bloodstream of society.

Adapted from ‘On Conformity’ by Colin Barth

4
TAMPINES JUNIOR COLLEGE
Preliminary Examination 2010

GENERAL PAPER 8806/02


Paper 2 30 August 2010
1 hour 30 minutes
ANSWER BOOKLET
Additional Materials: 1 Insert

READ THESE INSTRUCTIONS FIRST

Write your name, Civics Group and GP Tutor’s name in the spaces provided below.
Write in dark blue or black pen on both sides of the paper.
Do not use staples, paper clips, highlighters, glue or correction fluid.

Answer all questions.


The Insert contains the passage for comprehension
Note that 15 marks out of 50 will be awarded for your use of language.

At the end of the examination, fasten all your work securely together.
The number of marks is given in brackets [ ] at the end of each question or part question.

NAME: _________________________________________ CIVICS GROUP: __________

GP TUTOR’S NAME: ______________________________

FOR EXAMINERS’ USE:


Qn 1 Qn 7 MARKER 1

Qn 2 Qn 8 MARKER 2

Qn 3 Qn 9 MARKER 3

Qn 4 Qn 10 MARKER 4

Qn 5 Qn 11 MARKER 5

Qn 6 Qn 12 MARKER 6
SUB- AVE LANG
TOTAL ……………/35 MARK ……………/15

TOTAL: .................... /50

This document consists of 8 printed pages including the cover page.


Read the passages in the Insert and then answer all the questions. Note that up to fifteen marks
will be given for the quality and accuracy of your use of English throughout this Paper.
NOTE: When a question asks for an answer IN YOUR OWN WORDS AS FAR AS POSSIBLE
and you select the appropriate material from the passage for your answer, you must still use
your own words to express it. Little credit can be given for answers which only copy words or
phrases from the passage.

Questions on Passage 1

1. Explain why the author uses the word ‘crudely’ in line 6.

_________________________________________________________________________

_________________________________________________________________________

______________________________________________________________________ [1]

2. From your reading of lines 13 – 17, explain how the examples are used to develop the
author’s argument in paragraph 2. Answer in your own words as far as possible.

_________________________________________________________________________

_________________________________________________________________________

_________________________________________________________________________

______________________________________________________________________ [2]

 
 
3. In paragraph 3, the author argues that ‘success is the result of bending if not breaking the
rules.’

(a) Why does the author repeat the word ‘yes’ in lines 20, 21 and 22?

_________________________________________________________________________

_________________________________________________________________________

______________________________________________________________________ [1]

2
(b) Why does the author begin the sentence in line 24 with the phrase ‘And yes’?

_________________________________________________________________________

_________________________________________________________________________

______________________________________________________________________ [1]

4. In paragraph 4, the author argues that ‘fundamentally...the greatest risk is not taking a risk.’
(lines 29- 30)

Explain in your own words as far as possible the irony in this phrase.

_________________________________________________________________________

_________________________________________________________________________

______________________________________________________________________ [2]

5. In paragraph 5, the author cites an argument made by John Stuart Mill.

Explain the importance of the phrase ‘so long as it harmed no one’ to Mill’s argument in
paragraph 5.

_________________________________________________________________________

______________________________________________________________________ [1]

6. How does the material from paragraph 1 justify the question ‘Radical, no?’ in line 44?

_________________________________________________________________________

_________________________________________________________________________

_________________________________________________________________________

_________________________________________________________________________

_______________________________________________________________________ [2]
3
Questions on Passage 2

7. From your reading of paragraph 1, explain in your own words as far as possible the key
reason why people reject conformity.

_________________________________________________________________________

_________________________________________________________________________

_________________________________________________________________________

_______________________________________________________________________ [2]

8. From your reading of lines 15 – 18, explain why the author ends the sentence with three
dots (...)?

_________________________________________________________________________

_________________________________________________________________________

_______________________________________________________________________ [1]

9. In paragraph 4, the author raises a ‘blunt example’ to support his argument. (line 31)

From your reading of lines 31 – 33, how does the author illustrate the concept of ‘blunt’?

_________________________________________________________________________

_________________________________________________________________________

_______________________________________________________________________ [1]

4
10. Using material from paragraphs 2 to 4, summarise what the author has to say about the
criticism of conformity, the reasons why conformity is necessary, and the advantages of
conformity. Write your summary in no more than 150 words, not counting the opening
words which are printed below. Use your own words as far as possible.

Most criticism of conformity


_________________________________________________________________________

_________________________________________________________________________

_________________________________________________________________________

_________________________________________________________________________

_________________________________________________________________________

_________________________________________________________________________

_________________________________________________________________________

_________________________________________________________________________

_________________________________________________________________________

_________________________________________________________________________

_________________________________________________________________________

_________________________________________________________________________

_________________________________________________________________________

_________________________________________________________________________

_________________________________________________________________________

_________________________________________________________________________

_________________________________________________________________________

_________________________________________________________________________

_______________________________________________________________________ [8]

5
From both passages

11. Give the meaning of the following words as they are used in the passages. Use one word or
a short phrase.

a) tightly (Passage 1, line 10) ________________________________________________

b) exercise (Passage 1, line 46)_______________________________________________

c) misses (Passage 2, line 8) _________________________________________________

d) deliberation (Passage 2, line 16) ____________________________________________

e) possibility (Passage 2, line 40) _____________________________________________

[5]

6
12. Marcus Emery has grave reservations about conformity, whereas Colin Barth defends the
need for conformity.

With which author are you most in sympathy with? How far is conformity a matter of
concern to you and your generation?

_________________________________________________________________________

_________________________________________________________________________

_________________________________________________________________________

_________________________________________________________________________

_________________________________________________________________________

_________________________________________________________________________

_________________________________________________________________________

_________________________________________________________________________

_________________________________________________________________________

_________________________________________________________________________

_________________________________________________________________________

_________________________________________________________________________

_________________________________________________________________________

_________________________________________________________________________

_________________________________________________________________________

_________________________________________________________________________

_________________________________________________________________________

_________________________________________________________________________

7
_________________________________________________________________________

_________________________________________________________________________

_________________________________________________________________________

_________________________________________________________________________

_________________________________________________________________________

_________________________________________________________________________

_________________________________________________________________________

_________________________________________________________________________

_________________________________________________________________________

_________________________________________________________________________

_________________________________________________________________________

_________________________________________________________________________

_________________________________________________________________________

_________________________________________________________________________

_________________________________________________________________________

_________________________________________________________________________

_________________________________________________________________________

_________________________________________________________________________

_________________________________________________________________________

_________________________________________________________________________

_________________________________________________________________________

_______________________________________________________________________ [8]

8
9
Tampines Junior College English Department

General Paper Preliminary Examination 2010: Paper 2 Answer Key

Questions on Passage 1

1. Explain why the author uses the word ‘crudely’ in line 6. [1]

 In paragraph 1, the author uses the word ‘crudely’ to describe the


definition of conformity.
Requirements

 Candidates must explain the author’s intention of using the word


‘crudely’ by showing how it substantiates the writer’s argument in
paragraph 1.
 Therefore, necessary to relate the use of the word back to the
paragraph’s claim and writer’s point of view.
guidelines

As far as possible, candidates must explain both the writer’s intention, and
Marking

how the writer achieves his intention, in order to secure the full mark.

‘The individual who sacrifices his individual way of life to conform to his
From the
passage

neighbours’ lifestyle is lessening the quality of his life. [...] Conformity then,
crudely, is the willingness to put one's own personal beliefs, values and
morals aside in exchange for someone else's in order to better fit in.
Suggeste

The author uses the word ‘crudely’


d Answer

 to emphasise how conformity compromises the rights of the


individual. [1]

2. From your reading of lines 12 – 15, explain how the examples are used to develop
the author’s argument in paragraph 2. Answer in your own words as far as possible.
[2]

 The examples of ‘Little Suzy and Little Christy’ are used to develop the
writer’s claim that ‘the non-conformist doesn’t really have an
Requirements

alternative [to conformity’, especially since ‘society reinforces the


code through the use of sanctions’.
 Candidates must show how the examples provided illustrate the writer’s
argument in paragraph 2.
 Therefore, necessary to relate the use of the examples back to the
paragraph’s claim and writer’s point of view.
guidelines

As far as possible, candidates must explain how the examples illustrate the
Marking

writer’s argument in paragraph 2.

  1
Tampines Junior College English Department

‘Realistically, the non-conformist doesn't really have an alternative,


does he? As our societies are organized fairly tightly – from wearing
From the passage clothes that society does not agree with, to choosing a lifestyle which is
deemed deviant – it is sometimes too easy to appear to be outside the
normal behaviour of the community. And not to mention how society
reinforces the code through the use of sanctions. A microcosmic
example of sanctions may be found at the local child care centre. Little
Suzy has performed each activity of the day obediently and with a
positive attitude. She earns a shiny gold star next to her name on the
board. Conversely, Little Christy threw a tantrum during snack time
and was sent to time-out.’

The examples of Little Suzy and Little Christy


Suggested


Answer

demonstrate how society operates on a reward and punishment


system to correct unacceptable behaviour. [1]
 Hence, the non-conformist has no choice since he will be
punished into submission.[1]

3. In paragraph 3, the author argues that ‘success […] is the result of bending if not
breaking the rules.’

(a) Why does the author repeat the word ‘yes’ in lines 20, 21 and 22? [1]

 In paragraph 3, the author argues that according to the non-conformist,


Requirements

‘success [...] is the result of bending if not breaking the rules.’


 Give reasons to explain the how the repeated use of the word ‘yes’
substantiates the writer’s argument.
 Therefore, necessary to relate the use of the word back to the
paragraph’s claim and writer’s point of view.

 As far as possible, candidates must explain both the writer’s intention,


guidelines:
Marking

and how the writer achieves his intention, in order to secure the full
mark.
 Accept answers which lift ‘rules’

‘Yet, advocates of the present consensus do not necessarily enamour the


From the passage

eyes and mind of the non-conformist. After all, success, he would


contend, is the result of bending if not breaking the rules. Yes, he
knows that many of the highly successful cyberspace megabuckers are not
college graduates. Yes, he knows that the public education system is
more driven by creative thought than common knowledge. Yes, he
remembers that those who succeed in school in his day were those who
could and did think for themselves...’
Suggeste
d Answer

The writer intends


 to emphasise his mounting pride in those who had succeeded in
school and in life in spite of breaking the rules. [1]

  2
Tampines Junior College English Department

(b) Why does the author begin the sentence in line 24 with the phrase ‘And yes’? [1]

 In paragraph 3, the author argues that according to the non-conformist,


‘success [...] is the result of bending if not breaking the rules.’
 After explaining how non-conformists do succeed in school and in life,
Requirements

the author goes on to stress how non-conformists actually were also


schooled in deportment.
 Candidates must give reasons to explain the how the phrase ‘And yes’
substantiates the writer’s argument.
 Therefore, necessary to relate the use of the word back to the
paragraph’s claim and writer’s point of view.
guidelines

As far as possible, candidates must explain both the writer’s intention, and
Marking

how the writer achieves his intention, in order to secure the full mark.

‘Yet, advocates of the present consensus do not necessarily enamour the


eyes and mind of the non-conformist. After all, success, he would contend,
From the passage

is the result of bending if not breaking the rules. Yes, he knows that many of
the highly successful cyberspace megabuckers are not college graduates.
Yes, he knows that the public education system is more driven by creative
thought than common knowledge. Yes, he remembers that those who
succeed in school in his day were those who could and did think for
themselves. And yes, they were even given a grade on "Deportment".
(That means manners and obedience).’

The writer intends


Suggested


Answer

to criticise those who see non-conformists as unreasonable rule-


breakers [1/2]
 by implying sarcastically how non-conformists can abide by basic
rules of social decorum. [1/2]

4. In paragraph 4, the author argues that ‘fundamentally...the greatest risk is not taking
a risk.’ (lines 29- 30)

Explain in your own words as far as possible the irony in the italicised phrase. [2]

 In paragraph 4, the author argues that the most successful people


Requirements

always ‘buck the odds’.


 Candidates must explain the irony in the phrase by explaining how
expectations are met with the opposite in reality.
guidelines

As far as possible, candidates must explain how what happens in reality


Marking

is the opposite of expectations in order to secure the full marks.

  3
Tampines Junior College English Department

‘Indeed, study the life of successful people and you will see they always
buck the odds. Medical science said it was physically impossible for a
From the
passage
person to run a mile in under four minutes. People conformed to the word of
the experts until Roger Bannister broke this myth. Fundamentally, non-
conformity coexists with risk and the greatest risk is not taking a risk.
How can we understand our potential without taking risks? ...’

 When one takes a risk, he logically faces some element of danger


Suggested
Answer

or uncertainty. [1]
 Yet, the author claims that to not take a risk in fact incurs the most
amount of danger or uncertainty.[1]

5. In paragraph 5, the author cites an argument made by John Stuart Mill.

Explain the importance of the phrase ‘so long as it harmed no one’ to Mill’s argument
in paragraph 5. [1]

 In paragraph 5, the author cites Mill’s argument that ‘so few now dare to
Requirements

be eccentric, marks the chief danger of the time.’


 Candidates must explain how the phrase ‘so long as it harmed no one’ is
important to Mill’s argument.

 As far as possible, candidates must explain how the phrase is a


guidelines:

condition which governs the argument in order to secure the full


Marking

mark.
 Accept answers that lift ‘harm’ as long as they answer the conditional
clause.

‘And those are just some of the personal benefits. In 1859, the anti-slavery
From the passage

campaigner and women's rights advocate John Stuart Mill wrote in On


Liberty, ‘That so few now dare to be eccentric, marks the chief danger of the
time.’ A man who hated any kind of tyranny, Mill particularly despised the
sneering, curtain-twitching, self-elected arbiters of social conformity.
For him, they were tyrants responsible for enslaving the soul. Instead,
Mill saw nonconformity (so long as it harmed no one) as not only a matter
of personal freedom, but a boon to society.’
Suggeste
d Answer

Mill’s argument relies on the condition that non-conformity does not


harm anyone in order for his argument to be valid. [1]

  4
Tampines Junior College English Department

6. How does the material from paragraph 1 justify the question ‘Radical, no?’ in line 44?

[2]

 In paragraph 6, the author argues how non-conformists do abide by


Requirements

basic rules.
 The question ‘Radical, no?’ then is used sarcastically.
 Candidates must explain how material in paragraph 1 justifies the
sarcasm in the question.

 Accept answers which quote parts from the paragraphs succinctly.


guidelines:
Marking

 Accept answers that lift common words as long as they explain the
connection between the paragraphs.

‘But does being a non-conformist mean that one lives by no standards at


From the
passage

all? Not really; one can conform to the norms to the best of his ability. But to
conform to the nonsense of the moment, at a social and monetary cost that
is both damaging and unnecessary, is irresponsible. Non-conformists can
act responsibly. Radical, no?...’

 Since conformists accuse non-conformists of threatening social


stability owing to their refusal to put their ‘beliefs, values, and
morals aside in exchange for someone else’s’, it is logical that they
will find it difficult to accept that ‘non-conformists can act
responsibly’. [1]
 Hence the author poses the question ‘Radical, no?’ to imply
sarcastically his disapproval of the myopic viewpoint of
Suggested Answer

conformists. [1]
OR
 Conformity is necessary for the smooth running of society.
However, conformity also results in an individual having to
compromise on his ideals and principles. [1]
 The non-conformist on the other hand, also tries to adhere to
society’s expectations but is more discriminating, adhering to
these expectations only when it is reasonable and of value to
society. Hence, the non-conformist is actually more responsible.
This idea of the non-conformist being more responsible than the
conformist overturns our usual perceptions and is therefore
radical. [1]

  5
Tampines Junior College English Department

Questions on Passage 2

7. From your reading of paragraph 1, explain in your own words as far as possible
the key reason why people reject conformity. [2]


Requirem

In paragraph 1, the author argues how ‘it is sometimes easy to


ents

demonise conformity’.
 Candidates must summarise the key idea in paragraph 1.
guidelines

 Do not accept answers which lift ‘unusual’, ‘exception’, ‘different’,


Marking

‘position warranted’, ‘mediocrity’, ‘normalcy’, ‘convention’ and ‘novel’.

‘In the pursuit of individualism and individuality, it is sometimes easy to


From the passage

demonize conformity. Even if this is avoided, it is easier to dismiss


conformity as entirely disagreeable, so that one is always in the position of
wishing to argue for the unusual, for the exception, simply because it is
different. In most cases, this is a position warranted in order to correct
the heavy, combined weight of mediocrity that people have become
accustomed to endorse in the traditions of normalcy, convention,
expectation, and similarity.’

 To conform is often perceived to be synonymous to being ordinary.


Suggested

[1]
Answer

 It is therefore more appealing for people to reject a familiar idea for


one that is new. [1]

8. From your reading of lines 15 – 18, explain why the author ends the sentence with
three dots (...)? [1]

 In paragraph 3, the author implies how impossible it would be for us to


Requirements

consider every single step of the decision-making process.


 Candidates must explain how the three dots (...) substantiates the
author’s point.
 Therefore, necessary to relate the use of the word back to the
paragraph’s claim and writer’s point of view
guidelines

 Accept answers that lift common words.


Marking

 Do not accept answers which lift ‘established convention’

‘If we consider conformity as simply following the path established by others


From the

without deliberation, conformity would include every instance of


passage

following an established convention which has not been consciously


researched, examined, analysed, evaluated ...’

  6
Tampines Junior College English Department

The author wishes


 to mock those who define conformity as blindly following the norm
Suggested Answer
[1/2]
 by indicating how entrenched ideas/practices are actually the
products of an infinite number of intellectual processes. [1/2]

The above would be the best answer, but we also accepted the following
answer:
The author outlines
 an infinite/endless number [1/2]
 of intellectual processes [1/2]

9. In paragraph 4, the author raises a ‘blunt example’ to support his argument. (line 31)

From your reading of lines 31 – 33, how does the author illustrate the concept of
‘blunt’? [1]

 In paragraph 4, the author explains the advantages of conformity.


Requirements

 Candidates must explain how the examples raised are ‘blunt’.


guidelines

 Accept answers that lift ‘murder’.


Marking

 Do not accept answers which lift ‘bias’, ‘exceptions’.

‘If we are really honest in examining the value of conformity according to the
standard of what is life-advancing, we must admit that conforming can
From the passage

happen to be quite advantageous for those who follow it, and not only for
the rewards that have been built up around it ... A blunt example is the
nearly worldwide, if diversely expressed, bias against murder (at least,
outside of the allowed exceptions in war, honour-killings, and so
forth)...’

  7
Tampines Junior College English Department

The author illustrates the concept of ‘blunt’

 by choosing a very straightforward/obvious example [1/2]

 where, aside from the extreme cases, objection to murder is


universal. [1/2]
Suggested Answer

OR

The author illustrates the concept of ‘blunt’

 by giving an example that is not very accurate [1/2]

 by indicating how our seemingly united stand against murder is not


necessarily absolute since we do make allowances for some cases
of killing. [1/2] *

10. Using material from paragraphs 2 to 4, summarise what the author has to say about the
criticism of conformity, the reasons why conformity is necessary, and the advantages of
conformity. Write your summary in no more than 150 words, not counting the opening
words which are printed below. Use your own words as far as possible. [8]

From the passage Paraphrase

What the author has to say about the criticism of conformity

1. Most criticism of conformity consists of Most criticism of conformity is driven by


little more than a contempt for what is a disapproval of the commonplace. [½]
conventional merely because it is not
This disapproval is unfounded because it
different, without judging its possible
simply condemns that which is familiar,
merits.
[½] without any consideration of its
potential advantages. [½]
No lifting for:
 Contempt
 Conventional
 Different
 Judging
 Possible
 Merits

2. Always ‘rebelling against conformity’ The movement against conformity has


becomes a habit, an inevitable become unavoidably common.[1]
conformity in itself.
No lifting for:
 Habit

  8
Tampines Junior College English Department

 Inevitable

The reasons why conformity is necessary

3. Surely, if we stop to consider every It is impossible to accomplish anything if


convention we follow with every action we question all our norms.[1]
and every thought, we will get nothing
No lifting for:
done.
 Convention

4. Nor do we actually have the choice. Humans are not intellectually built to
Our mental faculties are incapable of question everything.[1]
pausing to reflect on everything,
No lifting for:
including the very elements of language
symbols and conceptual models we use  Choice
to formulate thoughts.
 Mental faculties
 Incapable
 Pausing
 Reflect

5. So, at the extreme generality of what we Conformity is natural to man.[1]


might mean by conformity, clearly it is
No lifting for:
innate, and it is necessary.
 Innate

The advantages of conformity

6. Positive conformity might take the form of Conformity perpetuates, and therefore
following already established and develops [½], prevalent progressive
quite popular life-advancing customs conduct and ideological standards. [½]
and ideological precedents in order to
No lifting for:
build upon them.
 Established
 Popular
 Life-advancing
 Customs
 Precedents
 To build upon

7. Or, positive conformity might take the Conformity may also help to propagate
form of following less mainstream, alternative or unfashionable conduct [½],
more unpopular life-advancing which may be progressive [½ ] .
traditions simply because of a greater
No lifting for:
exposure to them.
 Following
 Less mainstream

  9
Tampines Junior College English Department

 Unpopular
 Life-advancing traditions
 Greater exposure

8. Conformity is an evolved protective Conformity is a defence mechanism [½] ,


advantage of sorts. developed over time. [½]
No lifting for:
 Protective

9. Conforming to a norm can protect Conformity safeguards us [½] from


against change which might be more change which may be more harmful than
detrimental than advantageous to the beneficial [½] .
group of individuals.
No lifting for:
 Detrimental
 Advantageous

11. Give the meaning of the following words as they are used in the passages. Use one
word or a short phrase. [5]

Question 1mark

a) tightly (Passage 1, line 10) Rigidly (adverb)

b) exercise (Passage 1, line 46) Put into effect (verb)

c) misses (Passage 2, line 8) Overlooks (verb)

d) deliberation (Passage 2, line 16) Consideration (noun)

e) possibility (Passage 2, line 40) Likelihood (noun)

  10
Tampines Junior College English Department

12 Marcus Emery has grave reservations about conformity, whereas Colin Barth defends
the need for conformity.

With which author are you most in sympathy with? How far is conformity a matter of
concern to you and your generation?

Possible Responses

Marcus Emery has grave reservations about conformity...


Author’s Evaluation: Application:
argument With which author are you most in How far is conformity a matter of
sympathy with? concern to you and your
generation?
Definition of  Emery implies that to conform is to  In some cases, Generation
conformity is give up your rights i.e a crude Y is obsessed with a range
‘crude’. definition which is negative. of trivial causes, but with
very little consideration for
 Yet, he assumes that all the rights
the real impact these
of minorities should be preserved
causes have for the world.
on principle.
 Following trends without
 It is critical to counter argue that
really questioning why 
while it is important for Man to
the penchant to be
respect that the rights of minorities
different/unique/cool 
are important, some of these rights
reality TV, popularity of
are actually pursued or maintained at
talent contests which stress
the expense of other individuals
the need to be ‘unique’
within the minority community.
such as American Idol,
 What is important here is to select OMG, America’s Got
the appropriate rights to campaign Talent... what really is the
for, which will benefit as many point?
people as possible  a utilitarian
approach to conformity.

Society is  Emery implies that this is negative in  In some cases, Generation


organised nature. Y rejects the rules of
tightly and society which dictate social
works on a  Yet he assumes that this is true in
decorum / formality
system of all contexts.
especially those rules
sanctions  It is critical to counter argue that a pertaining to vices (e.g.
tightly organised society is not legalisation of marijuana,
necessarily a poorly organised one indulgence in pornography
i.e. it is not necessarily a negative and the sex trade) at the
phenomenon. expense of tradition and
actually what is right.
 There remains the universal
understanding that any society  In some cases, Generation
requires norms to be clearly Y conform to rules which
established for the safety and they set for themselves, or
security of all. rules instituted by
organisations /
 What is important here is the corporations, rules which
degree of acceptable bureaucracy really then have been
in individual societies, and how created at their expense.
appropriately different regimes The fashion industry and
mete out their systems of
  11
Tampines Junior College English Department

punishment and reward. After all, the media are notable


we need rules for a society to culprits of this.
function.

Success is the  Emery implies that success is the  In some cases, Generation
result of result of challenging convention. Y is a generation obsessed
bending if not with breaking the rules
breaking the  He assumes that all types of
rules success are gained from bending  But they break the rules
if not breaking the rules. He also without really achieving
implies that success is guaranteed anything new or anything
so long as one is NOT CAUGHT noteworthy. Therefore is
(and therefore punished for) such really success? Do the
bending or breaking the rules. means really justify the
ends then?
 It is critical to counter argue that not
all types of successes may or should  In addition, some rules
be reaped in this manner. (especially the law) should
not be tampered with.
 In addition, what are the
repercussions if one is caught and
punished for not towing the line? Can
this opportunity cost be borne in all
contexts?
 What is important here is that not
all rules should be bent or broken.

The greatest  Emery implies that the means justify  What about youths in
risk is not the ends. developing countries? Can
taking a risk they afford to fail if they
 He assumes that success is
really want to break out of
guaranteed or that the benefits
poverty? Consider the case
from challenging oneself will more
of young Serbian tennis
than outweigh the problems of
stars Djokovic and Ivanovic
defeat.
 It is critical to counter argue that the
opportunity cost of failure/defeat is
not one which everyone can bear.
 What is important here is that not
everyone should take risks; not
every project/plan should take the
road less travelled simply because
in some contexts, the ends are
more important than the means.
And conformity in the form of
established conventions does
guarantee success.

Colin Barth defends the need for conformity

The  Graves implies that this is an unfair  Generation Y certainly has


commonplace state of affairs. license to provoke change
has become over the violations of
associated  He assumes that the
human rights in the world,
with commonplace has value in all
especially women’s and
contexts.
children’s rights in the
  12
Tampines Junior College English Department

‘mediocrity’  It is critical to counter argue that in developing world.


some contexts, what is perceived as
the norm is actually primitive,
damaging and ultimately regressive
in nature. In such cases, to conform
to these norms is an act of complicity.
To not conform then would be an act
of just rebellion.
 What is important here is that the
individual must conscientiously
decide what he/she chooses to
reject or conform.

To not  Graves implies that this irony


conform has collapses his opposition.
ironically
become a  He assumes that this is the case
conformist universally, and that essentially,
tendency this is a negative trend.
 It is critical to counter argue that
everyone should strive to be unique,
and that even though that makes us
the same, the end result is technically
most important i.e we can all be
different, but we are different in
different ways. Such is particularly
important in today’s diverse
globalised / media-centric world.
 What is important here is that we
should acknowledge that this is an
emerging global trend which
cannot be halted or controlled.
Hence, rather than resisting this
trend, one should embrace firstly
the multitude of opportunities /
possibilities diversity presents to
the world, and secondly the spirit
of experimentation which typifies
our globalised world today.

Conformity  Graves implies that this is an


propagates advantage of conformity.
popular and
unpopular  He assumes that these long term
traditions benefits are worth the short term
which will lead repercussions.
to humanity’s  It is critical to counter argue that the
progress in short term repercussions will be too
the future. great for some to bear.
 What is important here is to note
that the ends do not always justify
the short term costs.

Conformity is  Graves implies that this is an


  13
Tampines Junior College English Department

an evolved advantage of conformity.


protective
 He assumes that everyone will
advantage of
benefit from this protection.
sorts
 It is critical to counter argue that
this protection may be
counterproductive, as it erodes the
spirit of adventure and the
acceptance that life is after all
unpredictable in some respects.

Ultimately to conform or not is entirely dependent on the context.


In the developing world, the following two scenarios are most common: in some cases, the
youth today cannot but conform to regimes not out of choice but out of circumstance. In
other cases, youth are fighting to break free of primitive practices in order to better their
lives.
In the developed world, the youth most influenced by wealth and the media resist conformity
but at the expense of worthy traditions. On the other hand, youth living in poverty in the
developed world have no choice but to conform to the pressures and expectations of life in
the developed world in order to survive, oftentimes resorting to means of survival which
compromises belief systems and values.

  14
VICTORIA JUNIOR COLLEGE
PRELIMINARY EXAMINATION

GENERAL PAPER 8806/1

Monday
30 August 2010 TIME: 1 hour 30 minutes

TIME 1 hour 30 minutes

INSTRUCTIONS TO CANDIDATES

Answer only one question from Paper 1.

Write your name and C.T. group in the spaces provided in your foolscap paper. Write
your question number in the left-hand margin of your foolscap paper.

If you use more than one sheet of paper for the question, fasten your sheets together.

At the end of the examination, hand in your answers to Paper 1.

If you are unable to answer a question in the Paper, send in a blank sheet, giving your
Name, C.T. group, and the number of that Paper.

INFORMATION FOR CANDIDATES

In Paper 1, all questions carry equal marks.

The total time of 1 hour and 30 minutes includes ten minutes for you to study the
question before you begin your answer. You may take notes during this time if you wish.

This question paper consists of 2 printed pages including this page.


Answer one question from this paper.

Your answer should be between 500 and 800 words.

1. How far is imagination valued in your society?

2. To what extent is your country an ideal place for both young and old?

3. Real change is always effected by the passion of individuals, never by the


actions of governments. To what extent do you agree with this statement?

4. Singles are a marginalised group in Singapore society. Do you agree?

5. We are devoting too much of our lives to formal education. To what extent is
this true?

6. Discuss the effectiveness of technology as a means of tackling social


problems.

7. The media have encouraged a herd mentality. Is this true?

8. “We cannot all afford to be saints.” To what extent is volunteerism a rich


man’s pastime?

9. The key to happiness is managing your expectations. Do you agree?

10. How far is the artist essential in contributing to the development of society?

11. How far would you agree that the only divide worth addressing is the rich-
poor divide?

12. "Decades of the good life have made Singaporeans soft... ... what they once
considered luxuries, they now think of as necessities." (Dr Lee Wei Ling). Is
this a fair comment?
VICTORIA JUNIOR COLLEGE
JC2 PRELIMINARY EXAMINATION

General Paper 8806/2

Monday
30 August 2010 TIME: 1 hour 30 minutes

INSTRUCTIONS TO CANDIDATES

Write your Name and Civics Class in the spaces provided in the answer
paper.

Answer all questions.

If you need to use extra sheets of writing paper for a question, fasten them
together with the answer booklet.

INFORMATION FOR CANDIDATES

The number of marks is given in brackets [ ] at the end of each question.

This question paper consists of 9 printed pages including this page.


The Rise Of The Consumer Society

1 Historians looking back at the 20th century may well conclude that it was the century of the
consumer society. What has undoubtedly had the most significant impact upon the way of life of
ordinary people in industrial societies over the last century has been the mass availability of
consumer goods. No aspect of everyday life has been left untouched by the arrival of the
consumer society: from the dwellings we inhabit; the modes of transportation and 5
communication we use to travel and converse with others; the foodstuffs we eat; the clothes we
wear; the ways in which we spend our leisure time; indeed, the very structure of daily time itself
– all of these changes are not simply the outcome of scientific and technological advancements,
but in fact reflect a profound and fundamental change in the way that, as societies, we organise
our very means to existence. 10

2 Our grandparents, and if not them then certainly their parents, would without doubt have lived in
a world which would have been just about as different from our own as it is possible to imagine.
It would not have been simply that their material needs – for food, clothing, and shelter – would
have been satisfied quite differently to our own, but indeed their very dreams, hopes, and
aspirations, for a long and healthy life, to have healthy and successful children, would actually 15
have been fulfilled under a very different framework of expectations. Consumer society is not
just about the satisfaction of needs, but in many ways it is about the forms through which we
view the world and our position within it. This is not to imply that our way is somehow an
improvement over that of our grandparents; simply that the arrival of consumer society during
the last one hundred years has transformed not only our material existence but also our 20
ontology, our very being itself.

3 The buying and selling of time is the central activity of the leisure industry in a capitalist
economy. This is what differentiates modern popular culture from the foIk culture which
preceded it. Football, for instance, developed in the 19th century into its various modern forms
out of local, traditional games, but by 1900 had become a professional sport. The players 25
earned their living by the game, and their spectators paid for the pleasure of watching.

4 Throughout the present century, adults have berated their children for preferring to buy the
products of popular culture rather than “make their own entertainment”. This offers a clear
distinction between foIk culture and popular culture: foIk culture is something you make; popular
culture is something you buy. 30

5 Among the many fundamental social changes brought by the Industrial Revolution was the way
in which leisure was systematised. The factory system regulated time in a new way, making
time-at-work different from time-not-working. In a sense that had not been true in preindustrial
culture, time-not working became an empty period that needed to be occupied. For much of the
19th century, leisure, which can be defined as the non-productive use of time, remained the 35
prerogative of the propertied classes. But by the early 20th century, the notion of leisure spread
down through the social system in Europe and North America and new activities came into
existence to occupy leisure time.

6 The city amusements of the Iate 19th century were prototypes for ephemeral consumption:
saloons, dance halls, pool rooms and roller-skating rinks; dime novels and illustrated papers, 40
circuses, amusement parks, burlesque shows and professional sports; melodrama and cheap
seats in the theatres and concert halls. Most spectacular of all were the great exhibitions of the
second half of the 19th century, beginning at London's Crystal Palace in 1851 and culminating in
the Chicago World's Columbian Exposition in 1893 and the Paris Exhibition of 1900. These
architectural extravagances, thrown up for a summer to display the new wonders of the worlds 45

2
of industry and commerce, were available to anyone who could pay.

7 This was not enough. By the turn of the century, industrial production had developed to the point
where the economy required consumption as well as production to be managed. 19th-century
industrialists had regarded their labour-force as a necessity for production, but in the early 20th
century, it was recognised that capitalism must encourage the workers to be purchasers as well. 50

8 Mass advertising developed out of a need to persuade people to buy. Manufacturers merely
made products, but advertisers "manufactured consumers". Advertising involved a shift in
cultural values away from a Victorian Protestant ethic which demanded that production,
property, and personal behaviour be controlled. It encouraged an ethic which permitted pleasure
and even sensuality. Advertising came to concentrate not on describing the product it was 55
selling, but on the emotional satisfactions that its consumption would afford its purchaser. It
preached the new, "therapeutic" doctrine of 20th century capitalism, that its citizens should seek
self-realisation through the intense experiences brought about through buying products for their
leisure time.

9 In 1899 the American economist Thorstein Veblen argued that "the conspicuous consumption of 60
valuable goods" became the principal means by which members of the Leisure Class
demonstrated their social standing to each other and to the rest of society. As he was describing
the nature and implications of a consumer culture, American capitalism was spreading that
culture, and the idea of leisure, to far larger sectors of the population. Several years later, a
writer on fashion noted that as wealth or social status were the basic selling points of most 65
clothes, "the styles should go as far as possible in proving that the owner does not have to work
for a living". From the 1920s onward, the idea of stylistic obsolescence in which annual models
introduce new season's fashions spread out from automobiles to other types of consumer goods
as the way to maintain a constant demand, through what Charles Kettering of General Motors
called "the organised creation of dissatisfaction". 70

10 In 1929 Christine Frederick wrote, "Consumptionism is the name given to the new doctrine; and
it is admitted today to be the greatest idea that America has to give to the world; the idea that
workmen and masses be looked upon not simply as workers and producers, but as
consumers.... Pay them more, sell them more, prosper more is the equation." This was the
American Dream: an economic perpetual motion machine which made everyone appear equally 75
prosperous. Fantastic visions of the American Dream drew immigrants, as novelist Michael Gold
described in 1930, "In the window of a store that sold Singer Sewing Machines in our
(Romanian) village. One picture had in it the tallest building I had ever seen. It was called a
skyscraper. At the bottom of it walked the proud Americans. The men wore derby hats and had
fine moustaches and gold watch chains. The women wore silks and satins, and had proud faces 80
like queens. Not a single poor man or woman was there; everyone was rich."

3
Name: ____________________ Content /35

CT Group: _________________ Language /15

Total /50

Paper 2 (50 Marks)

(Note that 15 marks out of 50 will be awarded for your use of language.)

Note: When a question asks for an answer IN YOUR OWN WORDS AS FAR AS
POSSIBLE and you select the appropriate material from the passage for your answer, you
must still use your own words to express it. Little credit can be given to answers which only
copy words or phrases from the passage

1) How has „the mass availability of consumer goods‟ (lines 3-4) had an impact on the way
of life in industrial society? Use your own words as far as possible. [2]

_________________________________________________________________________

_________________________________________________________________________

_________________________________________________________________________

_________________________________________________________________________

2) From paragraph 2, show how the arrival of the consumer society changed the way
people perceive themselves. Use your own words as far as possible. [2]

_________________________________________________________________________

_________________________________________________________________________

_________________________________________________________________________

_________________________________________________________________________

4
3) What does the word „managed‟ (line 48) suggest about the way consumption is handled?
[1]

_________________________________________________________________________

_________________________________________________________________________

4) In what way is 20th Century capitalism „therapeutic‟ (line 57) and why does the author put
inverted commas round this word? Use your own words as far as possible. [2]

_________________________________________________________________________

_________________________________________________________________________

_________________________________________________________________________

_________________________________________________________________________

5) Which two words in paragraph 8 suggest that 20th Century capitalism is like a religion? [1]

_________________________________________________________________________

_________________________________________________________________________

6) How does „stylistic obsolescence‟ (line 67) work in the fashion industry? What is its role in
„the organised creation of dissatisfaction‟ (line 70)? Use your own words as far as possible.
[3]

_________________________________________________________________________

_________________________________________________________________________

_________________________________________________________________________

_________________________________________________________________________

_________________________________________________________________________

5
7) In the passage, the author explains how the concept of leisure has changed over time
and the characteristics it has taken on. Using materials from paragraphs 3 to 6, summarise
his views in no more than 120 words, not counting the opening words printed below. Use
your own words as far as possible. [8]

The author claims that _______________________________________________

_________________________________________________________________________

_________________________________________________________________________

_________________________________________________________________________

_________________________________________________________________________

_________________________________________________________________________

_________________________________________________________________________

_________________________________________________________________________

_________________________________________________________________________

_________________________________________________________________________

_________________________________________________________________________

_________________________________________________________________________

_________________________________________________________________________

_________________________________________________________________________

_________________________________________________________________________

_________________________________________________________________________

_________________________________________________________________________

_________________________________________________________________________

6
8) Why is the American Dream described as an „economic perpetual motion machine‟ (line
75)? How does the Singer Sewing Machine advertisement illustrate this dream? Use your
own words as far as possible. [3]

______________________________________________________________________

______________________________________________________________________

______________________________________________________________________

______________________________________________________________________

______________________________________________________________________

9) Give the meaning of each of the following words as they are used in the passage. You
may write the answer in a word or short phrase. [5]

i) structure (line 7)

___________________________________________________________________

ii) berated (line 27)

___________________________________________________________________

iii) culminating (line 43)

___________________________________________________________________

iv) ethic (line 54)

___________________________________________________________________

v) afford (line 56)

___________________________________________________________________

7
10) The author examines the various effects of consumerism on society. To what extent is
your society reflective of the consumer culture as outlined in the passage? Do you think
there is cause for concern? Refer to some of the ideas raised in the passage as well as your
own knowledge and experiences. [8]

_________________________________________________________________________

_________________________________________________________________________

_________________________________________________________________________

_________________________________________________________________________

_________________________________________________________________________

_________________________________________________________________________

_________________________________________________________________________

_________________________________________________________________________

_________________________________________________________________________

_________________________________________________________________________

_________________________________________________________________________

_________________________________________________________________________

_________________________________________________________________________

_________________________________________________________________________

_________________________________________________________________________

_________________________________________________________________________

_________________________________________________________________________

_________________________________________________________________________

_________________________________________________________________________

_________________________________________________________________________

_________________________________________________________________________

_________________________________________________________________________

8
_________________________________________________________________________

_________________________________________________________________________

_________________________________________________________________________

_________________________________________________________________________

_________________________________________________________________________

_________________________________________________________________________

_________________________________________________________________________

_________________________________________________________________________

_________________________________________________________________________

_________________________________________________________________________

_________________________________________________________________________

_________________________________________________________________________

_________________________________________________________________________

_________________________________________________________________________

_________________________________________________________________________

_________________________________________________________________________

_________________________________________________________________________

_________________________________________________________________________

_________________________________________________________________________

_________________________________________________________________________

_________________________________________________________________________

_________________________________________________________________________

_________________________________________________________________________

End of Paper

9
VJC JC2 2010 Prelim Paper 2
Answer Scheme

1) How has „the mass availability of consumer goods‟ (line 3-4) had an impact on
the way of life in industrial society? Use your own words as far as possible. [2]

From the Text Paraphrase

no aspect of everyday life has been all-pervasive effects felt/ all areas of life
untouched (l. 4) affected [1/2]
…profound and fundamental change Deep/significant [1/2] and at the most
..the way we organise our very foundational level [1/2] transformed
means of existence (l. 9) the ways we live [1/2]

Examiners’ comments: Most students attempted to paraphrase all the


examples from l. 5 – 7 instead of identifying the topic sentence in l. 4 and
paraphrasing that. Their answers stopped after their list of examples.

2) From paragraph 2, show how the arrival of the consumer society changed the
way people perceive themselves. Use your own words as far as possible. [2]

From the Text Paraphrase

their material needs…very different Same things deemed as necessary


framework of expectations (l. 13-16) [1/2] now measured against new/higher
yardsticks [1/2]
…the forms through which we view the changed how we see [1/2] the world
world and our position within it (l.17- and our role/place [1/2] in it.
18)

Examiners’ comments: Few students really understood the author’s point


about ‘different expectations’ and there were clumsy attempts to
paraphrase the second part of the answer.

3) What does the word „managed‟ (l. 48) suggest about the way consumption is
handled? [1]

From the Text Interpretation

Manipulated/ carefully controlled with


an agenda in mind

1
Examiners’ Comments: A small number of students actually thought that
consumption needed to be reduced/reined in, which is the opposite of what
is suggested!

4) In what way is 20th Century capitalism „therapeutic‟ (line 57) and why does the
author put inverted commas round this word? Use your own words as far as
possible in your answer. [2]

From the Text Paraphrase/Interpretation

...emotional satisfactions… (l.56) self-awareness attained via the strong


self-realisation through the intense feelings of gratification [1/2]
experiences brought about through encountered from purchasing desired
buying products (l. 58-59) goods [1/2]
Answer to be inferred The author wants to show he is
skeptical about whether people can
attain self-awareness/ self-fulfilment
through purchasing commodities

Examiners‟ comments: While most students understood the question, their


answers „dropped‟ details or lacked accuracy. The sense of gratification
comes from purchasing goods, not merely consuming them. Author‟s
intent should be made clear for the second question.

5) Which two words in paragraph 8 suggest that 20th Century capitalism is like a
religion? [1]

From the Text Answer

preached (l. 57), doctrine (l. 57) [1/2


each]

Examiners’ comments: There were all kinds of strange answers offered that
have nothing to do with religion specifically – ‘ethics’ (which can be
secular), ‘self-realisation’, even ‘sensuality’! ‘Victorian Protestant’ in the
passage refers to the 19th C, not 20th C capitalism.

6) How does „stylistic obsolescence‟ (line 67) work in the fashion industry? What
is its role in „the organised creation of dissatisfaction‟ (line 70)? Use your own
words as far as possible. [3]

From the Text Paraphrase


…annual models introduce new latest trends/fads brought in [1/2]

2
season‟s fashions (l.68-69) render the older versions outdated
[1/2]
the way to maintain a constant results in a non-
demand (l.69) stop/continuous/sustained
craving/desire[1/2] for the
newest/latest [1/2]
through… the organised creation of by a systematic/deliberate/calculated
dissatisfaction (l.70) way [1/2] to induce a sense of
discontent [1/2]

Examiners’ comment: Many students explained the previous point – that


clothes styles should show that ‘the owner does not need to work for a
living’ (l. 66-67), when the explanation of ‘stylistic obsolescence’ actually
came after the sentence beginning with “from the 1920s onwards...
Note: the word ‘obsolete’ comes from the noun ‘obsolescence’! That itself
is a clue!
Strong scripts showed a very clear and well-explained link between
‘stylistic obsolescence’ and ‘organised creation of dissatisfaction’

8) Why is the American Dream described as an „economic perpetual motion


machine‟ (line 75)? How does the Singer Sewing Machine advertisement
illustrate this dream? Use your own words as far as possible. [3] [+1 bonus]

From the Text Paraphrase/Explanation


economic perpetual motion machine Wealth constantly created [1/2]
(l.75) which made everyone appear resulting in everybody as rich as
equally prosperous (l. 75-76) another [1/2]
…skyscraper…derby hats…gold Picture paints an image of wealth and
watch chains… silks and satins luxury attained by all [1]
(l.79-80)…everyone was rich (l.81)
…proud Americans…proud faces like Everyone enjoys sense of superiority
queens … (l. 79-80) because of high social status attained
[1]
Fantastic vision…everyone appears Picture creates an illusion and is an
equally prosperous (l.75-76) unrealistic portrayal, hence „Dream‟
[bonus mark]

Examiners’ comment: Each part of the answer needs to be carefully


explained in students’ own words. There was much lifting of words like
‘equally prosperous’ and ‘everyone was rich’. There was also too much
description of what the people in the ad were wearing instead of explaining
what they represent. Few got the bonus point, nor picked up the point
about the ‘proud’ people (significant enough for the author to repeat the
word).

3
9) Vocabulary

Word 1m ½m 0m

„…the very organisation; arrangement; form; system framework;


structure of configuration layout; model
daily time…‟
„…adults have scolded; rebuked; chided; criticised; condemned
berated their reprimanded complained
children…‟ (line about
27)
„culminating in reaching a high point; resulting; accumulating
the …‟ (line 43) concluding
„It encouraged value system; code of value; moral culture
an ethic…‟ (line principles; moral code conduct
54)
„…would afford give; bestow; grant; confer; Benefit; can pay for;
its purchaser…‟ provide reward access; permit
(line 56)

7) In the passage, the author explains how the concept of leisure has changed
over time and the characteristics it has taken. Using materials from paragraphs 3
to 6, summarise his views in no more than 120 words, not counting the opening
words printed below. Use your own words as far as possible. [8]

From the text Possible paraphrase


a. The buying and selling of time is the [The author claims that] the
central activity of the leisure industry in business of leisure principally
a capitalist economy. involves the trading of time.
b. Football [...] by 1900 had become a This would explain how sport
professional sport. The players earned became a full-time job for some,
their living by the game, and their funded by ticket sales for viewing
spectators paid for the pleasure of the matches.
watching.
c. Throughout the present century, adults Traditional forms of leisure
have berated their children for preferring involve personal effort / active
to buy the products of popular culture engagement / creation /
rather than “make their own construction / is more hands-on
entertainment”. This offers a clear whereas the contemporary forms
distinction between foIk culture and can just be purchased.
popular culture: foIk culture is
something you make; popular culture is
something you buy.
d. Among the many fundamental social The Industrial Revolution caused
changes brought by the Industrial leisure to become clearly

4
Revolution was the way in which leisure demarcated and designated time /
was systematised. The factory system time set aside for a specified /
regulated time in a new way, making particular purpose
time-at-work different from time-not-
working.
e. In a sense that had not been true in From then on, when one was not
preindustrial culture, time-not working at work, one had to fill the void /
became an empty period that needed use the free time by engaging in
to be occupied leisure.
f. For much of the 19th century, leisure, Industrialisation meant that the
which can be defined as the non- common people had such time
productive use of time, remained the away from work. OR The concept
prerogative of the propertied classes. of leisure was therefore
But by the early 20th century, the notion democratised / no longer
of leisure spread down through the restricted to the upper echelons of
social system society in the 1900s
g. new activities came into existence to Novel ways to engage in leisure
occupy leisure time were developed / appeared
h. The city amusements of the Iate 19th Some took the form of affordable
century were prototypes for ephemeral and fleeting opportunities for
consumption: saloons, [...] and cheap people to spend their money.
seats in the theatres and concert halls.
i. Most spectacular of all were the great Money was all one needed to
exhibitions of the second half of the enjoy viewing the monuments that
19th century [...]. These architectural celebrate mass production, trade
extravagances, thrown up for a summer and excess in the late 1800s.
to display the new wonders of the
worlds of industry and commerce, were
available to anyone who could pay.
(Maximum 9 points)

Examiners’ comments: The need to paraphase continues to be a problem


for a large number of students, with much lifting of keywords or inaccurate
or clumsy attempts at using own words. The same problem with
paraphrasing examples came up, especially when it for Paragraph 6;
instead of listing the examples, students need to identify what the author
wants to say about these examples – they provided fleeting enjoyment;
required people to spend money, but were affordable for the masses; they
represented the showy excesses of urban entertainment.

5
10) The author examines the various effects of consumerism on society. To what extent
is your society reflective of the consumer culture as outlined in the passage? Do you
think there is cause for concern? Refer to some of the ideas raised in the passage as
well as your own knowledge and experiences. [8]

Mark Range Descriptor

A 1. Students must make reference to the author‟s views


6-8 2. They must evaluate ideas from the passage with evidence of
balanced treatment
3. They must provide justification in the form of substantive evidence
drawn from current society. Justification provided should be
sound, directly relevant to the ideas from the passage and
demonstrate mature understanding of the various issues and
trends with regard to consumer culture in modern society
4. Students must take a stand by quantifying the extent of their
arguments with regard to the society‟s experience of consumer
culture
5. Students demonstrate high degree of coherence and clarity in their
arguments

Key ideas from the passage

 No aspect of everyday life has been left untouched by the arrival of


the consumer society. Consumer culture reflects a profound
change in the way we organize our very means of existence. It
transforms not only our material existence but also our ontology,
our very being self.
 Folk culture is something you make; popular is something you buy
 The notion of leisure spread down the social system and new
activities came into existence to occupy leisure time.
 The city amusements and the architectural extravagances are
constructed for ephemeral consumption
 Mass advertising developed out of a need to persuade people to
buy, creating a constant demand from consumers. It encouraged
an ethic which permitted pleasure and even sensuality.
 The conspicuous consumption of valuable goods became the
principal means by which members of the Leisure Class
demonstrated their social standing to the rest of the society

Evaluation of author‟s arguments

Application to Singapore:
 The pervasiveness of consumer culture in our everyday lives
in Singapore – the proliferation of consumer goods due to the
increasing spending power and demands of local consumers. This
social phenomenon is most evidently reflected in the IT and the
fashion industries. Eg: the annual Great Singapore Sale (GSS), the
various IT exhibitions, the frenzied promotion of the Apple‟s iPhone

6
series by different mobile phone operators

 How the consumption and possession of consumer goods


define our identities and reflect our social standing in sociey–
The types of consumer products that we purchase often go beyond
the value of consumption as they also represent our socio-
economic status and reveal who we are as consumers. Eg: certain
luxury goods are targeted at wealthy consumers in society (high-
end fashion labels such as Prada and Gucci), while some are
meant for the average consumer. There are also certain products
that are targeted at the youth market (eg: trendy IT gadgets), while
some are meant for the elderly buyers in society. Contemporary
popular culture seems to perpetuate the idea of ostentatious
consumption (as evident in the popular saying “If you‟ve got it,
flaunt it”) as a means of reflecting the extravagant and carefree
lifestyles of the rich and famous in society. (Eg: popular Hollywood
movies such as “Sex and the City” and “The Devil Wears Prada”
etc)

 The consumption of leisure has become democratised in


many societies today, including Singapore- One need not be
from the wealthy class to have access to certain forms of leisure in
society. Tickets for many arts-related performances and events are
made more affordable for the public in Singapore (eg. during the
Singapore Arts Festival). The increasing demands of consumers
have also lowered the production costs of many consumer goods
and services. For instance, in the IT industry, the prices of laptops
and mobile phones are also becoming increasingly affordable for
the masses during the annual IT exhibitions in Singapore.

 Some consumer goods are created for the ephemeral


consumptions of the buyers- Many fashionable items and trends
are simply passing fads among consumers who are driven by the
herd mentality to make similar consumer choices. The seasonal
and ever-changing trends of styles and designs in the apparel and
IT industries are reflective of the fact that many commercial
companies seek to capitalise on the ephemeral consumption of the
buyers. Eg: the transient nature of fashion styles according to
different seasonal trends and the fleeting trends of the different
models and designs of mobile phones and technological gadgets in
the local IT market.

 How mass advertising perpetually creates an insatiable desire


in every consumer – Advertisements constantly offer a flurry of
products and services that remind consumers of what they lack.
The marketing gimmicks employed through highly sensational
images and videos often seek to titillate the fancy of public
consumers. Eg: how the advertisements of a wide range of beauty
and counter-aging products tend to evoke a sense of imperfection
and insecurity among consumers. The popular idea of “retail

7
therapy” among many local shoppers is indicative of the fact of
some people derive great satisfaction from splurging on consumer
goods.

 However, one must also acknowledge the fact that while


consumerism is prevalent in Singapore, it is not an all-
pervasive phenomenon in society. Certainly not every local
consumer here pursues an extravagant lifestyle and many
individuals even reject the idea of consumerism. Many from low-
income families in Singapore still lead a frugal lifestyle. One also
cannot deny the fact that the recent financial recession has also
been a wakeup call to many as more and more people are now
more prudent when it comes to making consumer choices.

To argue there is cause for concern

 How consumer culture tends to conflict with our traditional


values with regard to our patterns of consumption – Traditional
values such as the importance of being thrifty and spending within
one‟s means were highly regarded by our forefathers of the
previous generations. Consumer culture tends to breed an
unhealthy mentality among young consumers that the more they
spend, the happier they will be. It encourages an ethic that permits
wasteful behavior especially among consumers. The various cases
of how young consumers in Singapore have fallen prey to credit
card debts suggest that more and more people have apparently
spent beyond their means due to their consumerist and
extravagant lifestyles.

 How unethical advertising has been a grave cause for concern


among local consumers – As much as advertisers seek to create
an insatiable desire among consumers, many of them have
violated ethical guidelines in their attempts to persuade the
consumers. Eg: The manipulation and distortion of facts in the
advertisements of some cosmetic treatments and spa services in
Singapore and how these cases have raised concerns from the
Consumers Association of Singapore (CASE).

 From the environmental point of view, green activists have


constantly lamented that consumerism in modern society
leads to excessive wastage of resources- It exacerbates the
“throw-away culture” among irresponsible consumers due to their
wasteful lifestyles and patterns of consumption. Eg: The “Bring
Your Own Bag” campaign at NTUC Fair Price bears testimony to
the fact that excessive use of plastic bags among shoppers is
indeed a cause for concern in local society as it leads to
environmental degradation.

8
Not a cause for concern

 Consumerism need not be seen as a negative phenomenon in


society. From the economic point of view, our active consumptions
are said to have a positive impact on the nation‟s economy. It is
through the vibrant consumer markets and the increasing spending
power of buyers that the domestic economy can be further
stimulated and sustained.

 Tighter regulations have been imposed by the Consumers


Association of Singapore to provide checks and balances on
the accuracy and reliability of commercial advertisements and
products. For instance, Case Trust, the local accreditation arm of
the Consumer Association, serves as an important platform to
ensure fair trading and transparency in the consumer market. The
recent recall of toxic toys in the local market serves as a
demonstration of how the Consumers Association is committed to
protecting consumer welfare.

 The idea of “ethical consumerism” has become more


prevalent among consumers in Singapore. Local consumers
are becoming more and more educated about the ill effects of
consumerism with regard to the various environmental concerns.
For example, Body Shop products appeal to the consumers who
are against the idea of animal testing in the production of many
cosmetic and facial products. *Scape, a new youth-centric
shopping mall located next to Cineleisure Orchard, is targeted at
young consumers who believe in ethical consumerism. Tenants
there possess a strong sense of corporate social responsibility and
are involved in raising funds and awareness about various social
causes. Besides, there have also been many initiatives
implemented to counter the adverse effects of consumerism. (eg:
recycling of paper and plastic bags in many shopping areas). The
National Environment Agency (NEA) has been working with local
partners such as the Singapore Environment Council and the
Public Waste Collectors to provide more recycling bins in various
shopping places to serve the public.

Mark Range Descriptor

B 1. Students refer to the passage, at least in passing


(3-5) 2. They identify ideas from the passage, possibly with minor
misrepresentation of the points
3. They raise issues, but discussion is limited or superficial and not
necessarily a balanced treatment given
4. Students attempt to evaluate the extent to which the ideas are
reflected, but not fully informed or convincing;
5. Answer shows an adequate level of understanding of the issues
but tends to be less thorough with limited development of ideas

9
6. Students show a fair degree of organisation, coherence and clarity.
Some minor logical gaps that do not lead to ambiguity in meaning

Mark Range Descriptor

C 1. Students fail to address the given requirements of the question


(1-2) 2. Fail to discuss ideas from the passage on the extent to which ideas
are reflected in society
3. Fail to relate evaluation to actual societal phenomena
4. Answer tends to be a mere summary or restatement of the text
rather than an evaluation of it
5. Answer shows very limited knowledge/understanding of the issues
and a higher incidence of misinterpretation with very thin support.
6. Coherence, organisation and clarity are in question

Examiners’ comment: Basic requirements were sometimes not met. There were
still AQ answers that read like a standalone mini-essay on Consumerism without
reference to passage or author’s views. A small number did not frame the answer
in the context of ‘your society’ but referred to the situation in a broad and general
context of ‘today’s society’. Yet another major problem is not answering both
parts of the question, and it is often the second part (‘cause for concern’?) that
gets short shrift.

Some students misrepresented the author. He does not believe, for example, that
everyone will be equally rich due to capitalism. Students are therefore maligning
him when they criticise him for being naïve in not seeing the problem of the rich-
poor divide in capitalistic societies! Other students included points that were not
from the passage such as Americanisation of our lifestyles. If those are part of the
students’ own knowledge and experiences, that must be clearly indicated so that
it does not sound like students are putting words in the author’s mouth.

Examples are really important as evidence. Some scripts lacked examples which
led to many unsubstantiated claims. Others gave inappropriate examples, which
became ineffective and did not support the point made. For example, on ads that
play on ‘emotional satisfaction’, an example chosen was ‘finger-licking good’ fried
chicken! This does not capture what the author wants us to understand – the way
ads play on our emotional needs and manipulate us psychologically, without
giving us any meaningful information about the product itself.

The better scripts met the basic requirements, covering a number of key ideas
from the passage and used them to apply to their society. They showed how some
of the signs of a consumeristic culture and its effects could be seen in their
society, with substantive evidence provided. Their evaluation of these effects
were well-considered, without sounding too alarmist or dismissive, when they
considered whether there was cause for concern.

10
Candidate’s Name CTG

YISHUN JUNIOR COLLEGE


JC2 PRELIMINARY EXAMINATION 2010

H1 GENERAL PAPER 8806/01


Paper 1 18 August 2010
0800 - 0930h

YISHUN JUNIOR COLLEGE YISHUN JUNIOR COLLEGE YISHUN JUNIOR COLLEGE YISHUN JUNIOR COLLEGE YISHUN JUNIOR COLLEGE
YISHUN JUNIOR COLLEGE YISHUN JUNIOR COLLEGE YISHUN JUNIOR COLLEGE YISHUN JUNIOR COLLEGE YISHUN JUNIOR COLLEGE
YISHUN JUNIOR COLLEGE YISHUN JUNIOR COLLEGE YISHUN JUNIOR COLLEGE YISHUN JUNIOR COLLEGE YISHUN JUNIOR COLLEGE
YISHUN JUNIOR COLLEGE YISHUN JUNIOR COLLEGE YISHUN JUNIOR COLLEGE YISHUN JUNIOR COLLEGE YISHUN JUNIOR COLLEGE
YISHUN JUNIOR COLLEGE YISHUN JUNIOR COLLEGE YISHUN JUNIOR COLLEGE YISHUN JUNIOR COLLEGE YISHUN JUNIOR COLLEGE
YISHUN JUNIOR COLLEGE YISHUN JUNIOR COLLEGE YISHUN JUNIOR COLLEGE YISHUN JUNIOR COLLEGE YISHUN JUNIOR COLLEGE
YISHUN JUNIOR COLLEGE YISHUN JUNIOR COLLEGE YISHUN JUNIOR COLLEGE YISHUN JUNIOR COLLEGE YISHUN JUNIOR COLLEGE
YISHUN JUNIOR COLLEGE YISHUN JUNIOR COLLEGE YISHUN JUNIOR COLLEGE YISHUN JUNIOR COLLEGE YISHUN JUNIOR COLLEGE
YISHUN JUNIOR COLLEGE YISHUN JUNIOR COLLEGE YISHUN JUNIOR COLLEGE YISHUN JUNIOR COLLEGE YISHUN JUNIOR COLLEGE
YISHUN JUNIOR COLLEGE YISHUN JUNIOR COLLEGE YISHUN JUNIOR COLLEGE YISHUN JUNIOR COLLEGE YISHUN JUNIOR COLLEGE
YISHUN JUNIOR COLLEGE YISHUN JUNIOR COLLEGE YISHUN JUNIOR COLLEGE YISHUN JUNIOR COLLEGE YISHUN JUNIOR COLLEGE
YISHUN JUNIOR COLLEGE YISHUN JUNIOR COLLEGE YISHUN JUNIOR COLLEGE YISHUN JUNIOR COLLEGE YISHUN JUNIOR COLLEGE
YISHUN JUNIOR COLLEGE YISHUN JUNIOR COLLEGE YISHUN JUNIOR COLLEGE YISHUN JUNIOR COLLEGE YISHUN JUNIOR COLLEGE
YISHUN JUNIOR COLLEGE YISHUN JUNIOR COLLEGE YISHUN JUNIOR COLLEGE YISHUN JUNIOR COLLEGE YSIHUN JUNIOR COLLEGE
YISHUN JUNIOR COLLEGE YISHUN JUNIOR COLLEGE YISHUN JUNIOR COLLEGE YISHUN JUNIOR COLLEGE YSIHUN JUNIOR COLLEGE
YISHUN JUNIOR COLLEGE YISHUN JUNIOR COLLEGE YISHUN JUNIOR COLLEGE YISHUN JUNIOR COLLEGE YSIHUN JUNIOR COLLEGE
YISHUN JUNIOR COLLEGE YISHUN JUNIOR COLLEGE YISHUN JUNIOR COLLEGE YISHUN JUNIOR COLLEGE YSIHUN JUNIOR COLLEGE
TIME
YISHUN JUNIOR1COLLEGE
hour 30 minutes
YISHUN JUNIOR COLLEGE YISHUN JUNIOR COLLEGE YISHUN JUNIOR COLLEGE YSIHUN JUNIOR COLLEGE
YISHUN JUNIOR COLLEGE YISHUN JUNIOR COLLEGE YISHUN JUNIOR COLLEGE YISHUN JUNIOR COLLEGE YSIHUN JUNIOR COLLEGE

INSTRUCTIONS TO CANDIDATES
Write your name and CTG in the spaces at the top of this page.
Answer one of the questions.
Write your answer on a separate answer paper.
Write the number of the question attempted on your answer script.
Place the question paper on top of your answer script and fasten all the sheets of paper
together.
If you are unable to attempt the paper, submit a blank sheet stating your name and CTG.

INFORMATION FOR CANDIDATES


All questions carry equal marks.
(Note that 20 marks out of 50 will be awarded for your use of language.)

Setters: Mdm Samsiah Bte Sanip FOR EXAMINER’S USE


Miss Beverly Low Content /30
Mr Edward Heng Language /20
Miss Mardiana Bte Rahmad TOTAL /50
Miss Michelle Neo

This question paper consists of 2 printed pages including this page.


[Turn over]
2

PAPER 1

Answer one question from this paper.

Answers should be between 500 and 800 words in length.

1 ‘Consumerism, and not terrorism, is the new evil.’ Comment.

2 How far should political decisions be allowed to influence sport?

3 ‘Happiness is merely an illusion.’ What is your view?

4 ‘Being entrepreneurial nurtures creativity but kills the environment.’


Discuss.

5 Can science and technology save the world?

6 Why bother going to school when there are other avenues for
learning?

7 ‘Competition brings about the best results but the worst in people.’
Discuss.

8 With reference to your society, is it realistic for parents to believe that


their children can be relied upon in their old age?

9 Assess the impact of new technologies on efforts to tackle crime in


your country.

10 ‘Music expresses that which cannot be said and on which it is


impossible to be silent.’ Do you agree?

11 ‘Advertisers constantly invent cures to which there is no disease.’ Is


this a fair comment?

12 ‘Values, not abilities, establish a career.’ What do you think?


Candidate’s Name CTG

YISHUN JUNIOR COLLEGE


JC2 PRELIMINARY EXAMINATION 2010

H1 GENERAL PAPER 8806/02


Paper 2 18 August 2010
1000h - 1130h
YISHUN JUNIOR COLLEGE YISHUN JUNIOR COLLEGE YISHUN JUNIOR COLLEGE YISHUN JUNIOR COLLEGE YISHUN JUNIOR COLLEGE
YISHUN JUNIOR COLLEGE YISHUN JUNIOR COLLEGE YISHUN JUNIOR COLLEGE YISHUN JUNIOR COLLEGE YISHUN JUNIOR COLLEGE
YISHUN JUNIOR COLLEGE YISHUN JUNIOR COLLEGE YISHUN JUNIOR COLLEGE YISHUN JUNIOR COLLEGE YISHUN JUNIOR COLLEGE
YISHUN JUNIOR COLLEGE YISHUN JUNIOR COLLEGE YISHUN JUNIOR COLLEGE YISHUN JUNIOR COLLEGE YISHUN JUNIOR COLLEGE
YISHUN JUNIOR COLLEGE YISHUN JUNIOR COLLEGE YISHUN JUNIOR COLLEGE YISHUN JUNIOR COLLEGE YISHUN JUNIOR COLLEGE
YISHUN JUNIOR COLLEGE YISHUN JUNIOR COLLEGE YISHUN JUNIOR COLLEGE YISHUN JUNIOR COLLEGE YISHUN JUNIOR COLLEGE
YISHUN JUNIOR COLLEGE YISHUN JUNIOR COLLEGE YISHUN JUNIOR COLLEGE YISHUN JUNIOR COLLEGE YISHUN JUNIOR COLLEGE
YISHUN JUNIOR COLLEGE YISHUN JUNIOR COLLEGE YISHUN JUNIOR COLLEGE YISHUN JUNIOR COLLEGE YISHUN JUNIOR COLLEGE
YISHUN JUNIOR COLLEGE YISHUN JUNIOR COLLEGE YISHUN JUNIOR COLLEGE YISHUN JUNIOR COLLEGE YISHUN JUNIOR COLLEGE
YISHUN JUNIOR COLLEGE YISHUN JUNIOR COLLEGE YISHUN JUNIOR COLLEGE YISHUN JUNIOR COLLEGE YISHUN JUNIOR COLLEGE
YISHUN JUNIOR COLLEGE YISHUN JUNIOR COLLEGE YISHUN JUNIOR COLLEGE YISHUN JUNIOR COLLEGE YISHUN JUNIOR COLLEGE
YISHUN JUNIOR COLLEGE YISHUN JUNIOR COLLEGE YISHUN JUNIOR COLLEGE YISHUN JUNIOR COLLEGE YISHUN JUNIOR COLLEGE
YISHUN JUNIOR COLLEGE YISHUN JUNIOR COLLEGE YISHUN JUNIOR COLLEGE YISHUN JUNIOR COLLEGE YISHUN JUNIOR COLLEGE
YISHUN JUNIOR COLLEGE YISHUN JUNIOR COLLEGE YISHUN JUNIOR COLLEGE YISHUN JUNIOR COLLEGE YSIHUN JUNIOR COLLEGE
TIME JUNIOR
YISHUN 1 hour 30 minutes
COLLEGE YISHUN JUNIOR COLLEGE YISHUN JUNIOR COLLEGE YISHUN JUNIOR COLLEGE YSIHUN JUNIOR COLLEGE
YISHUN JUNIOR COLLEGE YISHUN JUNIOR COLLEGE YISHUN JUNIOR COLLEGE YISHUN JUNIOR COLLEGE YSIHUN JUNIOR COLLEGE
YISHUN JUNIOR COLLEGE YISHUN JUNIOR COLLEGE YISHUN JUNIOR COLLEGE YISHUN JUNIOR COLLEGE YSIHUN JUNIOR COLLEGE
INSTRUCTIONS TO CANDIDATES
YISHUN JUNIOR COLLEGE YISHUN JUNIOR COLLEGE YISHUN JUNIOR COLLEGE YISHUN JUNIOR COLLEGE YSIHUN JUNIOR COLLEGE
YISHUN JUNIOR COLLEGE YISHUN JUNIOR COLLEGE YISHUN JUNIOR COLLEGE YISHUN JUNIOR COLLEGE YSIHUN JUNIOR COLLEGE
Write your name and CTG in the spaces at the top of this page.
Answer all the questions.
At the end of the examination, fasten the cover page, passages and all your work securely
together.
INFORMATION FOR CANDIDATES
The number of marks is given in brackets [ ] at the end of each question.
(Note that 15 marks out of 50 will be awarded for your use of language.)

Question 1 2 3 4 5 6 7 8 9 10 11
Marks 1 2 2 2 8 1 1 3 2 5 8
Total
Marks

For Examiner’s Use


Content /35
Language /15
TOTAL /50

This question paper consists of 10 printed pages including this page.


[Turn over]
2

Passage 1
Oliver Burkeman writes…

1 29 October 1969 has been asserted as the day “the infant Internet uttered its first words" 1
when a computer made contact with a second one, miles away at the Stanford Research
Institute, and an undergraduate, Charley Kline, tapped out a message. To say that the
rest is history is the emptiest of clichés but trying to express the magnitude of what has
really happened since is an undertaking that exposes the limits of language. Twelve 5
years after Kline's first message on the Arpanet, as it was then known, only 213
computers were on the network but 14 years later, 16 million people were online, and
email was beginning to transform the world. The first really utilizable web browser was
not launched until 1993, but by 1995 we had Amazon, by 1998 Google, and by 2001,
Wikipedia, at which point there were 513 million people online. Today, the figure is 1.7 10
billion.

2 Unless you are 15 years old or younger, you have lived through the dotcom bubble and
bust: the birth of Friendster, eBay, Facebook, Twitter and blogging, the browser wars,
Google Earth, and file-sharing controversies. You would also have seen the
transformation of the music industry, consumer banking, travel agencies, dating and 15
retail. In the entire history of mankind, nothing has changed so dramatically as computer
communications.

3 On the other hand, the breakthrough in 1969 was decidedly down-to-earth. The Arpanet
was not intrinsically intended as a secret weapon to humble the Soviets but simply to
enable researchers to access computers remotely, because computers were still vast 20
and expensive, and the scientists needed a way to share resources. Linking computers
by telephone lines was possible but glacially slow. As every computer in the network
must be connected, by a dedicated line, to every other, you could not connect more than
a handful of machines without everything becoming monstrously complex and costly. The
solution was “packet switching” – breaking data down in blocks that could be routed 25
around any part of the network that happened to be free, before getting reassembled at
the other end.

4 One of the most intriguing aspects about the growth of the Internet is this: to a select
group of technological thinkers, the surprise was not how quickly it spread globally,
remaking business, culture and politics – but that it took so long to get underway. Even 30
when computers were mainly run on punch-cards and paper tape, there were whispers
that they would work in a network, rather than individually. In 1945, the American
presidential science adviser, Vannevar Bush, was already envisaging the "memex", a
device in which "an individual stores all his books, records, and communications", which
would be linked to each other by "a mesh of associative trails", like web links. While 35
these seemed to have spurred on and shaped the evolution of the Internet, others had
frenzied visions of the world's machines turning into a kind of conscious brain.

5 Despite all these predictions, the arrival of the Internet was never a matter of inevitability.
It was a crucial idiosyncrasy of the Arpanet that its funding came from the American
military – but that the millions ended up on university campuses, with researchers who 40
embraced an anti-establishment ethic. Instead of smothering their research in the utmost
secrecy – as expected of a cold war project aimed at winning a technological battle
against Moscow – they publicised every step of their thinking, in documents known as
3

„Requests For Comments‟. Deliberately or not, these researchers helped encourage a


vibrant culture of hobbyists on the fringes of academia – students and rank amateurs 45
who built their own electronic bulletin-board systems and eventually FidoNet, a network
to connect them to each other. Arguably, these instances of unofficial tinkering did as
much to make the Internet a public platform. The hobbyists, by making unofficial
connections into the main system, were the first to open the Internet up to all comers.

6 And so without most of us quite noticing when it happened, the web went from being a 50
new curiosity to a background condition of everyday life: I have no memory of there being
an intermediate stage, when, say, half the information I needed on a particular topic
could be found online, while the other half still required visits to libraries.

7 It is absurd to compact the series of events into a few phrases: the dotcom boom, the
unprecedented dotcom bust, the growing "digital divide", and then the flourishing, over 55
the last seven years, of Web 2.0. It is only this latter period that has revealed the true
capacity of the web for "generativity": for the sharing of blogs, podcasts and videos by
anyone with access to the Internet, for the undermining of totalitarian regimes and
organisation of political resistance via radicalised websites, and for the use of sites such
as Twitter and Facebook to create (and ruin) friendships, and spread fashions and 60
rumours. But you almost certainly know all this: it is part of what, in many parts of the
world, we now call "just being alive".

Passage 2
Peter Singer writes…

1 Google‟s decision to stop acceding to the Chinese government‟s request to block its 1
citizens‟ access to websites deemed unacceptable has been greeted with enthusiasm in
liberal democracies worldwide. But the Australian government recently proposed
legislation to forbid websites containing material on child pornography, bestiality, incest,
graphic "high-impact" images of violence, instructions on crime, detailed descriptions of 5
the use of proscribed drugs, and how-to information on suicide for the terminally ill. A
readers' poll in the Sydney Morning Herald showed 96% opposed to those proposed
measures and only 2% in favour. More readers voted than in any previous poll shown on
the newspaper's website, and the result is the most one-sided.

2 The Internet, like the steam engine, is a technological breakthrough that changed the 10
world. Today, an Internet connection offers, at your fingertips, an amount of information
previously available only to those with access to the world's greatest libraries. Indeed,
what is available through the Internet dwarfs those libraries, and it is easier to find what
you need. Remarkably, this came about with no central planning, no governing body, and
no overall control, other than a system for allocating the names of websites and their 15
addresses.

3 That something so significant could spring up independently of governments and big


business led many to believe that the Internet can bring the world a new type of freedom.
It is as if an inherently decentralised and individualist technology had realised an
anarchist vision that would have seemed utterly utopian. That may explain why so many 20
people believe intensely that the Internet should be left completely unfettered. Perhaps
because Google has been all about making information more widely available, its
4

collaboration with China's official Internet censors has been seen as a deep betrayal.
The hope of Internet anarchists was that repressive governments would have only two
options: accept the Internet with its limitless possibilities of spreading information, or 25
restrict Internet access to the ruling elite and turn their backs on the 21st century, as
North Korea has done. Reality is more complex. The Chinese government was never
going to yield to Google's demand that it abandon Internet censorship. The authorities
will undoubtedly find ways of replacing the services that Google provided – at some cost,
and maybe with some loss of efficiency, but the Internet will remain fettered in China. 30

4 Even with censorship, the Internet is a force for change. Just the month before, when the
governor of China's Hubei province threatened a journalist and grabbed her recorder
after she queried about a local scandal, journalists, lawyers and academics used the
Internet to object. A web report critical of the governor's behaviour stayed up for 18 hours
before censors ordered it taken down. By then, the news was already widely dispersed. 35
Likewise, in Cuba, Yoani Sánchez's blog „Generation Y‟ has broken barriers that
conventional media could not. Although the Cuban government has blocked access to
the website on which the blog is posted, it is available around the world in many
languages, and distributed within Cuba on CDs and flash drives.

5 The new freedom of expression brought by the Internet goes far beyond politics. People 40
relate to each other in new ways, posing questions about how we should respond to
people when all that we know about them is what we have learned through a medium
that permits all kinds of anonymity and deception. We discover new things about what
people want to do and how they want to connect to each other. Do you live in an isolated
village and have unusual hobbies, special interests, or sexual preferences? You will find 45
someone online with whom to share them. Can't get to a doctor? You can check your
symptoms online – but can you be sure that the medical website you are accessing is
reliable? Technology can be used for good or for bad, and it is too soon to reach a
verdict on the Internet. In the 18th century, who could have foreseen that the
development of the steam engine would have an impact on earth's climate? Even if the 50
Internet does not fulfil the anarchist dream of ending repressive government, we are still
only beginning to grasp the extent of what it will do to the way we live.
5

Read the passages and then answer all the questions which follow. Note that up to fifteen For
Examiner’s
marks will be given for the quality and accuracy of your use of English throughout this Use
paper.

NOTE: When a question asks for an answer IN YOUR OWN WORDS AS FAR AS
POSSIBLE and you select the appropriate material from the passage for your answer, you
must still use your own words to express it. Little credit can be given to answers which only
copy words and phrases from the passage.

From Passage 1

1. Explain what the author means by “the infant Internet uttered its first words” (line 1).

[1]

2. What does the author imply by stating that “trying to express the magnitude of what has
really happened since is an undertaking that quickly exposes the limits of language”
(lines 4-5)?

[2]

3. Using your own words as far as possible, explain the author‟s claim in paragraph 6.

[2]
6

4. The author illustrates the capacity of the web for “generativity” (line 57). Choose any For
Examiner’s
two examples and explain how each reflects “just being alive” (line 62). Use your own Use
words as far as possible.

[2]

5. Using material from paragraphs 3 to 5 of the passage, summarise the factors


contributing to, and the misconceptions about, the development of the Internet.

Write your summary in no more than 130 words not counting the opening words
which are printed below. Use your own words as far as possible.

One of the factors contributing to the development of the Internet …..


7

For
Examiner’s
Use

[8]

From Passage 2

6. In paragraph 1, what do the findings of the poll indicate about the readers‟ attitude
towards the websites disapproved by the Australian government?

[1]

7. “…what is available through the Internet dwarfs those libraries” (line 13). Explain why
the writer uses the word „dwarfs‟.

[1]

8. How has Yoani Sánchez's blog “broken barriers that conventional media could not”
(lines 36-37)? Use your own words as far as possible.

[3]
8

9. “The new freedom of expression brought about by the Internet goes far beyond politics” For
Examiner’s
(line 40). Cite two forms of the new freedom of expression and their implications. Use

[2]

From both passages

10. Give the meaning of the following words as they are used in the passage.

Write your answer in one word or a short phrase.

a. dramatically (Passage 1, line 16)


-------------------------------------------------------------- [1]

b. compact (Passage 1, line 54)


------------------------------------------------------------------- [1]

c. unprecedented (Passage 1, line 55)


--------------------------------------------------------- [1]

d. anarchist (Passage 2, line 20)


------------------------------------------------------------------ [1]

e. fettered (Passage 2, line 30)


-------------------------------------------------------------------- [1]

[Total: 5]
9

11. With reference to the arguments from both passages, consider how the Internet has For
Examiner’s
been harnessed in your society and whether it has led to similar problems. Use
10

For
Examiner’s
Use

[8]
11

Yishun Junior College


General Paper
2010 JC2 Preliminary Examination
Suggested Answer Scheme

From Passage 1

1. Explain what the author means by “the infant Internet uttered its first words” (line 1).
[1 mark]
Text material Suggested answer
“the infant Internet uttered its first words" The author refers to the beginning of
when a computer made contact with a communication from one computer to
second one, miles away…Kline's first another [1]
message on the Arpanet… as when a baby starts to articulate
meaningful sounds. [bonus 1]
[Give „0‟ if only the second part is given.]

2. What does the author imply by stating that “trying to express the magnitude of what
has really happened since is an undertaking that quickly exposes the limits of
language” (lines 4-5)? [2 marks]
Text material Suggested answer
…trying to express the magnitude of what a. The growth in the number of Internet
has really happened since is an users (in terms of numerical evidence)
undertaking that exposes the limits of [½]
language. Twelve years after Kline's first
message on the Arpanet… only 213 b. and the emergence of new
computers were on the network… 14 years applications/ platforms (in terms of
later, 16 million people were online, and functions) [½] have developed
email was beginning to transform the [Give ½ if a. and b. are summarised as
world. The first really utilizable web progress/ developments.]
browser was not launched until 1993, but
by 1995 we had Amazon, by 1998 Google, c. to the extent that we are unable to
and by 2001, Wikipedia, at which point adequately grasp/ understand and
there were 513 million people online. represent in words the impact of the
Today, the figure is 1.7 billion. Internet on the users. [1]

3. Using your own words as far as possible, explain the author‟s claim in paragraph
6. [2 marks]
Text material Suggested answer (Inferred)
Without most of us quite noticing when it a. It is not discernible/ distinct/ evident at
happened, the web went from being a new which point in time (the time factor) [1]
curiosity to a background condition of
everyday life: b. we started to take the Internet for
granted in the conduct of our daily
I have no memory of there being an affairs, [½]
intermediate stage, when, say, half the
information I needed on a particular topic c. instead of treating it as a novelty that is
could be found online, while the other half not always essential. [½]
still required visits to libraries.
12

4. The author illustrates the capacity of the web for “generativity” (line 57). Choose any
two examples and explain how each reflects “just being alive” (line 62). Use your
own words as far as possible. [2 marks]

Text material Suggested answer


It is only this latter period that has revealed a. In blogging, podcasting and video-
the true capacity of the web for sharing users are engaged in
"generativity": for the sharing of blogs, producing/ conveying their views,
podcasts and videos by anyone with experiences and ideas to fulfil the need
access to the Internet, to reach out to/ communicate with
others. [1]

for the undermining of totalitarian regimes b. The websites of activists are used as a
and organisation of political resistance via platform to bring about change by
radicalised websites, challenging authoritarian rulers and
coordinating efforts to oppose/ defy
real or perceived lapses in governing.
[1]

and for the use of sites such as Twitter and c. On social networking sites, users build
Facebook to create (and ruin) friendships, or destroy relationships, and set
and spread fashions and rumours. trends. [1]

But you almost certainly know all this: it is Any two of the above to score full marks.
part of what, in many parts of the world, we
now call "just being alive".

5. Using material from paragraphs 3 to 5 of the passage, summarise the factors


contributing to, and the misconceptions about, the development of the Internet.

Write your summary in no more than 130 words not counting the opening words
which are printed below. Use your own words as far as possible. [8 marks]

One of the factors contributing to the development of the Internet…

Note: There must be at least 2 misconceptions to warrant a full 8 marks. Otherwise, deduct 1
mark if 8 or more points are provided out of which there is only 1 misconception.
No Text material Suggested answer
1 …simply to enable researchers to access a. There was a need to provide scientists an
computers remotely, because computers affordable means to contribute their
were still vast and expensive, and the expertise over long distances.
scientists needed a way to share
resources.
2 The Arpanet was not intrinsically intended b. The early Internet was misconstrued as a
as a secret weapon to humble the Soviets covert technology to outdo the Russians.

3 Linking computers by telephone lines... c. Developed to overcome the problems


glacially slow... must be connected, by a posed by traditionally connecting
dedicated line, to every other…connect computers - which was cumbersome and
more than a handful of machines… expensive -
everything… monstrously complex and
costly.
13

4 The solution was “packet switching” – d. by channelling data more efficiently,


breaking data down in blocks… routed
around any part of the network that
happened to be free, before getting
reassembled at the other end.
5 to a select group of technological e. experts had mistakenly expected the
thinkers, the surprise was… that it took so Internet to take off speedily.
long to get underway.
6 …whispers that they would work in a f. While visionary forecasting/ predictions
network… the American presidential might account for the Internet‟s capacity for
science adviser… was already saving massive data [1] and
envisaging… a device…"an individual
stores all his books, records, and g. collaboration/ networking [1],
communications"… linked to each other
by "a mesh of associative trails"... these
seemed to have spurred on and shaped
the evolution of the Internet,
7 …others had frenzied visions of the h. there were unfounded fears of the Internet
world's machines turning into a kind of evolving into an artificial intelligence/ to
conscious brain. have a mind of its own.
8 …the arrival of the Internet was never a i. Misunderstood to be a matter of time, the
matter of inevitability. advent of the Internet
9 …crucial idiosyncrasy of the Arpanet… j. was actually due to an unexpected/
funding came from the American unplanned rechanneling of grant from the
military… millions ended up on university military to the universities,
campuses, with researchers who
embraced an anti-establishment ethic… k. where liberal-minded researchers were
publicised every step of their thinking… open with their findings.
10 …helped encourage a vibrant culture of l. This created a conducive environment for
hobbyists on the fringes of academia – those who are technologically savvy and
students and rank amateurs who built outside the research circle to set up
their own electronic bulletin-board rudimentary/ early versions of the Internet.
systems and eventually FidoNet, a
network to connect them to each other.
11 Arguably, these instances of unofficial m. Their informal attempts to establish links to
tinkering did as much to make the Internet the central official system paved the way
a public platform. The hobbyists, by for making the Internet accessible to the
making unofficial connections into the public.
main system, were the first to open the
Internet up to all comers.
Sample summary
One of the factors contributing to the development of the Internet was the need to
provide scientists an affordable means to contribute expertise over long distances. But,
this was misconstrued as a covert technology to outdo the Russians. Developed to
overcome the cumbersome and expensive way of traditionally connecting computers, by
channelling data more efficiently, some experts mistakenly expected a speedy takeoff.
While visionary predictions account for the Internet‟s capacity for saving massive data
and collaboration, there were unfounded fears of it evolving into artificial intelligence.
Misunderstood to be a matter of time, its advent followed an unplanned rechanneling of
grant from the military to the universities, where liberal-minded researchers disclosed
their findings. A conducive environment emerged for the technologically savvy, outside
the research circle, to set up rudimentary versions of it. Informal attempts to establish
links to the central official system facilitated public access to the Internet.
(all 11 points in 137 words)
14

From Passage 2

6. In paragraph 1, what do the findings of the poll indicate about the readers‟ attitude
towards the websites disapproved by the Australian government? [1 mark]
Text material Suggested answer (Inferred)
…the Australian government recently a. Perhaps most readers feel that they
proposed legislation to forbid websites are discerning enough/ confident
containing material on child pornography, enough to recognise and ward off/
bestiality, incest, graphic "high-impact" manage the dangers/ the undesirable
images of violence, instructions on crime, influences posed by these websites. [1]
detailed descriptions of the use of
proscribed drugs, and how-to information Or
on suicide for the terminally ill. A readers'
poll in the Sydney Morning Herald showed b. Most readers do not feel immediately
96% opposed to those proposed measures threatened by/ worried about the
and only 2% in favour. More readers voted dangers posed by the websites.
than in any previous poll shown on the
newspaper's website, and the result is the
most one-sided.

7. „…what is available through the Internet dwarfs those libraries‟ (line 13). Explain
why the writer uses the word „dwarfs‟. [1 mark]
Text material Suggested answer
Today, an Internet connection offers, at a. The author wants to emphasise that
your fingertips, an amount of information the mass of data/ knowledge in the
previously available only to those with Internet far surpasses the collection of
access to the world's greatest libraries. information available through the most
Indeed, what is available through the reputable libraries in the world. [1]
Internet dwarfs those libraries…
Or

b. The author wants to highlight the


contrast between the amount of data
accessible via the Internet and the
massive but still limited amount in the
world‟s established libraries. [1]

8. How has Yoani Sánchez's blog „broken barriers that conventional media could not‟
(lines 36-37)? Use your own words as far as possible. [3 marks]
Text material Suggested answer
A web report critical of the governor‟s a. Unlike conventional media which could
behaviour stayed up for 18 hours before not avoid official censorship that easily,
censors ordered it taken down. By then, non-conventional media like the stated
the news was already widely dispersed. blog could circumvent government
censorship. [1]
Likewise in Cuba …blog „Generation Y‟
has broken barriers that conventional b. While the former could not disseminate
media could not. Although the Cuban information as extensively within a
government has blocked access to the relatively short period, the latter get
website on which the blog is posted… circulated quickly to other parts of the
15

world while also overcoming the


it is available around the world in many language barrier. [1]
languages,

…and distributed within Cuba in CDs and c. Its digital nature makes it easy to copy,
flash drives. store and transmit, unlike the usually
bulky format of the conventional media.
[1]

(Students must identify the barriers that


conventional media poses and explain how
the new media – through the blog example
– breaks such barriers/ restrictions.)

9. “The new freedom of expression brought about by the Internet goes far beyond
politics” (line 40). Cite two forms of the new freedom of expression and their
implications. [2 marks]
Text material Suggested answer
People relate to each other in new ways, a. People enjoy a novel way of
posing questions about how we should communicating with others. However,
respond to people when all that we know this form of communication may not
about them is what we have learned through always be truthful allowing all forms
a medium that permits all kinds of anonymity of dishonesty. [1]
and deception.

We discover new things about what people b. Individuals gather fresh insights into
want to do and how they want to connect to a whole range of desires, situations
each other. Do you live in an isolated village and inclinations and chances are
and have unusual hobbies, special interests, there will be a match. [1]
or sexual preferences? You will find
someone online with whom to share them.

Can't get to a doctor? You can check your c. Individuals may find answers to
symptoms online – but can you be sure that questions they have about almost
the medical website you are accessing is anything at all. However, the
reliable? credibility of such information is
questionable. [1]

Any two examples to receive full marks


for this question.
16

From both passages


10. Give the meaning of the following words as they are used in the passage.

Write your answer in one word or a short phrase.

Vocabulary 1 mark ½ mark 0 mark

a. dramatically (adverb) in a revolutionary greatly; surprisingly;


(Passage 1, line 16) way; radically; significantly; shockingly;
In the entire history of drastically considerably intensively
mankind, nothing has changed
so dramatically as computer
communications.
b. compact (verb) compress; summarise shorten;
(Passage 1, line 54) condense abbreviate;
It is absurd to compact the limit; restrict
series of events into a few
phrases…
c. unprecedented never seen/ unplanned;
(adjective) heard/ happened shocking;
(Passage 1, line 55) before extraordinary;
…the unprecedented dotcom unparalleled;
bust,,, unmatched
d. anarchist (adjective) anti-government; rebellious; political;
(Passage 2, line 20) against radical revolutionist
It is as if an inherently established rule/
decentralised and individualist central control
technology had realised an
anarchist vision that would
seem utterly utopian.
e. fettered (adjective) strongly controlled; tied down;
(Passage 2, line 30) restrained, regulated manipulated;
…but the Internet will remain restricted, chained;
fettered in China. censored limited

11. With reference to the arguments from both passages, consider how the Internet has
been harnessed in your society and whether it has led to similar problems. [8 marks]

You might also like